You are on page 1of 432

Principles of COST

ACCOUNTING

EDITED BY JOHAN HEFER


Oxford University Press Southern Africa (Pty) Ltd
Vasco Boulevard, Goodwood, Cape Town, Republic of South Africa
P O Box 12119, N1 City, 7463, Cape Town, Republic of South Africa
Oxford University Press Southern Africa (Pty) Ltd is a subsidiary of
Oxford University Press, Great Clarendon Street, Oxford OX2 6DP.
The Press, a department of the University of Oxford, furthers the University’s objective of excellence in research, scholarship, and
education by publishing worldwide in
Oxford New York
Auckland Cape Town Dar es Salaam Hong Kong Karachi Kuala Lumpur Madrid Melbourne Mexico City Nairobi New Delhi Shanghai
Taipei Toronto
With offices in
Argentina Austria Brazil Chile Czech Republic France Greece
Guatemala Hungary Italy Japan Poland Portugal Singapore South Korea Switzerland Turkey Ukraine Vietnam
Oxford is a registered trade mark of Oxford University Press in the UK and in certain other countries
Published in South Africa
by Oxford University Press Southern Africa (Pty) Ltd, Cape Town
Principles of Cost Accounting
ISBN 978 0 19 904548 8
eISBN 978 0 19 904874 8
© Oxford University Press Southern Africa (Pty) Ltd 2013
The moral rights of the author have been asserted
Database right Oxford University Press Southern Africa (Pty) Ltd (maker)
First published 2013
All rights reserved. No part of this publication may be reproduced, stored in a retrieval system, or transmitted, in any form or by any
means, without the prior permission in writing of Oxford University Press Southern Africa (Pty) Ltd, or as expressly permitted by law, or
under terms agreed with the appropriate designated reprographics rights organization. Enquiries concerning reproduction outside the
scope of the above should be sent to the Rights Department, Oxford University Press Southern Africa (Pty) Ltd, at the address above.
You must not circulate this book in any other binding or cover and you must impose this same condition on any acquirer.
Publishing manager: Alida Terblanche
Publisher: Marisa Montemarano
Development editor: Ilse Groenewald
Editor: Mark Ronan
Project manager: Sarah Floor
Indexer: Michel Cozien
Designer: Cecile Steenkamp
Cover design: Jade Benjamin
Cover images: Alamy
Typeset in 10 pt on 13 pt Adobe Caslon Pro by PH Setting cc
Digital conversion by Realmdigital (Pty) Ltd.
Unless otherwise indicated, certain questions and solutions are based on examination questions and solutions from the University of Cape
Town. The examination questions and solutions in their original examination form are the sole copyright of the University of Cape Town.
Adapted questions and solutions are copyright of the authors and/or Oxford University Press.
Abridged Contents
PART 1 Fundamental concepts
Chapter 1 Introduction to cost accounting
Chapter 2 Cost concepts, classification and behaviour
Chapter 3 Cost elements: Accounting for inventory
Chapter 4 Cost elements: Inventory management and control
Chapter 5 Cost elements: Labour
Chapter 6 Cost elements: Overheads
Chapter 7 Cost statements

PART 2 Costing systems


PART 2.1 Costing systems
Chapter 8 Job costing
Chapter 9 Contract costing
PART 2.2 Process costing
Chapter 10 Process costing: Single products
Chapter 11 Process costing: Joint products and by-products
PART 2.3 Other costing systems
Chapter 12 Standard costing
Chapter 13 Activity-based costing
Chapter 14 Direct and absorption costing systems

PART 3 Budgets
Chapter 15 Operational budgets
Chapter 16 Cash budgets
Chapter 17 Flexible and fixed budgets
Chapter 18 Capital budgeting

PART 4 Decision making


Chapter 19 Cost–volume–profit analysis
Chapter 20 Linear programming
Chapter 21 Relevant costing
Chapter 22 Pricing decisions
Contents
Preface
Contibutors

PART 1 Fundamental concepts


Chapter 1 Introduction to cost accounting
1.1 Introduction
1.2 Managing an organisation
1.3 Defining financial, management and cost accounting
1.3.1 Roles and objectives of cost and management accounting
1.3.2 Why is there a need for cost accounting?
1.4 Cost and management accounting guidelines
1.5 Ethical issues in cost and management accounting
1.6 Summary

Chapter 2 Cost concepts, classification and behaviour


2.1 Introduction
2.2 Cost objectives
2.2.1 Step 1: Categorising costs
2.2.2 Step 2: Assigning costs to cost objectives
2.3 Unit costs
2.4 Types of inventory
2.5 Job-costing and process-costing systems
2.6 Management accounting concepts
2.7 A brief note on value-added tax
2.8 Summary

Chapter 3 Cost elements: Accounting for inventory


3.1 Introduction
3.2 The accounting treatment of inventory
3.2.1 Cost of inventory
3.2.2 Valuation of inventory
3.3 Using an inventory ledger card
3.4 Summary

Chapter 4 Cost elements: Inventory management and control


4.1 Introduction
4.2 The inventory-management cycle
4.2.1 Setting norms for inventory levels
4.2.2 Identifying the need to order inventory
4.2.3 Ordering inventory
4.2.4 Inventory storage
4.2.5 Internal and external distribution and consumption of inventory
4.3 Economic order quantity
4.3.1 Cost of holding inventory
4.3.2 Cost of ordering inventory
4.3.3 Calculating the economic order quantity
4.3.4 Calculating the reorder point
4.4 Advanced inventory-management systems
4.4.1 Requirement planning systems
4.4.2 Just-in-time systems
4.5 Summary

Chapter 5 Cost elements: Labour


5.1 Introduction
5.2 Labour as a cost element
5.2.1 Salaries
5.2.2 Wages
5.2.3 Direct labour
5.2.4 Indirect labour
5.2.5 Normal hours
5.2.6 Overtime hours
5.2.7 Overtime premium
5.2.8 Idle time
5.2.9 Gross remuneration
5.2.10 Net remuneration
5.2.11 Cost to employer
5.3 Calculating remuneration payable
5.4 Recovery of direct labour cost
5.5 Labour cost as an element in determining total cost of production or service
5.6 Accounting entries for recording wages and salaries
5.6.1 Recording gross wages, payments to funds and payment of net wages
5.6.2 Recording wages as a cost element
5.7 Summary

Chapter 6 Cost elements: Overheads


6.1 Introduction
6.2 Types of overheads
6.2.1 Infrastructure overheads
6.2.2 Indirect materials
6.2.3 Indirect labour
6.2.4 Non-manufacturing overheads
6.3 Cost behaviour of overheads
6.3.1 Fixed manufacturing overheads
6.3.2 Variable manufacturing overheads
6.3.3 Semi-variable overheads
6.3.4 Step-fixed overheads
6.3.5 Cost behaviour of non-manufacturing overheads
6.4 Allocating overhead costs
6.4.1 Cost vs benefit of cost system
6.4.2 Timelines of a costing system
6.4.3 Cost centres
6.5 Process of allocating overhead costs to products and services
6.5.1 Step 1: Allocation of overheads to all cost centres
6.5.2 Step 2: Allocation of service department costs to production departments
6.5.3 Step 3: Computing overhead rates for each production cost centre
6.5.4 Step 4: Allocating overhead costs to products and services
6.5.5 Step 5: Calculating under- or over-absorption of overheads
6.6 Summary

Chapter 7 Cost statements


7.1 Introduction
7.2 The manufacturing process
7.3 Cost classifications in manufacturing firms
7.4 Flow of costs in manufacturing firms
7.5 Cost-accounting procedure in manufacturing firms
7.6 Cost of production and cost of sales statement
7.7 Statement of financial performance/income statement
7.8 Integrated and interlocking accounting systems
7.8.1 Integrated accounting system
7.8.2 Interlocking accounting system
7.9 Reconciliation of cost-accounting and financial-accounting profits
7.9.1 Reasons for difference in profits between cost accounts and financial accounts
7.9.2 Reconciliation process
7.10 Summary

PART 2 Costing systems


Chapter 8 Job costing
8.1 Introduction
8.2 Application of job costing
8.3 Cost flow and source documents
8.4 Job description and source documents used
8.4.1 Purchase and receipt of raw materials
8.4.2 Material issued
8.4.3 Recording labour
8.4.4 Allocation of overheads
8.4.5 Transfer to finished goods
8.4.6 Sale of finished goods
8.4.7 Actual overheads
8.5 Summary
Chapter 9 Contract costing
9.1 Introduction
9.2 Challenges encountered in contract costing
9.3 Definitions of key terms specific to contract costing
9.3.1 Certified work
9.3.2 Uncertified work
9.3.3 Retention money
9.3.4 Extra work
9.3.5 Material on site
9.4 Elements of contract costs
9.4.1 Contract costs
9.4.2 General head office costs
9.5 Recording contract costs
9.5.1 Contract account and contract debtor account
9.5.2 Completed contracts
9.5.3 Incomplete contracts
9.6 Recognising interim profit on incomplete contracts
9.6.1 Guidelines or principles for estimating interim profit to be recognised on incomplete
contracts
9.7 Summary

Chapter 10 Process costing: Single products


10.1 Introduction
10.2 Process costing
10.3 Expected production: Normal and abnormal losses, and abnormal gains
10.4 The process account
10.4.1 Process account: Normal losses with no scrap value
10.4.2 Process account: Normal losses with scrap value
10.4.3 Abnormal losses with no scrap value
10.4.4 Abnormal losses with scrap value
10.4.5 Abnormal gains with no scrap value
10.4.6 Abnormal gains with scrap value
10.4.7 Process costing for normal and abnormal losses and abnormal gains: Summary
10.5 Accounting for incomplete production (work in process)
10.5.1 Equivalent units
10.5.2 Opening work in process
10.5.3 Accounting for losses in a work-in-process system
10.6 Summary

Chapter 11 Process costing: Joint products and by-products


11.1 Introduction
11.2 Joint products
11.3 Methods of allocating joint costs
11.3.1 Physical measure
11.3.2 Sales value at split-off point
11.3.3 Net realisable value method
11.3.4 Constant gross profit percentage
11.4 By-products
11.5 Joint or by-products: Making sales decisions
11.6 Summary

Chapter 12 Standard costing


12.1 Introduction
12.2 Standard costs
12.2.1 Material standards
12.2.2 Labour standards
12.2.3 Standard overhead rate
12.3 Production cost variances
12.3.1 Standard cost cards and production budgets
12.3.2 Direct material variances
12.3.3 Direct labour variances
12.3.4 Variable overhead expenditure (rate) variance
12.3.5 Variable overhead efficiency variance
12.3.6 Total variable overhead variance
12.3.7 Fixed overhead expenditure variance
12.3.8 Sales margin price variance
12.3.9 Sales margin volume variance
12.4 Reconciliation statements
12.5 Advanced variances
12.5.1 Direct material mix variance
12.5.2 Direct material yield variance
12.5.3 Direct labour mix variance
12.5.4 Sales mix variances
12.6 Summary

Chapter 13 Activity-based costing


13.1 Introduction
13.2 Definitions relating to ABC
13.3 Advantages and disadvantages of ABC
13.3.1 Advantages of ABC
13.3.2 Disadvantages of ABC
13.4 Implementing an ABC system
13.4.1 Step 1: Identifying the organisation’s main activities and grouping them into cost
pools
13.4.2 Step 2: Identifying cost drivers for each activity
13.4.3 Step 3: Calculating activity rates
13.4.4 Step 4: Allocating costs of each cost pool to cost objects
13.5 Comparing ABC and traditional volume-based costing systems
13.6 When is it appropriate to introduce ABC?
13.7 ABC in service and retail organisations
13.8 Summary

Chapter 14 Direct and absorption costing systems


14.1 Introduction
14.2 Benefits and shortcomings of direct costing
14.3 Benefits and shortcomings of absorption costing
14.4 Drafting the statement of financial performance using direct and absorption costing
14.5 Reconciling profits determined by direct and absorption costing
14.6 Summary

PART 3 Budgets
Chapter 15 Operational budgets
15.1 Introduction
15.2 The budgeting process
15.3 Benefits and shortcomings of budgets
15.4 Master budgets
15.4.1 Revenue budget
15.4.2 Production budget
15.4.3 Direct-materials budget
15.4.4 Direct-labour budget
15.4.5 Manufacturing-overheads budget
15.4.6 Cost-of-sales budget
15.4.7 Operating-expenditure budget
15.4.8 Pro forma statement of comprehensive income
15.4.9 Other components of the master budget
15.5 Summary

Chapter 16 Cash budgets


16.1 Introduction
16.2 Basics of drawing up a cash budget
16.3 Purpose of a cash budget
16.4 Summary

Chapter 17 Flexible and fixed budgets


17.1 Introduction
17.2 Differences between fixed and flexible budgets
17.3 Preparing a flexible budget
17.4 Disadvantages of fixed budgets
17.5 Budgetary control: Responsibility accounting and performance reports
17.6 Flexible budgets and overheads
17.6.1 Variable overhead variances
17.6.2 Fixed overhead variances
17.7 Summary
Chapter 18 Capital budgeting
18.1 Introduction
18.2 Types of investment projects
18.2.1 Replacement projects
18.2.2 Expansion projects
18.2.3 Mutually exclusive projects
18.2.4 Independent projects
18.3 Determining accounting profits
18.4 Methods for making capital-budgeting decisions
18.4.1 Time value of money
18.4.2 Discount rates
18.4.3 Accounting rate of return
18.4.4 Payback period
18.4.5 Discounted payback period
18.4.6 Net present value
18.4.7 Internal rate of return
18.5 Summary

PART 4 Decision making


Chapter 19 Cost–volume–profit analysis
19.1 Introduction
19.2 CVP analysis (breakeven analysis)
19.2.1 Why is CVP analysis important?
19.2.2 CVP analysis and cost behaviour
19.2.3 Comparison of accounting and economist’s models of CVP
19.2.4 Other assumptions of CVP analysis
19.3 Marginal (variable) costing and absorption-costing income statements
19.4 Contribution (marginal) income
19.4.1 Total contribution
19.4.2 Contribution per unit
19.4.3 Contribution margin ratio
19.5 Breakeven point
19.5.1 Breakeven units (quantity or volume)
19.5.2 Breakeven value
19.6 Application of CVP analysis
19.6.1 Margin of safety
19.6.2 Expected or targeted profit
19.6.3 Breakeven analysis for multiple products
19.6.4 Changes in selling price, variable cost per unit and fixed costs
19.7 Other methods of conducting a breakeven analysis
19.7.1 Algebraic method
19.8 Limitations of CVP analysis
19.9 Summary
Chapter 20 Linear programming
20.1 Introduction
20.2 Concepts and principles of linear programming
20.3 Understanding limiting factors
20.4 The linear programming model
20.5 Limitations of linear programming
20.6 Summary

Chapter 21 Relevant costing


21.1 Introduction
21.2 Relevant financial factors
21.3 Making financial and non-financial decisions
21.4 Summary

Chapter 22 Pricing decisions


22.1 Introduction
22.2 Short-term and long-term pricing decisions
22.3 Market-based pricing
22.4 Cost-based pricing
22.5 Summary

References
Glossary
Index
Preface
Managing an organisation, whether in the manufacturing, retail, services, not for profit, or government
sector, requires important decisions to be made. These decisions could include whether to continue
or discontinue a product or service, determining the selling price of a product or service, or
measuring the performance of the organisation.
Relevant and accurate information is required to enable those responsible for making such
decisions to make good decisions. The discipline of cost and management accounting provides the
financial information to enable managers to do so.
Unlike financial accounting, cost and management accounting is not rules-based, but is rather
principles-based. This book, aimed at undergraduate students, attempts to explain the most important
principles of cost and management accounting to enable students to design and implement an
appropriate costing system.
The discipline of cost and management accounting is relevant, exciting and challenging, and the
authors of this book hope that it will assist students in preparing for their future careers.

Johan Hefer
Contributors
Reynier Boersma is a Senior Lecturer in the Department of Commercial Accounting at the University
of Johannesburg. He is the subject head of costing and also lectures cost accounting.

Marina Bornman is a lecturer in the Department of Commercial Accounting at the University of


Johannesburg. She obtained her M Com (South African and International Taxation) and is currently
furthering her studies in Taxation. She has lectured Cost Accounting at first-, second-and third-year
level at the Vaal University of Technology and from 2008 at the University of Johannesburg, and
currently lectures taxation.

Johan Hefer CA (SA) is a senior lecturer in the Department of Management, Rhodes University. He
mainly teaches financial management to third and fourth year students but has also contributed to the
Risk and Investment Management courses. His research interests include valuations and professional
ethics.

Christo Hurter is a lecturer in the School of Accounting at the Central University of Technology.

Joset Jordaan-Marais is currently a lecturer in Cost and Management Accounting in the Department
of Finance and Investment Management at the University of Johannesburg and has previously lectured
a variety of undergraduate Financial Management modules. She obtained her M Com (Financial
Management), cum laude, in 2013 at the University of Johannesburg and previously completed her B
Com (Accounting) Honours degree at the Rand Afrikaans University as well as a Post Graduate
Certificate in Education, cum laude, from UNISA.

Peter Kamala is currently acting Head of Department of Cost and Management Accounting at the
Cape Peninsula University of Technology. He has also acted as Senior Lecturer in the department,
teaching Management Accounting, Business Strategy and Cost Accounting. Peter obtained his M Com
in Activity-based costing at the University of the Witwatersrand, and is currently studying toward a
PhD in Environmental Accounting at the University of South Africa.

Jayan Cherukara Varughese Mathew obtained a B Com from the University of Kerala, India in
1977, a M Com from Agra University in 1981 and an HDE from the University of Fort Hare in 1989.
He joined the Ciskei Department of Education as an educator in 1987 and later the Griffiths Mxenge
College of Education as a lecturer in Accounting. He took over the responsibility of Registrar of the
College in 1996 and worked in that capacity until joining Border Technikon. Jayan became part of
Walter Sisulu University on its formation in 2005, and later was appointed Head of the Accounting
Department at Buffalo City Campus in 2010. He has acted as CIMA co-ordinator since 2005. He has
also co-authored a learning aid called Cost Accounting

Exposure which is used in the National Higher Certificate Accountancy (Extended) programme at the
University.

Melissa McGill qualified as a Chartered Accountant in 2009 after completing her articles at
PricewaterhouseCoopers. She stayed on as manager in the Private Company Services (PCS)
department at PricewaterhouseCoopers for three years after which she was appointed as Senior
Lecturer at the University of Johannesburg. She currently lectures Financial Management to
undergraduate students as well as Financial Accounting to Advanced Diploma and CIMA
professional exam candidates.

Jean Struwig is a Senior Lecturer in the Department Applied Accounting at Nelson Mandela
Metropolitan University. A qualified CA, she also gained her Masters in Commercial Taxation from
the University of Pretoria.

Peter Taylor is a Senior Lecturer in Accounting at Mangosuthu University of Technology where he


has been a member of staff since 1989. He has taught in the accounting field all in all for more than 30
years. He holds a BA from Wits completed 1975 majoring in Economics, a B Com from UNISA
majoring in Accounting, Business Economics and Commercial Law, a HDE with the main teaching
subjects being Accounting and Economics and an MBA from UDW (now UKZN). He has been a
member of SAIPA since the early nineties. His interests at present are mainly in the field of Cost
Accounting for undergraduate students and Financial Management on the University’s postgraduate
programme. In recent years he has also lectured in Financial Management at the Management College
of South Africa and Regent Business School on their MBA programmes and is currently a moderator
for the Financial Management discipline for a number of local business schools.

Minnette Vermaak is a Senior Lecturer in the Department of Commercial Accounting at the


University of Johannesburg. She graduated with an M Com in Financial Management from the
University of Johannesburg in 2012. Minnette lectures cost accounting to first- and second-year
students.
PART 1
Fundamental concepts
CHAPTER 1
Introduction to cost accounting

CHAPTER 2
Cost concepts, classification and behaviour

CHAPTER 3
Cost elements: Accounting for inventory

CHAPTER 4
Cost elements: Inventory management and control

CHAPTER 5
Cost elements: Labour

CHAPTER 6
Cost elements: Overheads

CHAPTER 7
Cost statements
1 Introduction to cost accounting
Jayan CV Mathew and Johan Hefer

LEARNING OUTCOMES
After studying this chapter, you should be able to:
• briefly describe the processes of organisational management, namely planning, organising, leading
and motivating, and controlling and monitoring;
• summarise the differences between the disciplines and practice of financial, management and cost
accounting;
• outline the key roles management accounting plays in the management of an entity and explain its
limitations;
• understand the importance of accounting information in terms of measuring performance;
• list the major cost and management accounting guidelines; and
• discuss the ethical requirements that cost and management accountants must adhere to.

PRACTICAL EXAMPLE
James runs a company that manufactures security gates. Although the factory is very busy, for some
reason the business is not making any money. James does not understand why. His newly appointed
accountant asks whether he has records that show how much each gate costs to manufacture. James
admits that he does not. He simply purchases the materials as required and keeps all the invoices
in a box. The accountant asks for the invoices and other bills, as well as the labour cost records.
The accountant explains that he will analyse this information to establish accurately the cost of
each security gate.

1.1 Introduction
Cost accounting has its origins in the history of manufacturing businesses. The Industrial Revolution
of 18th- and 19th-century Europe, and the concomitant mechanisation of industry, generated an
increasing need to accurately calculate the cost of production. It became imperative for all factory
expenses to be recorded using the double-entry principles. This gave rise to the branch of accounting
that later became known as managerial accounting. Managerial accounting uses reports produced by
cost accounting for effective cost control and efficient management of production resources.
Although cost accounting has its origins in the manufacturing sector, its modern-day applications
embrace a wide range of economic activities. Today cost accounting is practised by industries
involved in manufacturing, trading and services. Public-sector organisations, such as government
departments, also use cost accounting for the purposes of determining their effectiveness and
efficiency, and to improve their performance.
Cost accounting is a system used to determine the expenses incurred by a business or other
organisation. It can be defined as the system used to capture cost data and determine the cost of
producing a product or providing a service.
The information generated by cost accounting is used by managers to control costs and make the
business more profitable.
The cost of producing a product consists of the cost of the raw materials (e.g. the steel used in the
manufacture of the security gates in the opening example), the cost of labour (wages paid to the
metalworkers and other staff) and the cost of manufacturing overheads incurred (electricity charges,
depreciation of equipment, wages paid to cleaning staff, etc.). The cost of the product is the sum of
these three components, which are known collectively as the elements of the cost of production.

1.2 Managing an organisation


The managers of an organisation require information of various sorts to enable them to make
decisions that help improve the performance of the organisation. One of the main purposes of cost
accounting is to help provide this information. It is therefore important that the cost accountant
understands the purpose and process of managing an organisation.
The management process – which applies to any type of organisation – is composed of the
following elements:
• Planning. Managers are responsible for planning the activities of an organisation. When planning,
managers normally set the goals that they wish to achieve and compile plans to achieve those
goals.
• Organising. Managers need to organise the activities of the organisation to achieve its
organisational goals. These include organising production, determining staffing and managing the
day-to-day activities of the organisation.
• Leading and motivating. An important function of managers is to give direction to the activities of
the organisation through their leadership. This includes keeping staff motivated by means of an
appropriate reward system.
• Controlling and monitoring. Finally, managers are responsible for controlling all the activities of
the entity, which entails ensuring that the goals of the organisation that they set out in the planning
stage are achieved.

In the management process, financial information is of vital importance. Cost and management
accountants provide information during all the steps in the management process described above.
Firstly, they assist managers in planning the financial activities of the entity by drawing up budgets
that are aligned to the goals of the organisation. (The various types of budgets are covered in
Chapters 15–18). Secondly, they help management allocate the required resources to the various
functional areas of the organisation. Finally, cost and management accountants also play an important
role in motivating, controlling and monitoring an organisation by supplying information needed to
determine appropriate incentives, monitoring the financial performance of the entity and helping
managers make sound financial decisions.

1.3 Defining financial, management and cost accounting


Generally, accounting is divided into three related disciplines. The major division is between
financial accounting, and cost and management accounting.
Financial accounting focuses on the preparation of annual financial statements that fairly represent
the financial position and performance of the entity to external parties, such as the tax authorities,
banks, suppliers, shareholders and investors.

The financial statements are normally prepared using either International Financial Reporting
Standards (IFRS) (used in most countries, including South Africa) or Generally Accepted Accounting
Principles (GAAP) (used in the US). Financial accountants generally deal with past-oriented
information and report on what has happened.

Management accounting measures, analyses and reports financial and non-financial information
to management.

As mentioned, management-accounting information is then used by managers for the purposes of


financial and strategic decision making. Management-accounting information is normally only used
for internal purposes, and there are no set rules or standards that have to be followed. The key
questions that need to be asked are whether this information will help managers better achieve their
goals and whether the benefits of compiling the information outweigh the costs. Although it uses
historical information, management accounting tends to have a future-oriented outlook, in the sense
that the information provided informs future decision making.
Cost accounting provides information that can be used for the purposes of both financial and
management accounting.

Cost accounting measures, analyses and reports financial and non-financial information regarding
the costs incurred by the organisation.

In the manufacturing environment, a lot of effort may be spent on appropriately determining the cost of
manufacturing goods. This information may be used by financial and management accountants.
The distinction between cost and management accounting is often blurred, and therefore the terms
are often used interchangeably – or simply termed ‘cost and management accounting’. The differences
between cost and management accounting, and financial accounting are summarised in Table 1.1

Table 1.1 Summary of differences between cost and management accounting, and financial
accounting
Cost and management
Financial accounting
accounting
Provides financial information to
Helps managers make decisions
Purpose of information investors, banks and other
to achieve organisational goals
outside parties
Primary users of information Internal stakeholders External stakeholders
Focus and emphasis Future-oriented Past-oriented
Internal rules based on cost and
Reporting rules management accounting principles Has to comply with financial
and the needs of the organisation accounting standards (IFRS)

Varies according to the needs of


Annual, twice-annual or
the organisation from as short as
Time span of reports quarterly reports, as required by
daily to long-term reports of up to
law
20 years
One of the major professional bodies (to which many cost and management accountants belong) is the
Chartered Institute of Management Accountants, which defines management accounting as (CIMA,
2005: v):
‘The application of the principles of accounting and financial management to create, protect,
preserve and increase value for stakeholders of for-profit and not-for-profit enterprises in the public
and private sectors. Management accounting is an integral part of management. It requires the
identification, generation, presentation, interpretation and use of information relevant to:
• Inform strategic decisions and formulate business strategy
• Plan long-, medium- and short-term operations
• Determine the capital structure and fund that structure
• Design reward strategies for executives and shareholders
• Inform operational decisions
• Control operations and ensure the efficient use of resources
• Measure and report financial and non-financial performances to management and other
stakeholders
• Safeguard tangible and intangible assets
• Implement corporate-governance procedures, risk management and internal controls.’

From this definition, it is evident that management accountants have a major role to play in the
organisation.
We will now discuss some of the key roles, as well as advantages and disadvantages, of the
various types of accounting.

1.3.1 Roles and objectives of cost and management accounting


The cost and management accountant is part of the group that organises the operations of profit and
non-profit organisations. It is the responsibility of the accountant to provide the group members with
information that will help them to perform their tasks. Management accountants do not make business
decisions, but the information that they provide, together with the expert advice they give, helps
management make strategic, operational and financial decisions. Cost and management accountants
perform the following functions:
• Allocating costs between cost of sales and inventories for internal and external profit
measurement and inventory valuation.
• Providing relevant information to help managers make better financial and strategic decisions.
• Providing information for planning, control and performance measurement.

One of the main objectives of business entities is to maximise profit. To achieve this goal, the costs of
the business’s activities need to be controlled. Thus, cost and management accounting is an essential
service for businesses. The main objectives of cost and management accounting can be summarised as
follows:
• Determining the cost per unit of the products manufactured or the services rendered by the
company.
• Providing a fair analysis of cost to management.
• Establishing and communicating to management sources of wastage.
• Providing the necessary data to assist in price determination.
• Establishing the profitability of each product and advising management on how to improve
profitability.
• Avoiding stocking excessive inventories.
• Obtaining and providing data needed for planning, decision making and controlling.
• Assisting in the preparation of budgets and implementation of budgetary controls.

1.3.2 Why is there a need for cost accounting?


Financial accounting serves a statutory function, but has certain limitations in terms of the financial
information it is required to provide. The following are some of the major limitations:
• Financial accounting provides only the net results of collective operations of an organisation as a
whole. It does not show the operating results of departments, jobs or processes.
• Financial accounting does not establish standards and, as a result, it cannot be used as a basis for
measuring performance.
• The objective of financial accounting is not to control costs, such as material, labour, etc.
• Information provided by financial accounting cannot help an organisation determine the
appropriate prices of products or services.

These limitations have been addressed through the development of cost and management accounting.
The following are some of the benefits of cost accounting:
• It can identify profitable and unprofitable activities, and help management either eliminate
unprofitable activities or take steps to improve the profitability of these activities.
• It assists in the careful recording of costs of materials and wages so that they can be controlled
and wastage can be avoided.
• It enables the cost and management accountant to draw up ad hoc cost statements, thereby
facilitating decisions when a choice has to be made between alternative actions.
• A costing system can lay down standards of performance as part of the budgeting process at the
beginning of the accounting period. This allows actual incurred costs to be compared with the
standards, and thus to determine whether the organisation has performed according to plan.
• It enables the establishment of costs involved in manufacturing a product or providing a service,
and thereby sets standards. These standards can then be used to determine accurately the price of
the product or service.
• It promotes profitability by determining the reason for profit fluctuations and isolates the elements
that cause these fluctuations.
• It discloses the relative efficiencies of different workers and thereby helps establish suitable wage
systems to reward efficiency.

Cost and management accounting is not an exact science, but has developed through theories and
accounting practices based on reasoning and common sense. Cost and management accounting uses
these practices to record transactions and develop reports. However, cost and management
accounting also has certain limitations or disadvantages:
• Cost and management accountants do not use uniform procedures, and as a result different cost
accountants may arrive at different results from the same data.
• The practices followed need to be dynamic, otherwise the information generated makes it less
useful when conditions change.
• The cost involved in establishing and running a costing system makes it unaffordable for many
smaller businesses.

Furthermore, the complexity of some costing systems means that business executives may not fully
understand the implications of certain statements. This can be detrimental, as management may make
decisions on the basis of certain information the implications of which are not fully understood. The
accountant must therefore be careful in the way he or she presents information to the management of
the organisation.

1.4 Cost and management accounting guidelines


Although, unlike financial accounting, cost and management accounting is not bound by rules or
standards, it is important that the following considerations are kept in mind to ensure that
practitioners fulfil their professional role:
• Cost–benefit approach. Cost and management accountants should ensure that the benefit of the
information that they provide exceeds the cost of providing the information. If the cost of
maintaining a complex costing system exceeds the benefits of the system, it should not be
instituted.
• Behavioural and technical considerations. A cost and management accounting system should give
managers the technical information they require in the appropriate format. Cost and management
accountants should, however, also consider the behavioural implications of the information they
provide. For example, the allocation of costs between departments may lead to dissatisfaction
among the workforce and a reduction in productivity. A well-functioning costing system should be
designed in a way that helps managers motivate and control their departments to ensure the most
efficient use of the resources available to the organisation.
• Purpose of information. Cost and management accountants should remember that information may
be required for different purposes. Information compiled that may be appropriate in one situation
may not be appropriate in another. Even the way in which costs are allocated may need to be
changed according to the purpose for which the information is required. If, for example, a decision
needs to be made whether to continue or discontinue a product, the allocation of fixed overheads
between products may not be relevant (see Chapter 6). If, however, the information is required for
costing purposes, the fixed overheads would then be relevant.

Cost and management accountants need to ensure that they provide the information to management
based on the above considerations, but they also need to consider their ethical responsibilities.

1.5 Ethical issues in cost and management accounting


An understanding of financial accounting is necessary to become a cost and management accountant.
In both professions, ethics play a pivotal role. According to the Concise Oxford Dictionary, ethics is
the science of morals in human conduct. It is the way humans act in terms of honesty and decency
towards their fellow humans. It is important that accountants merit the respect they command in the
commercial world and society in general. They should earn this respect through honesty and
trustworthiness in their professional activities and pursuits. Unfortunately, this is not always the case.
It is important that practitioners strive continually to maintain the image of their profession. It is in
this light that we turn to the role of ethics in cost and management accounting.
Cost and management accountants have an obligation to the organisations they serve, their
profession, the public, the nation as a whole and themselves, to maintain the highest standards of
ethical conduct. There is a code of conduct that applies to the profession, and cost accountants must
not contravene this code, nor condone anyone else doing so. Members of the profession are expected
to have a responsibility, among others, to:
• be competent in the commission of their duties;
• maintain confidentiality;
• act with integrity;
• be accountable for their actions;
• be reliable in the information they provide;
• be loyal to those with whom they are involved in their professional capacity; and
• be responsible citizens in all their actions.

Cost and management accountants can often be placed in difficult positions where they may be
expected to act unethically. For example, the manager of a department may pressurise the cost
accountant to change the method of allocating costs to create the impression that the department’s
financial performance is better than it is in reality. In such situations, the cost accountant needs to
remember that he or she has an ethical responsibility to ensure that the information provided is
accurate.

CLOSING PRACTICAL EXAMPLE


James runs a company that manufactures security gates, which is not making any money. He has
appointed an accountant to help him establish the reasons.
James’s accountant comes back to him after having analysed the financial information provided
to him. He has established each cost incurred by the factory in the manufacturing process and
believes that he now better understands the problems facing the company. He indicates that the
company is not making a profit because the costs are too high.
James now has the necessary information to make decisions regarding the factory’s various
costs to ensure that its profitability henceforth improves.

1.6 Summary
This chapter provided an introduction to the origins, development and practice of cost and
management accounting. Cost and management accounting is a branch of accounting that developed in
the Industrial Revolution in order to quantify costing in manufacturing businesses. In the management
process, financial information is key, and cost and management accounting provides this information
during all the steps in the management process. Hence, management and cost accounting has a major
role to play in today’s organisations.
Both financial and management accounting are part of the financial information system of an
organisation. The limitation of financial accounting in terms of developing specific-purpose reports
for internal uses led to the development of cost and management accounting. The advantages of cost
and management accounting have made it an important activity within accounting in terms of
providing information for decision-making purposes.
Unlike financial accounting, management-accounting information is normally only used for internal
purposes, and there are no standards governing how it is to be practised. However, there are
professional guidelines that should be followed. Cost and management accountants need to ensure that
the benefit of the information they produce exceeds the cost of its production. They should also
remember that the information they provide needs to be technically correct and may influence the
behaviour of others in the organisation. They need to ensure that they produce the information bearing
in mind the purpose for which the information is intended. Finally, they need to act ethically to ensure
that their professional integrity is upheld.

Questions
1. Briefly describe the process of managing an organisation.
2. List three major differences between financial accounting, and cost and management
accounting.
3. What are the key roles that managerial accounting plays in an entity?
4. ‘There are no set rules for management accountants like there are for financial accountants.’
Discuss whether this means that a cost accountant may act without regard for conventions and
ethics. Justify your answer.
5. Discuss why there is a need for cost and management accounting.
6. Briefly describe how cost-accounting information can positively and negatively impact on the
attitudes of managers.
2 Cost concepts, classification and behaviour
Christo Hurter and Johan Hefer

LEARNING OUTCOMES
After studying this chapter, you should be able to:
• define, and explain the characteristics of, the cost of a product or service;
• define cost objectives;
• understand the principles of assigning costs to cost objectives;
• categorise material, labour and overhead costs;
• distinguish between direct and indirect costs, and variable and fixed costs;
• discuss the meaning and uses of unit costs;
• differentiate between the different types of inventory;
• differentiate between job-costing and process-costing systems; and
• explain the management-accounting concepts of relevant costs, opportunity costs and sunk costs.

PRACTICAL EXAMPLE
Lindile runs a wood mill in the Eastern Cape. Although the business appears to be doing well,
Lindile is uncertain about how much profit he is making. He approached an accountant to help him,
but after the meeting he was even more confused. The accountant referred to concepts like
‘inventory’, ‘raw materials’, ‘work in process’, ‘finished goods’, ‘overhead costs’ and various
other terms Lindile did not understand.
This experience made Lindile realise that he will need to understand the terminology and
concepts used in cost accounting. This will help him determine whether or not his business is
profitable. He looks forward to working with the accountant to set up an appropriate costing
system to help ensure that he can run a profitable business.

2.1 Introduction
The purpose of cost and management accounting, as discussed in Chapter 1, is to give managers the
information they require to make business decisions. Many decisions are made on the basis of the
costs incurred in the production or resale of a product, or the rendering of a service. Put simply, if the
money received from the sale of a product or service does not exceed the money spent on producing
that product or service, a manager would normally decide not to produce or sell that product or
service. There are exceptions to this general principle – such as in the welfare sector and other
public-sector services – but in business, where one of the main aims is to make a profit, unprofitable
products or services should be discontinued.
The cost and management accountant has to be able to calculate with reasonable accuracy the
money received and the money spent on a product or service. This information allows managers to
make strategic decisions. Managers are responsible for the strategic decisions that need to be made in
an organisation. These decisions are the long-term plans and objectives set by the organisation.

The money received from the sale of a product or the rendering of a service is known as income.
More formally, an income can be defined as an inflow of economic resources into an entity.

The income that can be achieved from a product or service is equal to the value a customer is willing
to pay for that product or service. As an example, if a customer is willing to pay R8 for a loaf of
bread, the income from that product received by the retailer would be R8. The income from a product
or service is often difficult to control, as it is dependent on external factors when there is a
competitive market for that product or service. When there is not an active market, and the entity can
set its price for a product or service with reasonable independence, the cost of making the product
will often be used to set the selling price (see Chapter 22: Pricing).
Given this, it is crucial that managers control the money spent on producing or reselling a product
or rendering a service. The money spent is also referred to as the cost of that product or service.
Consequently, in cost and management accounting most attention is paid to the cost of products and
services to provide managers with the information they require to ensure that costs can be controlled,
decisions can be made regarding the continuation or discontinuation of a product or service and
selling prices can be set.

The cost of a product or service is the monetary outflow of resources used to manufacture a
product, purchase a product or render a service.

In layman’s language, monetary outflow refers to the amount paid by the business to outside parties in
the process of manufacturing or selling a product or rendering a service. Managers need to understand
what is meant by cost and how costs behave to enable them to manage and control the costs in an
entity.
This chapter discusses the various cost concepts used when managing and recording costs in an
entity, and the terms associated with them. These cost concepts and terms are important, as they form
the basis of cost and management accounting, and will be used throughout this book.

2.2 Cost objectives


The importance of providing accurate cost information to management was discussed in Chapter 1
and in the introduction to this chapter. To enable us to calculate the cost of a product or service, costs
are normally assigned to a cost objective and then allocated to specific products or services.
A cost objective is an activity for which separate measurements of costs are required. As an
illustration, a manufacturer may have three product lines and management require costing information
for each of the three product lines independently. Each separate product line, in this case, is a cost
objective. Cost objectives are also referred to as cost centres.
In the case of a retail company, cost centres could be different stores or separate departments
within an individual store (e.g. the bakery or fruit and vegetable department). In a service
organisation, costs centres could be different long-term projects or different divisions (e.g. in an
accounting firm, there may be an accounting, tax and audit division).
Costs are normally assigned to cost objectives in a two-step process, as illustrated in Figure 2.1.
The figure introduces a number of concepts that will be discussed in this section. Firstly, all costs are
assigned to categories like labour, materials and overhead costs, or allocated by their behaviour,
namely whether they are fixed or variable costs. These concepts will be discussed further in this
section.
The second step is to assign these costs that have been allocated to specific cost objectives. This
information can then be used to consider the profitability, cost control and future planning of a cost
objective. We will now discuss these two steps in further detail.

Figure 2.1 Cost allocation process

2.2.1 Step 1: Categorising costs


In the first stage of allocating costs to cost objectives, costs may be categorised according to their
nature. These include cost categories such as labour, material or overheads. Costs may also be
recorded by their behaviour, namely according to whether they are fixed or variable costs.
The following are examples of costs categorised by their nature:
• Materials refer to the physical components that make up a product. As an example, in the
manufacturing environment, a sofa may be made up of wood, leather, screws and filling material.
In the retail environment, materials are the retail products that the retailer purchases to resell to
customers. In other words, materials are the tangible components manufactured or purchased by an
entity to use in production or to resell to customers.
• Labour costs are the remuneration paid by an employer to the employees in exchange for the
services the employees provide to the employer. These services may include physical labour,
application or parting of knowledge and managing a process where a product is manufactured or a
service is rendered. Labour costs in a manufacturing environment would include the costs of
factory workers, supervisory staff, and cleaning and administrative staff. Similar types of labour
costs are incurred in the retail and service sectors.
• Overheads are, collectively, all the other costs incurred in the production or sale of a product or
the rendering of a service apart from material and labour costs. In a manufacturing environment,
overheads include factory costs (e.g. electricity and rent), administrative costs (e.g. stationery)
and machinery costs (e.g. repairs and maintenance). Similar costs are incurred in the retail and
service environments.

The following are costs categorised according to their behaviour – fixed or variable:
• Fixed costs are those that behave independently of the level of production. Therefore, whether a
single item is produced or several, the cost will remain constant. Factory rent is an example of a
fixed cost, as it does not vary according to the level of production.
• Variable costs are those that change according to the level of production. The materials used,
labour costs and certain overhead costs, like electricity, vary according to the level of production.
If more products are manufactured, these costs normally increase. Fixed and variable costs are
discussed in greater detail in Chapter 6.

Once the costs have been categorised based on their nature or behaviour, they can be assigned to
specific cost objectives.

2.2.2 Step 2: Assigning costs to cost objectives


In the previous section, we discussed how costs are categorised. The costs should now be assigned to
cost objectives to generate the information management require to make decisions. Costs are
generally treated as direct costs, indirect costs or overhead costs. The categorisation of a cost into
one of these three components will influence how it will be assigned to a cost objective.

Direct costs are those costs that can be assigned to a specific product or service.

Direct costs normally include the components or materials used in the manufacture of a product
and the cost of the labour to manufacture a product or provide a service. In cost accounting, as in
many other disciplines, it is important to use the correct terms. When assigning direct costs to cost
objectives, we refer to the process as tracing a cost to a cost objective. A cost can only be a direct
cost if the cost can be traced to a specific cost objective to the exclusion of any other cost objective.
As an example, the quantity and value of wood used in the production of a table can be calculated
fairly accurately and therefore this cost will be traced to the cost of the table. In the same way, the
cost of labour to produce the table can also be measured with reasonable ease and therefore will be
traced to the cost of the table.
Overhead costs are normally classified as manufacturing overheads and non-manufacturing
overheads. Manufacturing overheads are those directly, or largely, attributable to the manufacturing
process – for example, electricity or factory rental. Non-manufacturing overheads are not directly
attributable to the manufacturing process and would include administrative, executive and selling
costs. Overhead costs are discussed in more detail in Chapter 6.
Manufacturing overhead costs cannot normally be traced to a specific cost objective because they
are not normally a direct cost. Overhead costs are normally classified as indirect costs.

Indirect costs are those costs that cannot be traced to a specific cost objective in a cost-effective
manner.

As an example, the cost of electricity to run a factory where there are multiple cost objectives cannot
normally be allocated to a specific cost objective in an economically feasible way. Therefore, in a
factory that manufactures various types of furniture the electricity costs cannot be easily traced to a
specific product. These costs can, however, then be allocated to specific products using a relevant
basis.
Indirect costs normally also include indirect materials and indirect labour. Indirect materials are
those that are used in multiple cost objectives – for example, glue and nails used to manufacture
tables. Indirect labour is the cost of employing, for example, cleaning or maintenance staff. All the
indirect costs are normally allocated to the cost objectives as manufacturing overheads.
Whereas direct costs are traced to specific products, indirect costs are allocated, as they cannot
be traced to a specific cost objective. The differences between direct and indirect costs, and variable
and fixed costs are illustrated in Figure 2.2.
Direct material costs are often referred to as prime costs; direct labour and manufacturing
overhead costs are referred to as conversion costs.
Non-manufacturing overheads are often not taken into consideration when calculating the cost of
manufacturing a product. However, this depends on how management wish to allocate costs in the
entity. Non-manufacturing overheads may not be allocated to manufacturing costs for financial-
accounting purposes.
In the retail and service sectors, overhead costs may also be allocated to specific cost objectives
or simply treated as non-allocated overhead costs. This decision would largely be based on how
economically feasible or useful such an allocation is in terms of enabling managers to make
decisions.

Figure 2.2 Differences between direct and indirect costs, and variable and fixed costs

It may be useful to briefly state again that a cost-accounting system is not based on external rules like
financial accounting, but should be structured in a way that ensures that management are supplied with
the information they require. Therefore, a cost-accounting system should be customised to meet the
needs of the users of the information generated by the system.
Non-manufacturing or non-allocated overhead costs are examples of period costs. These costs are
allocated to expenses as soon as they are incurred. An expense is defined in financial accounting as
an economic outflow of resources (in most cases, cash) for which no future economic inflows are
expected. This is in contrast to product costs, which are those costs that are allocated to the cost of
inventory and only allocated to expenses when the product is sold. In other words, we expect to
receive a future benefit from a product cost when we sell the product. As an example, if a retail
company purchases a product for R15, the value will be attributed to inventory because the company
expects to sell the product for a price in the future. When it sells the product, the R15 is allocated to
expenditure, as it cannot be sold again.
It should be noted that the allocation of indirect costs is a challenging process in which managers
need to use a great degree of judgement. There is no one way of allocating indirect costs to cost
objectives, and different managers or cost accountants may choose to allocate indirect costs in
different ways. The information generated by a cost-accounting system should therefore be
interrogated by managers to ensure that they do not make an incorrect decision due to the way in
which indirect costs are allocated.
In this section, we discussed how costs are classified according to specific categories. This
information is summarised in Figure 2.3. The following section explains how these costs can be
allocated to specific units to calculate the unit cost.

Figure 2.3 Classification of costs related to a product

2.3 Unit costs


As discussed in the previous section, costs are allocated to all the entity’s cost objectives. This is
referred to as the total cost. Once these costs have been allocated to cost objectives, we can now
calculate the unit cost.
The unit cost is simply the total cost divided by the number of units produced (or purchased, in the
case of a retail environment) during the period. Therefore, if total costs are R600 000 and the
company produced 10 000 units, the unit cost is R60.

Unit costs should be used cautiously for decision making, as the unit cost includes a portion of fixed
costs and therefore the unit cost will fluctuate depending on the level of production. If the level of
production fluctuates, unit costs may fluctuate too and therefore decisions made based solely on unit
costs may obscure the influence of fixed costs.
Unit costs may also be calculated in different ways depending on the purpose for which the unit
cost is calculated. If unit costs are calculated for financial-accounting purposes, the requirements of
the accounting standards have to be followed, whereas unit costs for management-accounting
purposes may calculate the unit costs in a different way. Decisions should therefore normally be
based on total costs rather than unit costs.
The next section discusses the different types of inventory.

2.4 Types of inventory


Inventory in a manufacturing environment normally includes three categories: materials, work in
process (WIP) and finished goods.

Product costs are those costs that are allocated to inventory until it is sold, at which point the cost is
allocated to expenses.
Materials, often referred to as raw materials, are those materials that have not been introduced
into the manufacturing process. The materials are therefore still in the state in which they were
purchased and no conversion has taken place. For the purposes of cost accounting, material inventory
is normally valued at the cost of the material plus any delivery charges.
Once materials are introduced into the manufacturing process, the partially finished products are
referred to as WIP. When calculating the cost of WIP, the material costs already incurred and any
conversion costs (direct labour and manufacturing overheads) incurred up to the point of measurement
are included when calculating the cost of the WIP.
Once the product’s manufacturing process is complete, it is referred to as finished goods. At this
point, all direct materials, direct labour and manufacturing overhead costs incurred in its production
are allocated to the product. Once the product is sold, this cost is then allocated to cost of sales.
In the retail environment, the term ‘inventory’ or ‘retail inventory’ is used to describe the goods
that are available for sale. The cost of retail inventory normally only includes the original purchase
price of the goods plus delivery charges – although some overheads and labour costs may be included
under certain circumstances.
Inventory is discussed in detail in Chapters 3 and 4.
We have not yet provided a comprehensive definition of a cost objective. The reason for this is
that cost objectives are defined according to the type of costing system that the entity uses. These
systems are explained in the next section.
2.5 Job-costing and process-costing systems
Cost-accounting systems normally allocate costs to cost objectives. The nature of a cost objective
differs according to the industry an entity operates in, such as the manufacturing, retail or service
environment. Broadly speaking, cost objectives are either a specific item or order manufactured as a
single unit, or a large number of products manufactured in a process.
When costs are allocated to a specific item (for example, a car) or an order (for example, one
batch of soap customised for a hotel), we refer to the costing system as a job-costing system. In
contrast, when costs are allocated to a specific manufacturing process (for example, producing a cold
drink) where large numbers of homogeneous products are manufactured, we refer to it as a process-
costing system.
Job- and process-costing systems are discussed in more detail in Chapters 8, 10 and 11.
Example 2.1 illustrates a process-costing system.

EXAMPLE 2.1
Klinedale Butchery receives carcasses from an abattoir and cuts the meat into various products
for resale.
The purchase price of the carcasses is classified as a material cost; the salaries of the staff are
classified as labour costs. Electricity, rent, repairs and maintenance are classified as overhead
costs.
Klinedale rents its premises for R5 000 per month; the rent is a fixed cost. The quantity of
meat carcasses ordered is based on demand and this is therefore a variable cost.
Klinedale is well known for its boerewors, and the meat used to make them is classified as a
direct cost; the spices used in the manufacture of the boerewors are an indirect cost. The
telephone account of the butchery is not allocated to production and is therefore a non-allocated
overhead cost.
In March, Klinedale allocated and traced R5 000 of costs to the production of boerewors and
produced 100 kg of boerewors. Therefore, the total cost of the boerewors was R5 000 and the
unit cost of boerewors was R50 per kg.
Klinedale also makes biltong. In March the company had meat in the cold room to the value of
R900 allocated to the production of biltong. This meat forms part of the material inventory.
Klinedale also had biltong in the process of drying, to which R1 500 costs had been allocated.
The biltong that is in the process of drying is termed WIP. The butchery also had biltong
available for resale, to which R400 costs had been allocated. The biltong available for resale is
known as finished goods.
Klinedale accounts for its cost of sales per product line but costs are allocated to a specific
product over the whole period. This is an example of a process-costing system, whereby costs
are not allocated to specific orders or jobs, but to a manufacturing process.

2.6 Management accounting concepts


Although cost and management accounting are used interchangeably, there is a distinction worth
noting. The goal of cost accounting is to allocate costs to products or services, whereas management
accounting uses cost accounting information to make informed business decisions. There are three
important cost concepts used in management accounting that should be considered: relevant costs,
opportunity costs and sunk costs. These concepts are not relevant when calculating the cost of a
product but are important when making management decisions regarding a product.
Relevant cost is a term used when one is faced with a financial decision. A relevant cost is a cost
that needs to be taken into consideration in a decision due to the fact that the incurring of the cost is
dependent on the decision made. As an example, let us assume that we need to decide whether or not
to accept a special order from a customer. If we accept the order, we will need to purchase materials
that we do not usually use for R10 000. These materials constitute a relevant cost because this cost
will only be incurred if we accept the order. Our labour costs are R50 000 per month but this cost
will be incurred whether or not we accept the order. Therefore, the labour cost component of the
special order is not a relevant cost, as the cost will be incurred whether we accept or reject the offer
– as long as no extra labour is required.
Opportunity cost refers to a potential loss of profits as a consequence of making a decision that
allocates resources in a certain way. If we continue with the example of the special order, let us
assume that we will lose R5 000 in profits from our normal production if we accept the special order.
The R5 000 is an opportunity cost because by accepting the special order we will forego the profits
of our normal production in favour of the special order. Put another way, by making the decision to
accept the special order, the company would lose the ‘opportunity’ to make a profit of R5 000 from
normal production.
A sunk cost is a cost that has already been incurred and therefore will not influence the decision
to invest resources. If we continue with the special-order example, let us assume that we have already
spent R3 000 on developing the plans for the special order. The R3 000 cannot be retrieved; it is a
sunk cost and should be ignored in making the decision, as it has already been incurred and the
decision made should not be influenced by the fact that we have already spent the R3 000.
We will discuss these concepts again in later chapters.

2.7 A brief note on value-added tax


In most countries, including South Africa, a sales tax is levied. In South Africa this is referred to as
value-added tax (VAT). Although a detailed discussion of VAT falls outside the scope of this book, it
is important to note the effect of VAT on the costs of an entity. If a company is registered for VAT
(and therefore has to recover VAT when selling products or services), the entity can claim all of its
VAT on costs (referred to as inputs) as a deduction from the VAT collected from its customers
(referred to as outputs). In this case, VAT has no impact on the costs of an entity as the entity simply
acts as a collecting agent for the tax authorities and VAT is not an expense. If, however, an entity is
not registered for VAT (and therefore does not include VAT in the selling price of its products or
services), the VAT included by their suppliers in their purchases (or inputs) cannot be claimed and
therefore would form part of its costs.
Thus whether an entity is registered for VAT or not, for the purposes of cost and management
accounting VAT (and most other sales taxes) is irrelevant, as it will either be claimed back from the
tax authorities or included in the costs when they are recorded in the financial- or cost-accounting
system.
CLOSING PRACTICAL EXAMPLE
Lindile, who runs a timber mill in the Eastern Cape, has now done some research into cost
accounting to help him gain a better understanding of how his costs behave. He has found that
inventory is the term used for all the stock he has on hand. Inventory comprises raw materials (the
timber that has not been used), work in process (timber that has been partially converted) and
finished goods (timber that is ready for resale).
Lindile has also found that he should first categorise his costs as materials, labour and
overheads, and then identify cost objectives to allocate these costs too. He believes that most of
his products are homogeneous in nature and therefore he should allocate costs using a process-
costing system. He has identified five product lines, such as planks and handles, that he would like
to account for separately. Lindile is now in a much more informed position to allocate the costs
and calculate the profit he is making in his business.

2.8 Summary
This chapter defined and explained the characteristics of the cost of a product or service and cost
objectives. It then discussed how costs are allocated to specific cost objectives by first categorising
costs as material, labour or overhead costs, and then tracing direct costs and allocating indirect costs
to cost objectives.
The chapter differentiated between inventory in different stages of completion. Raw materials are
inventory still in the state they were purchased. WIP is a product that is partially completed. Finished
goods are ready for resale.
The chapter also discussed two types of costing systems: one where costs are allocated to specific
items or orders, known as a job-costing system, and one where costs are allocated to a process of
manufacture where large numbers of homogeneous products are produced, known as a process-
costing system.
Finally, the management-accounting concepts of relevant costs, opportunity costs and sunk costs
were explained.
As we have now covered the basic terminology you will use in cost accounting, the next chapters
look at the processes and procedures involved in cost accounting.

PRACTICAL ACTIVITY
Write a memorandum to the senior partner in the legal firm where you have started to work as an
accountant. In the memorandum, briefly define the following concepts as they apply to a legal firm
and give relevant examples of each in your work context:
• Cost objective
• Indirect costs
• Job-costing system
• Labour costs.
Short questions
1. _________ can be described as the direct and manufacturing overhead costs incurred in the
production process:
a. Conversion costs
b. Sunk costs
c. Non-allocated overhead costs
d. Fixed costs
2. In a clothing store, which of the following types of inventory are most likely to be found:
a. Raw materials
b. Work in process
c. Retail inventory
d. Finished goods
3. Briefly define the term ‘cost’.
4. Briefly define the term ‘income’.
5. What term describes a system where costs are allocated to a manufacturing process in which
large numbers of homogeneous products are produced?

Long questions
1. Define a cost objective and illustrate it by means of a relevant example in the manufacturing
environment and an example in the service environment.
2. Give two examples of non-manufacturing overheads incurred by a large computer-
manufacturing company.
3. Define an indirect cost and give two examples of indirect costs from two industries.
4. Define a prime cost and illustrate with an example of the prime costs of a jewellery
manufacturer.
5. Differentiate between retail inventory and materials.
6. Give examples of the direct costs incurred by an accounting firm.
7. You are advising a client who has a clothing business. Explain the concepts of materials,
work in process and finished goods to her, illustrating your explanations with examples from
her business.
8. Classify the following costs as material costs, labour costs or overhead costs for a
manufacturer of security gates:
• The supervisor’s salary
• Cost of metal purchased
• Electricity
• Cost of paint
• Welders’ salaries
9. Classify the following costs as direct or indirect costs for a building contractor who is
building a house for a client. Briefly justify your classifications.
• Bricks purchased for the house
• Cement purchased for the house
• The wage paid to a painter who was hired to work on this specific project
• The repairs to machinery that is used on multiple sites
• The cost of diesel used by the machinery on the site
• The salary paid to the project manager who is only responsible for one project at a time
10. Discuss the calculation of the costs of a garden-service business, indicating the process of
allocating costs to cost objectives. The manager of the business defines each client as a
separate cost objective. Also briefly discuss whether the manager’s definition of the cost
objectives appears reasonable.
3 Cost elements: Accounting for inventory
Johan Hefer

LEARNING OUTCOMES
After studying this chapter, you should be able to:
• define inventory and discuss the main types of inventory;
• explain the accounting treatment of inventory;
• differentiate between periodic and perpetual inventory systems;
• calculate the value of inventory using the following three methods:
• First in first out
• Weighted average
• Standard costing
• understand how to use and complete an inventory ledger card; and
• enter inventory transactions into the general ledger of the entity.

PRACTICAL EXAMPLE
Thando and her friends launched a clothing company. The firm manufactures items of clothing and
sells them in informal retail outlets. At the end of the financial year, Thando reviewed the
operations of the business. The main purchases included fabric and other consumables, like thread
in bulk. The company had a standard price list incorporating each type of garment, which Thando
compiled when she started the business.
When she analysed the profitability of the enterprise, Thando was shocked. She and her
partners had worked hard selling clothing to the value of R200 000. But she discovered they had
only made a profit of R2 000. Where had it gone wrong?
After adding up all their costs, Thando found that they had spent R170 000 on fabric alone for
the year. It then became apparent that henceforth she would need to set the selling price of the
products by taking into account the cost of manufacturing them. She had to consider ways to do this
and contacted her nephew, who had just completed a course in cost accounting.

3.1 Introduction
Inventory encompasses three categories of materials. It can be defined as the trading assets of an
entity that are available for sale to customers, those that are in the process of manufacture and the
components used to manufacture a product (raw materials).

Inventory is a difficult asset to control and protect, and this applies to manufacturing and retail
businesses alike. However, efficient inventory control provides opportunities for cost reduction and
cash-flow improvement. The management and control of inventory are more fully discussed in
Chapter 4. In this chapter, the focus is on accounting for inventory and the cost of inventory. The main
emphasis in this chapter is on manufacturing companies because costing the component parts of a
manufacturer’s inventory is more complex than for a retailer.
In the case of businesses that sell products, the key to making a profit is to ensure that they sell the
products at a reasonable markup. To be able to calculate the markup, and often the selling price, the
business needs to know how much it costs to purchase or manufacture a product.
The total cost of a product consists of the following three elements:
• Direct material
• Direct labour
• Manufacturing overheads

Direct material costs consist of all the costs relating to the purchase and holding of materials – for
example, the cost of material, transport costs and import duties.

Direct labour costs refer to the cost of the labour used to convert raw materials into finished
goods (see Chapter 5).

Manufacturing overheads include the cost of operating the factory, such as electricity (see Chapter
6).

The combined cost of these three components equals the total cost of finished goods.
What do we mean precisely by the term ‘inventory’? Inventory (or stock, as it was previously
known) is defined by International Accounting Standard No 2 (IAS 2) as follows: ‘Inventories
include assets (a) held for sale in the ordinary course of business (finished goods), (b) assets in the
production process for sale in the ordinary course of business (work in process), and (c) materials
and supplies that are consumed in production (raw materials) or in the rendering of services.’
It is important to understand the different components of inventory. These will be discussed
using the opening practical example and are illustrated in Figure 3.1 in the context of a manufacturing
environment. The most important items of inventory are:
• Raw materials. These are the component parts that are converted into finished products in the
production process. Raw materials include:
− Primary materials (or direct materials). These are the key components that are converted into
finished products. An example of a primary material is the fabric used to manufacture
garments.
− Secondary materials (or indirect materials). These are the components used in the conversion
of the primary material into a finished product, such as the thread and pins used in the
manufacturing of the garments. Due to their nature, secondary material costs cannot be
accurately allocated to particular products or production lines, or cannot be allocated to a
product in an economically feasible manner. As a result, secondary materials are regarded as
indirect materials, which, in turn, form part of manufacturing overheads.
• Work in process (WIP), or incomplete products. WIP includes products that are in the process of
being manufactured. The value of these products already includes the cost of the primary material
and a portion of the direct labour and manufacturing overheads incurred. However, as these
products are not completed they cannot be classified as finished goods. In the example of
Thando’s business, WIP includes garments that are being made, but have not been completed.
• Finished goods. These are completed, manufactured products. In our example, they include the
finished garments ready to be sold.

There is further type of inventory, known as retail inventory. This classification of inventory consists
of finished products obtained from suppliers for resale purposes. Retail enterprises stock retail
inventories. If a retailer were to buy garments from Thando then they would be stocked as retail
inventory.

Figure 3.1 The components of inventory in a manufacturing entity

From the discussion above, it is clear that inventory includes the raw materials (direct and indirect),
WIP and finished goods that a business holds, or stocks, at a given moment – and in the case of retail
enterprises, retail inventory. Investment in inventory is usually substantial and constitutes the major
component of an entity’s costs. For example, in 2012 cost of goods sold was 81 per cent of Pick n
Pay’s revenue and 52 per cent of revenue for Aspen Pharmaceuticals. This means that for one rand of
revenue generated by Pick n Pay, 81 cents of costs were incurred to obtain the inventory. The value of
inventories comprised 35 per cent of total assets for Amalgamated Appliance Holdings in the
company’s 2011 annual financial statements.
We will now look at the accounting treatment of inventory in more detail.

3.2 The accounting treatment of inventory


As discussed, inventory is one of the largest cost factors and assets for manufacturing and retailing
entities. It is therefore important that inventory costs and inventory valuation are carefully accounted
for to ensure the accuracy and reliability of the accounting information. If inventory is valued
incorrectly by even a small margin, it could have serious implications for the business’s profitability.

3.2.1 Cost of inventory


The initial recognition of all costs for inventory purchased, as well as accounting for the
manufacturing, or conversion, costs for WIP and finished goods, are very important. According to
IAS 2, paragraph 10, ‘the cost of inventories shall comprise all costs of purchase, costs of conversion
and other costs incurred in bringing the inventories to their present location and condition’.
It is important to note that inventory is not always valued in the same way for management
purposes and for financial-reporting purposes. Management may include the cost of goods sold from a
management perspective. However, this cost would not be allowed to be included for financial-
reporting purposes. As long as this is done consistently and logically, there is no problem with such
an approach. But management must ensure they know what the differences are between the two
accounting systems. In the rest of this chapter, we will assume that the cost of inventory is calculated
in accordance with the IAS 2 standards.
The following sections briefly discuss the IAS 2 types of inventory costs – purchase cost,
conversion cost and other costs. Remember that these costs may apply to raw materials, WIP, finished
goods and retail inventory. It may help to understand the concept better by bearing in mind that the
cost of inventory includes all costs incurred until the point that inventory is ready for sale.

3.2.1.1 Purchase cost


The purchase cost of inventory (according to IAS 2, paragraph 11) includes:
• Purchase price – the amount paid for the inventory.
• Import duties and other taxes incurred in the purchase of the inventory. Note that if the purchasing
entity is a VAT vendor, the VAT can be recovered from the authorities and therefore VAT is
excluded. If, however, the entity is not a VAT vendor, the VAT is included in the cost of purchase
of the inventory.
• Transport, handling and other costs. These include transport charges, storage costs while in transit
and any other costs incurred to transport the inventory to the entity.
• Trade discounts, settlement discounts and rebates. Trade discounts may be given to a customer
before or when invoicing for a purchase. Settlement discounts are given to a customer for early
payment of invoice. A rebate is a repayment or reduction based on quantity sold or other terms
over a period of time, and this figure should be deducted from the cost of purchase.

3.2.1.2 Conversion costs


The cost of conversion of inventory, according to IAS 2, includes:
• Direct labour. This is the labour cost of converting raw materials into finished goods.
• Production overheads, namely, a systematic allocation of fixed and variable production overheads
that are incurred in the process of converting raw materials into finished goods. These include
depreciation and maintenance of factory buildings and equipment; cost of factory management and
administration; secondary materials (indirect materials); and indirect labour (e.g. cleaners and
security guards).

Direct labour and production overheads are covered comprehensively in Chapters 5 and 6,
respectively.

3.2.1.3 Other costs


These include any other costs that may be incurred that meet the criteria to be allocated to the cost of
inventory. This could include the transport of the goods during the manufacturing process, for
example. Normally, however, the main components of inventory cost are the cost of purchase and cost
of conversion.
The IAS 2 statement specifically excludes certain costs from the cost of inventories, as follows:
• Abnormal amounts of wasted materials, labour or other production costs
• Storage costs, unless those costs are necessary in the production process before a further
production stage
• Administrative overheads
• Selling costs

Let us now consider how to calculate the cost of inventory by means of an example.

EXAMPLE 3.1
Kelly owns a company that manufactures sofas sold in retail outlets countrywide. There are two
main inventory costs in the company’s production process – the cost of fabric and the cost of
timber. The following figures show the main costs incurred by Kelly’s company.
Fabric is imported from China denominated in US dollars ($). Kelly has calculated that the
per unit cost of fabric (i.e. the cost of fabric required to manufacture one sofa) is as follows:
Fabric cost per unit $80
Current $:R exchange rate $1:R8,5
Import duties per unit of fabric R30
Transport cost per unit of fabric R45

Timber is purchased in South Africa. Kelly has calculated that the per unit cost of timber (i.e. the
cost of timber to manufacture one sofa) is as follows:
Purchase cost per unit R450
VAT @ 14% R63
Transport cost per unit of timber R55

Kelly receives a 5 per cent discount on settlement of the purchase price and VAT from the timber
supplier for early payment. The timber needs to be treated before it can be used in the
manufacturing process. The cost of the treatment is included in production overheads. Once the
timber is treated, it has to be stored in a specially ventilated storeroom for two weeks before it
can be used. The cost per unit of the storeroom is R35.
The direct labour cost for one sofa is R85 and production overheads per unit are R25. The
storage cost for finished goods is calculated at R35 per unit, administrative costs at R15 per unit
and selling costs at R10 per unit.
Kelly wishes to calculate the cost per unit of her sofas. She has decided to calculate the cost
of inventory in accordance with the requirements of IAS 2. Her company is not VAT registered.
Kelly can calculate the cost per unit as follows:
Fabric costs (R)
Purchase price
Import duties 680
30 ($80 × 8,50)

Transport 45
Timber costs
Purchase price 450
VAT @ 14% 63 If the company were VAT registered this would be excluded
Discount (26) (450 + 63) x 5%
Transport 55
This is included as an inventory cost, as it is part of the
Storage 35
production process
Direct labour 85
Production overheads 25
Storage of finished
0 Not included, as it occurs after production is complete
goods
Administrative costs 0 Excluded, as it is not a production cost
Selling costs 0 Excluded, as it is not a production cost
Cost per unit 1 442
Therefore, the total inventory cost per sofa is R1 442.

3.2.2 Valuation of inventory


The cost of inventory is not a static figure, as it changes with every receipt of new inventory. This
section deals with the ways in which the cost of inventory can be translated into the value of
inventory on hand and cost of sales.
This cost of inventory is used in two ways, depending on how we account for inventory.
Accounting for the cost of goods sold (or cost of sales) can be done using one of two systems, a
periodic or a perpetual system.
With a periodic system, acquisitions and reductions in inventory are not recorded in the inventory
account. Instead, all inventory acquisitions are recorded in a purchases account. When inventory is
sold or used, the cost of sales of that unit is usually unknown. At the end of the accounting period, a
physical count of inventory is taken and a value is assigned to goods on hand based on the unit price
paid for them (how to determine this value will be discussed later in the chapter). Cost of goods sold
or used is calculated by deducting the value of inventory on hand from the value of goods available
for sale (opening inventory plus purchases) during the period.
With a perpetual system, the inventory account contains a current record of all inventory
transactions. A perpetual inventory system allows the firm to charge inventory costs to production or
to cost of goods sold as soon as inventory is used. Each acquisition of raw material and each transfer
of inventory to WIP is recorded in the inventory account as it occurs.

Table 3.1 Accounting entries for periodic and perpetual systems


Entry Periodic system Perpetual system
Purchase of inventory DR Purchases DR Inventory
CR Bank/creditors CR Bank/creditors
Selling price:
DR Bank/debtors
DR Bank/debtors CR Sales
Sale of inventory
CR Sales Cost price:
DR Cost of goods sold
CR Inventory
DR Inventory
When stock is counted None
CR Closing stock

Examples of journal entries for the two systems are given later in the chapter.
How do we calculate the unit cost for the valuation of stock in the periodic system and the value of
cost of sales in the perpetual system? Inventory costs do not remain static and therefore we need to
determine a way in which to value the inventory when a transaction takes place. If we assume we
sold cans of beans and for our last two purchases we paid R7 per can and for the other purchase
R7,10 per can, how do we determine which of these two prices we should use to value our inventory
on hand? We will address this problem in this section.
As discussed previously, calculating the cost for cost-accounting and for financial-reporting
purposes may not be the same. For financial-reporting purposes, inventory is valued at the lower of
cost or net realisable value. For cost-accounting purposes we do not normally consider the net
realisable value. There are three methods that may be used to determine the value of inventory for
cost-accounting purposes:
• First in first out (FIFO)
• Weighted average method (WAM)
• Standard costing method

Because manufacturing businesses make significant investments in inventory, the valuation method
cannot be changed annually because of the cost associated with such a change as well as the volume
of transactions that needs to be taken into account.
With the FIFO method of inventory valuation, inventory is assumed to be issued in the order in
which it is received. Therefore, it is assumed that the inventory that is received first is issued first at
the cost of that item of inventory.
The WAM uses the actual average purchase price (weighted by the applicable quantities) to
calculate the issue price.
The standard costing method values inventory by means of a predetermined standard unit price.
The three methods are discussed next by means of an example.

3.2.2.1 First in first out method


When the FIFO method is used, we assume that inventory is issued in the order in which it was
received. To explain this concept, let us look at an example.
EXAMPLE 3.2
Tumi owns a wholesale business that sells food items to retail outlets. During the first two weeks
of March 2013, the transactions pertaining to her baking flour stock were as follows:
Date Unit cost (R) Units purchased Units sold Balance
05/03 15,00 20 20
08/03 5 15
10/03 10 5
12/03 16,00 30 35
We can see that the company has 35 bags of flour left on 12 March. If we use the FIFO method,
we would value five bags at R15 per bag and the remaining 30 bags at R16 per bag as follows:
(5 × R15) + (30 × R16). Therefore, the total value of this stock is R675.
FIFO could be said to be the most logical of the inventory-valuation methods, as it assumes a
natural flow of inventory through the entity on the assumption that the inventory purchased first is
sold first. Given that inventory cost normally goes up over time, the FIFO method normally leads
to a lower cost of sales and a higher value for inventory than would be achieved using the other
two methods.

3.2.2.2 Weighted average method


With the WAM, inventory is valued using the weighted average value of the inventory. The weighted
average is calculated by multiplying the cost of each purchase of inventory by the number of items
purchased, divided by the total number of items purchased during the period. To explain this concept,
we will use the data given in Example 3.2.

EXAMPLE 3.3
Tumi’s company has 35 bags of flour left on 12 March. We calculate the weighted average cost of
the inventory as follows:
The inventory on hand would therefore be valued as follows: 35 × R15,60 = R548
Using the WAM to calculate inventory value provides a reasonably accurate cost over a
period of time, but does not reflect an accurate value for the items physically present. As
inventory cost tends to increase over time, this method tends to give a higher cost of sales and a
lower value of inventory because earlier purchases are on a weighted average basis equal to later
purchases.
One of the disadvantages of the WAM is that after every receipt, a new weighted average
value needs to be determined. In a manual system, this can give rise to a difficult and repetitive
process for an item that has a large number of transactions. Computerised systems, however, have
made this process much easier, as the calculations are performed electronically.
Tumi’s company has 35 bags of flour left on 12 March. We calculate the weighted average
cost of the inventory as follows:
Some companies use a simple average as opposed to a weighted average (e.g. (R15 + R16) ÷
2 = R15,50). If this method is used, all purchases of inventory are weighted the same and
therefore small orders with a large difference in unit price may significantly impact on the value
of inventory. For this reason, using the simple average cost is not recommended.
Some companies use a simple average as opposed to a weighted average (e.g. (R15 + R16) ÷
2 = R15,50). If this method is used, all purchases of inventory are weighted the same and
therefore small orders with a large difference in unit price may significantly impact on the value
of inventory. For this reason, using the simple average cost is not recommended.

3.2.2.3 Standard costing


With standard costing, we assign to each inventory item a standard value that only changes
periodically. Differences between the actual price and the predetermined standard cost are identified
as price variances. The calculation and use of standard costing are discussed in more detail in
Chapter 12. For the purposes of this chapter, we will only consider the impact of standard costing on
the valuation of stock. It should also be remembered that standard costs may not be used for financial
reporting and normally not for taxation purposes. The standard-costing method should only be used
for cost-accounting purposes.
We will continue with the data given in Example 3.2 to illustrate the standard-costing method.
(Please note, however, that standard costing is normally used in a manufacturing environment, and not
in retail, as in this example.)

EXAMPLE 3.4
Tumi has 35 bags of flour left on 12 March. She uses a standard-costing system, and her standard
cost for a bag of flour is R14, determined at the start of the financial year in January.
The value of the inventory on hand would therefore be 35 × R14 = R490.

3.3 Using an inventory ledger card


The inventory ledger card, either in a manual or computerised form, is the basis for valuing each
inventory item. The inventory ledger card is used to monitor quantities and costs of inventory. In the
following discussion on the use of inventory ledger cards, it is assumed that the perpetual inventory
system is used.
An inventory ledger card normally looks as follows:
Date Received Issued Balance
Unit Total Unit Total Unit Total
Units Units Units
price amount price amount price amount

The steps for using an inventory ledger card to value inventory are as follows:
• Step 1: Prepare an inventory ledger card showing the date, received, issued and balance columns.
Each column contains the following information: unit quantity, unit price and total amount.
• Step 2: Transactions showing units received and units issued are entered into the appropriate
columns.
• Step 3: After each transaction, both unit and total amount balances are updated in the balance
column. Thus, with a perpetual inventory system, the inventory balances are available after each
transaction and the inventory ledger card must reflect this. With a periodic system, the total amount
columns are often not used.
• Step 4: The cost of sales value is the sum total of the issuing column and the balance is the value
of inventory on hand.

The received column represents transactions with suppliers – either units purchased from the supplier
or units returned to the supplier. You will note that the transaction on 7 January in Example 3.5 is
indicated as a negative figure, the reason being that the original purchases need to be adjusted. The
return should always be recorded as the original unit purchase price. The total of all the transactions
in the receipts column equals the purchases for the particular inventory item for the period.
The issued column represents ‘transactions’ with the factory (manufacturing organisation) or the
sales office (retail organisation). Example 3.5 does not include a return from the factory or sales
office. Such a return should be recorded in the issues column as a negative to reflect the fact that the
inventory items are back in the storeroom. To calculate the balance in such a case, add the return from
the factory/sales office to the balance of the previous day because negative plus negative equals
positive (formula of balance column: balance previous day + receipts – issues).
The balance column represents the inventory quantity that should be physically in the storeroom
and would be balanced periodically either daily, weekly or monthly. To determine the balance at the
end of each day, you should add any receipts to the balance of the previous day and then subtract any
issues that occurred on the particular day. Be aware that the cost prices will depend on the valuation
method (FIFO or WAM), and these are discussed in detail below.
How to use inventory ledger cards with both the FIFO and weighted average methods of inventory
valuation is illustrated by means of Example 3.5.

EXAMPLE 3.5
Transactions concluded in respect of a particular inventory item:
1 January 2012 Inventory on hand 50 units @ R5 per unit
3 January 2012 Issued 20 units (sold @ R10 per unit)
4 January 2012 Received 80 units @ R6 per unit
5 January 2012 Issued 20 units (sold @ R10 per unit)
6 January 2012 Issued 30 units (sold @ R10 per unit)
7 January 2012 Returned to supplier 10 units (received 4 January)
1. Completing inventory ledger card using FIFO method
Date Received Issued Balance
Unit Total Unit Total Unit Total
Units price amount Units price amount Units price amount

1/1/2012 50 R5,00 R250,00


3/1/2012 20 R5,00 R100,00 30 R5,00 R150,00
4/1/2012 30 R5,00 R150,00
80 R6,00 R480,00 80 R6,00 R480,00
5/1/2012 20 R5,00 R100,00 10 R5,00 R50,00
80 R6,00 R480,00
6/1/2012 10 R5,00 R50,00
20 R6,00 R120,00 60 R6,00 R360,00
7/1/2012 (10) R6,00 (R60,00) 50 R6,00 R300,00

The FIFO method is applied by issuing the prices first of the inventory items that were received
first. The inventory on hand on 1 January is only recorded in the balance column, as it has been
purchased in a previous period. This cannot be included in the received column, as it would
incorrectly add to the purchases of the new period.
The transaction on 3 January requires an issue of 20 units. Because we are applying the FIFO
method, we have to issue at the first price received, which is R5 per unit, and record the
transaction in the issued column. Please note that the issue to the factory/sales office always takes
place at the cost price, and not the selling price. The balance column must now be adjusted by
looking at the previous available day’s balance, which is 50 units at R5 each. Subtract the 20
units from the 50 units and you get 30 units, indicated in the balance column on 3 January.
Similarly, to obtain the value of the balance column, subtract the issued amount of R100 from the
balance amount of the previous day (R250), which gives the balance of R150.
On 4 January, 80 units were purchased at R6 each. This must be recorded in the received
column. You will note that the balance column indicates two different prices – the 30 units at R5
carried down from the previous day as well as the 80 units at R6 per unit. The reason for this is
that when applying the FIFO method, units received first must be issued first. One cannot combine
the two different unit prices.
See whether you understand the transaction on 6 January and can calculate the balance.
You will note that the issues on 6 January consist of two transactions: 10 units at R5 each and
20 units at R6 each. The reason is, again, because we are applying the FIFO method. We have to
issue 30 units on 6 January and the balance of the previous day reflects 10 units at R5 and 80
units at R6. The 10 units at R5 per unit must therefore be issued first, and the balance of the 30
units (30 – 10), 20 units, must be issued at the next available price of R6 each. Adjust the balance
column accordingly.
On 7 January, 10 units are returned to the supplier. Remember that any returned units must be
returned at the same price at which they were originally purchased. The 10 units must be returned
at R6 each because that was the original purchase price on 4 January. This transaction must be
recorded in the received column because it is a return to the supplier – indicated as a negative
figure because of the return. To determine the balance on 7 January, subtract the 10 units at R6
each returned from the previous day’s balance of 60 units at R6 each. The reason it is subtracted
is that a positive plus a negative equals a negative (formula of balance column: balance previous
day + receipts – issues).
2. Completing inventory ledger card using WAM
Date Received Issued Balance
Unit Total Unit Total Unit Total
Units Units Units
price amount price amount Price amount
1/1/2012 50 R5,00 R250,00
3/1/2012 20 R5,00 R100,00 30 R5,00 R150,00
4/1/2012 80 R6,00 R480,00 110 R5,611 R617,10
5/1/2012 20 R5,61 R112,20 90 R5,61 R504,90
6/1/2012 30 R5,61 R168,30 60 R5,61 R336,60
7/1/2012 (10) R6,00 (R60,00) 50 R5,582 R279,00
Notes
1. R5 × 50 ÷ (50 + 80) + R6 × 80 ÷ (50 + 80)
2. R5 × 50 ÷ 50 + (80 – 10) + R6 × 80 ÷ (50 + 70)

According to the WAM, a weighted average cost price of the inventory item must be calculated
each time a transaction is recorded in the received column. This weighted average cost price is
then used for the issued column as well as the balance column. Please take note that if a return
from the factory takes place, it must be returned in the issued column (as a negative) at the cost
price it was originally issued at, if known.
There is no difference in the recording of the transactions on 1 and 3 January between the
FIFO method and the WAM because both transactions use only a single cost price of R5.
On 4 January, 80 units are received at R6 each; you must record this in the received column.
To calculate the balance column, add the number of units from the previous day in the balance
column and the units received on 4 January (30 + 80) and record the 110 units in the balance
column under the unit’s sub-column.
The weighted average cost per unit should then be calculated. In total, the entity purchased
130 units (50 + 80); 50 units were purchased at R5; 80 units were purchased at R6. Therefore R5
× 50 ÷ (50 + 80) + R6 × 80 ÷ (50 + 80) equals R5,61 per unit. The value of the balance is
therefore 110 × R5,61 = R617,10.
The issues on 5 and 6 January will take place at the average cost price of R5,61. To calculate
the balance column, subtract the value of the issues from the value in the previous day’s balance
column and do a similar calculation for the units.
The return to the supplier on 7 January must take place at the original purchase price of R6.
Because a transaction was recorded in the received column, a new average unit cost price must
be calculated. To calculate the balance column, take the number of units from the previous day in
the balance column and subtract the units returned (remember a positive and a negative equals a
negative, as described above) on the 7 January (60 − 10) and record the 50 units in the balance
column under the units sub-column. The new weighted average cost per unit then needs to be
calculated as before. The total number of units purchased changes from 130 to 120. Fifty units
were purchased at R5, and 70 at R6, therefore R5 × 50 ÷ (50 + 70) + R6 × 70 ÷ (50 + 70) equals
R5,58 per unit. The value of the balance is therefore 50 × R5,58 = R279,00.
You will notice that the value of the inventory is calculated as a different figure when we use
these two methods. However, there should never be a difference in the number of units between
the FIFO and weighted average methods.

The ways in which we account for inventory in the general ledger in a manufacturing environment are
discussed in Chapter 7.
Example 3.6 shows how the journal entries would be processed in the general ledger using the
periodic and perpetual inventory systems.

EXAMPLE 3.6
Using the information from Example 3.5, the following journal entries would be made in the
general-ledger accounts using the WAM. The entries for the FIFO method would be the same, but
the amounts would differ. We will assume that a stock count was conducted on 8 January 2012,
which confirmed that 50 units were in stock and that closing entries were done on that day for the
periodic system.
Perpetual
Date Periodic Account Dr (R) Cr (R) Dr (R) Cr (R)
Account
No entry required; inventory opening
01/01/2012 Opening stock 250,00
balance = R250
inventory 250,00
03/01/2012 Bank/debtors 200,00 Bank/debtors 200,00
Sales 200,00 Sales 200,00
Cost of sales 100,00
Inventory control 100,00
04/01/2012 Purchases 480,00 Inventory control 480,00
Bank/creditors 480,00 Bank/creditors 480,00
05/01/2012 Bank/debtors 200,00 Bank/debtors 200,00
Sales 200,00 Sales 200,00
Cost of sales 112,20
Inventory control 112,20
06/01/2012 Bank/debtors 300,00 Bank/debtors 300,00
Sales 300,00 Sales 300,00
Cost of sales 168,30
Inventory control 168,30
07/01/2012 Bank/creditors 60,00 Bank/creditors 60,00
Purchases 60,00 Inventory control 60,00
08/01/2012 Inventory 279,00
Closing stock 279,00
Cost of goods sold using the periodic system was therefore:
Opening stock R250
Purchases (480-60) R420
Closing stock (R279)
Cost of goods sold R391

CLOSING PRACTICAL EXAMPLE


Thando established an inventory management system that calculated the selling price on each order
based on the actual costs of that order. As a result, Thando’s company performed much better in
the second year that it operated. The cost and selling price of the company’s last order were
calculated as follows:
Fabric R75
Labour R25
Delivery charges of fabric R35
Overhead costs allocated R10
Total cost R145
Selling price with 50% markup R217,50

Thando was also able to calculate the value of the direct materials (fabric), consumables and
finished goods that she had on hand after she instituted a perpetual inventory system using the FIFO
method. She and her business partners felt satisfied that they were now in control of their costs and
profits.

3.4 Summary
This chapter discussed the various principles of inventory and explained the terms referring to
inventory, including the four types of inventory – raw materials, WIP, finished goods and retail
inventory.
The importance of calculating the cost of inventory was highlighted, including the costs that should
be included this calculation.
The periodic and perpetual inventory systems were explained. The methods used to value
inventory were discussed and examples used to illustrate the three methods:
• FIFO – where it is assumed that inventory is issued in the same order it was received
• WAM – where the weighted average cost of inventory is calculated
• Standard costing method – where standard costs are used to value inventory

Finally, this chapter showed how to use an inventory ledger card and make the relevant journal
entries to record transactions in a general ledger.
PRACTICAL ACTIVITY
You are the store manager of a large retail chain. As part of your responsibilities, you have to keep
a computerised inventory ledger for each of the items you manage.
One of the inventory items you take care of is numbered CS023. There was a technical
malfunction on your computer and you are not able to access the inventory system. The stock
however has to be reconciled immediately.
You have reviewed your supporting documentation and the following transactions occurred
pertaining to CS023 for the last month:
Date/narration Received unit price No. of units received No. of units issued
01 Balance R65 25
07 15
09 R69 30
10 7
15 15
19 12
28 R67 20
1. Prepare a manual inventory ledger card to reconcile the CS023 inventory. Inventory is valued
using the FIFO basis and the perpetual inventory system. The inventory ledger card should
have the following columns:
Date Received Issued Balance
Unit Total Unit Total Unit Total
Units Units Units
price amount price amount price amount

Calculate the cost of goods sold and value of closing inventory for the month.

Short questions
1. Briefly describe the four types of inventory and give an example of each.
2. Briefly describe the differences between the periodic and perpetual stock methods.
3. Briefly describe the FIFO method of stock valuation.
4. A company uses the weighted average cost basis to calculate the unit costs of a product over
a 12-month period. Calculate the weighted average cost of Product JR42, where the
following transactions took place during the period:
Transaction date No. of items purchased Unit cost (R)
5 Jan 3 500 2,50
4 Mar 8 000 2,25
17 Aug 4 000 2,75
18 Nov 3 000 2,60
5. Calculate the cost of manufacturing a chair using the following information for a furniture
manufacturer:
Product Basis of allocation per chair Cost
Timber 15 units R28 per unit
Fabric 1 m of fabric R40 per m
Labour cost 2 labour hours R30 per labour hour
0,1% of total overheads for the R500 000 total overhead costs
Overhead costs
prior year in prior year

Long questions
1. Painpharm is a pharmacy that maintains its inventory records using the periodic system.
Assume the company measures inventory on the FIFO basis. On 1 June 2013, the opening
inventory for Granpain, an over-the-counter painkiller, was 20 units at a cost of R24 per unit.
Painpharm is not VAT registered.
The following transactions occurred during the month:
• Purchased 10 units of Granpain from a supplier for R25 each.
• Sold 25 units to customers.
• Purchased 25 units from a supplier for R26 each.
• Returned 5 units purchased at R26 per unit, as they were damaged in transit.
• Sold 10 units to customers.
• One customer returned a unit. This unit was returned to the supplier, which refunded the
purchase price.
• Ordered 50 units from a supplier on 25 June 2013 to take advantage of a reduced price.
These units were received and the supplier invoiced Painpharm on 3 July 2013.
Calculate the units of Granpain on hand as at 30 June 2013 and calculate the value in rands at
which inventory will be shown in the management accounts.

2. As the production manager of Elegance Manufacturing Jewellers, you have been asked to
calculate the selling price of a necklace for a client. The jeweller has estimated the time
taken to manufacture the necklace and the raw materials required. You have prepared the
following summary:
No. of units required Cost per unit
Gold 5 R600
Diamonds 3 R400
Labour 8 R200
Overheads ? R30
Overheads are allocated as one unit of overheads for every two labour units. Manufactured
jewellery is marked up by 40 per cent of cost.
Calculate the selling price of the necklace based on the above information.

3. As the raw-materials store manager for MM Manufacturers, you have received and checked
the following bin cards for the three raw materials your company uses:
Product XJ12 Received Issued Balance
Date Unit cost (R) Units Units Units
05/03/2012 5,50 200 200
09/03/2012 50 150
12/03/2012 60 90
15/03/2012 80 10
16/03/2012 5,60 500 510
18/03/2012 50 460
23/03/2012 180 280
28/03/2012 200 80
29/03/2012 5,65 300 380
31/03/2012
Physical stock 20 360
short
Product XD63 Received Issued Balance
Date Unit cost (R) Units Units Units
01/03/2012
35
Opening stock
03/03/2012 20 15
05/03/2012 10 5
07/03/2012 25,00 30 35
15/03/2012 5 30
24/03/2012 7 23
31/03/2012
Physical stock 1 221
short
Note: 1. 35 units of Product XD63 had been received but are still in the delivery area.
This should be included in the raw-materials store count.
Product JJ1812 Received Issued Balance
Date Unit cost (R) Units Units Units
15/03/2012 0,50 1 000 1 000
18/03/2012 25 975
19/03/2012 510 465
29/03/2012 0,45 2 000 2 465
31/03/2012
Physical stock 0 2 465
short
Calculate the value of inventory using the FIFO method.
4. Arcade Games is an entity that sells and rents out games to entertainment centres. The
company is not VAT registered. Arcade Games purchased 10 Car Racing Pro arcade games,
all of which were sold. The costs incurred in the purchase and sale of the games were as
follows:
Per unit (R)
Cost (excl. VAT) 2 500
VAT @ 14% 350
Transport from supplier 300
Early settlement discount from supplier 150
Sales commission 5% of selling price
The games were sold for the following prices to customers:
Per unit (R)
5 units 3 500
2 units 3 800
3 units 3 900
Calculate the total gross profit made on the sale of the Car Racing Pro games.

5. Progress Electrical Appliances (PEA) purchases television parts from Japan, which the
company assembles and sells locally. During the second quarter, PEA’s production
information was as follows:
Opening stock No. of parts Value per part (R) Total value (R)
Unassembled parts 200 900 180 000
Partially assembled
50 1 000 50 000
parts
Finished goods 120 1 100 132 000
Labour and production overheads are allocated as follows:
Partially assembled parts Finished goods
(R per unit) (R per unit)
Labour 80 160
Overheads 20 40
The total cost of labour was R91 200 and overheads were R22 800 for the quarter. During
the quarter, PEA received two orders of unassembled parts from Japan:
Order 1 Order 2
Number of parts 200 150
Purchase price per part in US$ 100 103
Exchange rate (R/$) 8,50 8,40
Import duties (total R) 5 000 3 750
Shipping fees (total R) 30 000 25 000
PEA sold 500 televisions during the quarter at R2 000 per television. On the last day of the
quarter, PEA had the following inventory on hand:
No. of parts
Unassembled parts 58
Partially assembled parts 60
Finished goods 160
PEA uses the periodic inventory system and values inventory using the FIFO method. Calculate
PEA’s gross profit for the second quarter showing the sales, opening stock, purchases, labour
cost, overhead cost and closing stock separately.
4 Cost elements: Inventory management and
control
Reynier Boersma and Johan Hefer

LEARNING OUTCOMES
After studying this chapter, you should be able to:
• discuss the inventory-management cycle, which includes:
– setting norms and standards for inventory levels
– identifying the need to order inventory
– ordering inventory
– storage of inventory
– internal and external distribution and consumption of inventory
• determine the economic order quantity;
• establish the correct reorder point; and
• briefly discuss some advanced inventory-management systems, including:
– materials requirement planning (MRP)
– just in time (JIT) purchasing
– enterprise resource planning systems.

PRACTICAL EXAMPLE
Jenny’s new company manufactures picture frames, which she normally sells at craft markets.
However, she has recently received a large order from a chain of photo shops. This order
necessitates a large amount of raw materials. Jenny is unsure when to order the materials, how to
manage the stocks and how to keep her level of inventory low, as she does not have much storage
space. This large order has made Jenny think carefully about how to manage her inventory
efficiently and effectively.

4.1 Introduction
The management of inventory is critical because inventory is often a large component of an entity’s
assets and is often a critical factor in determining profitability. An entity’s inventory incurs costs for
various reasons. There is not only the investment in the stock itself – there are also other costs
incurred by ordering and holding inventory. Therefore, inventory needs to be managed carefully. The
inventory-management cycle is the management of the flow of inventory through the organisation.
In Chapter 3, we explained the accounting treatment of inventory; the focus of this chapter is the
management of inventory by an entity.

4.2 The inventory-management cycle


The inventory-management cycle comprises various steps that an entity should follow when managing
its inventory. This process is illustrated in Figure 4.1 as it applies to a manufacturing environment.

Figure 4.1 The inventory-management cycle

Before the process can start, the entity needs to determine its inventory levels and requirements.
These will be discussed later in the chapter. The process starts with the entity identifying a need to
order inventory. Next, inventory is ordered and received. The inventory will then enter a storage
facility, or warehouse, from which it is issued into the manufacturing process. After the finished
goods have been completed, they are sent to a storage facility again and then issued to customers. It is
important to understand that, despite these general principles, the inventory process may differ from
business to business, especially when it comes to the storage of raw material, work in process and
finished goods – which may be stored in the same storage facility or in different storage facilities
during the manufacturing process.
The inventory-management cycle will now be discussed in more detail in the following sections.
Note that the generic term ‘inventory’ is used throughout these sections to include raw material, work
in process, finished goods and retail goods.

4.2.1 Setting norms for inventory levels


Management needs to establish the quantity of inventory required at any given point to ensure that the
entity, firstly, does not run short of stock (known as a stockout), as this could lead to a production
interruption, and, secondly, does not keep excess inventory, as excess inventory incurs holding costs.
The entity should determine the precise level of inventory that it needs to hold because of the
delay between the time of acquisition and the time of delivery of the inventory. Holding inventory
incurs purchase, opportunity and storage costs, therefore the cycle needs to be managed in a cost-
effective manner to ensure that unnecessary costs are avoided.
Determining and monitoring inventory levels are becoming increasingly important in a business
environment of globalisation and international competition. (This is especially the case if inventory is
ordered from overseas, which may have a longer delivery time than inventory ordered locally.) For
example, if a furniture manufacturer does not plan for its inventory requirements and does not have the
necessary wood, the entire production process would come to a standstill. As a result, the company
would become less competitive. Lost production and sales will lead to financial losses, which would
decrease profitability and put the company at risk.
When planning for and managing the ideal level of inventory, the following concepts are important
to consider. For the purpose of these examples, raw materials will be considered, but the same
principles also apply to work in process, finished goods and retail goods.
• Normal inventory: This is the normal amount of inventory used in the production process that
needs to be on hand to ensure that normal production can continue. For example, in a textile
company six rolls of fabric may be the normal level of inventory needed to allow standard
production.
• Safety inventory (safety stock): This is the amount of inventory that needs to be held to ensure
that production can continue in the event of an unforeseen situation that might inhibit the ability to
procure raw materials in the normal time frame, such as a labour strike in the company that
transports stock. Safety inventory can be defined as extra inventory carried to prevent stockouts in
unforeseen situations. In the example of the textile company, the safety inventory may be two rolls
of fabric, as management has established that the factory can continue with production for a
reasonable amount of time with that quantity if they were unable to procure new stock.
• Stockout: A stockout occurs when inventory is not available for production purposes or when
there is a demand for a product that is not available.
• Strategic inventory: This is inventory that is held for strategic reasons. As an example, the textile
company may decide to order four rolls of fabric in excess of its normal inventory level, as a
rumoured upcoming strike may interrupt supply.
• Speculative inventory: This is inventory that is held for economic reasons. As an example, the
textile factory may decide to order five rolls in excess of the usual order, as management believes
that the price of fabric may go up the following month.
• Inventory in transit: This is inventory that has already been purchased but has not yet been
received; in other words, it is still in the process of delivery.
• Consignment inventory: This refers to inventory that is owned by one party but is in the
possession of another party. As an example, the textile factory may send some finished goods to a
retail outlet on consignment. The goods remain the property of the manufacturer until they are sold,
but the goods are in possession of the outlet.
• Technical inventory: The value or quantity of inventory that is physically in the enterprise’s
possession.
• Economic inventory: The value or quantity of inventory that represents the amount of inventory
that the entity owns after adjusting for inventory already paid for but not yet received, or inventory
already sold but not yet delivered, for example.
• Overstocking: This occurs when the amount of inventory held is above the quantity justified by
the volume of production.
• Understocking: This occurs when the amount of inventory held is below the quantity justified by
the volume of production.
• Average inventory: The average inventory is a figure often used in calculations. Although it is
possible to determine the average inventory with mathematical precision, it is so cumbersome that
the following formulae are usually sufficient to calculate it:

Average inventory = (opening inventory + closing inventory) ÷ 2


Or alternatively:
Average inventory = (order size ÷ 2) + safety inventory

• Maximum inventory: This is the greatest possible quantity of a particular item that can be held by
the entity at a point in time. It may occur when a new order is delivered earlier than the normal
delivery period.

We will now consider an example to illustrate these principles.

EXAMPLE 4.1
Brett owns a small supermarket. He sells an average of 100 loaves of bread per day. He normally
orders 100 loaves of bread on a daily basis. The bread stays fresh for three days after delivery.
The delivery is sometimes late so he always keeps 20 extra loaves of bread in stock for that
eventuality.
Brett’s safety stock is therefore 20 loaves and his average inventory would be 70 loaves
(normal order size ÷ 2 plus safety stock). The maximum stock would be 300 loaves, as the bread
only stays fresh for three days.
If the delivery was late and he had no inventory on hand, Brett would experience a stockout.
If Brett hears on the news that the day after next all transport workers will hold a one-day
strike, he would decide to order 200 loaves of bread the next day to ensure that he does not run
out if no delivery takes place. The extra 100 loaves of bread would be strategic inventory.
He receives a notice from the bakery that the price of bread will increase on Wednesday. He
may decide to order 300 loaves of bread on Tuesday (as the bread will keep for three days) and
put in no order for Wednesday and Thursday. The extra 200 loaves of bread would be
speculative inventory.
At the end of a day, he counts his stock and finds that he has 30 loaves in stock. The delivery
truck broke down and therefore a delivery of 100 loaves that he has already paid for will only be
delivered the next day. An old-age home that had ordered and paid for 10 loaves of bread has not
picked them up yet. Brett has 100 loaves of inventory in transit. His technical inventory is 30
loaves, as that is the amount of stock actually in his store, and the economic inventory is 120
loaves (30 in stock +100 in transit – 10 sold but not delivered).
Brett would be overstocking if he kept an inventory of 150 loaves per day, as he should only
have 120 (daily sales plus safety inventory), and would be understocking if he kept 80 loaves of
bread.

The function of inventory management centres on the ability to calculate accurately how much
inventory needs to be purchased and by what time (because of the time delay between acquisition and
delivery). This function becomes increasingly important due to the increase in technology and the
competitive environment. If too little is purchased, the enterprise may run out of inventory. If too
much is purchased, capital is unnecessarily invested in inventory and storage space is wasted. If any
one of these inputs is calculated incorrectly, it may result in unnecessary losses. The same would
apply if materials are not received at the right time.
In order to achieve maximum inventory efficiency, there are a number of parameters relating to
inventory management that should be adhered to. The process of determining levels of inventory can
be summarised by the following question: how much of what is required, and when?
• Firstly, how much? The exact quantity to be purchased must be determined. Usage remains the
basic guideline. However, other factors need to be considered as well. Should an additional
quantity be purchased as safety inventory because a strike may occur, which could affect delivery
time? Or perhaps speculative inventory should be considered as a means of diminishing the effects
of a possible price increase. Sometimes certain stock can be so crucial, indispensable and
difficult to obtain that strategic inventory needs to be maintained.
• Secondly, what? It is obvious that specifications of what needs to be purchased must be prepared
before purchasing can proceed. These specifications must clearly indicate the quality, the
dimensions and the combination of materials required.
• Thirdly, when? How much time elapses between the date of placing an order and the date of
delivery? When must the order be placed so that the inventory will be delivered when it is
needed? It is essential that materials are supplied to the enterprise when required. Therefore, the
size of the order and the date the order is placed are of the utmost importance. Delivery time, also
called the order period, and the reliability of the supplier play an important role in the time
parameter.

4.2.2 Identifying the need to order inventory


The point at which inventory needs to be reordered can be identified in many ways and is related to
the size and structure of the entity. Often the need to purchase raw materials comes from an order for
finished goods. As an example, a company may receive an order for 100 dresses from the sales
personnel. The production department would then calculate the quantity of fabric, thread and other
consumables required to fulfil the order. These quantities are then sent to the procurement department
to order stock. There may be an entire department dedicated to the order function, or a single owner-
manager who performs the function.
Alternatively, often where large quantities of consumable products are sold, as is the case with
retailers, inventory levels may be monitored through a computerised or manual inventory system. If
the quantity on hand reaches a certain point, known as the reorder quantity, the system will inform the
ordering department that the level of that inventory is low and that an order needs to be placed.
The way in which the need to order inventory is determined differs from company to company, but
whatever the method, it is critical that the system functions well. If inventory is not ordered when
required it may place the company at financial risk.

4.2.3 Ordering inventory


Once the need to order inventory has been identified, the order should then be placed with the
supplier. Ordering is an important process and competent people need to be employed to fulfil this
function. Even though the requirement for the order has already been established, it is essential to
source the products from reliable suppliers and to achieve certain benefits, such as cash discounts for
goods paid for with cash and beneficial credit terms negotiated with suppliers. In many cases, the
order needs to be placed with the supplier offering the most favourable terms.
All types of entities invest a significant amount of financial resources in inventory and therefore it
is of the utmost importance to achieve the best possible terms. However, in certain environments
where the quality of goods and the efficient delivery of inventory are paramount, the relationship with
the supplier and inventory quality may take precedence over cost.
In a competitive environment, the method of ordering and the relationships with suppliers are
important. For example, when US retailer Wal-Mart purchased a majority stake in Massmart, local
producers and trade unions complained to the Competition Commission about the advantage that Wal-
Mart would have over other retailers because of Wal-Mart’s very competitive supply chain
management and ability to source products very cost effectively and deliver them timeously across the
globe. In such a competitive environment, it is essential that trustworthy relationships are established
with producers and suppliers.
Effective control must be exercised over the purchase function at all times and authorised
personnel must approve and sign all orders. This control extends to ensuring that all goods ordered
are received and that they are of the specified quality.

4.2.4 Inventory storage


The storage of inventory before it is sold is important for several reasons. Firstly, the layout of the
warehouse or factory needs to be conducive to the effective operation of the distribution network,
namely the timely entrance of products into the production process. An example of an efficient facility
layout is shown in Figure 4.2. The goods are received at the facility and enter the production process,
and end in a finished-goods store, from where they are dispatched. It is also vital that storage is
designed to protect stock from damage. If inventory has to be moved from one part of a factory to
another and stores are not in accessible places, the chance of goods being damaged is greatly
increased.

Figure 4.2 Layout of manufacturing facility with storage areas


It is also important to consider any special requirements of inventory that is stored. As an example,
hazardous materials may need to be securely locked away and provision made for appropriate fire
equipment and ventilation. Cold-storage facilities need to be available for perishable stock, like
certain food products.
It is important that controls are in place to identify and keep track of inventory. To keep control
over inventory, each item needs to be given a unique inventory code. These codes are updated by the
inventory system, which may be manual or computerised. Today most inventory systems are
computerised and often integrated with the accounting system.
In a manual system, an inventory ledger, also known as a bin card, may be used to keep a record
of the quantity of each type of material in the warehouse. All receipts and issues are recorded on this
card, which is a valuable mechanism for the efficient control of the physical inventory. Relevant
information is contained on the bin card, such as the reorder point, order quantity and safety
inventory. An example of a bin card is given in Figure 4.3.

Figure 4.3 Example of bin card


Super Supermarket
Bin Card
Bin Card No: 25 Maximum quantity: 300
Name of item: White Bread Minimum quantity: 20
Stores ledger folio: 52 Ordering quantity: 100
Received Issued Balance
Unit Total Unit Total Unit Total
Date Units Units Units
Price Amount Price Amount Price Amount

In a computerised system, movement of inventory is monitored and controlled by means of computer


terminals at various points in the receiving, production and dispatch processes. When inventory is
moved from one of these points to another, the movement is recorded. Bar code scanners are often
used to simplify this process and allow managers to identify the quantity of stock on hand at any stage.
The inventory records do not always form a direct part of the accounting records, but will often
form part of the source documents of the financial accounting system. Often this is to prevent
unauthorised personnel viewing the cost of inventory items. Normally, only the quantity of physical
inventory is known to production and storage staff.
In today’s electronic era, with centralised distribution networks, it has become increasingly
important to ensure that the physical inventory is in line with the inventory as specified in the
inventory records, and therefore a physical inventory count should be conducted from time to time.
The intervals between inventory counts can vary. However, it is common practice for inventory
counts to be conducted at year end. Discrepancies between the inventory records and the physical
inventory on hand should be investigated and followed up by appropriate management. Some of the
most common discrepancies can be ascribed to theft, incorrect calculations and transactions that have
not been entered into the system. Some of these errors cannot be prevented by a sophisticated system
and are simply due to human error.
We discussed the accounting entries for the receipt, movement and dispatch of inventory in
Chapter 3 and will discuss it further in Chapter 7.
4.2.5 Internal and external distribution and consumption of
inventory
The success of an entity may be closely related to the quality of its inventory management processes
and procedures. One of the most critical aspects in these processes is the distribution and
consumption of inventory within an entity. All movements of inventory, especially in the case of
higher-value items, should be monitored. This is particularly applicable for retailers, which may
experience large losses due to theft or breakages.
Efficient controls need to be in place to ensure the effective distribution of inventory. A
requisition is normally the basic document used for issuing inventory from the warehouse. A
requisition document has two purposes:
• To provide authorisation to the person in charge of the inventory to issue the material
• To serve as source documentation for the accounting process and show the reasons for the issue

Requisitions are often in the form of electronic documentation. As an example, let us assume that a
company has a storeroom for its raw materials. If the production personnel require raw materials to
use in production, a requisition should be completed and signed by authorised personnel. The store
manager then issues the raw materials and signs the requisition form. This document (or its electronic
equivalent) is then sent to the accounts department. The inventory records also have to be updated
with the movement of stock. An example of a stock requisition form is shown in Figure 4.4.

Figure 4.4 Example of stock requisition form


Super Supermarket
Inventory requisition
Requisition number:
Date: 20/03/2013
523
Please issue the following materials for use on order 58:
Quantities Description
52 White bread
Date issued
21/03/2013 Department Manager Storeman

It is also important to control the issue of finished goods, which should not be issued without
authorisation. In many cases, an order is received by the sales staff, which is entered into the
accounting system. The order would then be confirmed and authorised by a manager, and this will
generate a delivery note, or requisition. The delivery staff or client would then pick up the order
based on the delivery note or requisition. All goods should be checked before they are dispatched.
In this section, we discussed the issues surrounding the inventory-management cycle. The next
section considers the quantitative methods for determining the economic order quantity and reorder
point. It also discusses some of the advanced systems that are used to keep inventory costs to a
minimum.

4.3 Economic order quantity


One of the most frequently used methods to determine the optimal order quantity, and thereby the
optimal inventory holding and reorder point, is known as the economic order quantity (EOQ).

The EOQ calculates the order size that is most efficient for the entity. The EOQ takes into account
the cost of holding an item of inventory and the cost of ordering it to determine the optimal order
size.

We will briefly discuss these two costs and then show how to calculate the EOQ.

4.3.1 Cost of holding inventory


There are various costs associated with holding inventory. These include storage costs, labour costs
and inventory losses. As with most decision-making techniques in the financial area, one should only
consider incremental costs when determining inventory-holding costs. In other words, with inventory-
holding costs, only those costs that vary based on the order size should be considered. For example, if
a company’s warehouse rental is R15 000 per month, there is enough space to hold an order and the
extra space cannot be used to generate any income, the R15 000 is a sunk cost and should not be
considered when calculating the EOQ.
Holding costs normally include, but are not limited to:
• opportunity cost of investment in inventory;
• incremental insurance costs;
• incremental warehouse and storage costs;
• incremental material-handling costs; and
• cost of obsolescence and deterioration of inventory.

It should be borne in mind that these costs may be incremental at different levels, as incremental costs
may be incurred if specific order sizes are exceeded. As an example, if a warehouse can hold 100
000 units, the cost of the warehouse is not an incremental cost until a higher level is reached where
further storage space may be required.

4.3.2 Cost of ordering inventory


There are various costs incurred by ordering inventory. These may include the labour cost of the
personnel responsible for placing orders, receiving inventory and paying for orders received and
administrative costs, like telephone bills. Although these costs may be significant, only incremental
costs should be considered when calculating the cost of ordering, as with the cost of holding
inventory, discussed above. Normally, the incremental cost for ordering would not be very high
because a high proportion of ordering costs tend to be of a fixed nature.
Let us illustrate the cost of holding and ordering inventory by means of an example.

EXAMPLE 4.2
KKJ Wholesale Frozen Foods is a wholesaler of food to supermarkets. The company operates a
large warehouse and is considering the reorder levels for frozen chickens it receives from a
supplier. In the last year, it ordered in batches of 1 000 chickens but due to increased demand, the
company needs to reconsider its order size. The details of the chicken orders and other costs are
as follows:
Number of chickens purchased per annum 30 000
Average cost per chicken R15
Cost of warehouse per annum (includes all costs) R150 000
Cost of ordering department per annum R120 000
Insurance cover for all frozen goods R100 000
The sale of chickens remains even throughout the year. If the size of the order for chickens
increases above 1 000 units per order, the cost of the warehouse is expected to rise by R30 000
per annum because more fridges would be required. Ordering costs would be expected to rise by
R900 per annum. The company expects a normal level of inventory of 1 500 chickens on hand (1
000 chickens currently). The insurance policy currently covers, among others, the loss of 1 000
chickens on hand due to power and mechanical failures. The insurer has indicated that at a level
of 1 500 chickens an extra R0,50 per average unit on hand would be required in excess of the
current cost of coverage.

Required
Calculate the holding cost per chicken and order cost per chicken. The entity’s required rate of
return is 15 per cent.

Suggested solution
Firstly, we need to identify the relevant costs. The purchase costs of the chickens and the fixed
costs of the warehouse and ordering department would not be relevant, as they do not change with
the size of the order. Therefore we need to take into account only the incremental costs, as
follows:
Incremental warehouse costs R30 000,00
Incremental investment in inventory
R1 125,00
([500 units × R15] × 15%)
Incremental insurance costs (500 units × R0,50) R250,00
Total annual holding costs R31 375,00
Holding cost per unit (÷ 30 000) R1,05
Incremental order costs will be R900 ÷ 30 000 units = R0,03 per unit.

4.3.3 Calculating the economic order quantity


The EOQ is the order quantity that should result in the lowest total holding and ordering costs. The
EOQ is calculated by using the following formula:
Where Q is the EOQ   , D is the total demand for the period, normally annual demand, O is the
incremental cost per order and H is the incremental holding cost per unit.
We will now calculate the EOQ for Example 4.2.
The EOQ is calculated using the formula:

Therefore, orders of 1 449 units will minimise the costs incurred by ordering and holding.
The question may arise, what if a supplier offers a discount for larger orders? Based on the
information in Example 4.2, what would happen if the supplier offered a discount of 5 per cent for
orders larger than 2 000 units? To determine if the quantity discount should be accepted or not, we
should calculate the saving in unit price, the saving in ordering costs and compare that to the extra
holding costs. We will consider this scenario in Example 4.3.

EXAMPLE 4.3
Assume the same information as given in Example 4.2, but that the supplier offers a discount of 5
per cent per chicken for orders over 2 000 units. Assume that all incremental costs stay at the
same levels as in Example 4.2.

Required
Advise whether or not orders of 2 000 units should be placed to make use of the quantity
discounts.

Solution
The savings should first be considered:
Saving in purchase price
R22 500
(30 000 × 15 × 5%)
Saving in ordering costs
Immaterial

Total savings per annum R22 500


Note: The saving in ordering costs is calculating by multiplying the annual demand by the
ordering cost and dividing it by the quantity, and then minus the annual demand and order costs by
the EOQ. This will give the rand value of the saving.

We should then calculate the increased holding costs if the larger quantity is purchased by
calculating the difference between the EOQ and the quantity, and dividing that by the holding
costs. This is then divided by two, as it is based on the average quantity on hand:

As the savings far exceed the increased holding costs, management should decide to place orders
in batches of 2 000.

4.3.4 Calculating the reorder point


It is important to order inventory at the right point so that it is available when required and there is
not excess stock, which incurs costs. Carrying excess inventory is costly, for the reasons discussed in
Section 4.3.1. If, however, an entity runs out of inventory (stockout) this might give rise to losses. The
latter is especially true in the manufacturing sector, where a shortage of raw materials may bring the
plant to a standstill.

The point at which inventory needs to be ordered is known as the reorder point.

The main factor to consider when determining the reorder point is the lead time. This is the time it
takes from when inventory is ordered to when it is delivered. Let us assume that it takes two weeks
for a supplier to deliver. The company would need to reorder when the inventory on hand is sufficient
for two weeks’ worth of production or sales. When ordering from overseas, delivery may take
considerably longer, and there are many variables so the lead time cannot always be assessed. The
reorder point in such cases should be very conservative.
As production and sales are not always even over a period of time, a firm needs to protect itself
against fluctuations and normally keep a level of safety stock in case of emergencies. We will
illustrate the reorder process by means of Example 4.4.

EXAMPLE 4.4
KKJ Wholesale Frozen Foods is considering the reorder levels for the frozen chickens it receives
from a supplier. The company purchases 30 000 chickens per annum and sales occur evenly
throughout the year. KKJ believes that a safety inventory of 200 chickens needs to be kept at any
time. The supplier has a lead time of seven days.

Required
Calculate the inventory level at which an order for chickens should be placed.

Suggested solution
The company sells 30 000 chickens per annum over 52 weeks. The supplier takes one week to
deliver so when the inventory level reaches 577 chickens (30 000 ÷ 52) there should be enough
chickens on hand to last for the one-week lead time. The company, however, also requires a
safety inventory of 200 chickens. Therefore, the reorder point should be when the inventory level
reaches 777 chickens (577 + 200).

4.4 Advanced inventory-management systems


In today’s competitive business world, it is critical that an entity, whether it trades in the
manufacturing or retailing sector, manages its inventory in the most cost-effective and efficient way.
Various inventory-management systems have been created to minimise holding costs and maximise
efficiency. In this section, we briefly discuss the materials requirement planning (MRP) system, the
enterprise resource planning (ERP) system and the just-in-time (JIT) system.

4.4.1 Requirement planning systems


A requirement planning system manages the ordering process to ensure inventory is received in time.
Requirement planning systems have their origins in the MRP system developed in the 1960s, when the
manufacturing environment became more and more complex, and distances between suppliers,
customers and manufacturers increased. MRP uses a computerised system to plan production. This
system includes the subcomponents of all items and can therefore calculate the raw materials and lead
times required to ensure that delivery takes place before production commences.
The main goal of an MRP system is to generate a coordinated schedule of which materials are
required when taking into account lead times to ensure that inventory levels are kept to a minimum,
while at all times ensuring that inventory will be available when required.
The MRP system has also evolved into manufacturing resources planning, whereby all resources,
including labour and overheads, are included in the planning to ensure that the necessary labour and
overheads are in place when required for production.
This system has further evolved into the sophisticated ERP system, which is an integrated set of
databases and programmes covering financial accounting, distribution, manufacturing, purchasing,
ordering and human resources. ERP systems are especially useful for large multinational
corporations.
To give an example of an ERP system, if a customer in Japan places an order for a new VW Polo
assembled in South Africa with a specific set of specifications, the customer’s credit limit will
automatically be checked and the order confirmed. The ERP system will then update VW South
Africa’s production schedule so that the specific vehicle based on the required specifications will be
manufactured. This will update the ordering system to order any required components for the vehicle,
and the manufacturing process will begin. The system will also book a place on the vessel that will
ship the vehicle to Japan. These systems will communicate with and update the financial-accounting
system.
Many ERP systems are large, expensive and complex, and therefore only worth implementing in
multinational corporations where the benefits of the system outweigh the costs. There are, however,
smaller and cheaper ERP systems that make it possible for even medium-sized businesses to have an
ERP system.

4.4.2 Just-in-time systems


One of the major costs in a manufacturing environment is the resources spent on inventory storage and
handling. If we take the example of a vehicle manufacturer, thousands of components need to be
stored, protected and handled for assembly. This extensive inventory takes up a large amount of
space, which increases rental and incurs other costs, such as labour, insurance, municipal charges and
other overheads. Inventory can also be damaged or stolen while in storage.
The JIT system is a form of MRP that keeps inventory-holding costs to a minimum. This is
achieved by organising the delivery of stock and beginning manufacturing of a product at the
precise point they are required.

The objectives of a JIT system are to meet customer demands for inventory, increase the quality of
products and decrease costs.
A full discussion of JIT systems falls outside the scope of this book, but it is important to
understand the main goals of this type of system:
• To eliminate any non-value-added activities within the manufacturing environment (e.g. storage
costs).
• To eliminate inventory on hand – all inventory is received immediately before required and all
inventory is sold immediately after it is manufactured.
• To achieve zero defects in the manufacturing process, as the system does not allow for spare
components to remanufacture or fix a defect. If a defective product is manufactured, the whole
process would need to be started again.
• To manage production in batch sizes of one, as one order gives rise to one product.
• To eliminate breakdowns, as a breakdown causes the whole system to grind to a halt.
• To eliminate late or defective deliveries from suppliers, as the whole production system is
premised on receiving correct inventory when it is required, and not before or after.

As should be clear from the above discussion, a JIT system requires very good relationships between
manufacturers and suppliers. If a supplier cannot deliver on time the whole production process is
interrupted. A good supply-chain relationship is normally managed in two main ways. The first is that
suppliers are not necessarily chosen based solely on price, but on quality and security of supply.
Secondly, suppliers face large financial penalties if they are unable to deliver the supplies precisely
when required.

CLOSING PRACTICAL EXAMPLE


Jenny has just received a large order for picture frames from a chain of photo shops, which will
necessitate her ordering a large amount of raw materials. She needs to determine when to order the
materials, how to manage the materials and how to keep her level of inventory low, as she does not
have much storage space and because of the costs involved.
She calculated her anticipated requirements for inventory and the EOQ of the various raw
materials. She then weighed these up against her available space and determined the size of her
orders of raw materials.
Finally, she determined the reorder point for the various raw materials to ensure that she orders
the materials in time so that her manufacturing process will not be interrupted.

4.5 Summary
In this chapter, we discussed the effective management and control of inventory. Although inventory
management and control are often not the responsibility of the cost or management accountant, it is
nevertheless important to understand the processes used to manage and control inventory efficiently.
This chapter discussed the inventory-management cycle, which includes
• setting norms and standards for inventory levels;
• identifying the need to order inventory;
• ordering inventory;
• storage of inventory; and
• internal and external distribution and consumption of inventory.

It also explained how to determine the EOQ  , which should ensure that orders are placed in the most
efficient way. We also discussed how to establish the correct reorder point to ensure that production
or supply is not interrupted, and excess stock is not held.
Finally, we briefly discussed certain advanced inventory-management systems – MRP, ERP and
JIT – which are used to control and manage inventory requirements, often in large corporate
manufacturing environments.

PRACTICAL ACTIVITY
Write a memo to your managing director explaining the factors to consider with regard to inventory
management in the retail environment. Your discussion should include all the steps in the inventory
management process and illustrate the factors mentioned with suitable examples.

Short questions
1. ____________ inventory is defined as the amount of inventory that needs to be held to ensure
that production can continue:
a. Strategic
b. Speculative
c. Safety
d. Retail
2. If a retail business has 50 items in the warehouse, 10 in transit from a supplier (which have
been paid for) and 15 items on consignment with another retailer, the economic inventory of
the store is:
a. 50
b. 60
c. 65
d. 75
3. Using the information given in question 2, what is the technical inventory of the retailer?
a. 50
b. 60
c. 65
d. 75
4. Briefly define normal inventory.
5. Give an example of an inventory-holding cost.

Long questions

1. Describe using an example the steps of the inventory-management cycle.

2. Briefly discuss the difference between strategic and speculative inventory. Give examples of
each.

3. A company’s demand for an item of inventory is 100 000 units per annum. The incremental
order costs are R1,50 per unit and the holding costs are R2,75 per unit. Calculate the EOQ of
this item.

4. BJH Limited is considering its per unit holding costs for a specific product, as the company
is considering increasing the average amount of inventory it holds by 2 000 units.
Management believe that their incremental warehouse costs will be R200 000 per annum due
to the larger amount of space they will require.
The incremental investment in inventory is expected to be R150 000 per annum and the
required rate of return is 15 per cent per annum. The only other incremental costs are that
they believe that they will need to pay R0,50 per extra unit of average inventory per month.
Determine the incremental holding costs per unit of inventory if the company increases its
average amount of inventory.

5. The purchasing manager wants to order 5 000 extra units above the normal order due to an
expected increase of 15 per cent in the cost of the inventory over the next three months. There
is enough space in the warehouse, but the 5 000 units are equivalent to about three months’
normal sales. The company is also experiencing cash-flow issues. The CEO believes that
this order would lead to overstocking and has asked for your advice.
Differentiate between speculative stock and overstocking, and intuitively give your opinion
on whether the extra units should be purchased or not.

6. Briefly describe a JIT production system and give an example of an industry in which it is
frequently used.

7. Advise a client who is launching a clothing store on the key activities in the inventory-
management cycle, and illustrate some of the processes and procedures that his company
should have in place to manage the process accordingly.

8. ADH Limited is a chemical wholesaler. The company is currently assessing the most
efficient number of units to order and trying to determine the reorder point for product DX1.

The following information has been supplied to you regarding DX1:


Normal annual demand 30 000 units
Order costs R3,60
Holding costs R8,90
Lead time Two weeks
Required safety stock 100 units
Determine the product DX1:
a. the EOQ.
b. an appropriate reorder point.

9. Regi Limited has just received a letter from a supplier. The supplier is offering a 10 per cent
discount on orders of 300 units or more of Regi’s major raw material. The following
information pertains to the item:
Demand per annum 20 000 units
Order costs R3,95
Holding costs R6,00
Unit cost R1 500
Determine whether or not the minimum order quantity should be increased to 300 units.

10. LT Chemicals manufactures various products from chemical raw materials. Some of the
chemicals that the company uses are hazardous and others have to be stored at temperatures
below freezing. Most of the chemicals may have serious health-and-safety concerns for
employees handling the chemicals.
Write a memorandum to the CEO of the company discussing the effect of the nature of the raw
materials on the inventory management of LT Chemicals.
5 Cost elements: Labour
Marina Bornman

LEARNING OUTCOMES
After studying this chapter, you should be able to:
• understand the principles and describe the types of remuneration received by employees;
• use labour remuneration principles to calculate the net wages of an employee;
• calculate the labour cost to company and hourly recovery rate;
• understand the role of labour cost in determining production or service costs; and
• record the accounting entries in respect of labour.

PRACTICAL EXAMPLE
Mr Xuma, an experienced welder, works on the steel security doors manufactured by Heavy Metal
Security Doors (HMSD). He works five days a week, eight hours per day and is paid a wage of
R60 per hour. The company also employs Mrs Yule, the quality inspector, who is paid a salary of
R16 000 per month.
HMSD needs to take into account the costs of these employees to calculate the cost of the doors
produced, including the following elements:
• What Mr Xuma is paid per week.
• How Mr Xuma and Mrs Yule’s remuneration would be treated when calculating the costs of
finished goods.
• How Mr Xuma and Mrs Yule’s remuneration should be accounted for in the accounting records.

Figure 5.1

5.1 Introduction
Remuneration is the compensation paid by employers to their employees in exchange for the services
they provide to the employer. These services may include physical labour, application or parting of
knowledge, or managing a process where a product is manufactured or a service is rendered. The
remuneration system allows an employer to determine the salaries and wages payable to employees.
Information from this system is also used to calculate the labour cost assigned to specific products
and/or services. In order to allocate these costs appropriately, the costs have to be included in the
accounting or cost records of the entity.
Labour, as part of a costing system, touches on various aspects that fall outside the scope of this
chapter. These include labour taxation, and health-and-safety laws. In this chapter, we focus on cost-
accounting concepts related to labour and will not discuss the other, more complex, subjects that are
normally covered by labour law and taxation.
In the opening example, we considered the questions that Heavy Metal Security Doors needs to
address to properly account for its labour costs. In order to answer these and other questions, it is
important to understand the procedures involved in a remuneration system and the terms used. This
chapter introduces important terms in respect of labour and guides you through the process of
determining and recording wages and salaries.

5.2 Labour as a cost element


All manufactured products and services rendered necessitate labour activities.

Labour can take the form of the physical activities necessary to transform raw materials into
finished products (e.g. the welding of the steel in a security door, as in the opening example) or
creative thinking skills (e.g. to design the shape and features of the door).

The cost of labour is therefore one of the elements to take into account when determining the cost of a
manufactured product or a service. The other two elements are direct materials (see Chapter 3) and
manufacturing overheads (see Chapter 6). In the case of services, where no materials are transformed,
the cost elements may consist only of labour and overheads.
The next sections briefly describe a number of important concepts when dealing with a
remuneration system and the accounting process for these costs.

5.2.1 Salaries
A salary is generally defined as the remuneration paid to employees earning a fixed amount per
month, as determined by their contract of employment.

Salaries are usually paid at the end of the month by means of a direct deposit into the employee’s
bank account or sometimes by cheque.

5.2.2 Wages
Wages are the remuneration paid to employees determined by the hours they worked during a
certain period (e.g. one week) or as a predetermined amount for completing a certain task.
For example, a painter employed to paint a room may either be paid an hourly wage or a negotiated
amount for finishing the task. Remuneration based on the completion of a task is also referred to as
‘piecework’. Although many employers pay wages through a direct transfer, some pay wages in cash
to employees, especially in the case of part-time or short-term workers.

5.2.3 Direct labour


Direct labour refers to those activities performed by employees directly involved in the
production of goods and services (e.g. workers on a production line).

Direct labour costs are those wages and salaries that can be traced to a specific product or job order,
or the provision of a service.

5.2.4 Indirect labour


Indirect labour refers to those activities performed by employees not directly involved in the
production of goods and services (e.g. janitors, maintenance workers and quality inspectors).

Indirect labour costs cannot be directly traced in an economically feasible way to a specific product
or job order, or the provision of a service.

5.2.5 Normal hours


Normal hours are the minimum working hours that an employee needs to work in a specified
period, as determined by the contract of employment for the contracted wage or salary.

5.2.6 Overtime hours


Overtime hours are the hours worked in a specific period in excess of the normal hours.

If a welder, for example, normally works 40 hours per week but then works 45 hours in a week, he
has worked five hours of overtime. Overtime is remunerated at 1,5 times (i.e. 150 per cent) the
normal rate per hour under the terms of the Basic Conditions of Employment Act. On Sundays and
public holidays, the rate is double the normal rate per hour. It should be noted that an employee may
be required to work ‘normal hours’ on a Sunday or public holiday for no additional pay if the
employment contract has this provision.
5.2.7 Overtime premium
Overtime premium is the amount in excess of the normal rate per hour paid for overtime.

As discussed, this is normally 1,5 or twice the normal rate per hour. Therefore, if an employee’s
normal rate is R20 per hour and he or she is entitled to a payment at 1,5 times the normal rate,
overtime will be remunerated at R30 per hour. The overtime premium will be the extra R10 per hour
(the difference between R30 and R20 per hour).

5.2.8 Idle time


Idle time is time spent by an employee at work but which is not productive in terms of time spent
on the production process.

Idle time may refer to time spent in meetings, during training or time that is wasted as a result of
machine breakdowns. The worker is still remunerated at the normal tariff for these hours but the entity
does not receive any direct benefit.

5.2.9 Gross remuneration


All amounts received by employees in the form of normal wages or salaries, overtime wages or
salaries and any other allowances received by the employee are added together to determine the gross
remuneration.

Gross remuneration is the total amount earned by an employee before deductions, such as
contributions to the pension fund, medical-aid fund and employee’s tax, are made.

Allowances are often included in the remuneration of employees. For example, a vehicle fringe
benefits or travel allowance is often given to salaried staff who need to use their private vehicles for
work purposes.

5.2.10 Net remuneration


Certain deductions may be made from the gross remuneration, such as contributions to a pension
fund, a medical-aid fund, union membership fees, etc. The remaining amount after these deductions
are made is the net wage or net salary (also known as net remuneration), which is the amount
payable to the employee.

5.2.11 Cost to employer


The cost to employer of an employee is the total of the gross remuneration of the employee, plus
all other costs relating to labour that the employer incurs.

These costs include, for instance, contributions to a provident or pension fund by the employer, plus
the employer’s contribution to a medical-aid scheme, if applicable, plus the employer’s portion of the
government’s Unemployment Insurance Fund (UIF), plus a pro-rata portion of any bonus payment that
will be made.

5.3 Calculating remuneration payable


On a weekly or monthly basis, the net salary or wage of an employee needs to be calculated. The
basic framework for the calculation of net weekly wages is shown in Example 5.1. A monthly salary
is calculated in much the same way but would simply start with ‘monthly salary’ (in the place of
‘weekly wage’) and ‘overtime salary’ (in the place of ‘overtime wages’).
Most employers use a payroll computer program to calculate the net wages and salaries to be paid
to employees. This program automatically, once correctly set up, calculates the various components
of the net salary. These programs are also useful, as they can produce various reports that can be used
in costing, financial accounting and paying over the various deductions. Certain payroll programs can
also be used to interface with the banking system to pay the wages electronically.
Calculating the net remuneration for a period is a complex and often specialised field that falls
outside the scope of this book as it involves knowledge of correctly applying taxation and other legal
principles. It is, however, important that the management accountant understands the basic principles
involved, as this information is used to determine the cost of production.
The following brief notes explain some of the deductions made from gross wages when
calculating the net wage:
• The contribution to a pension fund is determined according to the employee’s contract of
employment. In Example 5.1, we will use 7,5 per cent of normal remuneration for a regular week.
This is the amount allowed by the South African Revenue Service (SARS) to be deducted from
gross remuneration before the employee’s tax is calculated.
• Pay as you earn (PAYE) is the employee’s tax that the employer withholds from the employee’s
remuneration and then pays to SARS. PAYE is calculated on the taxable remuneration, namely
the amount remaining after deducting the pension fund contribution from the gross wages. For an
employee earning weekly wages, PAYE will be calculated and deducted weekly, but only paid to
SARS as a total amount for the month. The employer must pay the employee’s tax to SARS within
seven days after the end of the month in which the tax was withheld. In practice, employee’s tax is
determined using tables provided by SARS. Normally in examples and questions you can expect
to be given a tax percentage to use that would be multiplied by the taxable income.
• Premiums payable to medical-aid schemes are determined by the medical-aid funds and are based
on the number of dependants of the employee. The employment contract determines what portion
of the premium is to be paid by the employee and how much the employer will contribute. Only the
employee contribution is deducted from the employee’s gross remuneration.
• The contributions payable by employers to the UIF on a monthly basis are calculated at 2 per cent
of remuneration paid or payable to each of their employees during that month. The regulations
regarding UIF state that 1 per cent must be contributed by the employee and deducted from the
gross remuneration; the remaining 1 per cent is contributed by the employer. Employees and
employers therefore contribute in equal shares. Employees earning more than a certain amount (as
determined by the Department of Labour) have an upper threshold to their contribution.
• The Skills Development Levy (SDL) is a compulsory levy scheme for the purpose of funding
education and training. It is payable by most employers on a monthly basis and is equivalent to 1
per cent of all their workers’ pay after certain allowable deductions are made. Employees make
no contribution to this levy and therefore the levy may not be deducted from the worker’s pay.
Notice, therefore, that the SDL does not form part of the calculation in Example 5.1 but is included
when calculating the cost to company of an employee.

Example 5.1 illustrates how to calculate the weekly net wage of an employee working in a
manufacturing environment.

EXAMPLE 5.1
The time sheet of an employee of Satina (Pty) Ltd, an entity manufacturing bed linen, is provided
below.

Satina (Pty) Ltd

TIME SHEET for the week ending: 14 April 2012

EMPLOYEE: R Milan
DEPARTMENT: Production
EMPLOYEE NUMBER: 2512
SUPERVISOR: S Tolo
Day Date Time in Time out
Monday 8 April 08:00 16:30
Tuesday 9 April 07:00 17:30
Wednesday 10 April 08:00 17:00
Thursday 11 April 07:30 17:00
Friday 12 April 08:00 17:00
Saturday 13 April 08:00 13:30
Sunday 14 April 09:00 12:00

Signature: R Milan

Supervisor: S Tolo___________

There is a compulsory lunch break every day except Saturday from 12:30 to 13:00. The lunch
break is not reflected on the time sheet. A normal work week consists of 45 hours (eight hours per
day on weekdays and five hours on Saturdays). The normal wage rate is R60 per hour and
overtime is remunerated at 1,5 times the normal rate, except for Sundays and public holidays,
when it is paid at double the normal rate.
The company makes the following deductions from the employee’s wages on a weekly basis:
Pension fund 7,5% of normal wages
Medical-aid fund R120 per week
UIF 1% of gross wage
Trade union fees R12 per week

Employee’s tax (PAYE) must be deducted for this employee at 18%.

Required
Calculate the net wage payable to R. Milan for the week ending 14 April 2012. Remember to
deduct the 30 minutes for lunchtime.

Solution
Day Total hours Normal hours Overtime hours Sunday overtime
Monday 8 8 -
Tuesday 10 8 2
Wednesday 8,5 8 0,5
Thursday 9 8 1
Friday 8,5 8 0,5
Saturday 5,5 5 0,5
Sunday 3 - - 3
52,5 45 4,5 3

Wage calculation:

Normal wage (45 hours × R60 per hour) 2 700,00


Overtime wage (4,5 hours × (R60 per hour ×
405,00
1,5))
Sunday overtime (3 hours × (R60 per hour × 2)) 360,00
Gross wages 3 465,00
Less pension fund contribution (R2 700 × 7,5%) (202,50)
Taxable remuneration 3 262,50
Less other deductions:
 Employee’s tax (PAYE) (R3 262,50 × 18%) (587,25)
 Medical fund (120,00)
 UIF (R3 465 × 1%) (34,65)
 Trade union fees (12,00)
Net wage payable to employee 2 508,60
5.4 Recovery of direct labour cost
In the introduction, we asked, how does Mr Xuma’s wage contribute to the total cost of the finished
product? This question can be answered by calculating the rate at which labour is remunerated (rands
per hour) and the time (number of hours) spent on manufacturing the product. This is known as the
total labour cost for a product.
Consider the following scenario: An employee earns a wage of R1 500 per week. During a week
he manufactures 40 units of a certain product. What is the labour cost per unit manufactured? Given
the information, the direct labour cost can be calculated as follows: R1 500 ÷ 40 = R37,50 per unit.
In practice, this calculation is more complex and it is necessary to calculate a labour recovery
rate to make provision for certain factors that are not taken into account when just stating an hourly
wage rate. These factors are:
• The non-productive hours that are paid but do not contribute to actual manufacturing hours (that is,
idle time, paid leave for the year and public holidays on which workers have to be paid).
• The employer contributions to the various funds (pension, medical, UIF), which are extra labour
costs to be taken into account.

When calculating the labour recovery rate, the following formula is used:

The total labour cost is calculated per department (or cost centre) and consists of the total amount of
gross wages for the year (including bonuses) plus the employers’ contributions to the various funds.
Total productive hours for the year are calculated as the total number of hours available for work
less annual leave, sick leave, public holidays and idle time.
Example 5.2 shows how to calculate the hourly recovery rate.

EXAMPLE 5.2
Kula Manufacturers operates for 52 weeks in a year. A normal work week consists of 40 hours, 8
hours per day. Other information with regard to labour in the production department for the year
is as follows:
Paid annual leave 3 weeks
Public holidays 6 days during the year that fall on working days
Sick leave 5 days
Idle time 15% of hours available for work
Total wages R 50 000 per year
Pension fund 15% in total – ratio of 60 : 40 employee : employer.
Medical-aid fund 10% in total – ratio of 60 : 40 employee : employer
Annual bonus 1/12 of normal annual pay
Required
Calculate the labour recovery rate per hour.
Calculation of productive labour hours:
Number of weeks per year 52
Annual leave (weeks) (3)
Weeks available per year 49
Hours per week × 40
Hours available per year 1 960
Less:
(48)
public holidays: 6 × 8
Sick leave: 5 × 8 (40)
Hours at work for 1 872
Less: idle time (15% × 1 872) (280,8)
Expected productive hours: 1 591,2

Calculation of cost to employer:

Normal annual pay R50 000


Bonus (1/12 × 50 000) 4 167
Employer contributions:
Pension fund (50 000 × 15% × 0,4) 3 000
Medical aid (50 000 × 10% × 0,4) 2 000
Total cost to employer R59 167

This rate can now be used to calculate the labour cost for manufacturing a product or rendering a
service. For example, if it takes two hours to manufacture a product and the labour recovery rate is
R37,18 per hour, then the labour cost will be two hours × R37,18 per hour = R74,36 per product.
The next section provides another example of how the labour cost is determined and how this
contributes to the final cost of the product.

5.5 Labour cost as an element in determining total cost of


production or service
The role of wages and salaries in determining the production cost of a product can be best explained
by means of an example.
EXAMPLE 5.3
A company that manufactures security gates has determined that the direct material cost used to
manufacture one gate is R125,00. It takes six hours of direct labour in the welding department and
one hour in the finishing department to complete one gate. Manufacturing overheads per gate are
estimated at R40,00 and include all indirect material and indirect labour spent on manufacturing.
The hourly labour recovery rates were determined by the company to be the following:
• Welding department – R27,50 per hour
• Finishing department – R21,00 per hour

The total production cost per gate is calculated as follows:


Direct material R125,00
Direct labour
R165,00
• Welding (6 × 27,5)
R21,00
• Finishing (1 × 21)
Manufacturing overheads allocated R40,00
Total manufacturing cost per gate R351,00

The same principles can be applied to the cost of rendering a service, with the exception that there
will be no direct material costs.

5.6 Accounting entries for recording wages and salaries


As mentioned above, many organisations use computerised payroll systems to capture wage and
salary payments, which makes the administration and recording of payments less time-consuming.
There are two aspects to consider when recording wages and salaries when using an integrated
costing system:
• The gross wages earned by the employees and the contributions to funds and net wages payable to
employees.
• The recording of labour cost as an element of production cost, with the focus on the distinction
between direct and indirect labour.

5.6.1 Recording gross wages, payments to funds and payment of net


wages
When salaries or wages are earned, an expense account – ‘salaries and wages’ (sometimes split as
two accounts) – is debited with the total labour cost incurred for the period. The balancing credit
entries go to all the funds to which the contributions are owed as well as an account ‘creditors for
salaries and wages’, which represents the employees as creditors to which the net salaries and wages
are due. As soon as the payment is made, the bank is credited and the various funds (pension, medical
aid, etc.) are debited. The creditors for salaries and wages account is also debited by the actual
amount paid to the employees as net.
This is illustrated in Example 5.4.

EXAMPLE 5.4 SALARIES AND WAGES EXPENSE ACCOUNT


A company has calculated its salaries and wages for the month, and has found the following:
Gross salaries and wages R120 000
Pension fund liability R5 400
Medical-aid fund liability R8 700
UIF liability R2 400
PAYE liability R25 300
Net salaries and wages R78 200
The salaries and wages, and all liabilities were paid over during the month.

Required
Record the above transactions in the general ledger of the company.
5.6.2 Recording wages as a cost element
The gross-wages account does not distinguish between direct and indirect labour. Direct labour is an
element of production cost, whereas indirect labour is regarded as part of manufacturing overheads
and contributes to production cost as a manufacturing overhead cost.
The accounting entries for the flow of the cost will be discussed in more detail in Chapter 7, but a
short description of the process is given here. From the gross-wages account, the direct and indirect
labour amounts are transferred to the manufacturing account and manufacturing overheads account,
respectively, as shown in Figure 5.2. Any administrative or other non-production salaries and wages
will remain in the salaries and wages expense account or transferred to administrative or other
accounts depending on the accounting policy of the entity.

Figure 5.2 Recording labout cost as manufacturing cost element

Gross salaries and wages (or labour control)


Sundry wages xxx Manufacturing account xx (direct labour)
Manufacturing overheads account xx (indirect labour)

CLOSING PRACTICAL EXAMPLE


Heavy Metal Security Doors (HMSD) is a manufacturer of security doors. Mr Xuma, an
experienced welder, does the welding of the steel security doors. He works five days a week,
eight hours per day and he is paid a wage of R60 per hour. The company also employs Mrs Yule,
the quality inspector, who is paid a salary of R16 000 per month.
HMSD needs to consider the costs of these employees and have calculated that Mr Xuma is
paid R2 400 per week before deductions (R60 × 8 hours × 5 days = R2 400).
Mr Xuma’s salary will be a direct labour cost while Mrs Yule’s salary will be an indirect
labour cost.

5.7 Summary
This chapter introduced some terms and procedures used in the calculation of labour costs. It was
illustrated how hours worked by an employee and the rate at which these hours are remunerated
determine the gross wage earned by the employee. Deductions from the gross wage, such as
employee’s tax and contributions to funds, result in the final net wage payable to the employee.
Labour cost is one of the elements that make up the total cost of a product or service. By
calculating the hourly recovery rate, it is possible to determine the labour cost of a product or service
by applying this rate to the number of hours spent on manufacturing the product or rendering the
service.
Accounting entries for the transactions incurred in respect of labour are an integral part of a cost
system, and this chapter provided a brief overview of the general ledger entries for wage and salary
transactions.
PRACTICAL ACTIVITY
Goliath Builders CC often makes use of temporary workers for a two- to three-day period on
building projects. Explain the difference between paying these workers an hourly rate and a
piecework rate. You may use the internet as a source (referencing appropriately). Explain the
advantages and disadvantages of each method for the employer.

Short questions
1. Mahem CC produces garden pots from cement on order from garden nurseries. The company
has four full-time employees; it also employs some part-time employees at times when large
orders are placed. The employees’ job titles and job descriptions are provided in the table.
Classify the labour tasks listed in the table below as direct or indirect labour for Mahem CC.
Direct/indirect
Job title Job description
labour
Driver Delivers completed pots to nurseries.
Responsible for purchases of material, bookkeeping and
Accounting officer
sending accounts to customers.
Production Responsible for supervising the mixing and moulding of
supervisor cement and doing quality inspections.
Full-time cement Responsible for preparing the cement mix and pouring
mixer into the moulds.
• Responsible for preparing cement mixes and pouring
Part-time into moulds.
labourers • Responsible for glazing pots once removed from the
moulds.

2. Peter Big runs a small accounting firm from home. He employs a secretary and an accounting
clerk. The accounting clerk records the daily transactions of five of Peter’s clients using an
accounting software program. The clerk prepares a weekly time sheet keeping record of the
number of hours spent on each client. For the week ending 12 April 2013, the clerk submits
the time sheet below to Peter:
Client No. of hours Rate per hour Amount claimed
ABC (Pty) Ltd 12 R75
STR CC 14 R72
JKL (Pty) Ltd 8 R65
Ortis Brothers 6 R65
Dr Killian & Partners 4 R70
a. The total amount claimed for the week ending 12 April 2013 for services performed by
the clerk is:
i) R2 800
ii) R900
iii) R12 386
iv) R3 098
b. Are the services performed by the clerk regarded as direct or indirect labour for Peter’s
accounting firm?
c. What type of deductions should Peter make on the clerk’s wages before paying out his
wage for the week?
3. Explain the following terms:
a. Gross wages
b. Net wages
c. PAYE
d. UIF

Long questions
Note: these questions lend themselves to the use of Excel spreadsheets to determine the solutions;
students are encouraged to do the questions both manually as well as with a spreadsheet.

1. GHL Manufacturers submits the following information on its employees’ remuneration per
week:
Housing
Normal hourly
Name Position Normal hours allowance per
rate
week
Production-line
Jabu R40 40 R20
worker
Ray Machine operator R60 40 R50
David Supervisor R85 40 R100
Simon Security R45 40 R22
Notes
1. All the employees work five days per week as follows: Monday–Friday, eight hours per day.
2. Overtime remuneration on weekdays = normal rate plus 50%.
3. Hours worked on Sundays and public holidays are remunerated at double the normal rate.
4. Pension fund contribution: 7,5% of normal wage.
5. Medical fund contribution: R50 per week.
6. PAYE payable to SARS: 20% of taxable income.
7. UIF is deducted at 1% of normal wage.
8. Fees payable to the union, to which all the workers belong: R10 per employee per week.
The following relates to the machine operator for the week ending 15 November 2012: Actual
hours worked: Monday Tuesday Wednesday Thursday Friday Saturday Sunday 6 10 8 8 9 5 3
Monday Tuesday Wednesday Thursday Friday Saturday Sunday
6 10 8 8 9 5 3
Determine the net wage payable to the machine operator for the week ending 15 November 2012.
2. Mrs Boyle has employment details for the year ended 31 December 2012 as follows:
Staff number 4218256
Position held Administrative assistant
Department Marketing
Normal hours per week 40 hours
Normal wage rate per hour R45
Contribution to pension fund (% of normal
5%
wage)
Contribution to UIF (% of normal wage) 1%
Weekly contribution to medical aid R100
The company pays time and a half for the normal overtime, and double time for Sundays and
public holidays. The company’s contribution to Mrs Boyle’s pension fund is 7,5 per cent, to UIF
1 per cent and an additional R150 per week to medical aid. The average PAYE rate is 15 per
cent.
Mrs Boyle worked 50 hours during the week ending 25 October 2012. There were no public
holidays during this week. Her time sheet revealed the following:
Day Number of hours
Monday 8
Tuesday 10
Wednesday 8
Thursday 8
Friday 8
Saturday 4
Sunday 4
a. Calculate Mrs Boyle’s net wage payable for the week ending 25 October 2012.
b. Assume that Mrs Boyle’s annual normal wage is R121 680 and she is entitled to two
weeks’ paid leave per annum. There are nine public holidays that fall within the work
week during the year. The policy of the company stipulates that all workers are
remunerated in full should a work day be a public holiday. Calculate the hourly labour
recovery rate.

3. Bowie Manufacturers’ normal work week is Monday to Friday, eight hours per day.
Saturdays are regarded as normal overtime, but Sundays and public holidays are regarded as
special overtime.
The company’s salary structure is as follows:
Normal time Normal overtime Special overtime PAYE
Post level
(R per hour) (R per hour) (R per hour) (%)
1 R22 R35 R45 18
2 R28 R45 R60 20
3 R36 R60 R75 22
All employees must belong to the pension fund and contribute 7,5 per cent of their weekly wage
to the fund. PAYE is deducted as shown in the salary structure and the UIF contribution is
deducted at the standard rate of 1 per cent.
The production manager of Bowie Manufacturers sent you, the salary clerk, the following
email:
To: Salaries
CC: T Miles
Re: Correction of wages payable
Attention – Salaries Dept
Employee T Miles (employee no: 420772) has queried his net wages paid for the week of 20–26
February.
Mr Miles is on post level 2.
Please recalculate based on the time sheet below and reply showing the detailed calculation of
his net wages. Mr Miles also requests that you explain to him what the deduction PAYE and UIF
is for and why it is deducted from his wages.

Thank you

TIME SHEET
EMPLOYEE: T Miles 420772
WEEK ENDING: 26 Feb 2012
DAY TIME IN TIME OUT TIME IN TIME OUT
Monday 8:00 12:30 13:30 18:00
Tuesday 7:30 12:30 13:00 17:30
Wednesday 8:00 12:15 12:45 17:30
Thursday 8:30 - - 16:30
Friday 8:00 12:15 13:15 18:00
Saturday 8:00 12:00
Sunday 9:00 12:00

You are requested to:


a. Reply to the email, showing the calculation of Mr Miles’s net wage and answer the
questions asked by Mr Xaba.
b. Record the accounting entries for the wages and the contributions to the funds.

4. A company manufacturing garden pots uses two production processes. In process 1, all the
raw materials are combined and moulded into a pot; in process 2 the pots are decorated and
baked.
It has been established that in process 1 it will take an employee two hours to
manufacture one batch of pots, and the labour allocation rate in this department is R44 per
hour.
In process 2 it will take an employee three hours to decorate the batch of pots. The
following information is available for process 2:
Gross annual wage cost of the manufacturing
R286 000
department
Employer’s contribution to medical aid 5% of gross wage
Employer’s contribution to pension fund 7% of gross wage
Weeks per annum 52 weeks
Paid leave per annum 4 weeks
Public holidays that fall on working days 6 days
Working week (5 days x 9 hours/day) 45 hours
Idle time 15%
The company asks for your assistance in determining the following:
a. The labour recovery rate for process 2
b. The total labour costs per batch of pots

5. Jumble Stationers print personalised invoice books for small businesses. On a typical day,
12 invoice books are prepared and printed. The details of the production cost incurred per
day are as follows:
Raw material used R240
Direct wages paid ??
Indirect materials used R30
Indirect wages paid R54
Two people are directly involved with the typesetting and printing of the books. Mr A (the
typesetter) works on average eight hours per day, earning R25 per hour. Mr B, the print
machine operator, works on average six hours per day and earns R15 per hour.
You are required to calculate the direct wages paid on a typical day. Thereafter calculate
the total production cost incurred per day and then determine the production cost of one
invoice book.

6. Jona Philander’s time sheet for the week ending 15 August 2012 indicates a total of 49
working hours. Normal working hours are from Monday to Friday, 8:00 to 17:00, with a one-
hour lunch break. Jona’s time sheet showed that he also worked five hours on Saturday and
three hours on Sunday that week.
Normal remuneration is R40 per hour and overtime during the normal work week is paid
at time and a half. Hours worked on Saturdays and Sundays are remunerated at double the
normal wage.
Employees contribute 20 per cent of the normal wage to the pension fund. Of this, 10 per
cent is paid by the employer. Pension fund contribution is only deductible for PAYE
purposes to a maximum of 7,5 per cent of the normal wage.
Medical contributions amount to 10 per cent of the normal wages. This is paid by the
employer for all employees. The normal UIF contributions are made weekly.
Calculate the net wage payable to Jona Philander for the week ending 15 August 2012.

7. Multi-Bricks CC has provided you with the following information on employment conditions:
No. of weeks in operation 52
No. of weeks in operation 52
No. of weeks’ holiday leave 3
No. of public holidays on working days 8
Work hours per day (6 days per week) 8
Idle time is estimated at 5 per cent of available working hours
All employees are paid a Christmas bonus in December equal to 75 per cent of their
monthly gross wage. Performance bonuses to the value of R10 000 are paid on merit only;
bonuses are paid to deserving employees after a performance review annually in July.
Paula Davis and Jimmy Schulz are the only two employees working in the finishing
department, where they are responsible for the quality control of the finished bricks. The
finishing department is regarded as a cost centre, and management would like to determine
the hourly recovery tariff of this cost centre.
Paula’s monthly salary is R6 000 and Jimmy, being the senior controller, has a monthly
salary of R7 500. With the performance review during June, Paula scored a rating of 4 and
qualified for the performance bonus. Jimmy only scored a rating of 3 and did not qualify for
the performance bonus.
The employer contributes to the employees’ pension fund. For Paula this contribution was
R6 500 for the year, and for Jimmy R8 000.
Calculate the hourly recovery rate for the finishing department of Multi-Bricks CC.
6 Cost elements: Overheads
Jean Struwig and Johan Hefer

LEARNING OUTCOMES
After studying this chapter, you should be able to:
• discuss the types of overhead costs businesses will incur;
• discuss the typical cost behaviours of overhead costs;
• explain the reasons for allocating overhead costs to products or services, and the factors to
consider when doing so;
• describe and implement the process of allocating overhead costs to products and services,
including:
– the principles of allocating costs to all cost centres based on appropriate bases of allocation;
– the principles of allocating service department costs to production departments by using the
repeated distribution, simultaneous equation, specified order, direct allocation or
mathematical models;
– the principles for computing the appropriate overhead rates for each production cost centre;
– the principles for allocating overhead costs to production; and
• calculating the over- or under-absorption of overheads for a specified period.

PRACTICAL EXAMPLE
Lisa was looking at her grandfather’s factory production records and so-called management
accounts. He had decided that he wanted to retire at the end of the year, and Lisa would take over
the factory after she finished her B.Comm degree. Lisa analysed the activities of the company from
a financial point of view.
Lisa’s grandfather owned and managed Kenton Manufacturers. Lisa had to give her grandfather
credit – his production records were meticulous when it came to the direct labour and direct
material costs. However, he never even considered allocating overhead costs to production. When
she raised the question, he simply said: ‘The factory building costs have nothing to do with what
we produce and therefore how can I add these costs to the items I produce?’
The worst part was that he determined the selling prices of his products based only on the
material and labour costs. Lisa examined the financial statements for the previous financial year
prepared by the accountant and found the following:
2012

R
Sales 780 000
Cost of goods sold (450 000)
Gross profit 330 000
Factory costs costs
Administrative (250
(150 000)
000)

Net loss (70 000)

From the financial statement, she could clearly see that factory overheads were not being
appropriately considered when costing products and this was having a detrimental effect on the
profitability of the business. She could see that she would have to improve the costing system to
ensure that products were sold at sustainable prices.

6.1 Introduction
One of the main aims of cost and management accounting is to be able to cost specific products,
projects or services. This enables a manager to ensure that these products, projects or services are
profitable to the business. The main cost elements of any product are the direct material costs, the
direct labour cost and overhead costs. The main cost elements of a service are normally the labour
and overhead costs. Direct material and labour costs have been discussed in Chapters 3–5. In this
chapter we will discuss overhead costs.

Overhead costs are all costs incurred in the production process that cannot be traced
economically directly to specific products, projects or services. Examples of overhead costs are
rent, municipal charges, administrative staff’s salaries, and depreciation of equipment and
machinery.

It is important to understand the implication of overheads. The overhead costs of a business may be
substantial so the incorrect calculation or treatment of overhead costs will have a negative impact on
the profitability of the business, as sales prices are often calculated based on cost. The process of
allocating overheads to products and services can be very difficult, as the method used to allocate
costs can be arbitrary. If the method of allocation is not carried out accurately, it could lead to the
abandonment of profitable products if too great an overhead cost is allocated to a product line, or,
conversely, the continuation of an unprofitable product line if insufficient overheads are allocated to
it.
As discussed, overheads are costs that cannot be allocated to a specific product line in an
economically feasible way. Therefore, we allocate overheads to cost objects using an appropriate
method. What is ‘economically feasible’ has to be decided upon by the cost or management
accountant on a case-by-case basis. The direct material and direct labour costs are easily identified
and calculated. But it is not easy to allocate overhead costs accurately until the end of the financial
period. However, it is normally impractical for a business to wait until the end of the financial year to
calculate the cost of products manufactured, as this information is needed to determine selling prices
and for management purposes. Therefore, businesses allocate manufacturing overhead costs to
production based on their predetermined budgeted overhead rate, a term that we will explore later in
this chapter.
The difference between financial accounting and cost accounting should also be considered. In
financial accounting, only direct manufacturing overheads can be included in the cost of products.
When management or cost accounting is used in an organisation for internal reporting, however, non-
manufacturing overheads, like selling expenses, may also be included in the cost of inventory.
It should be remembered that overheads do not only occur in manufacturing contexts, although this
chapter will mostly focus on the manufacturing environment. Overheads and the allocation of
overheads also apply to service organisations, like accounting firms, legal practices and dry-
cleaners. It is just as important to consider the impact of overheads on costs for these types of
businesses as it is in the manufacturing context. Generally, the same principles and procedures would
inform the treatment of overheads in service organisations.

6.2 Types of overheads


Overheads consist of all costs incurred by organisation, except direct material costs and direct labour
costs. Two main types of overheads are usually treated separately, these are manufacturing overheads
and non-manufacturing overheads.

Manufacturing overheads are all overheads incurred in the process of manufacture; non-
manufacturing overheads are all other costs not incurred in the process of manufacture.

Manufacturing overheads include:


• infrastructure overheads – depreciation of machinery; rental of machinery or factory premises;
repairs and maintenance of equipment; electricity and water costs, etc.;
• indirect materials – materials that are used in the production process but cannot be allocated to
specific products, like oil used to lubricate a machine; and
• indirect labour – employee costs of those employees not directly involved with production.

Non-manufacturing overheads include:


• selling costs;
• distribution costs; and
• administrative costs.

It is important at this point to again differentiate between the financial-accounting and management-
accounting treatment of inventories. The financial-accounting treatment was more fully discussed in
Chapter 3 but to recap, the cost (or value) of inventory should include the cost of conversion and
other costs (IAS 2(10)). These costs should be allocated to production in a systematic way. The cost
of conversion would include manufacturing overhead charges and the other costs would include costs
‘… to the extent that they are incurred in bringing inventories to their present location and condition’
(IAS 2(15)). Generally, therefore, non-manufacturing overheads will not be included in the cost of
inventories for financial-accounting purposes. These costs may, however, be included for
management-accounting purposes if management deems it necessary. Management should, however,
not forget that the management accounts and the financial accounts will reflect different values if
different basis are used to value inventory.
We will now briefly discuss the different categories of overheads.
6.2.1 Infrastructure overheads
Infrastructure overheads are all costs incurred in running and maintaining the factory premises
and equipment.

These costs include various items and relate mainly to the business’s property, the production
equipment and the support functions/departments.
The main expenses that are incurred on a property are rental (if the property is not owned),
municipal charges, insurance charges and depreciation on buildings. These costs could form a
significant portion of the expenses of a business. These costs also tend to be constant and do not vary
with production levels. (Fixed and variable overhead costs will be discussed in the next section.)
Equipment-related costs include repairs and maintenance, and the depreciation, rental and
insurance of equipment. These costs are also often reasonably constant and not based on production
levels, although repairs and maintenance will normally increase when production increases.
Support functions include stores, information technology, maintenance departments and the canteen
for use by employees. Support departments generally support multiple production departments and
service departments. Support costs vary from highly fixed to highly variable, depending on the type of
support service offered. As an example, the stores expenses would tend to remain quite stable
independently of production, whereas the maintenance department costs may be more variable.

6.2.2 Indirect materials


Indirect materials are materials that are used in the production process of a product but cannot be
identified with a specific product because the materials are used for all the products and not for
one specific product.

In certain situations, the costs of the material used are so small that it is not worth the effort to
allocate the cost to a specific product. Examples in a furniture factory may include items such as glue,
nails, sandpaper and varnish.

6.2.3 Indirect labour


Indirect labour is labour costs that cannot be specifically traced to or identified with a specific
product.

The wages and salaries of employees involved in the manufacturing process, but who do not directly
work producing the specific products, are classified as indirect labour. Examples of indirect labour
may include the employment costs of a factory supervisor, stores employees, maintenance workers
and cleaners.
6.2.4 Non-manufacturing overheads
As discussed earlier, non-manufacturing overheads are not normally allocated to production costs for
financial-accounting purposes but may be allocated to production costs for management purposes.
Non-manufacturing overheads tend to be treated as period costs (in other words, included directly in
the income statement of a business) and may include items such as:
• selling costs (commission, sales staff salaries);
• marketing costs (advertising, sponsorships and marketing staff salaries);
• distribution costs (delivery charges, vehicle costs, drivers’ salaries); and
• administrative costs (audit fees, bank charges, administrative salaries).

These costs may be allocated to production costs for the purposes of cost and management
accounting. If this is the case, it should, however, be done with a certain amount of circumspection. It
should also be stressed that the line between manufacturing overheads and non-manufacturing
overheads is often arbitrary and situation-dependent.
As an example, the human resources (HR) department could be treated as a non-manufacturing
cost. If a portion of the HR staff’s role involves factory workers’ affairs, however, a case could be
made to include those costs in manufacturing overheads. The decision whether to include or exclude
specific costs in manufacturing overheads should be done on a case-by-case basis and based on facts.
Again, remember that most non-manufacturing overheads may not be included in the cost of
inventories for financial-accounting purposes.

6.3 Cost behaviour of overheads


Different overhead costs may have different behaviours based on manufacturing volumes, as
mentioned in Chapter 2. Certain costs, like factory rent, are normally fixed. Therefore, whether 1 or 1
000 items are manufactured, the rent will remain constant. Other costs can be totally variable. As an
example, let us assume a machine needs to be serviced after every 100 hours of production. The
service costs of this machine will therefore be directly related to the amount of production.
We can generally identify four distinct cost behaviours of overheads:
• Fixed manufacturing overheads
• Variable manufacturing overheads
• Semi-variable overheads
• Step-fixed overheads

It is important for a business to analyse each of its overhead costs to establish their behaviour, as this
information is important for budgeting purposes. These behaviour types are discussed in the
following sections.

6.3.1 Fixed manufacturing overheads


Fixed manufacturing overheads are fixed within a specific range for a specific period and are not
linked to the number of units manufactured during that period.
Examples of fixed manufacturing overheads are factory rent, building insurance, depreciation and the
salary of a production manager. As an example, the factory may have the capacity for the production
of 5 000 units. Let us assume rent is charged at a fixed rate per month: the rent will be fixed for the
period, whether the company produces 100 units or 5 000 units.

6.3.2 Variable manufacturing overheads


Variable manufacturing overheads vary directly in relation to the number of units produced. If
the number of units manufactured increases or decreases, the cost per unit stays constant.

Indirect materials tend to be variable manufacturing overheads, as the amount of materials used in the
manufacturing process would be based on the amount of production.

6.3.3 Semi-variable overheads


Semi-variable overheads are overhead costs that have both a variable and a fixed component. The
costs can be divided into their fixed and variable components.

Examples of semi-variable manufacturing costs may be the electricity, water and maintenance of the
factory. These costs typically have a fixed component, such as an electricity connection fee and a
variable component, such as the variable metered electricity usage.

6.3.4 Step-fixed overheads


Step-fixed overheads are overheads that remain constant within a specific range of production but
increase to a new constant value if the number of units produced increases to a specific range.

Indirect labour often falls within this category of overheads. Let us assume a company needs 10
workers if production is less than 1 000 units per month. As soon as the level of production is higher
than this, the company needs to employ one more employee for every 100 units produced. Therefore,
if the company were to produce 1 500 units in a particular month, it would require 15 workers. Once
a worker is employed, it is often difficult to retrench that worker if production subsequently reduces.
The cost of workers will therefore normally move in a stepped fashion.
These four behaviours of overhead costs are shown in Figure 6.1. Each business should evaluate
its costs according to its own requirements because how it treats a specific cost may be different from
another business. For example, the cost of electricity may be a fixed item if most of the electricity is
used for lighting in a warehouse, as it would be remain constant regardless of the level of units
processed. However electricity usage may vary directly with the level of units manufactured if most
of a business’s electricity is consumed by production machinery.
In the next section, we briefly discuss the cost behaviour of non-manufacturing overheads.
Figure 6.1 Cost behaviour of four types of overhead costs

6.3.5 Cost behaviour of non-manufacturing overheads


In the same way as manufacturing overheads, the different categories of non-manufacturing overheads
also exhibit different behaviours. We will now briefly give some examples of each of the cost
behaviours.
Many non-manufacturing overheads are fixed, as they are often not significantly influenced by the
level of production, like administrative salaries. Non-manufacturing overheads, like selling expenses,
may be variable – for example, the commission paid to agents may vary with the level of production
(and therefore sales).
An example of a semi-variable non-manufacturing overhead is sales personnel labour costs, as
they often have a fixed salary plus a commission based on their sales performance. An example of a
step-fixed overhead is the salaries paid to administrative personnel. An increase in production will
lead over time to an increase in the administrative burden, and extra administrative staff may need to
be appointed.

6.4 Allocating overhead costs


In this section, we discuss the factors to consider when a business decides how to allocate overhead
costs to various cost centres.

6.4.1 Cost vs benefit of cost system


The first question that should be considered if costs need to be allocated to cost centres is how
sophisticated the costing system should be. This can be a difficult question, as costing systems can
range from simplistic to highly sophisticated. In simple systems, overhead costs are treated as period
costs or allocated arbitrarily and not allocated to specific products or services in a systematic way.
The advantage of a simple system is that it is inexpensive to operate; the disadvantage is that it does
not give accurate management information.
Highly sophisticated systems allocate costs using complex methods, often using cause-and-effect
cost allocations. The characteristics of a highly sophisticated system are that they are expensive to
operate, but provide very accurate management information.
Each business is unique and in each situation management need to weigh up the cost of the
allocation system against the benefits of the increased accuracy of information provided by it.

6.4.2 Timelines of a costing system


Another factor that should be kept in mind is that costs need to be allocated on a continuous basis
during the year, even though information may not always be available. This means that allocations
through the year are often based on historical and budgeted information, and not the actual overhead
costs incurred. At the end of the year, there would normally be an over- or under-absorption of the
overheads because the budgeted overheads will seldom be exactly the same as the actual overheads.
The allocation process, therefore, has the following approximate timeline, which will be
discussed in more detail in the next section.

Before the start of the financial year:


• Step 1: Assign all the budgeted manufacturing overheads to the production and service cost
centres.
• Step 2: Reallocate all the budgeted service cost-centre costs to production cost centres.
• Step 3: Calculate separate budgeted overhead rates for each production cost centre.

During the financial year:


• Step 1: Allocate manufacturing overheads to cost of production based on the predetermined
budgeted factory overhead rate.
• Step 2: Allocate all actual manufacturing overheads to the ledger account.

At the end of the year:


• Calculate the under- or over-applied overheads and allocate them to the cost of sales.

6.4.3 Cost centres


A cost centre is a production or service unit, or department, whose costs can be allocated to that
department.

The calculation of the overheads that should be allocated to production cost centres is a very
important part of any costing system. As discussed previously, it should be remembered that these
allocations are generally made on the basis of budgeted amounts. These budgets will normally be
based on the previous year’s actual expenses. It is important that the behaviour of the costs (fixed vs.
variable) is taken into consideration when budgeting to ensure that the impact of any changes in
production levels is correctly adjusted in the overhead budget.
The business should identify its different cost centres. In a manufacturing environment, they may be
both production and service cost centres. Production cost centres could include assembly lines,
workshops or specialist manufacturing departments. Service cost centres include maintenance
departments, stores and supervisory departments.
It is important to consider the basis of allocation. The allocation is normally made based on the
activity levels or physical attributes of a cost centre. These attributes are often termed cost drivers.

Cost drivers are the activity level or physical attribute of a cost centre or department that best
reflects its cost characteristics.

These activity levels could include direct material costs, direct labour costs, direct labour hours,
machine hours and the number of units manufactured. Physical attributes of a cost centre could include
the floor area, number of employees and value of assets in use.
Overhead costs will normally be allocated on an individual basis. For example, factory rental may
be allocated based on floor area, whereas electricity costs may be allocated based on machine hours.
It is important that management find the appropriate basis to allocate the various costs.

6.5 Process of allocating overhead costs to products and services


The process of allocating overhead costs to specific products, services or projects is an important
one. Care should be taken to ensure accuracy and fairness. The process may differ slightly from one
business to another depending on how the businesses are structured. In this section, we will deal with
examples in a production- or project-based environment, but it should be remembered that these
principles remain the same in a service environment.
In general, there are five basic steps for allocating costs to products or services:
• Step 1: All manufacturing overheads should be allocated to all cost centres, both production and
service cost centres.
• Step 2: All service cost centre costs should be allocated to production cost centres.
• Step 3: Based on the costs of each cost centre, the appropriate overhead rate should be calculated.
• Step 4: Based on the overhead rates, costs should be assigned to products or services.
• Step 5: At the end of the period, compare actual overheads to allocated overheads and determine
any over or under recovery.
This process is illustrated in Figure 6.2.

Figure 6.2 Process for allocating overhead costs


In the example shown in Figure 6.2, the business’s overhead costs are allocated first to all
departments, both the two production departments and the service department. Then the service
department’s costs are allocated to the two production departments. The third step is calculating an
appropriate overhead rate for the two production departments; and, finally, the overheads are
allocated to specific product lines. It is important to note that a product may only carry overhead
charges from one department (as in Products 1 and 3 in Figure 6.2). It can only carry overhead
charges from more than one production department if both departments spend time in developing that
product (as in Product 2).
In this chapter, we will only consider the process of determining the allocation. The accounting
entries will be discussed in Chapter 7. We will now look at these five steps in more detail in the
following sections.

6.5.1 Step 1: Allocation of overheads to all cost centres


In this step all overheads are allocated to cost centres. This is termed the primary apportionment, and
overheads are allocated to both production and service departments.
The first decision that should be made is on what basis the overheads will be allocated to cost
centres. To reach this decision, management should consider the most appropriate cost driver of a
cost centre and the nature of the overhead. If, for example, one needs to appropriately allocate the
factory rental, this will normally be directly linked to the floor space each cost centre uses. If, for
example, the monthly rental of the factory is R100 000 and the factory has three cost centres that use
60 per cent, 30 per cent and 10 per cent of the floor space, the most appropriate allocation of the
factory rental would be R60 000, R30 000 and R10 000, respectively.
A summary of possible overhead costs incurred by a business and the appropriate basis of
allocation are given in Table 6.1. Note that this list is not exhaustive and the basis of allocation
indicated may not always be appropriate. Many of these costs could be allocated on different bases.

Table 6.1 Summary of overhead costs and basis of allocation


Cost Basis of allocation
Depreciation, insurance on assets, maintenance of
Value of items of plant and equipment
assets
Indirect labour and supervisory/management staff, Number of employees or employee costs
protective clothing
Insurance of property, rental of property, property
Area
maintenance costs, property taxes, security
Electricity Kilowatts used; machine hours
We will now illustrate these principles by means of Example 6.1.

EXAMPLE 6.1
Lisa has analysed her grandfather’s business and believes that she is now in a position to start
allocating overhead costs to production. Kenton Manufacturers have four departments. Two are
production departments; the other two are a stores department and a maintenance department.
Service Service
Basis of Production Production
department 1: department 2:
allocation department 1 department 2
Stores Maintenance
Number of
20 15 7 4
employees
Direct labour
3 900 1 700 N/A N/A
hours
Floor space 120 m2 170 m2 30 m2 15 m2
Machinery value R218 400 R111 000 R20 400 R 35 000
Material
R134 500 R69 900 N/A R11 600
requisitions
Machine hours 1 900 2 000 N/A N/A
The following budgeted costs were extracted from the operational budget for the next financial
year:
Cost R
Depreciation of machinery 61 000
Electricity and water 35 600
Factory rent 75 000
Insurance on building 12 400
Maintenance 57 600
Municipal rates 35 800
Supervisory staff 220 000
Indirect labour 53 500
Indirect material 14 900
Lisa has determined the following bases for allocating overhead expenses:
• Electricity and water expenses are allocated according to machine hours.
• Maintenance and depreciation are allocated according to machinery value.
• Municipal rates, factory insurance and factory rent are allocated according to floor space.
• Indirect materials and indirect labour are equally divided between the two production
departments.
• Supervisory staff costs are allocated based on the number of employees in each department.

Required
Allocate the budgeted overheads to the four departments.

Suggested solution
Serv. dept: Serv. dept:
Cost Basis Calculation Prod. dept 1 Prod. dept 2
Stores Maint.
Depreciation Value of
34 622 17 596 3 234 5 548
of machinery machinery
Electricity and Machine
17 344 18 256 0 0
water hours
Factory rent Floor space 26 866 38 060 6 716 3 358
Insurance on
Floor space 4 442 6 293 1 110 555
buildings
Value of
Maintenance 32 692 16 615 3 054 5 239
machinery
Municipal
Floor space 12 824 18 167 3 206 1 603
rates
Supervisory No. of
95 652 71 739 33 479 19 130
staff employees
53 500 ×
Indirect labour 50:50 26 750 26 750 0 0
50%
Indirect 14 900 ×
50:50 7 450 7 450 0 0
materials 50%
Total primary
258 642 220 926 50 799 35 433
allocation
Note: 1 ‘D’ refers to the department’s portion of the total of the basis.

6.5.2 Step 2: Allocation of service department costs to production


departments
In Step 1, the total overhead costs were allocated to the business’s various production and service
cost centres. Remember that the goal of the process is to allocate overhead costs to the various
products or projects of the business. Service department costs should also be borne by production
departments, as they form part of the costs incurred in the production process. This is also called
secondary apportionment. In this section, we will consider how this allocation should be done.
The process of allocating service department costs to production departments can be quite simple,
and it is especially simple if a service department only serves production departments. In this case,
the basis of allocation would be chosen that is similar to the primary allocation and the costs thus
distributed. Examples are given in Table 6.2. Again note that this list is not exhaustive, and
allocations should be made according to the specific circumstances of the business.

Table 6.2 Examples of allocating service department costs


Department Basis of apportionment
• Maintenance hours spent
Maintenance • Machine hours
• Value of machinery
• Number of requisitions
Stores • Value of requisitions
• Number of units issued
• Number of employees
Human resources
• Employee costs
• Number of units produced
• Number of employees
Quality control
• Value of units produced
• Number of inspections performed
• Support hours spent
Information technology • Number of consoles (or computers)
• Number of service calls

As a simple example, let us assume a factory has two production departments and one stores
department. The stores department’s costs are allocated based on the number of requisitions. Let us
assume that Department A made 250 requisitions and Department B 190 requisitions during the
previous year. Therefore, 57 per cent of the store’s costs will be borne by Department A and 43 per
cent of the costs by Department B.
This principle is seemingly very simple but can become complex. This complexity arises if
service departments also perform services for other service departments. This may not be a serious
problem if there are only one or two service departments but in a large manufacturing plant, such as a
vehicle manufacturer, the allocation can be a highly complex problem. There are five ways to deal
with this problem, the first two being the most commonly used:
• Repeated distribution: In this method, service department costs are allocated repeatedly in
specified percentages until the remaining amount is too small to concern oneself about.
• Simultaneous equation: With this method, an algebraic equation is formulated to solve the amount
of overheads that should be allocated to each production cost centre.
• Specified order of closing: In this method, management determines in which order service
department costs are closed off to the production cost centres.
• Direct allocation method: In this method, inter-service department allocations are ignored and all
service department costs are directly allocated to production departments.
• Mathematical models: This is used in very complex manufacturing departments and computers are
used to calculate the costs that should be allocated to production departments.
The principle is normally to apportion the service department with the highest overhead costs first, no
matter which method is followed. The first four of these models are illustrated in Example 6.2.

EXAMPLE 6.2
Limpopo Manufacturing has four departments – two service departments and two production
departments. The following information has been provided:
% to be allocated % to be allocated
Department from service from service Primary allocation
department A department B
Production department
50% 40% R250 000
A
Production department
30% 30% R220 000
B
Service department A 30% R40 000
Service department B 20% R60 000

Required
Calculate the amount to be allocated to production departments A and B from the service
departments using the following methods:
• Repeated distribution
• Simultaneous equation
• Specified order of closing, with service department A being closed off first
• Direct allocation method

Suggested solutions
1. Repeated distribution
With this method, the service department’s costs are allocated to each other until the remaining
costs are immaterial. In this example, service department B’s costs would be allocated to
production departments A and B, and service department A as follows:
Prod. Dept A Prod. Dept B Serv. Dept A Serv. Dept B
Primary allocation 250 000 220 000 40 000 60 000
Allocation of 24 000 18 000 18 000
(60 000)
Serv. Dept B (40%) (30%) (30%)
Subtotal 58 000 0
The total costs of service department A are R58 000. This will now be allocated to the
production departments and service department B. This will have the effect of allocating costs
again to service department A, which would then be distributed until an immaterial amount is left
that would be allocated between the two production departments. This process can be seen
below:
Allocation of Prod. Dept A Prod. Dept B Serv. Dept A Serv. Dept B
Serv. 29 000 17 400 11 600
(58 000)
Dept A (50%) (30%) (20%)

Allocation of
4 640 3 480 3 480
Serv. (11 600)
(40%) (30%) (30%)
Dept B
Allocation of
1 740 1 044 696
Serv. (3 480)
(50%) (30%) (20%)
Dept A
Allocation of
278 209 209
Serv. (696)
(40%) (30%) (30%)
Dept B
Allocation of
105 62 42
Serv. (209)
(50%) (30%) (20%)
Dept A
Allocation of
17 13 12
Serv. (42)
(40%) (30%) (30%)
Dept B
Final allocation of
8 4
Serv. (12)
(50%/80%) (30%/80%)
Dept A
Total overheads 309 788 260 212 0 0

2. Simultaneous equation
Assume that:
A = total overheads of service department A
B = total overheads of service department B
The total overheads that will be transferred from service departments A and B are therefore:
A = 40 000 + (30% × B)
B = 60 000 + (20% × A)
If we rearrange the above equations, it will give us:
A – 0,3B = 40 000 (1)
–0,2A + B = 60 000 (2)
We need to have either A or B the same in the two equations but with opposite signs so they
cancel out. The easiest will be to multiply equation (2) by 5 and equation (1) by 1, giving:
A – 0,3B = 40 000
–A + 5B = 300 000
We can now add the two equations together to give us:
A – 0,3B –A +5B = 40 000 + 300 000
Therefore 4,7B = 340 000
And B = 72 340
Substituting the value of B in equation (1) will give us:
A – 0,3(72 340) = 40 000
A = 40 000 + 21 702
A = 61 702
We will now apportion these values to four departments in the portions given. Please note that
this method will give the same value as the repeated distribution method above, the small
difference is due to rounding.
Prod. Dept A Prod. Dept B Serv. Dept A Serv. Dept B
Primary allocation 250 000 220 000 40 000 60 000
Allocation of 30 851 18 511 12 340
(61 702)
Serv. Dept A (50%) (30%) (20%)
Allocation of 28 936 21 702 21 702
(72 340)
Serv. Dept B (40%) (30%) (30%)
Total overheads 309 787 260 213 0 0
3. Specified order method
In this example, we were told that department A should be closed off first. Department B’s costs
will therefore only be written off to the production departments and not to service department A.
The costs of service department B will be multiplied by the percentage above divided by the
remaining percentage, excluding service department A’s allocation.
Prod. Dept A Prod. Dept B Serv. Dept A Serv. Dept B
Primary allocation 250 000 220 000 40 000 60 000
Allocation of
20 000 12 000 8 000
Serv. (40 000)
(50%) (30%) (20%)
Dept A
0 68 000
Allocation of 38 857 29 143
Serv. (68 000 × 40% ÷ (68 000 × 30% ÷ (68 000)
Dept B 70%) 70%)
Total overheads 308 857 261 143 0 0
4. Direct allocation method
In this method, the costs are allocated only to the production departments in the ratios excluding
the service departments.
Prod. Dept A Prod. Dept B Serv. Dept A Serv. Dept B
Primary allocation 250 000 220 000 40 000 60 000
Allocation of 25 000 15 000
Serv. (40 000 × 50% ÷ (40 000 × 30% ÷ (40 000) 0
Dept A 80%) 80%)
Allocation of 34 286 25 714
Serv. (60 000 × 40% ÷ (60 000 × 30% ÷ (60 000)
Dept B 70%) 70%)
Total overheads 309 286 260 714 0 0
6.5.3 Step 3: Computing overhead rates for each production cost
centre
In the previous section, we calculated the total budgeted overhead cost that needs to be allocated to
production cost centres. In this section, we consider the different bases that can be used to compute
the overhead rates for each production cost centre.
The overhead rate will then be used to allocate these costs to specific products or services. As an
example, if overhead rates were allocated based on labour hours and we calculated that the overhead
rate is R20 per hour, we could then allocate overhead costs to a product according to the number of
labour hours used to manufacture the product. Let us, therefore, assume that 15 labour hours were
used to manufacture a product. The figure of R300 (R20 × 15 hours) of overheads should be added to
the product’s cost of manufacture.
The applied or budgeted overhead rate is calculated using the following formula:

It is important that the basis of allocation is carefully chosen to represent the actual overhead cost of a
specific product. If this is not done on a consistent and fair basis, the overhead costs allocated to a
specific product may be incorrect. Different cost centres within the same business may also use
different bases. There are various possible activity levels or bases that can be used. Table 6.3 gives
some examples and the types of departments these allocation bases are best suited to. Again, the
allocation basis should be determined by the management accountant based on the business’s specific
needs.

Table 6.3 Examples of allocation bases


Allocation basis Type of department
Labour-intensive cost centres with a homogeneous
Direct labour hours labour force where most workers earn similar
wages.
Machine hours Machine-intensive cost centres.
Number of units manufactured Homogeneous products with very similar costs.
Direct material costs Cost centres where the cost of materials is high.
Labour-intensive cost centres with a
Direct labour costs heterogeneous labour force that includes various
types of labour with very different cost profiles.
Cost centres where prime cost forms the basis of
Prime costs (direct material + direct labour costs)
pricing a product.

We will now illustrate this process in Example 6.3.

EXAMPLE 6.3
Budgeted information of the welding department for the following year:

Direct material cost R880 000


Direct labour cost R546 000
Direct labour hours 24 000
Machine hours 36 000
Units manufactured 80 000
Manufacturing overheads R469 000

Required
Using the above data, calculate the predetermined overhead rates based on:
• Direct material cost
• Direct labour cost
• Direct labour hours
• Prime cost (direct labour + direct material)
• Machine hours

Solution
• Budgeted overhead rate based on direct material cost
= Budgeted overhead cost for period ÷ direct material cost

= 53% of material cost or R0,53 per R1 of direct material cost


• Budgeted overhead rate based on direct labour cost
= Budgeted overhead cost for period ÷ direct labour cost

= 86% of direct labour cost or R0,86 per R1 of direct labour cost


• Budgeted overhead rate based on direct labour hours
= Budgeted overhead cost for period ÷ direct labour hours
= R469 000 ÷ 24 000
= R19,54 per labour hour
• Budgeted overhead rate based on prime costs
= Budgeted overhead cost for period ÷ prime cost

= 33% of prime cost or R0,33 per R1 of prime cost


• Budgeted overhead rate based on machine hours
= Budgeted overhead cost for period ÷ machine hours
= R469 000 ÷ 36 000
= R13,03 per machine hour

The overheads thus calculated can now be allocated to the actual production for a period.
6.5.4 Step 4: Allocating overhead costs to products and services
Finally, the overhead costs can now be allocated to actual production. We would use the production
for a period and multiply it by the overhead rate to calculate the actual overhead cost that has to be
allocated to a product. We will explain this process in Example 6.4 by continuing with the figures
given in Example 6.3.

EXAMPLE 6.4
The company has calculated its overhead rates using various bases and made the following
summary:
Basis of allocation Rate Actual activity
Material cost R0,53 per R1 R6 000
Labour cost R0,86 per R1 R7 000
Labour hours R19,54 per hour 300 hours
Prime cost R0,33 per R1 R13 000
Machine hours R13,03 per hour 350 hours

Required
Calculate the cost of the manufactured goods using all five allocation bases.

Solution
Basis of allocation
Material cost Labour cost
Labour hours Prime cost (R) Machine hours
(R) (R)
Direct material 6 000 6 000 6 000 6 000 6 000
Direct labour 7 000 7 000 7 000 7 000 7 000
Applied 3 180 6 020
5 862 4 290 4 561
overhead costs (6 000 × (7 000 ×
(300 × R19,54) (13 000 × 0,33) (350 × R13,03)
(R) R0,53) R0,86)
Product cost
16 180 19 020 18 862 17 290 17 561
(R)

As can be seen from Example 6.4, it is very important to ensure that the basis of allocation chosen
best reflects the allocation of overhead costs within the department. It should also be remembered that
not all these bases would be calculated, but a single basis would be chosen for these allocations.

6.5.5 Step 5: Calculating under- or over-absorption of overheads


At the end of the financial year, and sometimes also in quarterly or monthly accounts, a business
needs to calculate whether production has over- or under-absorbed its overheads. This is an
important step, as the cost of goods sold may need to be corrected for the under- or over-absorption.
Under-absorption can be very problematic for a business, as this may mean that its selling prices may
have been based on lower than actual costs.
A simple example illustrates how to calculate over- or under-absorption.

EXAMPLE 6.5
Excitement Kites makes various types of kites. It has three cost centres: a fabric cost centre that
cuts and sews the fabric; a frame cost centre that makes the frames; and an assembly cost centre
that assembles and packages the kites. The following information pertains to the company’s
production for the last financial year:
Fabric Frame Assembly
Overhead rate R15 per labour hour R5,50 per machine hour R3 per unit
Activity for the year 1 800 labour hours 2 500 machine hours 5 000 units
Actual overheads for
R27 500 R12 000 R17 000
the year
Based on the above information, we can now calculate the over- or under-absorption of
overheads per department for the year as follows:
Fabric (R) Frame (R) Assembly (R) Total (R)
Overheads 27 000 13 750 15 000
55 750
absorbed (15 × 1 800) (5,50 × 2 500) (3 × 5 000)
Actual overheads 27 500 12 000 17 000 56 500
Over-/under-
(500) 1 750 (2 000) (750)
absorption
Therefore, Excitement Kites’ fabric and assembly units under-absorbed and the frame unit over-
absorbed overheads. Overall, the company under-absorbed R750, which means that the
company’s costs were R750 higher than planned and therefore gross profit will be R750 less than
expected and reflected in the management accounts.

As illustrated by the calculations in the examples in this chapter, the process of accurately allocating
overhead costs can be a very complex mathematical exercise. Therefore, a spreadsheet program like
Microsoft Excel will often be used to do these calculations.

CLOSING PRACTICAL EXAMPLE


Before we conclude this chapter, we will look at a comprehensive example of the whole process
from start to finish illustrating the principles covered in this chapter. We will continue with
Example 6.1, which covered the first step of allocating overheads to all departments and then show
the rest of the process.
To recap from Example 6.1 and the opening practical example, Kenton Manufacturers have four
departments. Two are production departments; the other two are a stores and a maintenance
department.
Service Service
Production Production
Basis department 1: department 2:
department 1 department 2
Stores Maintenance
Number of
20 15 7 4
employees
Direct labour
3 900 1 700 N/A N/A
hours
Floor space 120 m2 170 m2 30 m2 15 m2
Machinery value R218 400 R111 000 R20 400 R 35 000
Material
R134 500 R69 900 N/A R11 600
requisitions
Machine hours 1 900 2 000 NA NA
The following budgeted costs were extracted from the operational budget for the next financial
year:
Cost R
Depreciation of machinery 61 000
Electricity and water 35 600
Factory rent 75 000
Insurance on building 12 400
Maintenance 57 600
Municipal rates 35 800
Supervisory staff 220 000
Indirect labour 53 500
Indirect material 14 900

Management have determined the following bases for allocating overhead expenses:
• Electricity and water expenses are allocated according to machine hours.
• Maintenance and depreciation are allocated based on machinery value.
• Rates, insurance and rent are allocated based on floor space.
• Indirect materials and indirect labour are equally divided between the two production
departments.
• Supervisory staff costs are allocated based on the number of employees in each department.

Step 1
Based on the above information, the following primary allocations have been made (see Example
6.1 for one complete solution):
Serv. Dept: Serv. Dept:
Cost Prod. Dept 1 Prod. Dept 2 Stores Maint.
Total primary
258 642 220 926 50 799 35 433
allocation (R)

Step 2
Next, the service department overheads have to be allocated to the production departments. Let us
assume that the allocation from the stores department is based on the value of requisitions, and the
allocation from the service department is based on the value of machinery. Clearly, we would
exclude the value of the machinery of the maintenance department, as it would not allocate its costs
to itself. We can now draw up the following allocation table:
Basis Prod. Dept 1 Prod. Dept 2 Stores Maintenance
R value of 62,3% 32,4% 5,3%
Stores requisitions – (134 500 ÷ 216 (69 900 ÷ 216 0 (11 600 ÷ 216
Total R216 000 000) 000) 000)
R value of
62,5% 31,7% 5,8%
Machinery –
Maintenance (218 400 ÷ 349 (111 000 ÷ 349 (20 400 ÷ 349 0
Total R349 800
800) 800) 800)
(excl. Maint.)
Using the repeated distribution method, we can now allocate the service department costs as
follows:
Prod. Dept 1 Prod. Dept 2 Stores Maintenance
Primary allocation 258 642 220 926 50 799 35 433
Allocation of 31 648 16 459 2 692
(50 799)
stores (62,3%) (32,4%) (5,3%)
Subtotal 0 38 125
Allocation of 23 828 12 086 2 211
(38 125)
maintenance (62,5%) (31,7%) (5,8%)
Allocation of 1 378 716 117
(2 211)
stores (62,3%) (32,4%) (5,3%)
Allocation of 73 37 7
(117)
maintenance (62,5%) (31,7%) (5,8%)
5 2
Final allocation (7) 0
(62,3% ÷ 94,7%) (32,4% ÷ 94,7%)
Total overheads
315 574 250 226 0 0
(R)

Step 3
We now need to compute the overhead rates for each of the production cost centres. Lisa feels that
Production Department 1 is labour-intensive and therefore the overhead rate should be based on
labour hours, while Production Department 2 is machine-intensive and therefore the department’s
overheads should be allocated based on machine hours.
We can now calculate the overhead rates for the two departments:
Prod. Dept 1 Prod. Dept 2
Basis Labour hours Machine hours
Calculation 315 574 ÷ 3 900 250 226 ÷ 2 000
Overhead rate R81 per labour hour R125 per machine hour

Step 4
The overhead costs can now be allocated to production based on the overhead rates calculated. Let
us assume the business produced the following four projects for the year with the labour hours
spent in Production Department 1 and the machine hours spent in Production Department 2.
Project A Project B Project C Project D
Labour hours
700 1 800 600 1 300
(Prod. Dept 1)
Machine hours
300 500 280 700
(Prod. Dept 2)
Prime cost (direct
material + direct R350 000 R740 000 R250 000 R640 000
labour)
We can now calculate the cost of each of the four projects as follows:
Project A Project B Project C Project D Total
Prime cost R350 000 R740 000 R250 000 R640 000 R1 980 000
Prod. Dept 1
(Labour hours 56 700 145 800 48 600 105 300 356 400
@ R81)
Prod. Dept 2
(Machine hours 37 500 62 500 35 000 87 500 222 500
@ R125)
Total costs (R) 444 200 948 300 333 600 832 800 2 558 900
As can be deduced from the above tables, the level of production in Production Department 1 was
higher than expected and therefore more overhead costs were allocated to projects than originally
planned, whereas the level of production in Production Department 2 was lower than expected and
therefore lower overhead charges were allocated to projects than originally planned. It should be
highlighted here, though, that this does not necessarily mean that overhead charges were under- or
over-absorbed. We need to compare the allocated overhead costs to actual overhead costs to
calculate this (see Step 5).

Step 5
The over- or under-absorption of overheads can now be calculated for the year. Let us assume that
the company re-performed the allocations as given in Steps 1 and 2 based on the actual results for
the year, and found the following:
Prod. Dept 1 Prod. Dept 2
Total expected overheads (see R315 574 R250 226
step 2)
Total actual overheads R346 084 R247 325
Total overheads absorbed (see
R356 400 R222 500
step 4)
If we analyse the above table, we find that Production Department 1 over-absorbed its overheads
by R10 316 (R346 084 – R356 400) and Production Department 2 under-absorbed its overheads
by R24 825 (R247 325 – R222 500). Therefore, R10 316 will need to be deducted from cost of
goods sold for Production Department 1 and R24 825 added to cost of goods sold for Production
Department 2.
An in-depth analysis of these results fall outside of the scope of this chapter, but it would
appear that the difference experienced in Production Department 2 is due to a lower level of
production than expected but similar overhead costs. A possible explanation for this could be that
the overhead costs of Production Department 2 may be fixed in nature.
This information is critical for Lisa to inform future cost allocations, and the information can
allow her to identify areas where cost reductions may be appropriate. She should now be in a
much better position to run her grandfather’s business.

6.6 Summary
In this chapter, we discussed the techniques and principles involved in allocating overhead costs in a
systematic, economical way to products or services. The types of overheads (manufacturing and non-
manufacturing) and the cost behaviour of overheads (fixed, variable, semi-variable and step-fixed)
were discussed. The principles of the economic viability of a costing system, the timelines involved
in a costing system and the identification of cost centres were then discussed. These factors are
critical in designing an appropriate system for allocating overhead costs to cost centres.
The process of allocating overheads to specific products and services was then discussed. This
process basically comprises five steps, as follows:
• Step 1: Allocating costs to all cost centres based on appropriate bases of allocation.
• Step 2: Allocating service department costs to production departments by using the methods of
repeated distribution, simultaneous equation, specified order, direct allocation or mathematical
models.
• Step 3: Computing the appropriate overhead rates for each production cost centre.
• Step 4: Allocating overhead costs to production using the overhead rates.
• Step 5: Calculating the over- or under-absorption of overheads for the period.

The allocation of overhead charges to products or services is an important process, especially in a


large manufacturing environment, and therefore it is critical that a cost or management accountant
considers these aspects carefully. The overhead allocations will now be used in the accounting
system (see Chapter 7) for management accounting purposes and the costing system, depending on the
type of costing system used (See Chapters 8–14 for discussion on the various costing systems).
PRACTICAL ACTIVITY
Write a memo to Lisa based on the closing example, in which you discuss the issues she should
consider when calculating the overhead allocation rates for next year based on this year’s results.

Short questions
1. When will over-absorbed overheads occur?
2. What is the purpose of an overhead absorption rate?
3. Is the predetermined budgeted manufacturing overhead rate calculated by using actual or
budgeted values?
4. Calculate the predetermined budgeted overhead rate from the following:
• Budgeted overheads for the
R150 000
year
• Budgeted labour hours for the
60 000 hours
year
• Actual labour hours for the
62 000 hours
year
5. When you calculate the allocation of overheads, do you multiply the predetermined overhead
rate by the actual activity level or by the budgeted activity level?

Long questions
1. The following information was supplied by Suffield Manufacturing:
• Budgeted overheads for the
R376 600
year
• Actual overheads for the year R388 600
• Budgeted machine hours for
28 000
the year
• Actual machine hours for the
28 690
year
a. Calculate the overhead absorption rate based on machine hours.
b. Calculate the absorbed overheads.
c. Calculate the under- or over-absorbed overheads for the year.

2. The following information was taken from the records of Sam Manufacturers, which
produces three products.
Budgeted data for the year is as follows:
• Direct material costs R580 000
• Direct labour costs R260 000
• Manufacturing overheads R310 000
• Direct labour hours 10 400
• Machine hours 8 200
Details about the three jobs that were carried out during January are as follows:
Job no. Material costs Labour costs Machine hours Labour hours
401 R15 000 R8 400 250 280
402 R16 000 R6 200 310 250
403 R14 000 R7 100 220 300
Calculate the predetermined manufacturing overhead rate based on:
a. Material cost
b. Direct labour cost
c. Direct labour hours
d. Machine hours
e. Prime costs

3. Using the predetermined manufacturing overhead rates calculated in Question 2, calculate the
total costs of:
a. Job 401 if overheads are allocated on the basis of prime costs.
b. Job 401 if overheads are allocated on the basis of direct labour hours.
c. Job 402 if overheads are allocated on the basis of machine hours.
d. Job 403 if overheads are allocated on the basis of material cost.
e. Job 403 if overheads are allocated on the basis of direct labour costs.

4. Mbam’s accountant supplied the following information about production costs for the current
financial year, which ended two weeks ago:
• Direct materials R600 000
• Direct labour R415 000
• Production overheads R501 000
The accountant also supplied the following information about estimates for the next financial
year:
• The purchase price of direct materials will increase by 16 per cent.
• The direct labour rate per hour will increase by 16 per cent from the previous rate of R20
per hour.
• The production level will stay the same as the previous year.
• The production overheads will increase by 10 per cent.
a. Calculate the budgeted manufacturing overhead rate for the next financial year on the
basis of direct labour hours.
b. Calculate the total budgeted cost of production for Mbam for the next financial year.
c. Calculate the total cost of job 42B if 2 800 labour hours were worked and direct material
costs were R7 500.

5. The following budgeted data for the current financial year is supplied:
• Direct material R489 000
• Direct labour R222 400
• Overheads R589 000
• Direct labour hours 1 112
• Machine hours 650

The following actual costs for March were supplied:


Direct material Direct labour Direct labour
Job number Machine hours
cost cost hours
545 R9 500 R2 590 18 9
546 R4 300 R1 900 10 6
547 R2 850 R1 690 8 5
a. Calculate the predetermined manufacturing overhead rate based on direct labour cost.
b. Calculate the predetermined manufacturing overhead rate based on direct material cost.
c. Calculate the predetermined manufacturing overhead rate based on direct labour hours.
d. Calculate the predetermined manufacturing overhead rate based on machine hours.
e. Calculate the predetermined manufacturing overhead rate based on prime costs.
f. Calculate the total cost of job 547 if overheads are recovered on a direct labour basis.
g. Calculate the total cost of job 545 if overheads are recovered on a machine-hour basis.

6. The following information was taken from the budgets and records of Lincks Manufacturers:
Service Service
Production Production
Basis Department A: Department B:
Department A Department B
Stores Maintenance
Number of
80 45 14 8
employees
Direct labour
290 130 – –
hours
Floor space 100 m2 150 m2 40 m2 15 m2
Machinery value R388 400 R329 000 R20 400 R35 000
Material
R38 500 R29 500 – R8 600
requisitions
Machine hours 240 150 – –
The following budgeted details for manufacturing overheads were obtained for the current year:
R
Factory rent 48 000
Depreciation of equipment 84 000
Rates 18 800
Maintenance of equipment 15 900
Cafeteria for staff 12 890
Insurance on building 2 400
Electricity and water 29 900
Indirect labour 16 600
Maintenance of building 24 900
The following budgeted details for manufacturing overheads were obtained for the current year:
R
Factory rent 48 000
Depreciation of equipment 84 000
Rates 18 800
Maintenance of equipment 15 900
Cafeteria for staff 12 890
Insurance on building 2 400
Electricity and water 29 900
Indirect labour 16 600
Maintenance of building 24 900
Additional information:
• The reallocation of service department overheads takes place on the basis of:
– Stores (material requisitions)
– Maintenance (machine hours)
• Electricity and water expenses are allocated according to machine hours.
• Indirect labour costs are allocated equally to the four departments.
• Lincks decided that overhead rates for production departments must be calculated based on
direct labour hours.

Required
Allocate the overheads to Production Departments A and B by using the appropriate basis, and
calculate applicable overhead allocation rates for both departments. All calculations must be
done to the nearest rand, except for the overhead allocation rates, which must be rounded to two
decimals.
7 Cost statements
Jayan CV Mathew

LEARNING OUTCOMES
After studying this chapter, you should be able to:
• explain the costing process;
• discuss how costs are classified in a costing system;
• understand, and draw a diagram to illustrate, the flow of costs in a costing system;
• record costs according to the accounting procedure used in a costing system;
• draw up a cost of production and sales statement;
• draw up a statement of financial performance (income statement);
• explain the differences between integrated and interlocking accounts;
• prepare ledger accounts using cost ledger and financial ledger accounts systems; and
• reconcile profits as given in cost and financial ledgers, and identify possible reasons for the
difference between the profits shown in cost ledgers and financial ledgers.

PRACTICAL EXAMPLE
After passing his grade 12 studies in 2009, Eben Marcus decided to study for a three-year diploma
in fashion design. He finished the diploma in 2012 and started a shirt-making business. Eben has
started with a planned production of 150 shirts per month. He has employed three stitching
workers and one cleaner, and he does the fabric-cutting work himself. He has calculated the
amounts paid for materials, labour and overhead costs for the month, but is unsure how to record
these in the company’s general ledger.

7.1 Introduction
In the previous chapters, we discussed the elements of cost of production. In a manufacturing entity,
materials are transformed in the production process. Direct labour is employed in the transformation
process in order to convert the materials into a final product. And this process also incurs a number
of other expenses, known as manufacturing overheads. Overhead costs cannot be directly traced back
to the product and are therefore commonly known as indirect costs. As overheads are linked to
production but cannot be traced directly to the product, they are allocated to production using an
appropriate basis (see Chapter 6). The costs of direct material, direct labour and the indirect costs
(manufacturing overheads) are then added together in order to determine the total cost of producing a
product or providing a service.

Cost of production is the sum of all the direct and indirect costs incurred in the manufacturing of a
product or the rendering of a service.
The principles of a costing system can also be used in service and retail environments. For ease of
explanation and understanding, this chapter restricts itself to the flow of costs within a manufacturing
environment and discusses the different types of manufacturing costs. It should, however, be noted
that the same principles can be applied in non-manufacturing environments.
Consider the example of a typical manufacturing firm. Figure 7.1 illustrates the product cost flow
in such an environment.

Figure 7.1 Product cost flow

EXAMPLE 7.1 DETERMINING COST OF PRODUCTION FOR A


MANUFACTURING FIRM
The following information is taken from the records of KK Limited during the month of February:
Raw materials purchased and used R150 000
Direct labour utilised in production R202 500
Indirect material R3 750
Indirect labour R75 000
Consumables R60 000
Property rates and levies R27 000
Factory insurance R6 750
Plant hire and depreciation R142 500
Manufacturing overheads incurred are absorbed in production.

Required
Determine the cost of production of KK Limited for the month of February.

Solution
Cost of production of KK Limited
Raw materials R150 000
Direct labour R202 500
Prime cost R352 500
Manufacturing overheads R315 000
Total cost of production R667 500

Example 7.1 is a simple illustration of how production cost is calculated. From the previous chapters,
you have learnt that the elements of cost of production are direct materials, direct labour and
manufacturing overheads. Cost of production is the total of all the three cost elements.
The cost elements incurred in the production of a product or the provision of a service are added
together to arrive at the cost of production or the cost of providing the service. In the above example,
all the indirect costs that are incurred in the manufacturing activity are called manufacturing
overheads. This is to enable you to understand that manufacturing overheads are not just one item, but
a combination of items that are indirectly incurred in the production of a product or provision of a
service.

7.2 The manufacturing process


There are various types of business activities, but the main areas of operation are manufacturing,
retail and services (see Figure 7.2). The distinctions between these have been discussed in previous
chapters.
A manufacturing business transforms raw materials into products that can be marketed. In the
transformation of raw materials, labour is employed and manufacturing overheads are incurred. The
costs incurred by these three elements become, collectively, the cost of production, as discussed in
Example 7.1. Manufacturing firms employ different techniques in their manufacturing activities. Due
to technological changes and efforts to make production more cost-effective, these techniques are
regularly assessed and developments are introduced.
Manufacturing firms have three different types of inventories: raw materials, WIP and finished-
goods inventories. The nature of their inventories is one of the main differences between a
manufacturing business and a trading business. Trading businesses usually have only one inventory
account, namely the retail inventory account. Manufacturing is an activity in which raw materials go
through several stages before they become final products and are ready to be sold. Manufacturing
businesses therefore have these three different inventory accounts. The total inventory of the
manufacturing firm will be at different stages of completion at the end of the accounting period due to
the nature of the firm’s manufacturing activities. A firm’s inventory may be valued by applying any of
the stock valuation methods used by the firm. For the purposes of this chapter, we will use the first in
first out and weighted average methods, as discussed in Chapter 3, which deals with inventory
valuation.

Figure 7.2 Three main business activities


7.3 Cost classifications in manufacturing firms
The costs incurred by a manufacturing firm can be broadly classified into manufacturing costs and
non-manufacturing costs. Manufacturing costs are those costs incurred in relation to the manufacture
of a product (e.g. the cost of manufacturing a shirt). The manufacturing costs include the three
elements of the cost of production, i.e. direct material utilised in production (fabric for the shirt);
direct labour allocated to production (cutting and stitching) and manufacturing overheads applied to
production (lubricating oil, electricity, cleaning labour, etc.). This was discussed more fully in
Chapter 3.

Direct material, direct labour and other direct expenses, in combination, are known as direct
costs.

The direct costs can be traced to the product or identified as part of the product. The fabric used in
the production of a shirt and the wages paid to the tailors who produce it are examples of direct costs.

Expenses that are part of the manufacturing overheads are known as indirect costs.

Indirect costs are not traceable to the product, as they do not form part of the product. For example,
the lubricating oil consumed by the sewing machines used to manufacture the shirt, the electricity
charges incurred to run the sewing machines and the depreciation written off on the sewing machines
are all examples of indirect costs, or manufacturing overheads. These categories of costs have been
more fully discussed in previous chapters.

7.4 Flow of costs in manufacturing firms


The costing activity in a manufacturing firm begins with the procurement of the elements of
production. The cost of the raw materials issued to factory/production is then transferred to the
production/WIP account. The labour allocated to production and the manufacturing overheads applied
to production are also transferred to the production/WIP account. When production is completed, the
cost of the completed goods is transferred from the production/WIP account to the finished-goods
inventory account.
Although different entities may account for manufacturing costs in different ways, the standard
accounting process for bringing these costs into record is as follows:
• The costs of material purchased, labour and manufacturing overheads are debited in the respective
control accounts.
• When materials are issued, labour is utilised and manufacturing overheads are incurred. These are
allocated to production. The costs of these elements are transferred from their respective accounts
by debiting the production/WIP account and crediting the individual control accounts.
• When the manufacturing activity is completed, the cost of the completed goods is transferred by
debiting the finished-goods inventory account and crediting the production/WIP account.
• When the finished goods are sold, the cost of goods sold is transferred from the finished-goods
inventory account by debiting the cost of goods sold/cost of sales account and crediting the
finished-goods inventory account.

At the same time, the debtors’/bank account is debited and the sales account is credited with the total
sales value. Figure 7.3 is an illustration of the accounting procedure detailed above.

Figure 7.3 Accounting process for manufacturing costs

In Figure 7.3, (a) represents the procurement of elements of cost of production; (b) represents the
transfer of cost elements of production to the production account; (c) represents the transfer of the
cost of completed goods from production to finished goods; and (d) represents the transfer of costs of
goods sold to cost of sales and bringing sales into effect.
From this process, we can draw up the following diagram for the flow of costs in a manufacturing
firm (see Figure 7.4).

Figure 7.4 Flow of costs in manufacturing process


7.5 Cost-accounting procedure in manufacturing firms
It is important that the flow of costs is recorded in the financial accounting records of the
manufacturing firm. The transactions that take place in regard to manufacturing are first recorded in
the accounting records by means of journal entries. The journal entries are then posted to the different
ledger accounts. The information thus recorded is then summarised to prepare reports such as
statements of cost of production and cost of sales. This process is illustrated by means of Example
7.2.

EXAMPLE 7.2 RECORDING MANUFACTURING COST


TRANSACTIONS
Ace Manufacturing Company manufactures and sells leather caps for men. They provide the
following information and transactions for January 2013:

1. Materials on hand at the beginning of the month: R25 000.


2. Materials purchased on credit: R85 000.
3. Direct material worth R70 000 and indirect material worth R5 000 were issued to production.
4. Labour cost totalling R90 000 for the factory employees consisted of the following:
• Direct wages R75 000
• Indirect wages R15 000
5. Direct labour costs incurred were absorbed into production.
6. Depreciation of plant and machinery amounted to R1 500.
7. Other manufacturing overheads paid by cheque amounted to R25 200.
8. Total manufacturing overheads incurred were applied to production.
9. Products with a manufacturing cost of R150 000 were completed and transferred to finished-goods inventory.
10. Products with a total cost of R120 000 were sold during the period on credit for R200 000.

Required
Record the above transactions in the general journal and general ledger of the entity.
Ace Manufacturing: General journal
1. Material control 85 000
Creditors’ control 85 000
2. Production/WIP control 70 000
Manufacturing overheads control 5 000
Material control 75 000
3. Wages control 90 000
Wages payable 90 000
4. Production/WIP control 75 000
Manufacturing overheads control 15 000

Wages control 90 000


Wages payable 90 000
Bank 90 000
5. Manufacturing overheads control 1 500
Depreciation 1 500
6. Manufacturing overheads control 25 200
Bank 25 200
7. Production/WIP control 46 700
Manufacturing overheads control 46 700
8. Finished-goods inventory control 150 000
Production/WIP control 150 000
9. Cost of sales 120 000
Finished-goods inventory control 120 000
Debtors 200 000
Sales 200 000
Once the transactions are recorded in the journal/book of first entry, the next step is to post them
to the ledger accounts. As discussed in Chapter 6, the manufacturing overheads applied and
actually incurred may not be the same, as manufacturing allocation rates are normally based on
estimates. In such instances, the difference is to be determined and whether it is over- or under-
applied should be established. When the applied overheads are greater than the actual overheads,
this is known as over-absorbed overheads, and when the applied overheads are less than the
actual overheads it is known as under-absorbed overheads. In either case, the difference is to be
brought into account. If the overheads are under-absorbed, the journal entry made is as follows –
debit: cost of sales and credit: manufacturing overheads. On the other hand, if overheads are
over-absorbed, the journal entry will be – debit: manufacturing overheads and credit: cost of
sales.

Recording journal entries in a ledger


The following ledger accounts may be opened in order to record the above journal entries:

Ace Manufacturing: General ledger


Statement of financial position accounts
7.6 Cost of production and cost of sales statement
A cost of production and sales statement is drawn up to determine the total cost of producing and
selling the product. The first part of this statement shows the calculation of the cost of production, and
the second part the cost of sales. As stated earlier, the cost of production is calculated by adding the
three cost elements that make up the total cost of production. Once the cost of production is
determined, the cost of one unit of the product can be established. Establishing the unit cost helps
financial decision making, such as the selling price and profitability, etc. As discussed in Chapter 2,
unit costs should however be used circumspectly. The cost per unit is calculated by means of the
following simple formula:

Cost per unit = total cost of production ÷ number of units produced

In a normal business situation, it is not necessary that all the goods produced are sold during the same
period in which they are produced. Therefore, in the cost of sales section of the statement we
calculate the cost of the number of units sold using the cost per unit established. The cost of sales is
calculated using the following formula:

Cost of sales = number of units sold × cost per unit

The cost of sales thus established is then matched against sales in order to determine the profit margin
generated by the sale of these products.
Alternatively, the calculation of cost of sales can be done as follows:

Opening inventory finished goods + cost of finished goods during the period – closing inventory
finished goods.

Example 7.3 illustrates how to calculate the cost of production and the cost of sales, taking into
account all the different elements that must be incorporated into the calculation.

EXAMPLE 7.3 Drafting cost of production and cost of sales statement


Required
Use the information provided in Example 7.2 to draft a cost of production and cost of sales
statement.
When drawing up the statement, there is no specific format that must be used, as is the case in
financial accounting. For the sake of neatness and uniformity, we are using here a vertical format.
The same information, however, can be presented in various ways, as required by management. A
cost of production and cost of sales statement may be drawn up as follows:
Ace Manufacturing: Cost of production and cost of sales statement
Direct material used 70 000
Opening inventory 25 000
Purchases 85 000
Closing inventory (40 000)
Direct labour 75 000
Prime cost 145 000
Manufacturing overheads 46 700
Total manufacturing cost 191 700
Plus Opening inventory WIP 0
Total WIP 191 700
Minus Closing inventory WIP (41 700)
Total cost of finished-goods/goods manufactured 150 000
Plus Opening finished-goods inventory 0
Finished goods available for sale 150 000
Minus Closing finished-goods inventory (30 000)
Cost of sales 120 000
It is evident from the above statement that several items of cost information are needed to
determine cost of production and cost of sales. The following calculations are used in order to
determine the different aspects in a cost of production and sales statement:
Calculation of direct material used:
Opening inventory direct material
+ Purchases of direct material
= Total direct material available
− Closing inventory direct material
= Direct material used
Calculation of manufacturing cost:
Direct material used
+ Direct labour
= Prime cost
+ Manufacturing overheads
= Total manufacturing cost
Calculation of cost of finished goods:
Total manufacturing cost
+ Opening inventory WIP
= Total WIP
− Closing inventory WIP
= Cost of finished goods
Calculation of cost of sales:
Total cost of finished goods
+ Opening inventory finished goods
= Total finished goods available
− Closing inventory finished goods
= Cost of sales/cost of goods sold

7.7 Statement of financial performance/income statement


The statement of financial performance (also known as the income statement) is drawn up by a
manufacturing business to determine the financial results of its activities. The results of these trading
activities are depicted in the income statement. The manufacturing firm incurs expenses as a result of
selling the products it manufactures. These expenses are entered into the income statement prepared
for the specific period. The financial statements are drawn up according to the prescriptions of
International Financial Reporting Standards (IFRS). If the statement is drawn up for internal
purposes, as is the case for management accounting, there are no such prescriptions. For management
purposes, reports and statements are drawn up as and when they are required by management, usually
on a monthly basis. This is demonstrated in Example 7.4.

EXAMPLE 7.4 DRAFTING STATEMENT OF FINANCIAL


PERFORMANCE/INCOME STATEMENT
In order to draw up the statement, we are provided with the following sales and administration
expenses in addition to the information given in Example 7.2:
Salary of sales personnel R10 000
Freight on goods sold R9 500
Travelling expenses of sales personnel R5 500
Advertising R15 000
Marketing manager’s salary R20 000

Required
Use the above information to draw up an income statement for Ace Manufacturers.
Ace Manufacturers: Income statement
Sales 200 000
Cost of sales 120 000
Gross profit 80 000
Sales and administrative expenses: 60 000
Salary of sales personnel 10 000
Freight on goods sold 9 500
Travelling expenses of sales personnel 5 500
Advertising 15 000
Marketing manager’s salary 20 000
Net profit 20 000

We will now look at how what we have discussed above is also applicable in a service business.
Just as determining the cost of production is important in the case of manufacturing businesses,
determining the cost of the services offered is crucial in a service environment. We will take the
example of a hairdressing salon as a service business and look at the cost of providing services to its
customers. Like a manufacturing entity, the costs the salon incurs can also be divided into direct and
indirect costs. The cost of the hair products and other materials the business uses and the amount of
time the hairdressers take to do the work, etc., are direct costs. The rent the salon pays for its
premises and the wages it pays the cleaning staff, etc., are indirect costs. The direct costs and the
indirect costs are added together to establish the total cost of the services the salon provides.
7.8 Integrated and interlocking accounting systems
In the above discussions, we have explained how to record transactions that are related to
manufacturing costs. In the following sections, we look at the two types of accounting systems used by
manufacturing firms to keep records – integrated and interlocking accounting systems. Which of these
accounting systems is used is greatly influenced by the size of the enterprise. Smaller manufacturing
enterprises generally keep basic integrated costing records where there is no separate set of costing
and financial records, i.e., the two sets of records are combined in the same book. Larger
manufacturing enterprises maintain separate interlocking costing and financial records.

7.8.1 Integrated accounting system


An integrated accounting system is one in which cost and financial accounts are kept in the same
set of accounting records.

With an integrated system, there is no distinction between financial and cost accounts. This system
keeps control accounts together with supporting ledger accounts for each of these accounts. This
system is usually used by manufacturing firms that have a small volume of transactions. This type of
system meets the information requirements of both costing and financial accounting, and prevents the
need for separate sets of accounts for costing and financial purposes.
The section of records that deals with the elements of cost of production in the accounting records
enables the manufacturing enterprise to determine the cost of production and consequently the cost of
goods sold. This information is eventually used to draw up management-accounting information that is
required for decision-making purposes.
The rest of the accounting records assist in providing financial accounting information. The cost of
goods sold is used in order to determine the gross profit, and then the selling and administrative costs
are entered in the profit and loss account.

7.8.1.1 Requirements of an integrated accounting system


In order for the system to work effectively, the company must usually have:
• a policy on the extent of integration of the two sets of books, i.e. either full integration or partial
integration;
• consistency in treating the accruals, prepaid expenses and other adjustments used in the
preparation of final accounts;
• harmonisation of the activities of the staff in charge of cost and financial aspects of the accounts;
and
• a coding system that serves both cost and financial accounts.

7.8.1.2 Advantages of an integrated accounting system


An integrated accounting system has the following advantages:
• There is no need for reconciling the cost and financial accounting profits because there is only one
set of accounting records.
• There is more consistency between the cost and financial accounting records as the same
inventory-valuation and depreciation methods are used because cost and financial accounts are
integrated.
• There is less administrative work and consequently lower administrative costs because there is
only one set of accounting records.
• There is reduced chance of error, as recording takes place in the same set of accounting records.
• Decisions can be made more quickly because using one set of accounting records helps generate
information more quickly.

The ledger accounts illustrated in Example 7.2 reflect how accounts are kept in an integrated
accounting system.

7.8.2 Interlocking accounting system


An interlocking accounting system is one in which two separate sets of accounting records, for
cost and for financial accounting, are kept.

This type of system is normally maintained by manufacturing businesses that have large volumes of
transactions. With an interlocking system, the cost accounts are distinct from the financial accounts.
This system is also called an independent system, a non-integrated system, a cost ledger system or a
traditional system. In order to reconcile the two sets of books, a control account is maintained in each
set of books.
A financial ledger control account is kept in the cost-accounting records and a cost ledger control
account is maintained in the financial-accounting records. The financial ledger control account in the
cost-accounting books is a control account in which all the financial-accounting entries are recorded.
The cost ledger control in the financial-accounting books, on the other hand, records all entries that
affect cost accounts.

7.8.2.1 Characteristics of interlocking accounting system


An interlocking accounting system has the following features:
• Double-entry relationship between the cost and financial accounts does not exist.
• Different profit figures may arise from the different sets of books and then a reconciliation of
profits is required.
• Inventory valuation methods and depreciation methods used may be different.
• Control accounts maintain the link between the two sets of accounting records.

An interlocking accounting system is illustrated in Example 7.5.

EXAMPLE 7.5
Assume that, in Example 7.2, Ace Manufacturing uses an interlocking accounting system.

Required

7.9 Reconciliation of cost-accounting and financial-accounting


profits
The transactions that determine the costing profit and the financial profit are, in essence, the same.
Therefore, the profit or loss determined in the profit and loss account prepared in both ledgers should
be the same. However, the two amounts usually differ and it becomes necessary to establish the
reasons for the difference and then reconcile the two recorded profits.

7.9.1 Reasons for difference in profits between cost accounts and


financial accounts
The following factors may cause the difference in profits shown in the cost and financial accounts:
• Items of income, such as profit on sale of non-current assets, interest income, rental income,
dividend income, and expenses – such as losses on sale of non-current assets, interest on loans,
rent expenses, dividends paid and bad debts written off – only appear in the financial-accounting
records, and not in the cost-accounting records.
• Items such as inter-departmental charges of rent and internal charges of interest on capital between
different departments or cost objectives only appear in cost-accounting records, and not in the
financial-accounting records.
• Inventory valuation methods used in cost accounting and those used in financial accounting may
differ.
• Depreciation may be calculated in different ways for cost-accounting and financial-accounting
purposes.
• Manufacturing overheads may be allocated differently in the two sets of books.

7.9.2 Reconciliation process


Reconciliation is done by recording items of incomes and expenses that cause the difference. For
example, if we start with the profits as recorded in cost accounts, we add or subtract items that cause
the difference in the profit. This process is illustrated by means of Example 7.6.

EXAMPLE 7.6 RECONCILIATION OF PROFITS


The financial-accounting records of Ace Manufacturing showed a profit of R84 653, whereas the
cost-accounting records for the same period showed a profit of R20 280.
The following variations were identified on examination of the two accounts:
Financial
Cost accounts (R)
accounts (R)
Marketing expenses 78 900 73 695
Opening inventory 191 250 206 325
Closing inventory 140 625 136 635
The following item appeared only in the cost-accounting
records:
Notional rent charges 24 375
The following items appeared only in the financial-
accounting records:
Profit on sale of non-current assets 6 375
Interest received 19 763

Required
Reconcile the two profit figures.

Solution
Reconciliation statement
Profit as per cost accounts 20 280
Add: Profit on sale of assets 6 375 69 578
Interest received 19 763
Notional rent charges 24 375
Overvaluation of opening inventory in cost accounts 15 075
Undervaluation of closing inventory in cost accounts 3 990
89 858
Less: Undercharge of marketing expenses in cost
5 205
accounts
Profit as per financial accounts 84 653

CLOSING PRACTICAL EXAMPLE


Eben Marcus started his new shirt-manufacturing business with a planned production of 150 shirts
per month. He has employed three stitching workers and one cleaner, and he does the cutting work.
The stitching workers are paid at the rate of R35 per hour; the cleaning worker is paid at the rate of
R8 per hour and Eben charges R40 per hour. They all worked 150 hours each during the first
month of production. The following additional expenses were incurred during the same period:
• 300 metres shirt fabric at R25 per metre
• Rent for the month: R1000
• Electricity charges: R750
• Telephone charges: R600

Eben has a policy of applying manufacturing overheads at the rate of 20 per cent of direct labour
cost. The production and sales for the month were 150 shirts. Shirts were sold at R250 each.
With the information he has gathered, Eben can now draw up a general journal entry to record
the transactions in the general ledger.
Firstly, the cost per shirt produced needs to be calculated as follows:
Production cost statement
Direct material (300 m x R25) 7 500
Direct labour 16 500
Stitching labour (300 hrs x R35) 10 500
Cutting labour (150 hrs x R40) 6 000
Primary cost 24 000
Manufacturing overheads (R16 500 × 20%) 3 300
27 300
Cost per shirt = total cost ÷ number of shirts
= R27 300 ÷ 150 shirts = R182
General journal Debit Credit
Material control (300 × 25) 7 500
Creditors control 7 500
Production control 7 500
Material control 7 500
Wages control (300 × 35 + 150 × 40 + 150 × 8) 17 700
Bank 17 700
Production control (300 × 35 + 150 × 40) 16 500
Manufacturing overheads control 1 200
Wages control 17 700
Manufacturing overheads control (1000 + 750 + 600) 2 350
Bank 2 350
Production control (16 500 × 20%) 3 300
Manufacturing overheads control 3 300
Finished-goods inventory (150 × 182) 27 300
Production control 27 300
Cost of sales 27 300
Finished-goods inventory 27 300
Debtors (150 × 250) 37 500
Sales 37 500

7.10 Summary
All entities, whether they are manufacturing, retail or service businesses, incur both direct and
indirect costs to enable them to sell their products or provide their services. These costs need to be
accounted for in the cost- and financial-accounting records in accordance with the principles
discussed in the previous chapters. It is very important to be able to calculate the cost of sales of
production in a manufacturing entity. The cost- and financial-accounting records need to be kept in a
consistent method.
Generally, either integrated or interlocking financial- and cost-accounting records are kept. With
an integrated accounting system, the financial- and cost-accounting records are kept in the same set of
records. On the other hand, when two separate sets of records are kept, it is known as an interlocking
accounting system.
Although this chapter has focused on manufacturing enterprises, we should remember that the
principles are equally applicable to service businesses. These costing records will be the basis for
the chapters that follow. Once the production costs have been accounted for, as discussed in this
chapter, these costs can be allocated to specific products using either product (job) or process
costing. Product costing is discussed in Chapter 8; process costing is discussed in Chapters 10 and
11.

PRACTICAL ACTIVITY
Eben Marcus has started a shirt-making business and has decided to employ you as his accountant.
He has asked you take the responsibility of looking after the cost-accounting side of the business.
He has also asked you to give him advice on cost and the methods of record keeping so that his
business is run efficiently.
Explain with the help of a diagram to Eben cost accumulation/cost flow in a manufacturing
enterprise.
Advise Eben on effective cost-control measures so he can manage the production activity in an
efficient manner.

Questions

1. The following information is available from Milton Ltd:


Direct material costs R12 000. The ratio between direct materials and direct labour is 3:2.
The prime cost represents 50 per cent of the total manufacturing cost and 30 per cent of the
manufacturing overheads are direct costs. The WIP at the beginning of the period was 10 per
cent of prime cost, and closing inventory of WIP is 70 per cent of opening inventory. Eighty-
five per cent of the production was sold.
Answer the following multiple-choice questions by choosing the correct alternative for
each question:
• The prime cost is:
A. R8 000
B. R20 000
C. R6 000
D. R12 000
• The total manufacturing costs are:
A. R40 000
B. R40 600
C. 14 000
D. R34 000
• The total direct costs are:
A. R20 000
B. R32 000
C. R22 000
D. R26 000
• The cost of goods manufactured is:
A. R28 000
B. R18 000
C. R23 400
D. R40 600
• The cost of goods sold is:
A. R46 000
B. R34 000
C. R34 510
D. R37 400

2. Use the information given below to choose the correct answer from the alternatives. There is
only one correct answer.
Africa Products Ltd currently uses an integrated accounting system. The company issued
direct and indirect materials to production.
• Which entry is made in its books for the direct materials issued?
A. Debit materials control and credit creditors’ control
B. Debit WIP control and credit material control
C. Debit financial ledger control and credit material control
D. Debit material control and credit manufacturing overheads control
• The following journal entry is made when indirect materials are issued to production:
A. Debit materials control and credit WIP control
B. Debit indirect materials and credit creditors’ control
C. Debit materials control and credit manufacturing overheads control
D. Debit manufacturing overheads control and credit materials control
Africa Products Ltd has decided to change its accounting system from an integrated to an
interlocking accounting system.
• Which is the journal entry it makes for the cost of completed goods?
A. Debit finished-goods control and credit WIP control
B. Debit finished-goods control and credit financial ledger control
C. Debit cost of goods sold and credit WIP control
D. Debit WIP control and credit finished-goods control

3. You are employed as a junior accountant at Milton Ltd. You received the following
information concerning production:
• Normal production: 14 400 units
• Direct materials: 2 kg per unit @ R40
• Direct labour: 3 hours per unit @ R30
• Other direct manufacturing expenses: R20 per unit
• Fixed manufacturing overheads: R518 400, which are applied on the basis of direct labour
hours
• Variable non-manufacturing overheads: R216 000
• Selling and distribution costs: R144 000
• Administration expenses: R115 200
• Actual production and sales: 14 000 units
• Selling price: R300 per unit

Required
Calculate the following:
• Predetermined overhead rate
a. Overhead application rate per unit
b. Prime cost for the units produced
c. Conversion cost for the units produced
d. Over/under-applied overhead if the actual overheads are R502 000
e. Total manufacturing cost for the units produced

4. Use the same information as in Question 3, except for the following:


Production 14 000 units and closing inventory 500 units.

Required
a. Draw up a cost of production and sales statement
b. Prepare a statement of financial performance

5. You have been appointed as accounts assistant at Africa Products Ltd. Africa Products Ltd
maintains an integrated accounting system. You were given the following information:
seventy-five per cent of the purchases are on credit, 40 per cent of the sales are on cash and
30 per cent of the wages are general wages.
R
Raw materials purchased 750 000
Raw materials issued to production 500 000
Wages paid 250 000
Factory overheads incurred 100 000
Factory overheads absorbed in production 125 000
Selling and distribution expenses 50 000
Sales 950 000
Received from debtors 350 000
Paid to creditors 362 500
All the materials placed in production during the period were completed.
Record the data in the general journal.

6. Africa Products Ltd has decided to change from an integrated to an interlocking accounting
system. As their accounts assistant, you have been asked to change the records that you made
using the information given in Question 5.

Required
Make journal entries using the information in Question 5 as per an interlocking accounting system.
PART 2
Costing systems
2.1 COSTING SYSTEMS
CHAPTER 8
Job costing

CHAPTER 9
Contract costing

2.2 PROCESS COSTING


CHAPTER 10
Process costing: Single products

CHAPTER 11
Process costing: Joint products and by-products

2.3 OTHER COSTING SYSTEMS


CHAPTER 12
Standard costing

CHAPTER 13
Activity-based costing

CHAPTER 14
Direct and absorption costing systems
8 Job costing
Christo Hurter

LEARNING OUTCOMES
After studying this chapter, you should be able to:
• explain what is meant by job costing;
• explain the difference between job costing and process costing;
• understand how and why a job-costing system would be applied;
• understand cost flow and compile the source documents of a job-costing system; and
• prepare the general ledger entries to account for jobs in a job-costing system.

PRACTICAL EXAMPLE
Hot Rod Cars (HRC) manufactures custom-made vehicles. Each vehicle is made to specifications
set by the customer. In most cases, a second-hand vehicle is either purchased on behalf of or
supplied by the customer. This vehicle is then transformed by adding custom components to it.
The manager of HRC is considering a vehicle that his team has just finished working on. They
had purchased the vehicle for R60 000 and bought various custom parts to the value of R100 000.
The manager is aware, however, that other costs were incurred in producing this vehicle for the
customer. The selling price for the vehicle agreed with the customer was R200 000. The manager
is trying to calculate whether HRC made a profit or not, but is unsure how to work out the cost of
producing the vehicle.

8.1 Introduction
Profitability and financial survival are the main aims of profit-seeking businesses. Calculating the
difference between what was earned (income) and what was sacrificed (cost) supplies the
information needed by the business – be it a car manufacturer, as in the example, or a service
provider, such as a transport company (e.g. a taxi firm) – to work out its profit. It is important that the
cost of a product or service is calculated accurately so that it can be compared with or used to
establish the final selling price.
Calculating the cost of a product or service is not only done to determine profit, but also for the
valuation of inventory. When information about costs and profitability is known, managers can make
strategic decisions, develop pricing plans and meet external reporting requirements.
A costing method is therefore needed to calculate the cost of producing a unit of output. There are
various types of costing systems, and it is important to apply the system that suits the way the products
are manufactured or how the service is provided.
This chapter discusses the job-costing system. Among the different costing systems, job costing is
mostly used where different products (with different characteristics, sizes and values) are made to
customers’ specifications. A job-costing system is used in businesses such as dressmaking, furniture
manufacturing and engineering firms. Job costing is also used by some service entities, such as repair
shops and consulting firms.
In job costing, the costs for a specific job are recorded on a job card.

For every job, the job card is the control document that records the direct material used (quantity
and value) on the job, the direct labour (hours and value) and the absorbed overheads.

The method to calculate costs for the job is to use the direct material plus the direct labour plus the
manufacturing overheads. This is known as normal costing.
Very large projects, like construction contracts, for example, are normally also accounted for in a
similar way to job costing but due to the specialised nature of the industry, they are very complex.
Contract costing will be dealt with in Chapter 9.
Process costing differs from job costing, in the sense that a large number of identical products are
produced from the same process. The units are identical and therefore have the same cost. The cost
per unit is calculated by dividing the total cost for a particular time period by the number of units
produced during that period. Process costing may be used by, for example, canning factories, clothing
manufacturers and breweries. Process costing is more fully discussed in Chapters 10 and 11.
The importance of thoroughly understanding a job-costing system cannot be overemphasised. The
job-costing system forms the foundation of the recording of transactions in a manufacturing
environment. It is the basis of recording entries into the factory account, be it known as work in
process, work in progress or incomplete work. This will then lead to the valuation of finished goods.
The knowledge gained in the understanding of the double entries used in the job-costing system
provides the basis for costing systems like process and standard costing.

8.2 Application of job costing


A job-costing system may be used where costs are calculated for a single unit of production or when
small quantities with identical specifications are manufactured. A job may consist, for example, of a
single chair or a complete dining set (e.g. six chairs and a table) manufactured for a specific
customer. In the latter example, each job will require a different amount of resources.
When applied in a service entity, job costing can also be used in the same way. A job card will be
used that serves as a record for direct labour and applied overheads sacrificed for the specific
service. There are not normally direct materials in a service entity. As an example, accounting and
legal firms normally have detailed costing records where the time spent on each client is allocated to
that client’s costing records as well as any printing, photocopying or other direct costs incurred by
servicing the client.

8.3 Cost flow and source documents


In this section, cost flow and accumulation of expenses through the cost-accounting system are
discussed using examples. Figure 8.1 shows a typical manufacturing enterprise, an example of an
environment where the use of a job-costing system may be relevant. Many of the ideas in this section
have been discussed in Chapters 2 to 7, and you may want to refer back to those chapters for further
clarification.

Figure 8.1 Manufacturing firms may make use of job-costing systems

Raw material is bought by the procurement function from the supplier(s). Upon arrival, a goods-
received note (GRN) is completed and the material is stored in the raw-materials storeroom. The
stock ledger cards are updated from the goods-received note.
When required for production by way of a requisition, a stores issue voucher is issued by the
store manager and the material is moved to the production area. The job card is updated with the
material received from the store using the stores issue voucher. Here labour (calculated by the use of
clock cards) is used to transform the raw material into a product that will look different from the
materials it originates from. During this transformation process, other costs (overheads), like
electricity, are incurred in the process of transforming the raw material into the final product. The
overheads are allocated to the job using a predetermined allocation rate, as they cannot be
economically traced to specific units of production.
Once finished, the job is moved to the finished-goods storeroom until it is sold. The total cost for
the job can be calculated from the job card.

8.4 Job description and source documents used


An important feature of job costing is that before any task or job can begin, a complete job
description is required.

A job description includes customer requirements as well as technical details of the job to be
done. These details will be attached to or written on the job card to direct the production
department to complete the job exactly as required. The description may be in the form of words
and drawings, and may include technical files.

It should be remembered that costing systems tend to be specifically tailored to the needs of the entity
and the requirements of management, and differ from business to business.
In the next section, we will discuss the various transactions and their related source documents.

8.4.1 Purchase and receipt of raw materials


Let us take the example of a company named DEMO Manufacturers. Raw materials are ordered from
the suppliers, ABC Supplies. When raw material is received from the supplier at the goods-received
department of the company, a GRN is issued.
EXAMPLE 8.1: GOODS RECEIVED
One hundred units of material A at R50 each and 100 units of material B at R30 each were
received from the supplier on GRN G101.
DEMO Manufacturers
Goods-received note No: G101
Received by: DEMO Manufacturers Order no: PO 107
Received from: ABC Supplies Date: 1 March 2013
Quantity Description
100 Material A
100 Material B
Received by Date

The value of the delivery is normally not specified on the GRN. The goods-received department
will only verify the quantity and description. The value will be controlled by the creditors’
department.
The GRN is passed to the creditors’ department to inform them that the material has been
received and a liability must be recorded. When the invoice is received from the supplier, it is
matched with the corresponding order and the GRN. Reference was made on the GRN to the
order number and on the invoice referring to the GRN number.
DEMO Manufacturers
Goods-received note No: G101
Received by: DEMO Manufacturers Order no: PO 107
Received from: ABC Supplies Date: 1 March 2013
Quantity Description Rand/unit Amount
100 Material A 50,00 5 000,00
100 Material B 30,00 3 000,00

Total 8 000,00

The purchase is then recorded as shown in the general ledger below. Materials are an asset and
will be debited to the materials control account.

General ledger
Dr Materials control Cr
Creditors’ control 8 000,00
Simultaneously, the following will be recorded on the stock ledger cards:

Material A
Ref Receipts Issues Balance
Units R/u Value Units R/u Value Units R/u Value
G101 100 50,00 5 000,00 100 50,00 5 000,00

Material B
Ref Receipts Issues Balance
Units R/u Value Units R/u Value Units R/u Value
G101 100 30,00 3 000,00 100 30,00 3 000,00
Note: Take note that the total of the stock ledger cards (stock subsidiary ledger) balances with the
material control account in the general ledger.

8.4.2 Material issued


As the store manager is responsible for the material (asset) under his or her control, no stock will be
issued unless a stores requisition is received. This document indicates the job for which the material
is required. When the material is issued, the store manager will prepare a stores issue voucher, which
serves as the source document to update the records.

EXAMPLE 8.2: MATERIAL ISSUED


Ten units of material A and five units of material B were issued to Job 001 on stores issue
voucher IV101.
Job 001 is for the manufacture of one unit of final product X, which will be sold for R2
000,00 (ignoring VAT).
DEMO Manufacturers
Stores Issue Voucher No: IV101
Issued by: Materials store Requisition: RE007
Issued to: Job 001 Date: 1 March 2013
Quantity Description Rand/unit Amount
10 Material A 50,00 500,00
5 Material B 30,00 150,00

Total 650,00
The material (recorded previously as an asset) is now sent to the factory and is debited to the
work-in-process control account (asset). Control must always be kept over assets, therefore a
subsidiary ledger (job card) is used.

General ledger
Dr Materials control Cr
Work-in-process
Creditors’ control 8 000,00 650,00
control

Dr Work-in-process control Cr
Material control 650,00

Stock ledger cards


Material A
Ref Receipts Issues Balance
Units R/u Value Units R/u Value Units R/u Value
G101 100 50,00 5 000,00 100 50,00 5 000,00
IV001 10 50,00 500,00 90 50,00 4 500,00

Material B
Ref Receipts Issues Balance
Units R/u Value Units R/u Value Units R/u Value
G101 100 30,00 3 000,00 100 30,00 3 000,00
IV001 5 30,00 150,00 95 30,00 2 850,00

Job card
DEMO Manufacturers
Job card
Job 001
Description: 1 unit of product X
Customer: Finished-goods store
Date completed: 2 March
Date started: 1 March 2013
2013
Material Labour Overheads
Ref
Units R/u Value Hours Rate Value Hours Rate Value
10 50,00 500,00
5 30,00 150,00
Total 650,00
Note: The total of the stock ledger cards (stock subsidiary ledger) must balance with the material
control account in the general ledger (R7 350,00) and the total of job cards (work-in-process
subsidiary ledger) must balance with the work-in-process control account (R650,00).

8.4.3 Recording labour


Labour hours are recorded using a clock-card system (discussed in Chapter 5). In the factory there is
another clock-card system that is used for timekeeping of labour hours spent on a specific job. The
number of hours worked will now be traced to the job.

EXAMPLE 8.3: Recording labour


Employee Steven worked four hours on job 001. The recovery rate for labour is R45,00.

General ledger
Dr Work-in-process control Cr
Material control 650,00
Labour control 180,00

Dr Labour control Cr
Work-in-process
180,00
control

Job card
DEMO Manufacturers
Job card
Job 001
Description: 1 unit of product X
Customer: Finished-goods store
Date completed: 2
Date started: 1 March 2013
March 2013
Material Labour Overheads
Ref
Units R/u Value Hours Rate Value Hours Rate Value
10 50,00 500,00 4 45,00 180,00
5 30,00 150,00
Total 650,00 180,00

Note: Take note that the total of job cards (work-in-process subsidiary ledger) balances with the
work-in-process control account (R830,00).

8.4.4 Allocation of overheads


Overheads are applied to a job using a predetermined overhead rate (discussed in Chapter 6). No
source document is used. Most often, overheads will be applied at labour hours, although other
methods, like machine hours, are also used. The method used depends on the cost driver (discussed in
Chapter 2).

EXAMPLE 8.4 APPLYING OVERHEADS


Overheads are applied to job 001 at R20,00 per labour hour.

General ledger
Dr Work-in-process control Cr
Material control 650,00
Labour control 180,00
Overheads control 80,00

Dr Overheads control Cr
Work-in-process
Material control 650,00 80,00
control

Job card
DEMO Manufacturers
Job card
Job 001
Description: 1 unit of product X
Customer: Finished-goods store
Date completed: 2
Date started: 1 March 2013
March 2013
Material Labour Overheads
Ref
Units R/u Value Hours Rate Value Hours Rate Value
10 50,00 500,00 4 45,00 180,00 4 20,00 80,00
5 30,00 150,00
Total 650,00 180,00 80,00

Note: Take note that the total of the job card (work-in-process subsidiary ledger) balances with
the work-in-process control account (R910,00).

Chapter 2 discussed that the total cost of a product using normal costing is the total of direct material,
direct labour and applied overheads. The job card, together with the work-in-process control
account, is the accounting way to adhere to this principle.

8.4.5 Transfer to finished goods


Once the job is finished, it is moved to the finished-goods store. To keep control over these assets, a
stock ledger card system is used in the finished-goods storeroom. The job card is now used as a
source document, as it is removed from the work-in-process subsidiary ledger.

EXAMPLE 8.5: FINISHED GOODS


Job 001 is finished and moved to the finished-goods storeroom.

General ledger
Dr Work-in-process control Cr
Material control 650,00 Finished-goods control 910,00
Labour control 180,00
Overheads control 180,00

Dr Finished-goods control Cr
Work-in-process control 910,00

Job card
DEMO Manufacturers
Job card
Job 001
Description: 1 unit of product X
Customer: Finished goods store
Date completed: 2
Date started: 1 March 2013
March 2013
Material Labour Overheads
Ref
Units R/u Value Hours Rate Value Hours Rate Value
10 50,00 500,00 4 45,00 180,00 4 20,00 80,00
5 30,00 150,00
Total 650,00 180,00 80,00
Total value of job transferred to finished goods 910,00

Stock ledger cards – Finished goods


Final Product X
Ref Receipts Issues Balance
Units R/u Value Units R/u Value Units R/u Value
Job001 1 910,00 910,00 1 910,00 910,00

Note: Take note that the total of finished goods stock ledger cards (finished goods subsidiary
ledger) balances with the finished goods control (R910,00).

8.4.6 Sale of finished goods


With the sale of the finished product, the asset (finished goods) is lost and therefore must be
transferred to an expense account cost of sales. The value of the product was based on a normal
costing system (as explained in the Introduction), and on actual material, labour and applied
overheads.

EXAMPLE 8.6: SALE OF FINISHED GOODS


Final Product X is sold for R2 000,00 (ignoring VAT), and stores issue voucher FI011.
Dr Finished-goods control Cr
Work-in-process control 910,00 910,00

Dr Cost of sales Cr
Finished-goods control 910,00

Dr Sales Cr
Debtors’ control 2 000,00

Stock ledger cards – Finished goods


Final Product X
Ref Receipts Issues Balance
Units R/u Value Units R/u Value Units R/u Value
Job001 1 910,00 910,00 1 910,00 910,00
FI001 1 910,00 910,00 0 0,00 0,00

8.4.7 Actual overheads


The applied overheads need to be compared with the actual overheads. Under- or over- allocated
overheads must be corrected in the cost of sales account. Although for the purposes of this example
we will look at the over/under recovery for a specific job, this process is normally only undertaken
on a periodic basis (e.g. annually).

EXAMPLE 8.7: APPLIED AND ACTUAL OVERHEADS


The actual overheads for job 001 were R100,00.
Dr Overheads control Cr
Bank 100,00 Work-in-process control 80,00
Cost of sales (under- 20,00
applied)

Dr Cost of sales Cr
Finished-goods control 910,00 Income statement 930,00
Overheads control 20,00

The sales and cost of sales account is transferred to the income statement at the end of the
financial period.

CLOSING PRACTICAL EXAMPLE


Hot Rod Cars (HRC) manufactures custom-made vehicles. Each vehicle is made according to
specifications set by the customer. The manager of HRC is calculating the cost of a vehicle the
company has just completed. The following costs were entered into the job card:
Purchase price of vehicle R60 000
Custom parts purchased R100 000
Labour cost R25 000
Overhead costs allocated R10 000
Cost of vehicle R195 000

HRC agreed to sell the car to the customer for R200 000. It is therefore clear that the company only
made a profit of R5 000 on the vehicle. The manager is concerned, as that figure is well below the
10 per cent gross profit margin that the entity aims for. When considering the various costs
incurred, he realises that the labour cost traced to the vehicle was higher than expected because the
second-hand vehicle they purchased was in poorer condition than they had hoped. He now
believes that they may need to consider ways to ensure that this problem does not recur. He
decides that in future, two of his mechanics will check the vehicle before they purchase it to ensure
that the labour that a job may incur can be taken into account in advance.

8.5 Summary
Job costing is a widely used costing method whereby the costs of individual projects, or jobs, are
allocated to jobs on an individual basis. This is in contrast to a process-costing system, whereby
costs are allocated to large numbers of homogeneous products.
In a job-costing system, materials, labour and overhead costs are allocated to the specific job card
throughout the process of the manufacture of a product or the rendering of a service. These costs are
also allocated to the general ledger accounts for financial-accounting purposes.
Job costing is very important in a job environment and is very closely linked to contract costing,
which will be dealt with in Chapter 9. The type of costing system that an entity decides to use must be
closely aligned with its own processes.

PRACTICAL ACTIVITY
Zelda’s is a dressmaker that specialises in wedding dresses. The firm uses a job-costing system.
On 7 May 2013, Chenesai ordered a wedding dress to be delivered on 2 July 2013. The design
was supplied by Susan, who gave Zelda a full description.
The job was started on 9 May 2013.
Materials issued from the storeroom for the job were as follows:
Stores issue voucher SI1012
Cloth A352 3 metres at R350/metre
Cloth A819 5 metres at R290/metre

Stores issue voucher SI1019


Pearls grade 1449 80 at R15 each
The labour recorded for the job was as follows:
Cutters (recovery rate – R80 per hour)
Anna 3 hours
Jenny 2 hours
Seamstress (recovery rate – R130 per hour)
Penny 2 hours
Joyce 4 hours
Overheads were applied at R50 per direct labour hour.
The dress was finished on 28 June and sent to finished goods awaiting delivery on 2 July 2013.

Required
Complete the following job card for Zelda’s in respect of the dress they made for Chenesai.
Job card
Description: ........................ Job no: ........................
Customer: ........................ Date requested: ........................
Date started: ........................ Date completed: ........................
Material Labour Overheads
Ref
Units R/u Value Hours Rate Value Hours Rate Value

Total
Total value of job transferred to finished goods

Short questions
1. Briefly describe what is meant by a job-costing system and explain how it differs from a
process-costing system.
2. The _________ is normally used to record raw materials received from the supplier:
a. Job card
b. Goods-received note
c. Delivery note
d. Stores requisition
3. Job Z115 is sold for R35 000. The job card indicates that the total cost of the job was R25
000. Give the appropriate general journal entry to record the sale and the transfer out of
finished-goods control.
4. Give three examples of industries in which job costing is typically used.

Long questions
1. a. Shadrock the Wood Man is a carpenter who manufactures bar stools and tables. He uses
a job-costing system to record his production cost. On 2 April 2013, Shadrock ordered
200 units of wood material and 150 units of iron material from Handiman Supplies on
order number 123. The order was delivered on 5 April 2013 and recorded on GRN 222.
The supplier invoice 323 was received on 6 April 2013.
i. Record the above information on the GRN, invoice and stock ledger cards below.
ii. Enter the required transactions into the ledger.

Goods-received note ........................ No: ........................


Received by: ........................ Order no: ........................
Received from: ........................ Date: ........................
Quantity Description Rand/unit Amount

Total

Supplier invoice No: ........................


Received by: ........................ GRN no: ........................
Received from: ........................ Date: ........................
Quantity Description Rand/unit Amount

Total

Material – wood
Ref Receipts Issues Balance
Units R/u Value Units R/u Value Units R/u Value

Material – iron
Ref Receipts Issues Balance
Units R/u Value Units R/u Value Units R/u Value

b. On 9 April Shadrock issues 20 units of wood and 16 units of iron to production for job
A47 on stores issue voucher 99. The bar stools to be manufactured will be kept in
finished goods until sold.
i. Record the above information on the stock ledger cards used in Question 1 a).
ii. Open your own job card for job A47 and do the necessary entries.
iii. Update your ledger accounts used in Question 1 a).
2. A company manufactures to customer order and operates a job-costing system. Job X3
remained incomplete at the end of month 4 with the following production costs incurred:
Prime costs R4,360
Overheads R2,890
The company worked on two jobs in month 5. Prime costs incurred were:
Job X3 Job X4
R R
Direct materials issued from 1 660 8 240
stores
Direct materials returned to
0 (470)
stores
Direct materials transferred
(180) 180
between the two jobs
Direct labour 720 3,690
Direct labour is paid at a rate of R9 per hour. Production overheads are absorbed at a rate of
R17,50 per direct labour hour. 10 per cent of the total production cost of each job is added in
order to recover general administration costs. Job X3 was completed in month 5 and the
customer paid the agreed sum of R13,400.
a. Prepare a profit statement for Job X3.
b. Calculate the value of work in process for Job X4 at the end of month 5.
(Source: ACCA T4 Jun 2009 Sec B Q3)

3. SKA Manufacturers uses a job-costing system. The plant has a machining department and an
assembly department. Its job-costing system has two direct cost categories (Direct materials
and direct labour) and two manufacturing overhead cost centres. The machining department
overheads are allocated to jobs based on the actual machine hours and the assembly
department overhead are allocated to jobs based on the actual direct labour costs. The 2014
budget for the plant is as follows:
Machining department Assembly department
Manufacturing overheads R2 100 000 R4 100 000
Direct-labour costs R1 300 000 R2 200 000
Direct-labour hours 80 000 250 000
Machine hours 30 000 250 000
The company is currently finalising the costing of Job S035. The following information has
been extracted from the job card:
Machining department Assembly department
Direct materials R530 000 R210 000
Direct-labour costs R150 000 R230 000
Direct-labour hours 9 200 26 000
Machine hours 3 500 25 000
The agreed selling price for the job is R2 400 000.
a. Calculate the manufacturing overhead rate for each department.
b. Calculate the profit or loss made on Job S035.

4. KLM records and manufactures CDs for local artists. Each recording is treated as a unique
job. In June 2013 the company completed all records that were in progress and in July
worked on only two recordings, JZ Rapper and Just Flutes. The following information has
been gathered for the two recordings:
JZ Rapper Just Flutes
Direct materials R150 000 R4 000
Direct labour R55 000 R36 000
Studio hours 45 25
Direct materials only comprise the material cost of manufacturing the CDs. The direct-
material cost of manufacturing one CD is R10. Recording overheads are allocated to
recordings at a rate of R250 per studio hour, and production overheads at a rate of R12 per
CD manufactured. Only the JZ Rapper CD was completed during July 2013.
a. Calculate the total cost of the JZ Rapper job.
b. Prepare the journal entry transferring the JZ Rapper job to finished goods.
c. Determine the closing balance in the work in process Control Account.

5. Mbuza and Associates is a consulting firm specialising in local government work. It employs
10 people who work directly with the clients. The average budgeted total cost to company
per employee is R550 000 per annum. Each employee is budgeted to bill 1 800 hours per
year. All professional labour costs are included in a single direct cost category and traced to
jobs on a per-hour basis. All other costs of Mbuza and Associates are included in a single
indirect cost account and are allocated to jobs using professional labour hours as the
allocation base. The budgeted indirect costs for the year is R3 500 000.
a. Calculate the budgeted direct cost per professional labour hour.
b. Calculate the budgeted indirect cost per professional labour hour.
c. Mbuza and Associates is bidding on a tender for the Buffalo City municipality, which
will require 250 professional labour hours. Prepare a cost estimate for the job.
9 Contract costing
Peter Kamala

LEARNING OUTCOMES
After studying this chapter, you should be able to:
• define contract costing;
• describe the challenges faced when accounting for large contracts;
• define the key terms used in contract costing;
• define the elements of contract costs;
• prepare contract-costing accounting records appropriately, including:
– The contract account
– The contract debtor account
– Completed contracts
– Incomplete contracts; and
• discuss the issues to consider and the methods used to account for the expected profits on
incomplete contracts.

PRACTICAL EXAMPLE
In 2010 South Africa hosted the FIFA World Cup football tournament. In preparation for the
tournament, massive construction projects were undertaken, such as building various new stadiums
and the Gautrain, Africa’s first rapid rail system with underground stations. The stadiums included
Loftus Versfeld (Pretoria), Royal Bafokeng (Rustenburg), Mangaung Stadium (Bloemfontein),
Soccer City (Johannesburg), Green Point Stadium (Cape Town) and Moses Mabhida (Durban),
and others.
Such projects cost billions of rands. Have you ever wondered how the construction companies
were able to determine the exact cost of these big projects and whether they made a profit or a
loss? To determine the cost and profit on long-term construction contracts, the construction
companies use a method known as contract costing.

9.1 Introduction
Contract costing is a type of job costing used where the job is on a large scale and has a long
duration.

Contract costing is often used for jobs that are in the form of construction or engineering projects of
significant value, such as the construction of buildings, roads, stadiums, bridges, dams, aeroplanes
and ships, which may take many months, or even years, to complete. Large consulting contracts for the
long-term development of systems or other service contracts may also be accounted for using contract
costing.
A feature of contract costing is that a formal contract is signed between the contractor and the
customer. The amount receivable by the contractor according to the agreement is referred to as the
contract price. The work may be based at the customer’s site where the contractor may erect a site
office – in the case of constructing a building for a customer, for example. Or the work may be based
at the contractor’s premises, as in the case of the construction of a plane or other item.
For most contracts, a large percentage of costs are directly traceable to specific contracts. These
costs would include both the direct costs, like site labour and materials, and costs that would
normally be treated as indirect costs, such as supervision and depreciation. Due to the size of
contracts, indirect costs can be identified with a particular contract, as a supervisor or a specific
asset will often only be used on a specific contract. Accordingly, most costs can be treated as direct
costs.
Another characteristic of contract costing is that it is often used in industries such as the
construction industry, where heavy construction equipment, such as cranes, earth movers and tractors,
are used. The cost of this equipment can make up a large portion of the overall contract costs. If such
equipment is leased, then the leasing fees are charged directly to the contract. If they are owned, their
costs (depreciation and maintenance) can be allocated to the contract. Accordingly, most contracts
have low indirect costs, which would typically include an allocation of head office charges to
different contracts.

9.2 Challenges encountered in contract costing


Contracts can be very large and complex, and therefore the entities that are parties to a contract need
to spend a great deal of resources managing these contracts. Accounting, both from a financial-
accounting and cost-accounting perspective, also gives rise to challenges. Some of these challenges
are as follows:
• Difficulties in the control of costs. Due to the sheer size of some large contracts, and the fact that
some sites are remotely located (for example, in the construction of a nuclear power plant or a
bridge), it may be difficult to control material usage, wastage, pilferage, loss of tools and damage
to plant and equipment.
• Challenges in dividing profit between different accounting periods. Often a construction contract
takes several accounting periods to be completed. This means that an estimate must be made of the
interim profit earned on an incomplete contract at the end of each accounting period to avoid
excess fluctuation in reported profits. A contractor should estimate the interim profit
conservatively to allow for possible defects that may arise in the future. This is done by
transferring part of the interim profit to an account called reserve for contingencies. In short,
contract costing often uses financial-accounting rules to some extent to determine how contracts
are accounted for.

9.3 Definitions of key terms specific to contract costing

9.3.1 Certified work


It is common practice in large contracts for a client to make progress payments as the contract
proceeds, as opposed to a single payment on completion of the work. Such payments are made
according to certificates issued by a qualified professional, such as the client’s supervising engineer,
a quantity surveyor or an architect, who estimates the value of work done at specific stages of
completion.

Work for which a certificate has been issued is referred to as certified work. The certificate
specifies the contract value of approved work completed as of the date of the certificate, which is
referred to as value of work certified.

On the basis of the certificate, the contractor requests progress payments. The cost related to the value
of work certified is referred to as cost of work certified.

9.3.2 Uncertified work


The certificates of work completed are issued at time intervals agreed upon by the parties to the
contract, which is not necessarily at the end of the contractor’s accounting period. Accordingly, at the
end of a contractor’s financial accounting period, there is usually a portion of work that has
progressed since the issuing of the last certificate. This work is referred to as uncertified work.

9.3.3 Retention money


To protect themselves against defective work by contractors, it is usual for clients, based on the
contract agreement, to withhold a certain percentage of progress payments until work has been
completed in a satisfactory manner or for a specified period after the completion of the contract. The
withheld amount is known as retention money.

9.3.4 Extra work


It is common in the construction sector for a client to request a contractor to perform additional work
beyond that agreed upon in the contract price. For instance, a client might request a contractor to
install equipment in a building. Such extra work will not only increase the contractor’s contract
price, but will also result in an increase in the contract costs.

9.3.5 Material on site


At the end of each accounting period, there will often be unused material on the contract site for an
incomplete contract. The cost of this material is credited in the contract account and carried forward
to the next period. Accordingly, it is reflected as a current asset in the statement of financial position
at the end of the financial year.

9.4 Elements of contract costs


Costs incurred by contract projects can typically be classified as one of two categories, namely
contract costs and general head office costs (see Figure 9.1).

Figure 9.1 Elements of contract costs

9.4.1 Contract costs


Contract costs are those costs that are incurred in the contracts of the entity. Most costs can be
traced to specific contracts.

There are, however, times when a cost cannot be allocated to a specific project but are not head
office costs. These costs are referred to as indirect costs. Direct and indirect contract costs are
explained in the following sections.

9.4.1.1 Direct contract costs


Direct contract costs are all the costs that can be directly traced to a particular contract (cost
object) in an economically feasible way.

Because of the nature of contracts, most costs can be classified as direct costs. This is one of the
strengths of contract costing – the greater the proportion of costs that can be classified as direct costs,
the better the quality of information for financial decision-making purposes. Examples of costs
classified as direct costs include the following:
• Material purchased and delivered directly to the site of a project, and material issued to a specific
project from a central store
• Wages incurred for a specific project
• Cost of subcontractors’ work
• Costs incurred in hiring plant and equipment for a specific project
• Costs incurred in transporting plant and equipment to and from a site
• Depreciation of plant and equipment employed for a specific project
• Interest on funds borrowed to finance a specific project
• Design costs related to a specific project

Any incidental income, such as income from the sale of surplus material or disposal of plant and
equipment, may be used to offset the direct contract costs. However, setting off this income against
costs is discouraged, as the aim of costing is to provide good information to management, which is
better achieved when we keep track of incidental income and report on it separately.

9.4.1.2 Indirect contract costs


A contractor may have several projects running concurrently. In such a case, a contractor may incur
costs for the benefit of more than one project.

Costs that cannot be attributed to a specific project are regarded as indirect contract costs.

Similar to job costing, these costs are allocated to the various projects running concurrently in an
appropriate manner, such as on the basis of a fixed percentage or the number of employees assigned
to a contract. Examples of indirect contract costs may include the following:
• General warehouse costs
• General insurance for plant and materials
• General technical support that is offered to all current contracts
• Costs of processing the construction staff payroll

9.4.2 General head office costs


General head office costs are the costs that are incurred at the head office that are necessary for
the company to function, but are not project costs.

Accordingly, these costs are either allocated to the various contracts or treated as overhead costs,
similar to non-manufacturing overheads. Examples of such costs include the following:
• Head office administrative costs
• Advertising and marketing
• Research and development costs
• Depreciation of head office equipment and idle plant

9.5 Recording contract costs


In the previous section, we discussed the types of costs incurred by contract projects. We will now
discuss the method for accounting for these contracts.

9.5.1 Contract account and contract debtor account


The general objective of contract costing is to determine the profit or loss of each contract.
Accordingly, a separate ledger account is kept for each contract. Although several accounts may be
maintained in the ledger of each contract, in this chapter we will focus on two accounts:
• Contract account
• Contract debtor account

It is worth noting that some financial-accounting rules are often applied in contract costing to enable
comparability with the financial-accounting records, and because these rules are often appropriate.
One should, however, remember that a costing system needs to supply the financial information
required by the management of an entity. Therefore, a contract-costing system does not have to follow
the financial-accounting rules but can be designed according to the needs of the entity.

9.5.1.1 Contract account


Contract costs are accumulated in contract accounts. Each contract account is treated as a cost
objective (also referred to as a cost centre, which is a business unit for which costs are accounted for
separately) and is charged with the contract costs and allocated to general head office costs.
The contract account generally includes two sections, which may be accounted for in different
ways either with sub-accounts or separate ledger accounts. The first section is used to accumulate
costs that form the cost of sales attributable to the contract. The cost of sales is also referred to as
cost of work certified.
In the second section, the final profit or loss on the contract is determined by deducting the cost of
work certified (cost of sales) from the value of work certified (sales). For further clarification, refer
to Example 9.1. When complete, a contract account may have the entries listed in Table 9.1:

Table 9.1 Contract account


Debit Credit
First section
Direct costs:
• Direct material purchased for a contract
• Direct material transferred in
• Direct labour
• Plant hire
• Subcontractor work
• Architects’/engineers’ fees
• Cost of plant bought
• Book value of plant transferred in Value of plant transferred out
Indirect costs: Cost of material transferred out
• Allocated warehouse costs Closing cost of sales to compare with sales value
• Allocated general insurance
• Allocated transport costs
Head office charges:
• Allocated rent
• Allocated administrative expenses
• Allocated selling costs
• Allocated depreciation/idle plant
Second section
Sales value of completed contract (contract debtor
Cost of sales
account)
Final profit closed off to the profit/loss account

9.5.1.2 Contract debtor account


This is like any other debtor account, and therefore it is debited by the sales value of work certified
and credited when the debtor makes a payment.

Table 9.2 Contract debtor account


Debit Credit
Sales value of completed contract Amount received from debtor (bank)
(contract account) Amount outstanding balance c/d
Note: Balance c/d is the balance carried down to the next period.

9.5.2 Completed contracts


Some contracts can be started and completed in the same accounting period. For such contracts,
calculating profit is straightforward, as it is simply the contract price minus the cost of the contract
(direct costs + allocated indirect costs). For completed contracts, there is no uncertainty about a
project’s profitability, since all costs and income are brought into account in a single period.
Therefore no adjustments are required at the end of the year, as the risk for recognising profits that
will not realise is eliminated.

EXAMPLE 9.1: COMPLETED CONTRACT


Jose Konzolo Construction Ltd completed the construction of a new building for J.S. Vohra
during the accounting period ended 31 December 2013. Further details in respect of the project
are as follows:
Details R
Material bought on credit for contract 200 000
Materials transferred from storeroom 120 000
Materials returned to supplier 20 000
Materials returned to storeroom 24 000
Wages paid 250 000
Direct overheads 30 000
Machinery transferred to site (01/01/2013) 250 000
Machinery transferred to another contract PK/4 200 000
(31/12/2013) 1 000 000
Value of work certified 920 000
Cash received from J.S. Vohra
Required
Draw up the contract account and contract debtor (J.S. Vohra) account for the year ended 31
December 2013, and determine the profit/(loss) on the contract.

Solution
Contract account: J.S. Vohra
R R
Accounts payable Accounts payable
200 000 20 000
(material) (materials returned)
Material control Material control
120 000 24 000
(issued) (returned)
Wages control (wages
250 000 Contract PK/4 200 000
paid)
Cost of sales (balance
Machinery (to site) 250 000 606 000
c/d)
Direct overheads 30 000
850 000 850 000
Balance b/d cost of Contract debtor (sales
606 000 1 000 000
sales value)
Profit and loss 394 000
1 000 000 1 000 000

Contract debtor account: J.S. Vohra


R R
Contract account (sales
1 000 000 Bank (amount received) 920 000
value)
Balance c/d 80 000
1 000 000 1 000 000

9.5.3 Incomplete contracts


Large contracts often take several years to complete. As a result, a large project might be incomplete
by the end of an accounting period. A company does not have to wait until the contract is completed,
however, to recognise profit on the contract, and it could be argued that it should forecast the
expected profits throughout the process to ensure that costs can be controlled. However, the profit
should normally only be recognised if the contract is in a reasonable state of completion. This is
especially true for financial accounting, but may be relevant for cost-accounting purposes as well.
Most of the entries in the contract account will be as indicated in Example 9.1 for completed
contracts, except that there will be a balance in the account for the following:
• Machinery and tools on site
• Material on site
• Uncertified work (work in process), which should be reflected as a current asset in the statement
of financial position under inventory
• Accrued and prepaid expenses

EXAMPLE 9.2: INCOMPLETE CONTRACT


Nancy Gee Construction Ltd started a long-term contract to build a factory in Cape Town on 1
March 2012, which was expected to be completed on 30 June 2013. The contract price was R2,8
million and had reached a stage at which profit could be recognised. The following information
relates to the year ended 28 February 2013:
Details R
Material issued to site from store 1 200 000
Materials returned to store 100 000
Materials remaining on site at 28 February 2013 40 000
Wages paid 500 000
Wages accrued 60 000
Subcontractors’ charges paid 50 000
Plant bought on credit and delivered to site for contract work 200 000
Value of plant on site at 28 February 2013 120 000
Overheads allocated to contract 50 000
Certified work at 28 February 2013 2 400 000
Cost of work not yet certified by architect 75 000
Cash received from client in respect of certified work 2 000 000
Required
1. Prepare the contract account in the ledger of Nancy Gee Ltd, showing the profit or loss on the
contract for the accounting period ended 28 February 2013.
2. Prepare the contract debtor account in the ledger of Nancy Gee Ltd, showing the amount
owed by the debtor on 28 February 2013.
3. Prepare an extract of the statement of financial position to show how the balances should
appear as of 28 February 2013.

Solution
1. Contract account in the books of Nancy Gee Ltd
Contract account
R R
1 200
Materials control (material issued) Materials control (returned) 100 000
000
Wages control (wages paid) 500 000 Balances: c/d
Creditors (plant purchased) 200 000 Materials on site 40 000
Overheads (allocated) 50 000 Plant (value at site) 120 000
Balance c/d: accrued wages 60 000 Uncertified work 75 000
1 725
Subcontractors’ work 50 000 Cost of sales (balance c/d)
000
2 060 2 060
000 000
1 725 2 400
Balance b/d cost of sales Contract debtor (sales value)
000 000
Profit and loss 675 000
2 400 2 400
000 000
Balances: b/d Balances: b/d
Materials on site 40 000 Accrued wages 60 000
Plant (value at site) 120 000
Uncertified work 75 000
2. Contract debtor account in the books of Nancy Gee Ltd
Contract debtor account
R R
2 400 2 000
Contract account (sales value) Bank (amount received)
000 000
Balance c/d 400 000
2 400 2 400
000 000
3. Extract of the statement of financial position as at 28 February 2013
R
Non-current assets:
120 000
Property, plant and equipment
Current assets:
Inventory: 40 000
Material on hand 75 000
Work in process (uncertified work)
Receivables 400 000
Current liabilities:
Wages accrued 60 000

As explained in Section 9.3.3, it is common, if provided for in an up-front agreement, for clients to
withhold a certain percentage of progress payments (retention money) until work has been completed
in a satisfactory manner. Example 9.3 illustrates the treatment of retention money.
EXAMPLE 9.3: INCOMPLETE CONTRACTS WITH RETENTION
MONEY WITHHELD
Fantamansions Ltd, which builds state-of-the-art houses, works with well-known architects and
interior designers to meet the particular needs of its demanding, wealthy clients. On 1 July 2012,
the company undertook its most ambitious project to date to construct a mega mansion for Paul
Green, which was expected to take at least two years to complete. The contract entitled Mr Green
to a retention amount of 10 per cent of the work certified by an architect. On 28 February 2013,
the end of the company’s accounting period, details of this contract were as follows:
Details R (000s)
Materials purchased from a creditor and delivered to site 2 300
Materials remaining on site at 28 February 2013 200
Wages paid 1 750
Wages accrued 150
Plant delivered to site for contract work 500
Value of plant on site at 28 February 2013 375
Direct expenses incurred 225
Hire of crane 300
Quantity surveyor’s fees 300
Allocated overheads 40
General head office overheads apportioned to contract 350
Certified work at 28 February 2013 5 950
Extra work certified at 28 February 2013 50
Cost of work not yet certified by architect 175
Cash received from client in respect of certified work ?

Required
1. Prepare the contract account in the ledger of Fantamansions Ltd, showing the profit or loss on
the contract for the accounting period ended 28 February 2013.
2. Prepare the contract debtor account in the ledger of Fantamansions Ltd, showing the amount
owed by the debtor on 28 February 2013.
3. Prepare an extract of the statement of financial position to show how the balances should
appear as at 28 February 2013.

Solution
1. Contract account: Paul Green
R (000s) R (000s)
Creditor (material) 2 300 Balances: c/d
Wages control (wages paid) 1 750 Materials on site 200
Plant 500 Plant (value at site) 375
Direct expenses incurred 225 Uncertified work 175
Balance c/d: accrued wages 150 Cost of sales (balance c/d) 5 165
Hire of crane 300
Quantity surveyor’s fees 300
Allocated overheads 40
Head office overheads 350
5 915 5 915
Balance b/d cost of sales 5 165 Contract debtor (sales value) 5 950
Profit and loss 835 Contract debtor (extra work) 50
6 000 6 000
Balances: b/d Balances: b/d
Materials on site 200 Accrued wages 150
Plant (value at site) 375
Uncertified work 175
2. Contract debtor account, Paul Green
R (000s) R (000s)
Contract account (sales value) 5 950 Bank (amount received) 5 4001
Contract account (extra work) 50 Balance c/d 600
6 000 6 000
Note 1: 10% of work certified is the retention amount. Therefore, 90% × 6 000 = R5 400.

3. Extract of the statement of financial position as at 28 February 2013


R (000s) R (000s)
Non-current assets: 375
Property, plant and equipment 375
Current assets:
Inventory: 200
Material on hand 175
Work in process (uncertified work)
Receivables 600
Current liabilities:
150 150
Wages accrued

9.6 Recognising interim profit on incomplete contracts


As discussed, contract costing can be complex, especially when projects are incomplete. Accounting
in a cost-accounting environment for these costs and calculating the expected profit of a contract can
present major challenges. Although cost accounting has no rules governing its procedure, some
financial-accounting rules are often borrowed to allow cost accountants to determine the expected
profits of a project when it is still in progress.
Theoretically, as in the above examples, the interim profit (sales value of work certified – cost of
work certified) was fully recognised. In practice, however, this would be imprudent, as there are
many unforeseen risks and uncertainty with regard to incomplete contracts that could easily result in
additional costs. Therefore, a contractor will often not recognise all the profit on an incomplete
contract but rather set aside part of the interim profit to provide for possible future defects that he
could be held liable for, according to the prudence concept. The amount set aside is transferred to a
reserve or provision for contingencies account.
In financial accounting, according to the prudence concept, a contractor does not have to wait until
a contract is completed in order to recognise profit, as some contracts may take several years to
complete. They can recognise profits on a contract provided the outcome of such a contract can be
assessed with reasonable certainty before its conclusion (refer to International Accounting Standard
11 – IAS 11). Where the outcome of a long-term contract cannot be assessed with reasonable
certainty before the conclusion of the contract, no profit should be recognised. By contrast, if it is
expected that there will be a loss on a contract as a whole, the full amount of the loss should be
recognised as soon as it is foreseen (i.e. immediately) and reflected as such in the profit and loss
account.
These financial accounting rules will often be followed by the persons preparing cost accounts. If
a cost accountant decides to follow the financial-accounting rules in recognising the interim profit on
incomplete contracts, the following steps should be followed:
• Step 1: Calculate estimated profit or loss expected on each contract. If a loss is expected on the
contract, then recognise the full loss immediately.
• Step 2: Calculate the percentage of completion on each of the contracts.
• Step 3: Apply the guidelines or principles for recognising interim profit.

The guidelines discussed in the next section are provided here for continuity purposes.
• If a project is less than 30 per cent complete, no profit is normally recognised.
• If a project is between 30 and 80 per cent complete, normally use the work certified method.
• If a project is over 80 per cent complete, normally use the percentage of completion method.

9.6.1 Guidelines or principles for estimating interim profit to be


recognised on incomplete contracts
When estimating the interim profit to recognise on an incomplete contract, the overriding factor is the
percentage of completion of the contract. This is usually determined by expressing the cost to date of
a contract as a percentage of total estimated costs to completion.

Once the percentage of completion is determined, one of three options for estimating the interim profit
may be used, often depending on the percentage calculated. This is done in accordance with the
concept of prudence, according to which profits may be underestimated, but not the losses (IAS 11).
According to this principle, the closer a project is to completion, the more profit can be recognised,
as the risks reduce. Therefore, for a project that is less than 30 per cent complete, no profit should
normally be recognised. For a project that is between 30 and 80 per cent complete a reasonable
amount of profit is normally recognised. For a project that is more than 80 per cent complete, a larger
amount of profit may be recognised.
The discussion that follows for cost-accounting purposes should be treated as guidelines that
could be used as three methods for accounting for incomplete contracts.

9.6.1.1 For a project that is less than 30 per cent complete


Where the percentage of completion is less than 30 per cent, no profit should normally be recognised,
as the contract is still at a premature stage. Therefore, the risks are great and the result of the project
cannot be determined.

9.6.1.2 For a project that is between 30 and 80 per cent complete


(substantial work has been done)
If a contract’s percentage of completion is between 30 and 80 per cent, and it is reasonably certain
that profit will be earned, the following formula may be used to estimate the interim profit:

Notional profit = value of work certified – cost of work certified

This method is also referred to as the work certified method. It continues to allow for the risks still
inherent in a partially completed project and therefore remains prudent.

9.6.1.3 For a project that is more than 80 per cent complete


For a contract that is nearing completion (i.e. more than 80 per cent complete) and it is reasonably
certain that profit will be earned, the level of prudence is lowered. Accordingly, the following
formula may be used:

Estimated total profit on completion = contract price – (actual cost to date + estimated costs to
complete)

This method is referred to as the percentage of completion method. These principles of estimating
profit on incomplete contracts are illustrated by means of Example 9.4.

EXAMPLE 9.4: ESTIMATING INTERIM PROFIT ON AN INCOMPLETE


CONTRACT
Gianlucca Construction Ltd had four contracts with different customers running concurrently on
31 December 2012. Details regarding these contracts are provided below:
W X Y Z
R R R R
1 000 1 800 1 000
Contract price 000 800 000 000 000
Cost to date 600 000 320 000 400 000 520 000
1 000
Estimated costs to completion 100 000 500 000 280 000
000
Work certified to date 500 000 300 000 400 000 450 000
Progress payments received to date 440 000 180 000 300 000 400 000
Cost of work certified 480 000 200 000 350 000 425 000
The incomplete contracts were to be completed by 31 December 2013.

Required
Determine whether a profit or loss is expected on each of the contracts.
• Calculate the percentage of completion on each of the contracts.
• Apply the principles for recognising interim profits for each of the contracts and calculate the
profit or loss to be recognised in the current year by Gianlucca Construction Ltd for the year
ended 31 December 2013.

Solution
The following steps are recommended when answering this question:
Step 1: Calculate estimated profit or loss expected on each contract.
Determine whether a profit or loss is expected on each contract. If a loss is expected on the
contract, then recognise the full loss immediately. Therefore, in the case of a loss, no further
calculation is required. In the case of an expected profit, proceed to step 2.
W X Y Z
R R R R
Contract price 1 000 000 800 000 1 800 000 1 000 000
Total expected
(700 000) (820 000) (1 400 000) (800 000)
cost
Cost to date 600 000 320 000 400 000 520 000
Estimated costs to
100 000 500 000 1 000 000 280 000
completion
Expected profit 300 000 (20 000) 400 000 200 000
For project X, a loss of R20 000 should be recognised immediately in conformity with the
prudence concept.
Step 2: Calculate the percentage of completion on each of the contracts.
Determine the percentage or stage of completion of the contract:

W Y Z
R R R
Percentage of
completion =

Percentage 85,71% 28,57% 65%


Step 3: Apply the principles for recognising interim profits, and calculate the profit to be
recognised for each contract.

Contract Y
If a contract is less than 30 per cent complete, do not recognise profit. Therefore, no profit is
recognised for contract Y, as it is 28,57 per cent complete.

Contract Z
If the contract is between 30 and 80 per cent complete, we should recognise profit using the work
certified method. Accordingly, the interim profit of project Z, which is 65 per cent complete,
should be recognised using this method, as follows:

Notional profit = value of work certified – cost of work certified


Notional profit = R450 000 – R425 000 = R25 000

Contract W
If the contract is over 80 per cent complete, recognise profit using the percentage of completion
method. Accordingly, the interim profit of project W, which is 85,71 per cent complete, should be
recognised using this method, as follows:

Estimated total profit on completion = contract price – (actual cost to date + estimated costs
to complete)
Estimated total profit on completion = R1 000 000 – (R600 000 + R100 000)
Estimated total profit on completion = R300 000

Having looked at the various contract costing concepts and related examples, we now look at a
comprehensive example to bring all these concepts together.

EXAMPLE 9.5: COMPREHENSIVE EXAMPLE OF CONTRACT


COSTING
Imtiaz Loghdey (Pty) Ltd, a construction company, specialises in the construction of bridges in the
Western Cape. SANRAL, the national road agency, awarded the construction of a footbridge over
the N1 highway in Woodstock, Cape Town, to this company. The company began construction
during the financial year ending 30 April 2011.
The following information was made available for Contract Account Bridge CB/02/2010:
R
Original contract price 4 200 000
Total estimated costs 3 120 000
Cash received (progress payments) until 30 April 2011 2 000 000
Work certified until 30 April 2011 3 250 000
Work completed, but not yet certified -
Material purchased during the year 1 635 000
Material sold at the site – at selling price (see also note 1) 216 500
Material on hand at 30 April 2011 432 000
Labour costs paid for the year 1 200 000
Direct overheads (excluding depreciation) (see also note 3) 90 000

Notes
1. The cost price of the material sold was R192 000.
2. Interest was paid on a loan of R1 500 000 that was granted on 1 October 2010 to finance the
building cost of the project. The interest was paid on 30 April 2011. Interest was charged at
a rate of 20 per cent per annum.
3. On 1 July 2010, machinery to the value of R575 000 was purchased and transferred to this
contract. The life expectancy of this machinery is five years, when it will have an estimated
residual value of R90 000. This machinery was transferred to another contract (Bridge
CB/04/2010) on 31 December 2010, where it was used until 30 April 2011.
4. No reserve/provision for contingencies must be provided on this contract, as the contract is
substantially completed.
5. The work certified is payable on the issue of the certificate, subject to retention money of 10
per cent for a period of three months, until all the stipulations of the agreement are met.

Required
1. Prepare the contract account CB/02/2010 and determine whether a profit or a loss is
expected on the contract for the year ended 30 April 2011.
2. Calculate the percentage of completion of the contract.
3. Apply the principles of recognising interim profits and calculate the profits to be recognised
by Imtiaz Loghdey (Pty) Ltd in the year ended 30 April 2011 (round off to the nearest whole
number).
4. Prepare the contract debtor account (SANRAL) CB/02/2010 for the financial year ended 30
April 2011.

Solution
1. Prepare the contract account CB/02/2010 and determine whether a profit or a loss is
expected on the contract for the year ended 30 April 2011.
Contract account: N1 bridge CB/02/2010
R R
Machinery (to site) 575 000 CB/04/2010 526 500
Creditors (material) 1 635 000 Material disposal 192 000
Bank (wages paid) 1 200 000 Balance c/d
Direct overheads 90 000 (material on hand) 432 000
Bank (interest paid) 175 000 Cost of goods sold c/d 2 524 500
3 675 000 3 675 000
Contract debtor
Cost of goods sold b/d 2 524 500 CB/02/20x10 3 250 000
(SANRAL)
Profit and loss 725 500

3 250 000 3 250 000


Balance b/d
(material on hand) 432 000

A profit of R725 500 is expected for the year ended 30 April 2011.
2. Calculate the percentage of completion of the contract.

3. Apply the principles of recognising interim profits and calculate the profits to be recognised
by Imtiaz Loghdey (Pty) Ltd in the year ended 30 April 2011.
The contract is nearing completion, as it is more than 80 per cent complete. Therefore, the
percentage of completion method is used.

Estimated total profit on completion =


contract price – (actual cost to date + estimated costs to complete).

Estimated total profit on completion = 4 200 000 – (2 524 500 + 595 500) = R1 080 000
Note: 3 120 000 – 2 524 500 = 595 500.
Therefore, the contract account will now appear as follows:
Contract account: N1 bridge CB/02/2010
R R
Machinery (to site) 575 000 CB/04/2010 526 500
Creditors (material) 1 635 000 Material disposal 192 000
Bank (wages paid) 1 200 000 Balance c/d
Direct overheads 90 000 (material on hand) 432 000
Bank (interest paid) 175 000 Cost of goods sold c/d 2 524 500
3 675 000 3 675 000
Contract debtor
Cost of goods sold b/d 2 524 500 3 250 000
CB/02/2010
Profit and loss 514 286 (SANRAL)
Reserve 211 214
3 250 000 3 250 000
Balance b/d
(material on hand) 432 000
4. Prepare the contract debtor account (SANRAL) CB/02/2010 for the financial year ended 30
April 2011.
Contract debtor account (SANRAL) CB/02/2010 (N1 bridge)
R R
N1 bridge CB02/2010 3 250 000 Bank (amount received) 2 925 000
(work certified) Balance c/d 325 000
(10% retention money)
3 250 000 3 250 000

CLOSING PRACTICAL EXAMPLE


In 2010 South Africa hosted the FIFA World Cup tournament, which necessitated several major
construction projects. The construction companies used the contract-costing methods discussed in
this chapter to account for the billions of rands of costs incurred by these projects. Key among
these projects was the construction of the Green Point Stadium (now known as the Cape Town
Stadium), which took two years and nine months to complete.
The stadium, which was constructed by South African construction contractor Murray and
Roberts, was completed at a cost of 4,4 billion rands. The major costs were materials, labour,
subcontractors’ fees, depreciation of equipment and bank interest, all of which were debited in the
contract account. Any material remaining, returned to the stores, or disposed were credited to the
contract account. The balance in the contract account was then allocated to the cost of goods sold,
which was carried down to compute the profit. To determine the income, the architect’s certificate
was used to provide the value of work certified. The profit was then determined by subtracting the
cost of goods sold (cost of work certified) from the value of work certified. Since the contract was
not completed in one year, the profit recognised in the first year was calculated using a percentage
of completion method. The remaining unrecognised profits would then be sent to a reserve. The
contract debtor account was then prepared by debiting the account by the value of work certified
and crediting it by any amount received from the client (the FIFA organising committee). Any
percentage retained by the client would also be credited in the contract debtor account. This
process was repeated in the second and third years, when the contract was completed.
9.7 Summary
Contract costing is commonly used for long-term contracts, such as the construction of buildings,
aeroplanes, machines and ships. In contract costing, all costs are traced to specific contracts and
accounted for over the life of the project. The contract income can then be offset against the contract
costs to determine whether a profit or loss was made by the contract.
Due to the unique nature of each contract, most contract costs can be classified as direct costs.
Indirect costs and head office costs incurred are allocated to the contracts using various criteria, such
as a fixed percentage, or based on the number of employees assigned to a contract. Each contract is
accounted for separately. Although some contracts can be started and completed within the same
financial year, most large contracts are by their nature long term, and may take several years to
complete. This means that the date on which they are started and the date on which they are completed
often fall within different financial years, which creates a problem of matching the revenue to the
related costs.
Given the need to determine interim profit on incomplete contracts, companies employ three
methods to estimate the profit to be recognised in line with the prudence concept, based on cost to
date as a percentage of the total estimated cost of a contract. Where a contract is less than 30 per cent
complete, no profit is recognised. For contracts that are between 30 and 80 per cent complete, the
work-certified method is used to determine the profit to be recognised. For contracts that are over 80
per cent complete, the percentage of completion method is used, a less conservative method that can
be applied as the contract nears completion. Despite these rules, cost and management accountants
need to report the full picture (regardless of matching, prudence, etc.) to make sure that management
have all the information they need.

Short questions
1. What is meant by notional profit?
2. How are accounts prepared when a contract lasts for more than a year?
3. What is work in process?
4. What is uncertified work?

Long questions
1. On 31 December 2009, Marigold Builders supplied the following information regarding a
building contract for a client, Limpopo Education Department:
Details R
Material delivered to the contract site 433 624
Material issued to site from store 5 256
Material transferred to another site in Gauteng 12 720
Excess material sold to the public (cost price
46 000
R23 000)
Machinery delivered to site 100 000
Machinery transferred to site from North West
42 000
Province
Site wages paid 337 240
Wages accrued on 31 December 2009 7 360
Work done by subcontractor 156 640
Sundry site expenses 40 608
Sundry site expenses prepaid for 2010 as at 31
4 056
December 2009
Work not yet certified 57 692
Head-office charges (calculated at 10% of cost
?
of material used)
Overheads charged at 5% of site wages ?
Value of machinery on site at 31 December
118 000
2009
Value of material on site at 31 December 2009 146 400
Value of work certified 940 000
Cash received from Limpopo Education
920 000
Department
a. Prepare a contract account in the ledger of Marigold Builders at 31 December 2009.
Using the value of work certified, show the profit to be taken by the contractor and the
reserve for contingencies for the period.
b. Prepare the ledger account for Limpopo Education Department (client account) in the
books of Marigold Builders.
c. Prepare a balance sheet for Marigold Builders, only showing the entries applicable to the
building contract.
d. Round up all calculations to the nearest R1 where applicable.

2. Mambo Contractors Ltd started working on a contract during the course of the year. By the
end of the year, the following items had been charged to the contract account:
R
Material issued 8 000
Wages paid 11 200
Plant purchased for contract 4 000
Work done by subcontractors 1 800
Other site expenses 600
Hire of special machinery 800
Work certified by architect 26 000
Work not yet certified 2 000
Progress payments received from client 25 200
Value of plant at end of year 2 800
Value of unused material at end of year 1 600
Contract price 2 000 000
a. Prepare the contract account for Mambo Contractors Ltd for the year end if the profit to
be taken by the contractor is calculated using the work certified method.
b. Prepare the client account for the same period.
c. Show the relevant entries in the balance sheet of Mambo Contractors for the year end.
d. If the estimated cost to complete the contract is R150 000, calculate the profit to be taken
by the contractor using the percentage of completion method.

3. Canyon Ltd started working on a contract on 1 April 2006. The contract price was R2 000
000 and the target completion date was 31 May 2007. It was agreed with the client that the
retention amount would be 4 per cent of the work certified by the architect. At 28 February
2007 (the end of the company’s accounting period), the cost accountant collected the
following information regarding this contract:
R
Material sent to site 700 000
Material returned to the storeroom 80 000
Plant sent to site 1 000 000
Material transferred to another contract 40 000
Material on site at 28 February 2007 75 000
Plant hire charges 200 000
Labour cost incurred 300 000
Overhead costs 75 000
Direct expenses incurred 25 000
Sales value of work certified by architect 1 500 000
Cost of work not yet certified 160 000
Cash received from client 1 440 000
Depreciation is charged on plant at a rate of 12% p.a.
Prepare a contract account in which you clearly indicate which amount should reasonably
be transferred to the profit and loss account at 28 February 2007. Use the work certified
method to calculate profit taken.

4. Best Build Contractors have been commissioned to build a high rise by DTU Bank. It is
estimated that the building will take three years to complete. The contract price has been set
at R9 000 000 and no extra work is anticipated. You are provided with the following
information for the financial year ending 28 February 2009:
R
Material delivered to site by Fed Building
100 000
Supplies
Material transferred to site from another site in
50 000
Gauteng
Material delivered to site from storeroom 20 000
Material returned to storeroom 5 000
Excess material sold to public (cost price R4
6 000
000)
Machinery delivered to site 500 000

Machinery transferred from site back to


50 000
storeroom
Wages paid on site 400 000
Wages accrued 5 000
Architect’s fees 20 000
Head office costs apportioned at 6% of direct
?
wages
Uncertified work 30 500
Overheads allocated at 10% of cost of material
?
used
Cash received to date (amounting to 90% of
540 000
value of work certified)
Value of work certified (to be calculated as
?
above)
Value of material on hand at 28 February 2009 30 000
Value of machinery on hand at 28 February
450 000
2009
Best Build Contractors uses the work certified method to calculate the profit to be taken at the end
of the financial year.
a. Prepare a contract account for Best Build Contractors for the financial year ended 28
February 2009.
b. Prepare a client account for the year ended 28 February 2009.
10 Process costing: Single products
Johan Hefer

LEARNING OUTCOMES
After studying this chapter, you should be able to:
• discuss the concept of process costing and distinguish it from job costing;
• explain and account for:
– expected production;
– normal losses;
– abnormal losses and gains;
– losses or gains that have a scrapping value; and
• account for incomplete production (work in process), including:
– opening work in process;
– weighted average method;
– first in first out; and
– accounting for losses in a work-in-process system.

PRACTICAL EXAMPLE
James has just returned from college, where he has studied job costing in his management-
accounting class. His father runs a chemical company that manufactures various household
chemicals on a small scale. James explained to his father how he should be accounting for his
production using a job-costing system.
His father tells him that he does not believe that a job-costing system would work in his
business environment. He mixes various chemicals in bulk then packages them into bottles. How
would he distinguish one job from another? On what basis would he allocate costs?
James wondered how you account for costs in a process environment. He was to find out
following week, when his course was due to cover process costing.

10.1 Introduction
Different industries have different cost drivers in their processes, and the underlying management-
accounting system must cater for the financial information needs of managers. There are many types of
systems used for the costing of production but, generally, costs are either allocated to production of a
specific product (i.e. on a job, batch, contract or project basis) or to processes. The product-costing
methods have been discussed in Chapters 8 and 9. This chapter and Chapter 11 discuss process
costing.
It may be useful to explain the differences between the two types of costing systems. These are
summarised in Figure 10.1. With a product (or job) system, direct materials, direct labour and
overhead costs are allocated to specific products. A product-costing system is usually used in an
environment where products are unlike in nature, or heterogeneous. Some examples are engineering
firms, construction companies and any business that manufactures specialised products to order. In the
service industry, examples of this type of system are the consulting environment, like attorneys and
accountants, or cleaning services, where direct labour costs and overheads can be allocated to
specific projects or clients.

Figure 10.1 Comparison of job and process costing

In a process system, on the other hand, direct materials, direct labour and manufacturing overheads
are allocated to a specific process. This system is often used where products are alike or similar in
nature (i.e. homogeneous) and it would be almost impossible to allocate costs to specific products.
Examples of industries where process costing is often used are the chemical, textile, distillation
and mass-production industries. In the service sector, process systems may be used, for example, in
the insurance industry where the costs of call centre operators, assessors and other staff are almost
impossible to link to specific insurance contracts. Therefore, the cost of the process of providing
insurance is calculated instead.
This chapter considers the various aspects of calculating process costs and discusses the
accounting entries used in a process-costing system. Firstly, we will define and discuss process
costing.

10.2 Process costing


As discussed in the previous section and illustrated in Figure 10.1, we use process costing in
industries where homogeneous (i.e. similar) products are manufactured on a large scale. Consider a
facility that manufactures soft drinks or bars of soap, or a bakery. In a bakery, it would be illogical to
allocate costs to every single loaf of bread. In an automated environment such as a bakery, precisely
the same amount of flour and other ingredients are used for each type of the same loaf. Therefore, in
this type of production or service environment, process costing is normally used.
In process costing, the total cost (i.e. direct material plus direct labour plus overhead costs) of
producing the product over a period is allocated to the products produced for that period.

The methods used to calculate direct material, direct labour and overhead costs were discussed in
Chapters 3 to 6. In this chapter, we focus on determining the expected production, how to deal with
any unexpected gains or losses and how to calculate the unit cost of production to enable us to value
both work in process (WIP) and finished goods.
It should be remembered that process systems can be very simple (e.g. one direct material is
converted into a product, as in the case of manufacturing candles), or very complex (e.g. multiple
processes using multiple materials that are added at different points in time, as in the case of
manufacturing motor vehicles). For the purposes of explaining the principles, we will look at
reasonably simple manufacturing processes, but one needs to bear in mind that many manufacturing
processes are complex – and so are their costing. The principles, however, are the same.
We will first use a basic example to illustrate a process-costing system.

EXAMPLE 10.1
Tina owns a bakery that bakes bread on order from supermarkets, health shops and individuals.
She bakes and delivers about 300 loaves a day and does not keep any finished loaves on hand.
Tina is reviewing her monthly accounts for the previous month and has found the following:
Loaves of bread produced 6 000
Material costs R30 000
Labour costs R10 000
Overhead costs R8 000
We can therefore see that Tina’s total process cost for the period to produce 6 000 loaves was
R48 000. We can therefore calculate her cost per unit for the period to be R8 per loaf.

10.3 Expected production: Normal and abnormal losses, and


abnormal gains
As discussed in the introduction, process costs are allocated to expected production for a period. We
will now discuss how to calculate these expected outputs.
In this section, at certain times we will concern ourselves with units and at others with values.
Make sure that you know whether we are dealing with units or values at any point in this discussion.
Example 1 was a very simply one but illustrates the most important aspect of process costing, namely
that the cost per unit is calculated as follows:

As you can see, the top part of this equation is a value (the cost of inputs) and the bottom part is units.
Let us briefly discuss what is meant by ‘expected outputs in units’.

The expected outputs in units are the number of units that should be produced based on the number
of units that were put into the process.

To give an example, if Tina uses 0,5 kg of flour to produce one loaf of bread, she would expect to
produce 10 loaves of bread if she uses 5 kg of flour. Thus her expected outputs in units would be 10
loaves.
However, in reality process production is often more complex than this. A loss or wastage is
normally expected. In a bakery, some flour is bound to be lost in various ways (such as falling on the
floor). Flour is also not always equally absorbent, and slightly more or less flour may be consumed to
bake a loaf.
These losses may be classified as either normal or abnormal losses. A normal loss is an expected
loss. Tina knows that, due to wastage, she may only bake 19 loaves of bread from a 10 kg sack of
flour, although she only uses 0,5 kg per loaf. Tina’s normal loss is therefore expected to be 0,5 kg
(i.e. 10 – (19 × 0,5 kg)).
An abnormal loss is one that is higher than expected. Let us assume Tina is only able to bake 18
loaves from a 10 kg bag of flour. Her normal loss is one loaf, or 0,5 kg, as calculated in the previous
paragraph. She may, however, experience an abnormal loss of 1 loaf (or 0,5 kg), if she was only able
to bake 18 and not 19 loaves from a 10 kg bag of flour.
Tina may also find that she could bake 20 loaves from a 10 kg bag of flour and therefore she did
not incur any losses. This would be considered an abnormal gain, as it is expected that a certain
amount of loss will occur. Tina’s normal loss will remain one loaf, or 0,5 kg, but will now show an
abnormal gain of one loaf, or 0,5 kg.
It is important that you understand the terms ‘expected output/production’, ‘normal loss’,
‘abnormal loss’ and ‘abnormal gain’ for the next section of this chapter. To illustrate this further, we
will consider another simple example.

EXAMPLE 10.2
Fragrant Candles manufactures gift candles with a logo impressed on the candles. This is the only
product manufactured at the company’s Alberton factory.
Wax is received from a supplier in pellet form, melted and poured into moulds. Once the
candles have hardened, they are removed from the moulds, packed into boxes and sent to the
warehouse. A box contains 10 kg of candles. The factory expects to use 12 kg of wax to produce
every box of candles (therefore, 12 kg of wax is expected to produce 10 kg of candles).
The Alberton factory’s production manager is reviewing the week’s production. Production is
recorded and accounted for on a daily basis, as follows:
Wax input Candle output Gain/loss
(kg) (kg) (kg)
Monday 120 100 20
Tuesday 120 110 10
Wednesday 120 115 5

Thursday 120 100 20


Friday 120 90 30
Based on the above information, we can calculate the normal losses, abnormal losses and
abnormal gains.
Wax input Candle Gain/loss Normal loss Abnormal Abnormal
(kg) output (kg) (kg) (kg) loss (kg) gain (kg)
Monday 120 100 20 20 – –
Tuesday 120 110 10 20 – 10
Wednesday 120 95 25 20 5 –
Thursday 120 100 20 20 – –
Friday 120 90 30 20 10 –
As can be seen in the above table, on Monday and Thursday the units of wax put into the system
produced the expected amount of candles. On Tuesday more candles were produced than
expected (after deducting the normal loss), therefore there was an abnormal gain. On Wednesday
and Friday, however, fewer candles were produced than expected and therefore there was an
abnormal loss.

10.4 The process account


One of the main tools used to manage a process-costing system is the process account. The process
account works similarly to a general ledger account (and it may be integrated into the general ledger).
There is a separate process account for every process in a system. An example of a process account
is given in Figure 10.2, which explains the main features of the account.

Figure 10.2 Example of a process account


We will now modify the details given in Example 10.2 to illustrate the use of a process account in its
most basic form.

EXAMPLE 10.3
Continuing with the example of Fragrant Candles, the factory expects to use 10 kg of wax to
produce one box of candles (therefore 10 kg of wax is expected to produce 10 kg of candles). The
direct labour cost per day is R5 000 and the overhead costs per day are R15 000. Wax costs R40
per kg.
Production for Monday was as follows:
Wax input Candle output
Monday 100 kg 100 kg
A process account can now be drawn up for Monday’s production. Please note that the R/kg of
output was calculated as R24 000 divided by 100 kg.
Process account
Amount Amount
Description Kg R/kg Description Kg R/kg
(R) (R)
Transfer to
Material 100 40 4 000 finished 100 240* 24 000
goods
Labour 5 000
Overheads 15 000
100 24 000 100 24 000
* Note: R24 000 ÷ 100
As can be seen, using a straightforward process account is not difficult, but it is important that
you understand how it works, as we will now add to it various factors in the form of normal
losses, abnormal losses and abnormal gains, and how they should be accounted for.

10.4.1 Process account: Normal losses with no scrap value


As discussed, most production processes have a certain amount of waste. In the example of the candle
manufacturer, some wax will be lost due to spillage, evaporation and damaged stock. It is important
to determine whether or not the waste has any value. If the waste is simply lost, it has no value and
therefore will not impact on the calculation of the total costs of the process. If, however, the scrap can
be reused or sold in some way, this should be accounted for.
We will firstly deal with a situation where waste has no value, using details from Example 10.2 to
illustrate how the process account should be applied.

EXAMPLE 10.4
In Example 10.2, the production for Monday was as follows:
Candle Abnormal Abnormal
Wax input Gain/loss Normal loss
output loss gain
Monday 120 kg 100 kg 20 g 20 kg – –
The direct labour cost per day is R5 000; the overhead costs per day are R15 000; wax cost is
R40 per kg. The process account for Monday can now be completed as follows:
Process account
Amount Amount
Description Kg R/kg Description Kg R/kg
(R) (R)
Material 120 40 4 800 Normal loss 20 – –
Transfer to
Labour 5 000 finished 100 248 24 800
goods
Overheads 15 000
120 24 800 120 24 800
Remember that the formula for unit cost is total production cost ÷ expected output. Expected
output is equal to total input less normal losses. This example provides that for every 12 kg the
company expects 10 kg of production. Therefore, the expected output will be 120 kg – 20 kg =
100 kg and the unit cost will be 24 800 ÷ 100 = R248 per kg.

10.4.2 Process account: Normal losses with scrap value


As explained in the previous section, most production processes incur some loss of material.
Sometimes this material can be sold or reused. The value of these sales should therefore be brought
into account when calculating the cost of production. Therefore the unit cost will be calculated as:

The value recovered from the sale of normal losses is deducted from the process account (which is
essentially part of cost of sales). Firms normally either ‘sell’ these losses internally or sell them to a
customer who purchases the scrap material at a specified rate. Therefore, either another department
or an external party will ‘owe’ the department for the scrap material. It should be remembered that
the scrap material does not have any value if there is no customer, internal or external, to purchase it.
For the purposes of this discussion, we will assume that an outside party purchases the scrap. If an
internal department purchases the scrap and that department also uses process costing, its process
account would be debited.
You should also bear in mind that not necessarily all scrap will be sellable. If Fragrant Candles
has 20 kg of lost wax, it may only be able to recover and sell 10 kg. For the purpose of this example,
however, we assume that all of the lost materials are recoverable. In reality, though, only the
recoverable losses would be accounted for in value terms.
When selling scrap material, the accounting entries will credit the process account and debit the
‘accrued income from normal loss account’, as the cash may or may not have been received as yet. As
cash is received for the normal losses, the bank would be debited and the ‘accrued income from
normal loss account’ would be credited, making it zero. We will now use the same information as in
Example 10.4 to illustrate this situation. We will illustrate the cash or interdepartmental transfers in
Example 10.5.

EXAMPLE 10.5
The details can be found in Example 10.2. The production for Monday was as follows:
Candle Abnormal Abnormal
Wax input Gain/loss Normal loss
output loss gain
Monday 120 kg 100 kg 20 kg 20 kg
The direct labour cost per day is R5 000 and the overhead costs per day are R15 000. Wax costs
R40 per kg. Normal wax losses can be sold for R15 per kg. The process account for Monday can
now be completed as follows:
Process account
Amount Amount
Description Kg R/kg Description Kg R/kg
(R) (R)
Material 120 40 4 800 Normal loss 20 15 300
Transfer to
Labour 5 000 finished 100 245 24 500
goods
Overheads 15 000
120 24 800 120 24 800
Remember that the formula for unit cost with normal losses and a scrap value is (total production
cost less value of scrap) ÷ expected output. Expected output is equal to total input less normal
losses. Therefore, the unit cost will be (24 800 – (20 × 15)) ÷ 100 = R245 per kg.
The accrued income from normal loss account (part of accounts receivable) will look as
follows:
Accrued income from normal loss account
Amount Amount
Description Kg R/kg Description Kg R/kg
(R) (R)
Process
20 15 R300
account

10.4.3 Abnormal losses with no scrap value


In the previous section, we discussed normal losses, or, in other words, losses of materials that can
be expected in the normal manufacturing process. These are not, however, the only types of losses
incurred by a manufacturing process – there are also abnormal losses, as explained in Section 10.3.
We will first deal with abnormal losses without a scrap value and then abnormal losses with a scrap
value.
As discussed previously, abnormal losses are losses that could not have been expected and would
normally occur because of human error or an accident. Such losses may be caused by large spillages,
problems with machinery during the manufacturing process or negligence on the part of personnel. It
is best practice that abnormal losses should not be included in the normal process cost but should be
accounted for separately. This is true for both financial accounting and management accounting. The
reason for the separate accounting is that it is important to highlight these losses for management
purposes.
Abnormal losses are accounted for as a separate item in the process account and then debited to
the abnormal losses account. The value of abnormal losses is determined by multiplying the cost per
unit calculated in the same way as the cost of normal outputs, multiplied by the abnormally lost units.
We will illustrate this process with another example.

EXAMPLE 10.6
The details can be found in Example 10.2. The production for Wednesday was as follows:
Candle Abnormal Abnormal
Wax input Gain/loss Normal loss
output loss gain
Wednesday 120 kg 95 kg 25 kg 20 kg 5 kg
The direct labour cost per day is R5 000 and the overhead costs per day are R15 000. Wax costs
R40 per kg. The process account for Wednesday can now be completed as follows:
Process account
Description Kg R/kg Amount Description Kg R/kg Amount
(R) (R)
Material 120 40 4 800 Normal loss 20 – –
Abnormal
Labour 5 000 5 248 1 240
loss
Transfer to
Overheads 15 000 finished 95 248 23 560
goods
120 24 800 120 24 800
The unit cost is calculated in the same way as in Example 10.4. The example states that for every
12 kg of material, the firm expects 10 kg of output. Therefore, the expected output is 120 kg – 20
kg = 100 kg and the unit cost will be 24 800 ÷ 100 = R248 per kg.
The business would also have an abnormal loss account. This is an expense account that will
be used to monitor the amount of abnormal losses incurred. This account will be included in the
income statement of the company and will look as follows:
Abnormal loss account
Description Kg R/kg Amount Description Kg R/kg Amount
Process
5 248 R1 240
account

10.4.4 Abnormal losses with scrap value


In the previous section, we discussed the accounting treatment for abnormal losses where there is no
scrap value. We will now consider the scenario where the lost materials have a scrap value. Please
refer again to Section 10.4.2 on the scrap value of normal losses for some of the factors that need to
be considered.
When abnormal losses have a scrap value, the abnormal loss in value terms can be reduced by the
income from the sale of these items. This income will also be transferred into the accrued income
from normal losses account, as they will normally be sold. We will illustrate this situation with an
example using the information from Example 10.6, but in this case, the abnormal losses have a scrap
value.

EXAMPLE 10.7
The production for Wednesday was as follows:
Candle Abnormal Abnormal
Wax input Gain/loss Normal loss
output loss gain (kg)
Wednesday 120 kg 95 kg 25 kg 20 kg 5 kg
The direct labour cost per day is R5 000 and the overhead costs per day are R15 000. Wax costs
R40 per kg. All wax losses can be sold for R15 per kg. The process account for Wednesday can
now be completed as follows:
Process account
Amount Amount
Description Kg R/kg Description Kg R/kg
(R) (R)
Material 120 40 4 800 Normal loss 20 15 300
Abnormal
Labour 5 000 5 245 1 225
loss
Transfer to
Overheads 15 000 finished 95 245 23 275
goods
120 24 800 120 24 800
The unit cost is calculated in the same way as in Example 10.6. Therefore, the unit cost will be
(24 800 – (20 × 15)) ÷ 100 = R245 per kg.
The business would also have an abnormal loss account, which is the same as the one in
Example 10.6, except that the scrap value can be deducted.
Abnormal loss account
Amount Amount
Description Kg R/kg Description Kg R/kg
(R) (R)
Accrued
Process
5 245 R1 240 income from 5 15 R75
Account
normal loss
Finally, the accrued income from the normal loss account will look like this:
Accrued income from normal loss account
Amount
Description Kg R/kg Description Kg R/kg Amount
(R)
Process
20 15 300
account
Abnormal
5 15 75
loss

10.4.5 Abnormal gains with no scrap value


Initially, we discussed manufacturing processes where no losses of materials were incurred, then we
considered normal losses and abnormal losses. In the case of both normal and abnormal losses, we
looked at the approaches where there was a scrap value or not. The final case to consider is when
there is an abnormal gain.
An abnormal gain normally occurs when the actual loss is smaller than the normal loss, as
discussed previously. The abnormal gain is calculated by comparing the expected output (total inputs
less normal losses) to the actual output. If the actual output is more than the expected output, then there
is an abnormal gain.
Abnormal gains are accounted for by debiting (or ‘increasing’) the inputs and accounting for
normal losses as usual. The unit cost is calculated in the same way. The abnormal gain will be
credited. Note that abnormal gains and abnormal losses will be accounted for separately in the
general ledger, and not offset. We will illustrate this using the data from Example 10.2.

EXAMPLE 10.8
The details can be found in Example 10.2. Production for Tuesday was as follows:
Candle Abnormal Abnormal
Wax input Gain/loss Normal loss
output loss gain
Tuesday 120 kg 110 kg 10 kg 20 kg 10 kg
The direct labour cost per day is R5 000 and the overhead costs per day are R15 000. Wax costs
R40 per kg.
The process account for Tuesday can now be completed as follows:
Process account
Amount Amount
Description Kg R/kg Description Kg R/kg
(R) (R)
Material 120 40 4 800 Normal loss 20 - -
Transfer to
Labour 5 000 finished 110 248 27 280
goods
Overheads 15 000
Abnormal
10 248 2 480
gain
130 27 280 130 27 280
The unit cost is calculated in the same way as in Example 10.4. The example states that for every
12 kg, the firm expects 10 kg of production. Therefore, the expected output is going to be 120 kg –
20 kg = 100 kg and the unit cost will be 24 800 ÷ 100 = R248 per kg.
The business would also have an abnormal gain account. This is an income account that will
be used to monitor the amount of abnormal gains. This account will be included in the income
statement of the company. This account will look as follows:
Abnormal gain account
Amount
Description Kg R/kg Description Kg R/kg Amount
(R)
Process
10 248 R2 480
account
10.4.6 Abnormal gains with scrap value
The final scenario that could occur when accounting for process costs is the factor of abnormal gains
when the waste has a scrap value. This has the impact of changing the unit prices to account for the
value of normal losses and changing the accounting entries slightly. In this case, we will debit the
abnormal gain account and debit the accrued income on normal loss account, as the amount of scrap
available for sale is reduced. This is illustrated in Example 10.9.

EXAMPLE 10.9
The details can be found in Example 10.8. The production for Tuesday was as follows:
Candle Abnormal Abnormal
Wax input Gain/loss Normal loss
output loss gain
Tuesday 120 kg 110 kg 10 kg 20 kg 10 kg
The direct labour cost per day is R5 000 and the overhead costs per day are R15 000. Wax costs
R40 per kg. All wax losses can be sold for R15 per kg.
The process account for Tuesday can now be completed as follows:
Process account
Amount Amount
Description Kg R/kg Description Kg R/kg
(R) (R)
Material 120 40 4 800 Normal loss 20 15 300
Transfer to
Labour 5 000 finished 110 245 26 950
goods
Overheads 15 000
Abnormal
10 245 2 450
gain
130 27 250 130 27 250
The unit cost is calculated in the same way as shown in Example 10.5. Therefore, the unit cost
will be (24 800 – (20 × 15)) ÷ 100 = R245 per kg.
The business would also have an abnormal gain account. The units of normal gains available
for sale are reduced and therefore this reduction in units will be accounted for at resale value.
This account will look as follows:
Abnormal gain account
Description Kg R/kg Amount Description Kg R/kg Amount
Accrued
income Process
10 15 R150 10 245 R2 450
from normal account
loss
Finally, the accrued income from the normal loss account will look like this:
Accrued income from normal loss account
Description Kg R/kg Amount Description Kg R/kg Amount
Process Abnormal
20 15 R300 10 15 R150
account gain account

10.4.7 Process costing for normal and abnormal losses and abnormal
gains: Summary
In this section, we considered the various possible permutations when accounting for the units of
production in a process system. We also illustrated the accounting entries required. As always with
management accounting, the system should be tailored to the specific requirements and environment of
the entity. Each question in an exam or practical situation needs to be carefully analysed and
considered before determining the most appropriate way to calculate unit cost and account for the
process costs.
This section concludes with a comprehensive example to illustrate all the principles covered in
this section. This example involves two further possibilities: where multiple materials are used to
manufacture a product and where there is more than one process involved. Where multiple materials
are used in a process environment, one would treat the direct material in the same way as a single
direct material. When there is more than one process involved, instead of closing off the process
account to finished goods, it is closed off to the second process. If there are still more processes, this
continues until the products are transferred to finished goods.

EXAMPLE 10.10
Grace Chemicals produces a range of chemical products. The manufacturing process of one of
these products, Handy Cleaner, can be summarised as shown in Figure 10.3.

Figure 10.3 Multiple processes and multiple materials

Handy Cleaner is manufactured by means of two processes. In process A, 10 litres of material A


are combined with 15 litres of material B to manufacture 23 litres of the product of process A.
The normal loss is therefore 2 litres per 25 litres for process A. In process B, the 23 litres made
during process A are combined with 5 litres of material C to produce 27 litres of the finished
product. The normal loss for process B is therefore 1 litre per 28 litres.
The production manager has supplied the following production records for the month:
Units Cost
Process A
Material A inputs 500 litres R25 per unit
Material B input 750 litres R35 per unit
Labour costs R9 500
Overhead costs R6 000
Process A output 1 100 litres
Process B
Material C inputs 240 litres R60 per unit
Labour costs R10 000
Overhead costs R5 000
Finished goods 1 300 litres
The losses in process A can be sold for R25 per litre to an outside company; the losses in
process B have no scrap value.

Required
Complete the process accounts for the two processes.

Solution
When attempting a question like this, it is important to familiarise yourself with the question. It is
then useful to calculate the normal losses and abnormal losses in units. This can be done as
follows:
Materials Product Normal Loss Abnormal Abnormal
Loss (l)
input (l) output (l) (l) loss (l) gain (l)
100
1 250
Process A 1 100 150 (1 250 × 2 ÷ 50
(500 + 750)
25)
1 340 48
Process B (1 100 + 1 300 40 (1 340 × 1 ÷ 8
240) 28)
We can now complete the process accounts for the two processes.
Process A account
Amount Amount
Description Litres R/litre Description Kg R/litre
(R) (R)
Material A 500 25 12 500 Normal loss 100 25 2 500
Abnormal
Material B 750 35 26 250 50 45 2 250
loss
Transfer to
Labour 9 500 1 100 45 49 500
process B
Overheads 6 000
1 250 54 250 1 250 54 250
The unit cost A is calculated as follows: (54 250 – (100 × 25)) ÷ (1 250 – 100) = R45
Process B account
Amount Amount
Description Litres R/litre Description Litres R/litre
(R) (R)
Process A 1 100 45 49 500 Normal loss 48 - -
Transfer to
Material C 240 60 14 400 finished 1 300 63 81 900
goods
Labour 10 000
Overheads 7 496
Abnormal
8 63 504
gain
1 348 81 900 1 348 81 900
The unit cost B is calculated as follows: 81 396 ÷ (1 340 – 48) = R63
The other accounts that will be used, showing all entries, including closing entries, are as
follows:
Abnormal gain account
Description Litres R/litre Amount Description Litre R/litre Amount
Income Process B
R504 8 63 R504
statement account
Abnormal loss account
Description Litres R/litre Amount Description Litres R/litre Amount
Accrued
Process A
50 45 R2 250 income from 50 25 R1 250
account
normal loss
Income
R1 250
statement
Accrued income from normal loss account
Description Litres R/litre Amount Description Litres R/litre Amount
Process A 100 25 R2 500 Bank R3 750
Abnormal
50 25 R1 250
loss

10.5 Accounting for incomplete production (work in process)


In the previous section, we discussed how to account for normal and abnormal losses, and abnormal
gains. In the examples, we assumed that all units produced during the period were fully completed at
the end of the period. This is, however, not always the case. In most processes there are opening
and/or closing inventories of items that have been partially completed. These items are referred to as
work in process. In this section, we consider the various aspects of, and accounting treatment for,
WIP.
We will firstly discuss how we determine the number of units produced during a period and then
the way to value this production. Remember that the objective is to determine the value of closing
WIP and finished goods, and the correct cost of sales.
We can either use process accounts, as explained previously, although a number of calculations
would have to be made separately – or we can use three statements:
• The equivalent production statement. In this statement, the equivalent units are calculated
(explained in Section 10.5.1).
• Unit cost statement. In this statement, the unit cost of direct materials, direct labour and
manufacturing overheads will be calculated.
• Production cost statement. In this statement, the production cost for the period will be calculated.

In the rest of this chapter, we will use these three statements; the examples illustrate how they work.
One of the key principles when accounting for WIP is that we normally assume one or more so-
called inspection points. These are stages, or points, in the process where we count units of WIP. Let
us assume that a process has two inspection points, A and B. At point A, the process is 30 per cent
complete; at point B, the process is 80 per cent complete. This is shown in Figure 10.4.

Figure 10.4 Inspection points in a process

When counting units, we should remember that this is a process and, as such, units move from one
point in the process through to finished goods. Units cannot move backwards in a process. (In certain
cases, units may be wasted and then reintroduced, as in the candle-manufacturing example above.
This should, however, be accounted for as waste and not seen as units moving back in the process.)
Let us assume that in the day we introduced 100 units into the process. All 100 units passed
inspection point A; 90 units passed inspection point B; and 85 units were transferred to finished
goods. There are no losses in this process. In this example we can see that the following represents
the production for the day:
Units introduced 100
Units completed 85
Units still in the production process 15
The above indicates that we need to account for 15 units that are still in the production process.
Therefore, as 90 units had passed point B and only 85 units were finalised, we can see that five units
are between 80 and 100 per cent complete. The remaining 10 units have passed point A but not point
B, and we can therefore see that these units are between 30 and 80 per cent complete.
In practice, many businesses only have one inspection point – at 50 per cent complete. This may,
however, vary according to the needs of the organisation.
This information is used to calculate what is known as equivalent units.

10.5.1 Equivalent units


The cost incurred in the partial manufacture of WIP items should be allocated to them and not to the
production of the goods transferred to the next process (or finished goods). As an example, assume all
costs of a process are R100 000, that 8 000 units were transferred to finished goods and 2 000 units
were partially completed. A portion of the R100 000 was incurred in the partial completion of the 2
000 units and therefore we cannot simply take R100 000 divided by 8 000 completed units to
calculate the cost per unit.
We must use equivalent units as the basis to allocate production costs to units produced and
partially produced during the year. Equivalent units convert partially completed units to the
equivalent fully completed units. As an example, if the 2 000 units in the example above were 50 per
cent completed, they would be equivalent to 1 000 completed units. Therefore the equivalent units for
the period are calculated as 8 000 units plus 1 000 units, or 9 000 units. The cost per unit would then
be R100 000 divided by 9 000 units, or R11,11 per unit.
It is also true that in a production process costs are not incurred equally throughout the process. It
may therefore be necessary to calculate the equivalent units of the three cost categories – direct
materials, direct labour and overheads – separately and then divide those costs by the equivalent units
for each of the types of costs. We can then use these unit costs to calculate the value of WIP. Let us
use an example to illustrate this process.

EXAMPLE 10.11
Holly manufactures potpourri for use by other manufacturers and retailers. Her production
process can be described as follows.
Flower petals and spices (the potpourri mix)
Step 1 20% complete
are purchased and put onto shelves.
The potpourri mix is dried in a drying oven for
Step 2 60% complete
a day at low temperatures.
Scent oil is then added to the dried potpourri
Step 3 mixture and left for a day so the scent can 80% complete
permeate the mix.
The potpourri is packed into bags of 1 kg and
Step 4 100% complete
put into finished goods.
The end of each step is an inspection point. In the production process, the potpourri mix costs are
all incurred before they are dried, but the scent oil costs are only incurred at the start of step 3.
Direct labour costs are incurred equally throughout the process, but all of the overhead costs have
been added at the end of step 2, as the overhead costs mostly relate to the cost of electricity
consumed by the oven.
Holly’s production records reflect the following production at the end of the month:
Potpourri completed 200 kg
Potpourri completed to step 2 10 kg
Potpourri completed to step 3 0
Cost of potpourri mix R6 000
Cost of scent oil R1 000
Labour costs R4 000
Overhead costs R1 500
As a first step, we should calculate the equivalent units of direct materials (in this case, potpourri
mix and scent oil), direct labour and overhead production using the equivalent production units.
In the equivalent production statements, the number of units put into production (we will deal
with opening WIP later) is included in the left column. On the right of the descriptions there are
columns for total units, direct material units (often only a single column but in this example there
are two direct materials) and conversion costs (as direct labour and manufacturing overhead
costs are not incurred equally through the process, they are separated here).

Equivalent production statement


Output units
Input units Description Potpourri Labour Overhead
Total Scent oil
mix costs costs
210 Started
Completed 200 200 200 200 200
Closing WIP 10 10 Nil1 62 103
210 210 210 200 206 210
Notes: 1. Scent oil is only added in step 3; 2. Labour costs at step 2 are 60% of completed units;
thus 10 × 60%; 3. Overhead costs = 100% incurred at the end of step 2.

We can now calculate the unit costs using the unit cost statement. In the unit cost statement, the
total costs of the various direct materials, direct labour and manufacturing overheads are
introduced as costs. The total of these costs is then divided by the equivalent units to obtain the
total unit costs. Note that the total unit cost (i.e. a completed unit) is only equal to the total costs
divided by the total equivalent units if the equivalent units for the different components are the
same. Generally, this will not be the case, and therefore the unit cost is equal to the unit cost of
materials plus the unit cost of conversion costs.

Unit cost statement


Description Total Potpourri mix Scent oil Labour Overheads
Current period
Potpourri mix 6 000 6 000
Scent oil 1 000 1 000
Direct labour 4 000 4 000
Manufacturing
1 500 1 500
overheads
Total (R) 12 500 6 000 1 000 4 000 1 500
Unit cost (R) 60,13 28,57 5 19,42 7,14

We can now calculate the value of WIP and the costs of the units produced for the period. We use
the production cost statement to show the costs of production for the period, as follows:
R R
Finished goods (200 × 60,13) 12 026,00
Closing WIP 473,62
Potpourri mix (10 × 28,57) 285,70
Scent oil (nil × 5) 0
Direct labour (6 × 19,42) 116,52
Manufacturing overheads (10 × 7,14) 71,40
12 499,62
The difference between R12 499,62 and the total cost of production of R12 500 is due to
rounding.
This example is a bit more complex given that we had to account for potpourri mix, scent oil,
direct labour and manufacturing overheads separately. Normally, it is possible to only account for
direct material and conversion costs (direct labour plus manufacturing overheads). Equivalent
units and WIP are important in the context of process costing, so make sure that you understand
the above example.
In a process where there is closing WIP in one month, there will also be opening WIP the next
month. We will now consider how to account for opening WIP.

10.5.2 Opening work in process


Up to this point, we have assumed that there is no opening WIP. But, clearly, if there is closing WIP
there must also be opening WIP in the following month. Opening WIP brings forward costs from the
previous period for uncompleted work and will incur further costs to bring them to completion.
There are two methods for accounting for costs brought forward and conversion costs: the
weighted average method and first in first out (FIFO) method.
With the weighted average method, we assume that the units of WIP at the end of the previous
period become completely merged with the units that started during the current period and cannot be
identified independently. Essentially, therefore, we divide the costs of the opening WIP plus the
current period costs by the equivalent units started during the period plus the equivalent units of
opening WIP.
If we use the FIFO method, we account for opening WIP separately from the new production of the
new month. Essentially, therefore, we account for the opening WIP separately and then divide the
period production costs by the equivalent units produced in the month and the equivalent units of WIP
completed during the month. Therefore, if 1 000 units were 40 per cent completed in the previous
month, we would add 600 (1 000 × 60 per cent) equivalent units to the current month’s production.
These can be quite complex processes, so let us use an example to illustrate the difference
between the two methods.

EXAMPLE 10.12
JLS Electronics manufactures CD players on a large scale and uses a process-costing system to
account for the costs of manufacture. The production for the month was as follows:
Opening WIP
Units 2 500
Direct materials R137 500
Direct labour R50 000
Manufacturing overheads R18 750
Production for the month
Units started 18 000
Direct materials R2 016 000
Direct labour R789 250
Manufacturing overheads R308 000
Closing WIP 3 200 units
WIP comprises 50 per cent of completed units with all of the material costs having been incurred
but only 50 per cent of the conversion costs (labour and overheads).
The weighted average method is used for inventory valuation.

Required
Draw up the following statements:
• Equivalent production
• Unit cost
• Production cost

Solution
Firstly, we would complete the equivalent production statement. In this statement we include the
opening WIP as an input plus the 18 000 units started during the month. We know that the closing
WIP was 3 200 units and therefore the number of units finished must be 17 300 (2 500 + 18 000 –
3 200). Our WIP closing materials are 3 200 units but the conversion costs are only 1 600
equivalent units (3 200 × 50 per cent).

Equivalent production statement


Output (units)
Input (units) Description Total Material Conversion

2 500 Opening WIP


18 000 Started
Completed 17 300 17 300 17 300
Closing WIP 3 200 3 200 1 600
20 500 20 500 20 500 18 900
We can now complete the unit cost statement. Firstly, we enter the costs brought forward for the
WIP and then add the current period costs. We can then calculate the unit cost. Note that the unit
cost of material and the conversion cost are calculated separately. We must then add the two unit
costs calculated to find the total unit cost. As the conversion equivalent units and the total
equivalent units are not the same, the total costs cannot simply be divided by the total equivalent
units.

Unit cost statement


Total
Description Materials (R) Conversion (R)
(R)
Opening WIP
Direct materials 137 500 137 500
Direct labour 50 000 50 000
Manufacturing
18 750 18 750
overheads
Current period
Direct materials 2 016 000 2 016 000
Direct labour 789 250 789 250
Manufacturing
308 000 308 000
overheads
Total 3 319 500 2 153 500 1 166 000
Unit cost 166,74 105,05 61,69
Based on the equivalent units and the unit cost statements, we can now complete the production
cost statement. Please note the small difference is due to rounding:

Production cost statement


R R
Finished goods (17 300 ×
2 884 602
166,74)
Closing WIP 434 864
Materials (3 200 × 105,05) 336 160
Conversion (1 600 × 61,69) 98 704
3 319 466
Using the details from Example 10.12, we will now use the FIFO method for inventory valuation.

EXAMPLE 10.13
Use the information provided in Example 10.12. The FIFO method is used for inventory
valuation.

Required
Draw up the following statements:
• Equivalent production
• Unit cost
• Production cost

Firstly, we complete the equivalent production statement. In this statement we include the opening
WIP as an input plus the 18 000 units started during the month. Under output units, we enter the
opening WIP as an output in the total column. No materials have to be added so nothing will be
entered in the material output column but the remaining conversion of these items (1 250
equivalent units (2 500 units × 50 per cent remaining to be completed)) will be included in the
conversion column. We know that the closing WIP was 3 200 units and therefore the number of
units started and completed must be 14 800 (18 000 – 3 200). Our WIP closing materials are 3
200 units but the conversion cost is only 1 600 equivalent units (3 200 × 50 per cent).

Equivalent production statement


Output (units)
Input (units) Description
Total Material Conversion
2 500 Opening WIP 2 500 1 250
18 000 Started
Started and
14 800 14 800 14 800
completed
Closing WIP 3 200 3 200 1 600
20 500 20 500 18 000 17 650
We can now complete the unit cost statement. As we are using the FIFO method, we ignore the
opening WIP costs and account for them separately in the production cost statement. We can then
calculate the unit cost. Please note that the unit cost of material and conversion cost will be
calculated separately. We must then add the two unit costs calculated to find the total unit cost, as
in Example 10.12.

Unit cost statement


Total Materials Conversion
Description
(R) (R) (R)
Current period
Direct materials 2 016 000 2 016 000
Direct labour 789 250 789 250
Manufacturing
308 000 308 000
overheads
Total 3 113 250 2 016 000 1 097 250
Unit cost 174,11 111,94 62,17
Based on the equivalent units and the unit cost statements, we can now complete the production
cost statement.
Production cost statement
R R
Finished goods 2 860 791
Opening WIP costs (50 000 + 18 750 + 137 500) 206 250
Direct materials (14 800 × 111,94) 1 656 712
Conversion costs ((14 800 + 1 250) × 62,17) 997 829
Closing WIP 457 680
Materials (3 200 × 111,94) 358 208
Conversion (1 600 × 62,17) 99 472
3 318 471

10.5.3 Accounting for losses in a work-in-process system


When accounting for WIP equivalent units, we have to consider expected and abnormal losses to
ensure that costs are allocated fairly. In this section, we look at how to account for these losses. It
should also be noted that the accounting process differs according to whether the weighted
average or FIFO method is used.
When we account for normal and abnormal losses and gains in a process-costing system and
there is WIP, the most critical aspect is where these losses occur. Losses are normally accounted
for at a specific inspection point (see Section 10.5). If WIP has passed the inspection point, we
account for the normal losses on those items; if it has not, we only account for the normal losses
when the units pass that inspection point. It is important to note that the treatment shown in Table
10.1 is required when dealing with losses in a process-costing system.

Table 10.1 Treatment for dealing with losses in process-costing system


Account for losses in this Account for losses in
period next/previous period
Closing WIP has not passed
inspection point
Closing WIP has passed
inspection point
Opening WIP had not passed
inspection point
Opening WIP had passed
inspection point

Let us use a brief example to illustrate these four treatments.

EXAMPLE 10.14
JZN manufactures carpets. The company has two product ranges – Home and Office. JZN uses a
process-costing system, accounting separately for the two product ranges. Both processes are
nearly identical and normal losses amount to 10 per cent of units introduced. These losses are all
incurred before inspection point A, where units are 60 per cent complete in terms of conversion
costs but 100 per cent complete in terms of materials. Conversion costs are incurred equally
throughout the process.
The summary for the previous month’s production is as follows:
Home units Office units
Opening WIP (50% and 80%
6 000 5 200
complete, respectively)
Units introduced 60 000 80 000
Units completed 56 100 69 200
Closing WIP (90% and 40%
3 000 8 000
complete, respectively

Required
Draw up the equivalent production statement for the month using the weighted average method.

Equivalent production statement


a. Home
Output (units)
Input (units) Description
Total Material Conversion
6 000 Opening WIP
60 000 Started
Started and
56 100 56 100 56 100
completed

Normal losses 6 9001 6 900 4 140


Closing WIP 3 000 3 000 2 700
66 000 66 000 66 000 62 940
Note: 1. Normal losses:
Opening WIP 50% completed thus not passed inspection point 6 000
Units introduced 60 000
Closing WIP 90% complete so did pass inspection point 3 000
Total 69 000
Thus normal losses = 69 000 × 10% = 6 900 units

b. Office
Output (units)
Input (units) Description
Total Material Conversion
5 200 Opening WIP
80 000 Started
Started and
69 200 69 200 69 200
completed

Normal losses 8 0001 8 000 4 800


Closing WIP 8 000 8 000 3 200
85 200 85 200 85 200 77 200
Note: 1. Normal losses:
Opening WIP 80% completed thus passed inspection point 0
Units introduced 80 000
Closing WIP 40% complete so did not pass inspection point 0
Total 80 000
Thus normal losses = 80 000 × 10% = 8 000 units.
This example showed how to calculate equivalent units where there are losses and WIP in the
process. We will now look at a comprehensive example showing the treatment based on the
weighted average and FIFO methods.

EXAMPLE 10.15
A company uses the process-costing system to account for its production costs. Production for the
previous week was as follows:
Opening WIP (kg) 20
Raw material input (kg) 100
Closing WIP (kg) (10)
Finished goods (kg) (85)
Gain/loss (kg) 25
Normal loss (kg) 20
Abnormal loss (kg) 5
The opening WIP comprised material costs of R790 and conversion costs of R3 500, and had not
passed the inspection point. Any losses on those units are included in normal losses. The
conversion costs for the week were R15 000. Closing WIP material costs have been incurred at
100 per cent, but only 50 per cent of conversion costs have been incurred. The closing WIP had
already passed the inspection point. The raw-material costs are R40 per kg. All losses are
incurred before the inspection point.
The weighted average method is used for inventory valuation.

Required
Draw up the following statements:
• Equivalent production
• Unit cost
• Production cost

Equivalent production statement


Output
Input
Description (units)
(units)
Total Material Conversion
20 Opening WIP
100 Started
Completed 85 85 85
Normal losses 20 20 10
Abnormal losses 5 5 2,5
Closing WIP 10 10 5
120 120 120 102,5
We can now complete the unit cost statement. Firstly, we will enter the costs brought forward
for the WIP and then add the current period costs. We can then calculate the unit cost.

Unit cost statement


Total Materials Conversion
Description
(R) (R) (R)
Opening WIP
Direct materials 790 790
Conversion costs 3500 3 500
Current period
Direct materials (100 ×
4 000 4 000
40)
Conversion costs 15 000 15 000
Total 23 290 4 790 18 500
Unit cost 220,41 39,92 180,49
Based on the equivalent units and the unit cost statements, we can now complete the production
cost statement. Note that the small difference is due to rounding.

Production cost statement


R R
Finished goods 21 338,15
85 × 220,41 18 734,85
Normal loss 2 603,30
Materials (20 ×
798,40
39,92)
Conversion costs
1 804,90
(10 × 180,49)
Closing WIP 1 301,65
Materials (10 ×
399,20
39,92)
Conversion (5 ×
902,45
180,49)
Abnormal losses 650,83
Materials (5 ×
199,60
39,92)
Conversion (2,5 ×
451,23
180,49)
23 290,63
We will now take the same example but use FIFO to illustrate how this method is applied when
there are normal losses.

EXAMPLE 10.16
Use the information provided in Example 10.15. The FIFO method is used for inventory
valuation.

Required
Draw up the following statements:
• Equivalent production
• Unit cost
• Production cost

Firstly, we need to complete the equivalent production statement. In this statement, we include the
opening WIP as an input plus the 100 units started during the month. We will now enter under
output units the opening WIP as an output in the total column. No materials have to be added, so
nothing will be entered in the material output column, but the remaining conversion of these items
will be included in the conversion column. We know that the closing WIP was 10 units, and
normal and abnormal losses were 25 units. Therefore, the number of units started and completed
must be 65 (100 – 25 – 10). The WIP closing materials = 10 units but the conversion costs are
only five equivalent units.

Equivalent production statement


Input Output (units)
Description
(units) Total Material Conversion
20 Opening WIP 20 10
100 Started
Started and
65 65 65
completed
Normal losses 20 20 10
Abnormal losses 5 5 2,5
Closing WIP 10 10 5
120 120 100 92,5
We can now complete the unit cost statement. As we are using the FIFO method, we ignore the
opening WIP costs and account for them separately in the production cost statement. We can then
calculate the unit cost. Please note that the unit cost of material and the conversion cost will be
calculated separately. We must then add the two unit costs calculated to find the total unit cost, as
shown in Example 10.15.

Unit cost statement


Total Materials Conversion
Description
(R) (R) (R)
Current period
Direct materials
4 000 4 000
(100 × 40)
Conversion costs 15 000 15 000
Total 19 000 4 000 15 000
Unit cost 40 162,16
Based on the equivalent units and the unit cost statements, we can now complete the production
cost statement.

Production cost statement


R R R
Finished goods 21 473,60
Opening WIP costs
4 290,00
(790 +3 500)
Direct materials (65
× 40) 2 600,00
Conversion costs
12 162,00
((65 + 10) × 162,16)
Normal loss 2 421,60
Materials (20 × 40) 800,00
Conversion costs (10
1 621,60
× 162,16)
Closing WIP 1 210,80
Materials (10 × 40) 400,00
Conversion (5 ×
810,80
162,16)
Abnormal losses 605,40
Materials (5 × 40) 200,00
Conversion (2,5 ×
405,40
162,16)
23 369

CLOSING PRACTICAL EXAMPLE


The next time James returned home from college, he had a solution for accounting for the costs in
his father’s chemical company, which manufactures various household chemicals on a small scale.
A process-costing system is ideal in an environment like James’s father’s business, where bulk
homogeneous products are manufactured. James advised his father that he should account for
production based on the actual production for any period. He should also calculate what his
normal losses are due to evaporation and normal spillage when packaging the chemicals. He also
explained to his father that abnormal losses should be accounted for differently.
Lastly, he discussed his father’s business and whether or not there was opening and closing
WIP; his father indicated that there was. He then explained how his father should account for this
WIP as a portion of the direct material, direct labour and manufacturing overhead costs incurred in
manufacturing these WIP units.
His father smiled and said, ‘And I was wondering whether they were going to teach you
anything useful!’

10.6 Summary
In this chapter, we discussed accounting in a process-costing environment. Process costing is widely
used in the industries for which it was designed, namely where bulk homogeneous products are
manufactured. Process costing gives management critical costing information to ensure that decisions
are made on the basis of the best available information.
In a process-costing system, it is important to account for direct material, direct labour and
manufacturing overheads separately, as the equivalent units of production are not necessarily the
same. Furthermore, normal and abnormal losses, and abnormal gains should be accounted for. Any
scrapping value of these costs should also be taken into account appropriately.
Partially finished products were taken into consideration and we explained how they can be
valued using the weighted average or FIFO methods. The method for accounting for losses and WIP
were finally considered.

PRACTICAL ACTIVITY
Write a memo to the managing director of the law firm where you work, in which you explain to
him:
• what a process-costing system is; and
• how such a system could be implemented in a legal firm.

Short questions
1. Compare and contrast process costing and job costing, giving examples of industries where
they are typically used.
2. Explain the following:
• Normal losses
• Abnormal losses
• Abnormal gains
3. Simonswood Cheese Company makes cream cheese. For every 10 kg of the raw cheese
mixture the company expects to sell 8 kg of cream cheese. Complete the following table
based on production for the previous month:
Normal Abnormal Abnormal
Input Output
loss loss gain
(kg) (kg)
(kg) (kg) (kg)
Week 1 100 85
Week 2 120 92
Week 3 90 72
Week 4 80 66

Long questions
1. Zera Glass manufactures glass bottles. It receives a mixture from a supplier, which is melted
and cast to make the bottles. Normal losses are 5 per cent of inputs, and losses can be sold as
scrap at R2 per kg. Due to the nature of the manufacturing process, there is no WIP.
The production schedule for the previous month is as follows:
Units introduced (kg) 20 000
Units produced (kg) 19 000
Glass mix cost (R/kg) 10
Conversion costs R35 000

Draw up the following statements for the month:


• Equivalent production
• Unit cost
• Production cost

2. James Ncube Chartered Accountants have five staff members, each earning R8 000 per
month. Each staff member works 165 hours per month, of which 15 hours are set aside for
office administration and other non-billable hours. James uses a process-costing system to
account for the firm’s costs.
In June 2013, R7 300 hours were billed and total overhead costs were R15 000. All
billable hours were already billed during the month and overhead costs are allocated based
on billable hours. Draw up the following statements for June 2013:
• Equivalent production
• Unit cost
• Production cost

3. B&A Natural Products produces aromatherapy oils. The firm’s production for the first six
months of 2013 was as follows:
Material Labour Overhead
Input Output Waste sold
costs costs costs
(units) (units) (units)
(R) (R) (R)
January 1 000 990 0 120 000 45 000 30 000
February 1 050 1 010 20 127 050 46 000 31 000
March 985 980 0 117 215 46 000 31 050
April 1 020 1 010 6 125 460 50 600 30 900
May 1 100 1 095 3 132 550 51 600 32 500
June 1 350 1 200 110 170 100 50 300 32 500
Normal losses are 1 per cent of units input; when waste is recoverable, it is sold at R3 per
unit.
Prepare a production account for the production of B&A Natural Products for the first six
months of 2013. Balance the account on a monthly basis.

4. ABC Ltd manufactures tables and makes use of a process-costing system. The company uses
two materials to make the tables. Wood is added at the beginning of the process and steel is
added when the process is 40 per cent complete. Conversion costs are incurred evenly
throughout the process and no losses are incurred.
The following cost and production details are available for August 2013:
Production
Opening WIP (45% complete) 4 500 tables
Closing WIP (30% complete) 5 500 tables
Completed and transferred 59 000 tables
Costs
6 300
Material (R) Wood
Opening WIP 2 350
Conversion costs (R) Steel
4 850
83 200
Material (R) Wood
Current costs 31 900
Conversion costs (R) Steel
71 150
Draw up the following statements using the weighted average and FIFO methods for August
2013:
• Equivalent production
• Unit cost
• Production cost

5. Disney Land Ltd manufactures two toys, Mickey Mouse and Donald Duck. Disney Land has
made the following information available to you:
Materials are added to the product as follows –
• Mickey Mouse: Material A is added at the beginning of the process; Material B is added
at the end of the process; conversion costs are incurred evenly through the process.
• Donald Duck: Material A is added evenly throughout the process; conversion costs are
incurred evenly throughout the process.
Production information for July 2006:
Mickey Mouse (units) Donald Duck (units)
Opening stock: WIP 5 000 6 000
% Completion 60% 60%
Units introduced (started) 100 000 90 000
Closing stock: WIP 15 000 3 000
% Completion 33% 30%
Costs Mickey Mouse (R) Donald Duck (R)
Opening stock: WIP 5 000 10 800
Material A 9 000 18 000
Conversion costs 100 000 270 900
Current costs
180 000
Material A
275 850
Material B
Conversion costs 451 500
Draw up the following statements for Mickey Mouse based on the FIFO method and for
Donald Duck using the weighted average method:
• Equivalent production
• Unit cost
• Production cost
6. Tidy Tiles CC manufactures tiles in two departments. The output from Department 1 becomes
the input into Department 2.
Department 1
Clay is mixed and fast baked. This takes place from the start of the process and continues up
to 30 per cent of the process.
At the 70 per cent point of processing, the pattern is added and the tile is then baked again
to set the pattern.
The conversion costs are incurred evenly throughout the process.
Department 2
Glazing is added to the tile from 20 to 60 per cent of the process.
The tile is then baked to full hardness, cleaned and sent to the packaging department.
Conversion costs are incurred evenly throughout the process.
The following information is available for December 2013:
Opening Closing inventory
Process Units started Units transferred
inventory WIP WIP
16 000 (40% 14 000 (85%
Department 1 275 000 235 000
complete) complete)
47 000 (50% 54 000 (30%
Department 2 ??? 215 000
complete) complete)
Units in both departments are inspected when the units are 100 per cent complete. Normal losses
are expected to be 5 per cent of units that pass the completion point in any month.
Carried forward from Incurred in December 2013
November 2013 (R) (R)
Clay (Department 1) 28 000 3 850 000
Pattern (Department 1) - 3 735 000
Conversion costs (Department
96 000 2 125 600
1)
Glazing (Department 2) 188 000 1 019 500
Conversion costs (Department
131 000 597 700
2)
Transferred in (Department 2) 1 625 120 ???
Draw up the following statements for both departments using the weighted average method:
• Equivalent production
• Unit cost
• Production cost

7. Lone Hill Breweries makes two products, Lone Cider and Hill Beer. The brewery uses a
process-costing system, accounting separately for the two product ranges. Both processes are
nearly identical and normal losses amount to 5 per cent of units introduced. These losses are
all incurred before inspection point A, where units are 50 per cent complete in terms of
conversion costs but 100 per cent complete in terms of materials. Conversion costs are
incurred equally throughout the process.
The summary for the previous month’s production is as follows:
Lone Cider Hill Beer
Opening WIP
Units (80% and 40% complete,
10 800 10 520
respectively)
Direct material costs (R) 21 060 21 040
Conversion costs (R) 31 320 36 820
Current month
Units introduced 180 000 200 000
Units completed 170 675 191 094
Direct material costs (R) 360 000 460 000
Conversion costs (R) 558 000 696 000
Closing WIP
Units (60% and 30%
9 500 10 900
completed, respectively
Draw up the following statements for both products using the FIFO method:
• Equivalent production
• Unit cost
• Production cost
11 Process costing: Joint products and by-
products
Minnette Vermaak and Johan Hefer

LEARNING OUTCOMES
After studying this chapter, you should be able to:
• explain what joint and by-products are;
• allocate costs to joint products on the basis of:
– a physical measure (e.g. weight, units produced);
– the sales value at the split-off point;
– the net realisable value at the split-off point;
– a constant gross profit percentage being achieved for all joint products;
• account for the income of by-products as:
– additional sales revenue;
– other income;
– a reduction of the joint manufacturing costs;
– a reduction in the cost of goods sold; and
• make decisions regarding the further processing of products of specific orders based on the
incremental profit or loss.

PRACTICAL EXAMPLE
Impala Platinum Holdings Limited is in the business of mining, refining and marketing platinum
group metals (PGMs) and associated base metals. The group has extensive operations at the two
most significant PGM deposits in the world, namely the Bushveld Complex in South Africa and the
Great Dyke in Zimbabwe.
The six PGMs occur together alongside nickel and copper. Platinum, palladium and rhodium,
the most economically significant of the PGMs, are found in the largest quantities and are the key
minerals mined by Impala. The remaining PGMs are produced as by-products and are not seen as
key to the success of the company. (Impala Platinum Holdings Limited Annual Report 2012)
With a mix of products, the question therefore arises how costs should be attributed to the
different minerals that are mined, processed and sold by Impala.

11.1 Introduction
I n Chapter 10, we described the method of accounting for costs in a process-costing system. We
discussed the ways to account for normal and abnormal gains and losses, and how to account for
work in process using the weighted average and first-in-first-out methods. Process costing is
appropriate in an environment where multiple homogeneous products are manufactured or created
during a repetitive process. Common features of many such processes are joint and by-products.
Joint products are manufactured or excavated and are jointly seen as the company’s integral
products.

Examples are the products of oil refining, in which petrol, paraffin and several other petrochemicals
are produced during the same process. Mining also commonly excavates multiple minerals in the
same process. Platinum mines, for instance, commonly produce platinum, rhodium and several other
minerals as an integral part of their operations, as shown in the opening example.

A by-product is one that is created incidentally through the manufacturing process and is not seen
as an integral product of the company.

Examples of by-products are sawdust and offcuts of wood produced when planks of wood (the core
product) are manufactured.
A typical example of a process in which joint and by-products are created is shown in Figure
11.1.

Figure 11.1 The process of producing joint and by-products

This chapter discusses the ways to account for joint and by-products. The key question when joint
products are produced during a manufacturing or similar process is how to allocate costs to the
various products. Where by-products are produced, costs are normally not allocated to these products
but there are different ways in which to account for the gain from the sale of by-products.

11.2 Joint products


Joint products are produced together as an outcome of the same production process. Joint products
are to a large extent equal in importance to the organisation and all products are marketed and sold as
part of the main function of the business.
In a process-costing system, direct materials, direct labour and manufacturing overheads are
allocated to a process rather than to specific jobs or batches. In processes where joint products are
produced, costs should be shared in a fair way among the products to ensure that the products’ costing
is fair and to enable management to make decisions on resources, costs and other factors according to
the data from the costing system.
Remember that the allocation of joint costs between product lines may seem irrelevant. As an
example, if the Impala platinum mine allocates too large a portion of its costs to the income from
rhodium it will show, conversely, a larger than actual profit in the other minerals but a lower than
actual profit in rhodium. However, the overall profit from the joint products will be the same, no
matter how the costs are allocated to them.
It is also irrelevant from the point of view that joint products are normally the products of a
common process and production can seldom be stopped on any single product of a joint production
process. In other words, as part of its mining operations the platinum mine automatically extracts
rhodium and palladium, as these minerals occur together. However, there are exceptions to this, and
in certain industries it may be possible to choose which joint products to produce and which not to
produce.
The fact that, from an organisational level, the way in which the joint costs are allocated seems
irrelevant does not mean that it is not important to get the costing as accurate as possible. This is
especially true if selling-price decisions are made based on the cost information. If a specific product
is unfairly burdened with too large a proportion of the joint manufacturing costs, the profitability of
that product line may be impeded and an inaccurate picture of the profitability of the joint products
may be reported.
The next section discusses the ways in which joint costs can be allocated to joint products. Bear in
mind that, as with all costing systems, the circumstances will be unique to each environment and the
process-costing system should best reflect the economic reality of the process it aims to cost.

11.3 Methods of allocating joint costs


Joint manufacturing costs are incurred before the point where joint products are separated (or ‘split
off’, to use the technical term) from each other, either for sale or further processing. These costs may
include direct materials, direct labour and manufacturing overheads. The accounting for this part of
the process is done similarly to that discussed in Chapter 10 and will not be discussed further at this
point.
At this split-off point, different products have been produced that should jointly carry the joint
manufacturing costs. The question is, on what basis do we allocate these costs between the joint
products? Generally, this can be done in four ways:
• On the basis of a physical measure (e.g. weight or units produced)
• On the basis of the selling price at the split-off point
• On the basis of the net realisable value at the split-off point
• On the basis of a constant gross profit percentage being achieved for all joint products

We will briefly explain these four methods using the same example. The management of any costing
system would need to consider its own requirements to ensure that joint costs are allocated fairly.
The different methods may all have equal validity in different process environments. We will
however indicate that although the physical-measure method is widely used due its simplicity, the
other three methods could be said to be more logical. However, they may be far more complex to
apply for many reasons, including the need to take into account the complexity of the further processes
needed to finish a product for sale.

11.3.1 Physical measure


The first method of allocating joint manufacturing costs between joint products we will discuss is
where such an allocation is based on a physical measure. Examples of physical measures are the
number of units produced, kilograms output, litres output – or any other physical measurement that
may be appropriate.
Physical measures are especially useful where there is not a large discrepancy between the unit
prices of the different products. If there is a large difference in unit prices, however, this method
could prove problematic. Let us assume that two products, A and B, are produced in a process. The
production for the month was 1 000 units of both Product A and Product B, and the joint costs were
R10 000. That would mean that R5 000 will be allocated to each product based on the number of
units produced. Let us, however, assume that product A sells for R6 per unit while product B sells for
R20 per unit. Product A would then show a profit of R1 000 ((1 000 × R6) − 5 000), whereas product
B would show a profit of R15 000 ((1000 × R20) − 5 000). Hence, in this example, this method
would not appear to give a fair reflection of costs.
Another example where physical units can be a problematic cost-allocation method to use is when
the different joint products are in different forms. For example, in the extraction industry one joint
product may be a solid while another is a gas. It would be difficult to determine a basis to allocate
the costs between these two, as one product is measured in kilograms whereas the other is measured
in cubic centimetres.
The main benefit of using a physical measure is that it is often the easiest method to calculate. If a
process like the one described above has two products that are measured in the same units, allocating
costs between the different products based on the physical units of output is a very simple process.
The other methods that we will discuss later can be much more complex.
We will now look at an example of allocating costs to joint products. As indicated, we will use
the same example to explain all four methods.

EXAMPLE 11.1
Magix Mix produces three chemical products using a joint process. The products produced are
PX1, PX2 and PX3. All three are liquids measured in litres. The following information pertains
to production during the previous quarter:
PX1 PX2 PX3
Units produced (l) 10 000 20 000 40 000
Selling price per unit
20 36 14
(R)
Further processing
costs (after split-off) 150 000 160 000 Nil
(R)
Joint production costs were R700 000 for the quarter.

Required
Prepare the gross profit calculation for the three products and the total gross profit for the quarter
using the physical-measure method based on litres produced.
Suggested solution
PX1 PX2 PX3 Total
Sales
(units × selling 200 000 720 000 560 000 1 480 000
price)
Cost of goods sold (250 000) (360 000) (400 000) (1 010 000)
Joint costs (see
100 000 200 000 400 000 700 000
calculation 1)
Further costs 150 000 160 000 0 310 000
Gross
(50 000) 360 000 160 000 470 000
profit/(loss)
Gross profit (25%) 50% 29% 32%

Calculation 1
Joint costs are allocated based on the physical measure. In total 70 000 (10 000 + 20 000 + 40
000) litres of product were produced. We can now calculate the joint cost to be allocated as
follows:
PX1: R700 000 × 10 000 ÷ 70 000 = 100 000
PX2: R700 000 × 20 000 ÷ 70 000 = 200 000
PX3: R700 000 × 40 000 ÷ 70 000 = 400 000

As can be seen in Example 11.1, the physical measure method in this case does not appear to give
a fair reflection of the process, as PX1 shows a loss while PX2 shows a very large profit. It
should be remembered, however, that the overall gross profit does give a fair reflection of the
overall process.

11.3.2 Sales value at split-off point


Because the physical measure method of allocating joint manufacturing costs to joint products does
not always fairly reflect the costs of manufacturing the joint products where the selling prices are
very different, the second method calculates the allocation based on the sales value of the joint
products.
The main drawback of this method is where the products cannot be sold at the split-off point and
have to go through further processes to be finished for sale. If this is the case, calculating the selling
price can be very difficult or even impossible at the split-off point. This drawback is addressed by
the methods discussed in the two subsequent sections.

EXAMPLE 11.2
The same information is used as in Example 11.1, but product PX1 can be sold for R8 at the split-
off point, PX2 for R20 and PX3 for R14.
Required
Prepare the gross profit calculation for the three products and the total gross profit for the quarter
using the selling price at split-off point method.

Suggested solution
PX1 PX2 PX3 Total
Sales
(units × selling 200 000 720 000 560 000 1 480 000
price)
Cost of goods sold (203 846) (429 231) (376 923) (1 010 000)
Joint costs (see
53 846 269 231 376 923 700 000
calculation 1)
Further costs 150 000 160 000 0 310 000
Gross
(3 846) 290 769 183 077 470 000
profit/(loss)
Gross profit (2%) 40% 33% 32%

Calculation 1
Joint costs are allocated based on the sales value at split-off point.
Estimated sales revenue at split-off point (units produced × unit sales price):
PX1 (10 000 × R8) R80 000
PX2 (20 000 × R20) R400 000
PX3 (40 000 × R14) R560 000
Total R1 040 000
We can now calculate the joint cost to be allocated as follows:
PX1: R700 000 × 80 000 ÷ 1 040 000 = R53 846
PX2: R700 000 × 400 000 ÷ 1 040 000 = R269 231
PX3: R700 000 × 560 000 ÷ 1 040 000 = R376 923

As can be seen from Example 11.2, the total gross profit remains the same even though different
methods are used to allocate costs between products. This approach appears to give a better
reflection of the gross profits of the products, although it can still be argued that Product PX1 is
carrying too large a portion of the joint costs.

11.3.3 Net realisable value method


The net realisable value method is useful in cases where there are no selling prices available for
products at the split-off point. This method calculates a net realisable value, which is equal to the
sales value less further processing costs to determine the net realisable value at the split-off point.
The joint manufacturing costs are then allocated between the joint products based on the net
realisable value. We will illustrate the use of this method in Example 11.3.
EXAMPLE 11.3
The same data is used as in Example 11.1.

Required
Prepare the gross profit calculation for the three products and the total gross profit for the quarter
using the net realisable value method.

Suggested solution
PX1 PX2 PX3 Total
Sales (units ×
200 000 720 000 560 000 1 480 000
selling price)
Cost of goods sold (179 915) (495 043) (335 042) (1 010 000)
Joint costs (see
29 915 335 043 335 042 700 000
calculation 1)
Further costs 150 000 160 000 0 310 000
Gross
20 085 224 957 224 958 470 000
profit/(loss)
Gross profit 10% 31% 40% 32%

Calculation 1
Joint costs are allocated based on the net realisable value at the split-off point. We calculate the
net realisable value by calculating the sales value and then deducting the further costs to be
incurred and allocating the joint costs on this basis:
PX1 PX2 PX3
Sales value (R) 200 000 720 000 560 000
Further costs (R) 150 000 160 000 0
Net realisable value
50 000 560 000 560 000
(R)
The total net realisable value for the three products is therefore R1 170 000.
We can now calculate the joint cost to be allocated as follows:
• PX1: R700 000 × 50 000 ÷ 1 170 000 = R29 915
• PX2: R700 000 × 560 000 ÷ 1 170 000 = R335 043
• PX3: R700 000 × 560 000 ÷ 1 170 000 = R335 042

Note: PX2 and PX3 in the above calculation should be the same amount, as they have the same
net realisable value. The difference is due to rounding.
This method appears to be a much fairer allocation of costs, and the gross profits appear more
reasonable across the different products. One criticism that can be raised about this method is that
the gross profit percentages are so different, with PX1 achieving a gross profit of 10 per cent
compared to PX2 with 31 per cent and PX3 with 40 per cent. This criticism is dealt with in the
last of the four methods.

11.3.4 Constant gross profit percentage


It can be argued that the different products that are produced in a joint process should all have a
constant gross profit percentage. This is a fair statement, as the manner in which joint manufacturing
costs is allocated appears largely irrelevant because the total gross profit remains constant, as has
been shown in the previous three examples. Therefore, there is logic in the approach that the products
that are produced from the same process all achieve the same gross profit percentage. We will
illustrate this method in the next example.

EXAMPLE 11.4
The same data is used as in Example 11.1.

Required
Calculate the gross profit for the three products and the total gross profit for the quarter using the
constant gross profit percentage method.

Suggested solution
PX1 PX2 PX3 Total
Sales (units x selling price) 200 000 720 000 560 000 1 480 000
Cost of goods sold (136 480) (491 328) (382 192) (1 010 000)
Joint costs (balancing figure) (13 520) 331 328 382 192 700 000
Further costs 150 000 160 000 0 310 000
Gross profit (Calc. 1) 63 520 228 672 177 808 470 000
Gross profit % (Calc. 1) 31,76% 31,76% 31,75%* 31,76%
Calculation 1
Joint costs are allocated to achieve a constant gross profit percentage. To calculate the allocation
of joint costs for the three products, the total gross profit must first be calculated as follows:
Gross profit = sales − joint costs − further costs
= 1 480 000 − 700 000 − 310 000
= R470 000

The gross profit percentage can then be calculated as 31,76% (470 000 ÷ 1 480 000).
We can now calculate the gross profits required to achieve the gross profit of 32 per cent for
each of the products as follows:
PX1: 200 000 × 31,76% = 63 520
PX2: 720 000 × 31,76% = 228 672
PX3: 560 000 × 31,76% = 177 808*
Note: * Rounded to achieve R470 000
Using the constant gross profit percentage method, as shown in Example 11.4, does not give a
reasonable answer. Due to product PX1’s large further processing costs relative to its sales value, the
other two products have to ‘carry’ some of the further costs so that Product PX1 achieves the overall
process gross profit percentage.
These examples have illustrated some of the complexities and factors that need to be considered to
achieve a fair allocation between the different joint products. In this particular example, we would
suggest that the net realisable value method gives the fairest reflection. However, in reality every
situation is different and the allocation method should be chosen carefully according to the cost-
benefit trade-off present in any cost accounting system.

11.4 By-products
As explained, many production processes lead to products that are incidental to the manufacturing
process and that are not the manufacturer’s key products. These are known as by-products. Although
incidental, by-products have sales value. A sawmill, for instance, produces sawdust, a by-product
that can be sold.
It is important to distinguish between by-products, waste and scrap. A by-product is produced
incidentally to the production process but is sold in a systematic fashion. Waste is produced by the
manufacturing process but is generally simply thrown away or otherwise disposed of and has no
economic value. Scrap is the waste product of a process; it has some economic value but is not sold
in a systematic fashion and is accounted for as a normal or abnormal loss with a scrap value. (Scrap
value was discussed in more detail in Chapter 10.)
The key question that arises with by-products is how to account for the income received from the
sale of these products. Generally, no joint process costs are allocated to by-products, although there
may be exceptions to this rule. If joint costs are allocated to by-products, this is applied in the same
way that costs are allocated to joint products, as discussed previously in this chapter. Any costs
incurred in the further processing or sale of by-products would also be offset against the by-product’s
income.
In this section, we will only consider how to deal with the profit of a by-product (if costs are
allocated to it) or the income from the sale of a by-product (if no costs are allocated to it). In the rest
of this chapter, we will accept that no further or joint costs need to be allocated to the by-product.
There are generally four ways to deal with the income from the sale of by-products:
• Treat the income as additional sales revenue.
• Treat the income as other income.
• Treat the income as a reduction of the joint manufacturing costs.
• Treat the income as a reduction of the cost of goods sold.

The four methods used to deal with income from the sale of by-products are reasonably self-
explanatory, but the difference between reduction of the joint manufacturing costs and reduction of
cost of goods sold deserves some clarification. When the income reduces joint manufacturing costs,
this also affects the value of the work in process, as the costs that are divided by the equivalent
production (as discussed in Chapter 10) will change as the joint manufacturing costs are reduced.
If the income reduces the cost of goods sold, it will only be deducted when calculating the cost of
goods sold and will therefore not affect the unit costs of work in process.
We will illustrate the four methods by means of an example.

EXAMPLE 11.5
Jelly Beans Ltd produces jelly beans and accounts for the production costs using a process-
costing system. In the process of manufacturing the jelly beans, a by-product is created that is sold
to animal-feed producers, who use it. This by-product is not seen as integral to the business and
no joint costs are allocated. The following information pertains to the previous financial year:
Sales (R) 13 000 000
Process costs (R) 9 000 000
Income from the sale of by-products (R) 600 000
Production (units) 15 000 000
Work in process (units) 450 000
There was no opening work in process; the cost of work in process units are equal to 50 per cent
of a completed unit’s costs.

Required
Prepare the income statement for the company for the year, showing four scenarios in which the
income from the sale of by-products is treated as:
• Additional sales revenue
• Other income
• Reduction of the joint manufacturing costs
• Reduction in the cost of goods sold

Suggested solution
Reduction in joint
Additional sales Other income Reduction in cost
manufacturing
(R) (R) of goods sold (R)
costs (R)
Sales 13 600 000 13 000 000 13 000 000 13 000 000
Cost of goods sold (8 867 250) (8 867 250) (8 276 250) (8 267 250)
Cost of production 9 000 000 9 000 000 8 400 000 9 000 000
Sale of by-
(600 000)
products
Work in process (132 750)1 (132 750) 1 (123 750)2 (132 750) 1
Gross profit 4 732 750 4 132 750 4 723 750 4 732 750
Other income 600 000
Net profit 4 732 750 4 732 750 4 723 750 4 732 750
Notes: 1) WIP unit cost: 9 000 000 ÷ (15 000 000 + (450 000 × 50%)) = R0,59 per unit
WIP: (450 000 × 50%) × 0,59 = R132 750; 2) WIP unit cost: (9 000 000 − 600 000) ÷ (15 000
000 + (450 000 × 50%)) = R0,55 per unit; WIP: (450 000 × 50%) × 0,55 = R123 750
11.5 Joint or by-products: Making sales decisions
When dealing with joint or by-products, decisions often have to be made regarding the sale of the
products. These decisions may take into account whether a product should be sold at the split-off
point or processed further, or whether a specific order should be accepted or not. The process of
decision making is relatively simple if the costs are known. Essentially, the decision should be made
by comparing the incremental revenue from further processing or sales against the incremental costs
of further processing the product. If the incremental revenue is more than the incremental costs, it
makes sense to process the product, as the extra income would be larger than the costs of the further
processing. If the incremental income is less than the incremental costs, no further processing of the
product should take place.
It is important when calculating incremental costs that we remember to include all relevant
incremental costs. These costs could include extra direct material, direct labour and manufacturing
overhead costs incurred; impacts on other products; and increased non-manufacturing overhead costs.
We will now consider these factors by means of an example.

EXAMPLE 11.6
Du Cane uses a process-costing system to account for its manufacturing process. One of the
processes in its factory is Process A, in which three products are manufactured. These products
are labelled A1, A2 and X2. A1 and A2 are joint products and X2 is a by-product.
Du Cane needs to make a decision on whether to further process A1 and A2, and whether to
accept an offer for product X2 from a prospective customer. The details are as follows:
• Product A1: This product can either be sold at the split-off point for R8 per litre or can be
sold after further processing for R20 per litre. The extra processing involves extra direct-
material costs to the value of R3 per litre that have to be added, and extra conversion costs of
R2 per litre will be incurred.
• Product A2: This product can be sold at the split-off point for R12 per litre or can be further
processed at a cost of R6 per litre (which includes the direct material and conversion costs).
Once the further processing has been done, the product can be sold for R14 per litre.
• By-product X2: This product can be sold at the split-off point for R2 per litre. However, a
prospective customer has contacted the company and has offered to pay R6 per litre for 12
000 litres if the products are processed further. Du Cane has considered the costs and has
found that incremental material and labour costs would be R1,50 per litre. It has also
established that fixed overheads would increase by R15 000 per month. Variable overheads
are not expected to change.

Required
Advise Du Cane whether to further process products A1 and A2 and whether to accept the offer
for the by-product, X2.

Suggested solution
In the first two cases, with the two joint products, all figures are given per litre and therefore we
can simply calculate the incremental profit or loss per litre to make the decision as follows:

Product A1
Incremental revenue per litre (R20 − R8) 12
Incremental costs per litre (R3 + R2) (5)
Incremental profit per litre 7
Decision: As incremental profits are positive, process the product further.

Product A2
Incremental revenue per litre (R14 − R12) 2
Incremental costs per litre (6)
Incremental loss (4)
Decision: As incremental profits are negative, do not process the product further.
When a decision needs to be made for the by-product, X2, the fixed overheads of the company
will increase by R15 000 per month. The R15 000 therefore also needs to be considered. One can
now either multiply the per litre figures by 12 000 litres or divide the R15 000 by 12 000 litres to
find the incremental profit or loss.

By-product X2
Incremental revenue total (12 000 l × (R6 −
R48 000
R2))
Incremental direct costs total (12 000 l × R1,50) (R18 000)
Incremental fixed overheads (R15 000)
Incremental profit R15 000
Decision: As incremental profits are positive, accept the order.

CLOSING PRACTICAL EXAMPLE


As explained in the opening example, platinum, palladium and rhodium are processed by Impala
Platinum Holdings as joint products. Therefore, the process costs are allocated to these minerals
using one of the four methods discussed in this chapter. The other minerals (such as nickel and
copper) will be treated as by-products and the income generated will be allocated using one of the
four methods for accounting for by-product income discussed in this chapter.

11.6 Summary
In this chapter, we considered the methods used to account for joint and by-products in a process-
costing system. When joint and by-products are present in a process-costing system, it is very
important to ensure that they are accounted for correctly and that the allocation of costs among the
different products is fair.
Joint and by-products frequently occur in manufacturing processes and accounting for these is
important to ensure that the information is available for decision making by the managers of the entity.
We explained that joint products are jointly produced during the same process and are of equal or
similar importance to the entity. By-products are incidental to the process and are not as important to
the entity as the joint products.
We discussed how joint-product costs can be allocated to different products using four methods:
• Physical measure (e.g. weight, units produced)
• Selling price at split-off point
• Net realisable value at split-off point
• Constant gross profit percentage achieved for all joint products

We also discussed how the accounting treatment of the income of by-products can be applied using
four methods:
• Additional sales revenue
• Other income
• Reduction of the joint manufacturing costs
• Reduction of the cost of goods sold

Finally, we discussed how decisions should be made regarding the further processing of products of
specific orders based on their incremental profit or loss.

Questions
1. Joiner Ltd is a company that manufactures products L and M using a joint production process.
Product L requires further processing before being sold, while product M is sold at the split-
off point. The sales director of Joiner Ltd is concerned about the profitability of product L,
which has been showing a loss for some time, and has stated that a decision needs to be taken
about the future of this product.
Products L and M emerge from a joint process of two chemicals, A and B. Details of the
raw-material input are as follows:
Processing costs R
Raw materials
123 540
Chemical A
43 660
Chemical B
Direct labour costs 69 600
Factory overheads 46 400
• Joint costs are apportioned based on the physical standards method.
• The output of product L is processed further at R1,80 per kg.
• The output of the process delivers 20 000 kg of product L and 60 000 kg of product M.
• Product L is then available for sale at 6,80 per kg. Product M is sold for R5,00 per kg.
• There are no opening or closing inventories.
a. Calculate the profit/loss for products L and M.
b. Calculate the profit/loss if the relative market value at the split-off point is used to
allocate joint costs.

2. Rainbow Chickens incurs joint production costs of R405 000 for the production of three joint
products (whole chickens, braai packs and fillets). The runaways are sold as byproducts.
The income generated will be treated as other income. The whole chickens and braai packs
can be sold without further processing for R34,70 and R37 per kilogram, respectively. The
fillets require further processing at a cost of R5 per kg before being sold for R52 per kg.
The following information was made available for September 2012:
Output
Whole chickens 2 430 kg
Braai packs 1 944 kg
Fillets 1 026 kg
At the beginning of September the company had 500 kg of whole chickens, 400 kg of braai
packs and 50 kg of fillets in the storeroom. At the end of the month, it had 450 kg of whole
chickens, 550 kg of braai packs and 75 kg of fillets. The total value of opening finished goods
is R40 000 for whole chickens, R36 000 for braai packs and R15 000 for fillets. A FIFO
inventory valuation method is used.
a. Determine the unit flow of the products
b. Allocate the joint costs to the joint products according to:
i. Physical unit method
ii. Market value at split-off point
c. Calculate the gross profit and gross profit percentage if the market value at the split-off
point is used to allocate joint costs for fillets only.

3. Flintstone Dairy Farm manufactures a variety of dairy products. The company uses a joint
process to manufacture its products. The two main products are butter and cheese, plus a by-
product, buttermilk. The following information is available for October 2012:
• The opening inventory of cheese was 200 kg, and butter 1 000 kg.
• During October, 900 kg of cheese and 4 500 kg of butter were sold.
• Joint costs amount to R44 000, of which R24 000 is the variable component of the joint
costs.
• Production output for butter was 4 000 units; cheese was 1 000 kg.
• Cheese can be sold at the split-off point for R24/kg.
• Butter can be sold at the split-off point for R36/kg per unit or it can undergo further
processing at a cost of R12 per kg and can then be sold for R52 per kg.
a. Calculate the closing inventory for both products.
b. Assuming that Flintstone does not process butter further, allocate the joint costs (in total
and per unit) to the joint products according to:
i. Physical unit method
ii. Market value at split-off point
iii. Calculate the gross profit if the market value at split-off point is used to allocate
joint costs for butter only.
c. If Flintstone decides to sell butter after further processing, allocate the joint cost (in total
and per unit) according to the market value at split-off point, and calculate the gross profit
for butter.
d. Advise Flintstone on whether butter should be sold at the split-off point or undergo further
processing.

4. Green Plastics Ltd operates a process that produces two joint products, Alpha and Beta.
During the month the company used 10 000 kg of materials with a total cost of R1 million, 10
000 labour hours at an average cost of R50 per labour hour and 50 000 machine hours at a
cost of R10 per machine hour in the joint process.
The joint process yields 3 000 kg of Alpha, which can be sold at the split-off point for
R500 per kg and 7 000 kg of Beta. Beta needs to undergo further processing, during which an
additional total cost of R150 per kg will be incurred. Beta can then be sold for R700 per kg.
The following production and sales information is available:
Alpha (units) Beta (units)
Opening inventory 1 000 500
Production 3 000 7 000
Sales 3 500 7 500
a. Calculate the closing inventory for both products.
b. Allocate the joint costs (in total and per unit) to the joint products according to:
i. Physical unit method
ii. Market value at split-off point
c. Calculate the gross profit if the market value at the split-off point is used to allocate joint
costs for Beta only.

5. Wandi’s Ltd manufactures joint products A, B and C. Products A and C need to be further
processed before being sold. The joint costs are allocated to the three products using market
value at the split-off point.
The following information is available for December 2012:
A B C
Quantity produced
8 000,00 10 000,00 2 000,00
(units)
Sales price per unit (R) 40,00 25,00 18,00
Further processing
5,00 0,00 3,00
costs per unit (R)
At the beginning of December, Wandi’s had 2 000 units of product A and 2 500 units of
product B available. The sales units for December were as follows: 9 000 units of product
A, 10 000 units of B and 2 000 units of C. Joint costs amount to R206 000.
a. Determine the unit flow of the products.
b. Allocate the joint costs to the joint products according to market value at the split-off
point.
c. Allocate the joint cost to the products according to the physical unit method.
d. Calculate the gross profit and gross profit percentage if the market value at the split-off
point is used to allocate joint costs for product C only.
12 Standard costing
Peter Taylor

LEARNING OUTCOMES
After studying this chapter, you should be able to:
• explain the meaning of standard costs and how they are determined;
• explain how a standard costing system works;
• explain production cost variances, and calculate labour, material, overhead and sales variances;
and
• identify the reasons for these variances.

PRACTICAL EXAMPLE
In preparation for the annual general meeting of Promix Ltd, the CEO asks the senior cost
accountant why the company’s profits are 10 per cent lower than the budgeted figures.
The senior cost accountant is able to reply as follows: ‘Firstly, the company produces a range
of standardised products. Due to market volatility at present, the price of raw materials is apt to
change suddenly. We are therefore frequently caught out by price changes and the material used
often turns out to be more expensive than the budgeted price.
‘Secondly, although we budget carefully well in advance of the period in question, our profit is
always between 10 and 20 per cent less than the budgeted figure. We need to examine effectively
every element of cost more closely. This will hopefully determine more accurately the reasons for
the differences between actual and budgeted figures. Our actual figures are unacceptable. I will get
this analysis to you before the AGM.’

12.1 Introduction
Standard costs are predetermined estimates of the cost to manufacture a single unit under efficient
operating conditions and during a specific future period. Standard costs are the planned or target
costs of the product under current or anticipated conditions.

Normally, however, these standard costs are not the same as the actual costs incurred. Therefore, the
figures for the period end will show:
• the standard cost;
• the actual cost; and
• the difference between these costs, which is termed the ‘variance’.

The usefulness of standard costing has been widely recognised in the production field. It is widely
used in the textile, electrical, engineering, food processing and chemical industries. Standard costing
has also been adapted in the service industry, and can be applied in the public sector and in non-
profit institutions.

12.2 Standard costs


In a standard costing system, the standards of performance, usage of material, production quantity and
cost rates must be established. Standards can be set using an average of past historical costs or
engineering studies can be used to determine the best combinations of resources, production methods
and product quality.
Standard costs are normally ascertained for the following elements of cost:
• Direct material
• Direct labour
• Variable overheads
• Fixed overheads

Standard costs should not be seen as static over long periods of time. Although they will often be set
for a specific period, like a year, standard costs should be reconsidered when planning for the next
period. If standard costs are not adjusted as soon as new information about the cost elements becomes
available, the information supplied by the costing system could become more and more irrelevant.
We will now consider the different types of standard costs.

12.2.1 Material standards


Based on the production of one unit of output, the expected quantities of materials required can be
determined. The quantity of material needed is referred to as the standard material quantity.
Standard prices of materials are obtained from the purchasing department, which researches the
market for appropriate material suppliers and selects suppliers that can supply the required materials
in the required quantity and quality at the most competitive price.
Standard quantities and prices of all materials should be set for each product. A certain level of
wastage will normally also be considered when determining the amount of material required.

12.2.2 Labour standards


Once the most efficient production method has been identified, the expected number of hours required
to manufacture the product is determined under efficient operating conditions. These are referred to as
the standard hours required to complete the job.
Wage rates are determined according to grades of experience and the skill levels required to
complete the job. In certain industries, trade unions are usually involved in the negotiation of these
wage rates. This will also include the remuneration paid to part-time and full-time employees.
The standard rates of labour costs should be set for each type of product or process in the
organisation.

12.2.3 Standard overhead rate


When calculating the standard overhead rate, the total expected overhead costs will be calculated.
This will then be allocated using the standard overhead rate based on either direct labour hours or
machine hours. In some cases, the standard overhead rate may also be based on material cost and
labour cost.
If an operation is labour-intensive, the company would normally use direct labour hours as a
basis, whereas if the operation is capital-intensive, then machine hours would be used. The applied
or predetermined allocation rate is calculated in advance and the variable overhead standard set.
Variances occur if actual direct labour hours, machine hours, and material and labour costs differ
from the standard. Variances will also occur if the expected and actual total overheads differ.
As the term suggests, ‘fixed manufacturing overhead’ means that this element of cost does not vary
directly with production. At least in the short run, the total fixed overheads remain constant,
regardless of production. They need to be reassessed annually to take into account changes in these
costs. However, during the period, costs will be allocated according to the expected levels of
production and fixed costs.

12.3 Production cost variances


In the previous section, we discussed the various standards that would be set by the organisation
based on its expectations for the period. Once the actual results are available, the expected standards
should be compared to the actual outcomes. This comparison will reveal various differences, which
when analysed may help management to monitor and improve cost management.
The overall production cost variance is the difference between the standard production cost of
actual production volume and the actual production cost over a specified period. The actual
production volumes are used because if the expected production volumes were to be used, the
variance would not be relevant, as both the actual costs and the actual volumes would differ from
those expected.
The overall total manufacturing cost variance hides variances among materials, labour and
overheads, and therefore this variance needs to be broken down into smaller, more meaningful,
elements of variance.

Figure 12.1 Organisational hierarchy of manufacturing cost variances


12.3.1 Standard cost cards and production budgets
A standard cost card is normally drawn up for each product manufactured or manufacturing process.
An example of such a card is given in Example 12.1. Thereafter a production budget can be drawn up.

EXAMPLE 12.1
Standard cost card for product Beta
Direct materials: 3 kg of material S at R30 per
R90
kg
Direct labour: 4 hours at R100 per hour R400
Variable overheads: 4 hours at R20 per hour R80
Total standard cost permit R570
The standard usage of material S in the production of Beta is 3 kilograms, and the standard
price per kilogram of S is R30. Therefore, the cost of material S for Beta is R90. The standard
hours required to manufacture one unit of Beta is four hours and the standard rate per hour is
R100. Therefore the cost of labour is R400. The predetermined overhead rate in this case is
dependent on direct labour hours. The number of hours is the same as for the direct labour
calculation. The overhead allocation rate would be calculated in advance using the following
formula:

This formula has been discussed in detail in Chapter 6 on overheads.


The total standard cost per unit is therefore R570. If product Beta is now sold for R700 per
unit the standard contribution margin will be R130 per unit.

A production budget for Beta can now be prepared for the month of March using the standard
quantities and costs shown in the standard cost card (see Example 12.2). In addition, annual fixed
overheads are expected to be R840 000, which are expected to be incurred evenly throughout the
year. This will mean that fixed overheads per month will amount to R70 000.

EXAMPLE 12.2
This example is based on the information in Example 12.1.

Production budget for product Beta


R
Sales 1 000 units of Beta at R700 per unit 700 000
Direct material: 3 000 kg of S at R30/kg 90 000
Direct labour: 4 000 hours at R100 per hour 400 000
Variable overheads: 4 000 hours at R20 per
80 000
hour
Budgeted contribution 130 000
Fixed overheads 70 000
Budgeted profit 60 000

After the end of the reporting period, the production budget should be compared with the actual
results for the period to determine the reasons for variance that may occur between the budgeted
profit and the actual profit. We will continue with the information given in Example 12.1 to show the
actual results and then analyse the results. We will also consider the possible reasons for the
variances.

EXAMPLE 12.3
Actual results (product Beta)
R
Sales: 800 units at R750 per unit 600 000
Direct material: 2 500 kg at R35/kg 87 500
Direct labour: 3 000 hours at R110 per hour 330 000
Variable overheads: 3 000 hours at R25 per
75 000
hour
Contribution 107 500
Fixed overheads 60 000
Profit 47 500

Based on the budgeted and actual results, a set of variances can now be prepared showing the
differences between the actual data and the standard data. The management accountant, in consultation
with management, will examine the variances to determine the reasons for these differences.
Management should then implement strategies and processes to address the unfavourable differences.
The following abbreviations will be used in the preparation of the variances that follow:
AC = actual cost
AFO = actual fixed overheads
AH = actual hours
AP = actual price
AQ = actual quantity
AR = actual rate
AV = actual volume
AY = actual yield
BFO = budgeted fixed overheads
SC = standard cost
SCM = standard contribution margin per unit
SH = standard hours
SM = sales margin
SMP = standard margin price variance
SP = standard price
SQ = standard quantity
SR = standard rate
SV = sales volume
SY = standard yield

12.3.2 Direct material variances


This section looks at direct material price variance and direct material quantity variance.

12.3.2.1 Direct material price variance


This variance shows the difference between the standard price and the actual purchase price for the
actual quantity of material.

Material price variance = (AP – SP) × AQ


= (R35 – R30) × 2 500
= R12 500 (unfavourable)
The material price variance shows that the actual price per kilogram was R5 more than the standard
for the total amount purchased of 2 500 kg.
Where a variance is unfavourable, the result is effectively worse than the standard – and vice
versa. It is very important that each variance is evaluated in this way and the words ‘favourable’ (F)
or ‘unfavourable’ (U) are added after the rand total.

Reasons for the variance


There are various reasons that can explain the variance. Market conditions may cause an increase in
the price of raw materials. A further possibility is that original supply sources may become
unavailable and material may need to be purchased from more expensive sources. Thirdly, poor
inventory control may result in the purchase of emergency material at a higher price.

12.3.2.2 Direct material quantity variance


This variance shows the difference between the standard quantity specified for actual production
and the actual quantity used at the standard purchase price.

When calculating this variance, the actual number of units produced should be used rather than the
number expected. In this example, therefore, 800 units at a standard of 3 kg per unit were produced.
Therefore, the standard quantity used for this variance would be 2 400 kg.
Material quantity variance = (AQ – SQ) × SP)
= (2 500 kg – 2 400 kg) × R30
= R3 000 (unfavourable)
This variance is also referred to as the material usage variance.
The material quantity variance shows that 100 kg more than the standard amount of material has
been used and therefore the variance is unfavourable.

Reasons for the variance


More material than was budgeted for has been used to manufacture the number of units actually
produced. This may be caused by inefficient usage of material in the production process or poor
quality materials may have been supplied, which had to be discarded. Another reason is that
materials may have been stolen, giving the impression that material has been misused.
12.3.2.3 Total material variance
This variance shows the difference between the standard material cost for production and the
actual cost of material.

Again, the variance should be based on the actual number of units produced, therefore 2 400 kg × R30
= R72 000.
Total material variance = SC – AC
= (R72 000 – R87 500)
= R15 500 (unfavourable)
The price variance of R12 500 unfavourable added to the quantity variance of R3 000 unfavourable
provides the total material variance.

12.3.3 Direct labour variances

12.3.3.1 Direct labour rate variance


This variance shows the difference between actual and the standard direct labour rate per hour for
the total hours worked.

Direct labour rate variance = (AR – SR) × AH


= (R110 – R100) × 3 000 hours
= R30 000 (unfavourable)
The rate variance is unfavourable because R10 per hour more than the standard rate has been paid to
the labour force for 3 000 hours.

Reasons for the variance


Increased wage levels may have been negotiated between unions and management, which could
explain the increased rate per hour.

12.3.3.2 Direct labour efficiency variance


This variance shows the difference between the standard hours required for production and the
actual hours taken at the standard labour rate.

The actual production was 800 units at a standard time of four hours per unit. Therefore, 3 200 hours
is the number of calculated standard hours of actual production.
Direct labour efficiency variance = (AH – SH) × SR
= (3 000 hours – 3 200 hours) × R100
= R20 000 (favourable)
This variance is favourable because fewer production hours have been used than required by the
standard. In other words, the labour force has been more efficient than the standard in this case.

Reasons for the variance


In this case, the favourable variance may be caused by the efficient use of time by the labour force,
resulting in less than the budgeted number of hours being used to manufacture the output.
The opposite would be true if an unfavourable variance occurs. For example, strikes or illness
may disrupt the manufacturing process, resulting in more time being taken to manufacture the output.

12.3.3.3 Total labour variance


This variance shows the difference between the standard labour cost for production and the actual
labour cost.

Standard labour cost will be based on 3 200 hours at R100 per hour, which equals R320 000.
Total labour variance = (SC – AC)
= (R320 000 – R330 000)
= R10 000 (unfavourable)
The labour rate variance of R30 000 unfavourable plus the labour efficiency variance of R20 000
favourable gives the total labour variance of R10 000 unfavourable.

12.3.4 Variable overhead expenditure (rate) variance


This variance shows the difference between actual variable overhead figures and standard figures
determined according to the method of allocation.

In this case, direct labour hours are used. However, machine hours, direct material cost, direct labour
cost or even prime cost could have been used as a basis for allocation.
Variable overhead expenditure variance = (AR – SR) × AH
= (25 – 20) × 3 000
= R 15 000 (unfavourable)

Reasons for the variance


The reasons for this variance are similar to those that apply to labour variances because overheads
are allocated using direct labour hours in this case.
12.3.5 Variable overhead efficiency variance
This variance shows the difference between the actual production hours taken and the hours that
should have been taken to complete the required production units.

Variable overhead efficiency variance = (AH – SH) × SR


= (3 000 – 3 200) × R20
= R4 000 (favourable)

Reasons for the variance


The reasons for this variance – if labour hours are used – are similar to those discussed under direct
labour efficiency variance (Section 12.3.3.2).

12.3.6 Total variable overhead variance


This variance is the difference between the actual variable overheads incurred and the standard
variable overheads charged to production.

Total variable overhead variance = SC – AC


= (64 000 – 75 000)
= R11 000 (unfavourable)
The sum of the variable overhead expenditure variance (R15 000 unfavourable) and the variable
overhead efficiency variance (R4 000 favourable) agrees with this figure.

12.3.7 Fixed overhead expenditure variance


This variance is the difference between all items of budgeted expenditure that are constant and
therefore independent of the level of output, and the corresponding actual fixed overheads for the
same period.

Fixed overhead expenditure variance = AFO – BFO


= 60 000 – 70 000
= R10 000 (favourable)

Reasons for the variance


Fixed costs may change due to an increase in factory rental; extra supervisors may be employed; or
the salaries of existing supervisors may be increased. There may be a need to change the standard in
line with these increases.
Thus far, we have dealt with cost and quantity variances. However, companies also look at sales
variances to note changes in expected revenues. These variances are discussed in the next sections.

12.3.8 Sales margin price variance


This variance notes the difference between the actual selling price per unit and the standard selling
price per unit for the actual quantity sold.

Sales margin price variance = (AP – SP) × AQ


= (R750 – R700) × 800
= R40 000 (favourable)
Note that because of a R50 per unit increase in the selling price, the overall revenue from sales will
increase and therefore the variance will be favourable.
The sales price may be increased to maintain a margin required in the event of an increase in input
costs.

12.3.9 Sales margin volume variance


So that the impact of the sales effort on profit can be taken into account, we consider this variance in
terms of the contribution margin rather than the selling price.

This variance shows the difference between the actual quantity of units sold and the standard
quantity expected to be sold for the standard contribution margin.

Sales margin volume variance = (AV – SV) × SM


= (800 – 1 000) × R130
= R26 000 (unfavourable)
The actual sales volume of 800 units is less than the planned sales volume of 1 000 units and,
accordingly, the resulting variance will be unfavourable because revenue will be lower than
expected.

12.4 Reconciliation statements


Management will be interested in the reason for the actual profit being less than the budgeted profit. A
reconciliation statement showing all the causes for the variance between actual profit and budgeted
profit summarises the reasons for the difference (see Example 12.4).

EXAMPLE 12.4
Information is based on previous examples.

Reconciliation statement showing differences between actual and budgeted variable costing
systems
Budgeted net profit R60 000
Sales variances
Sales margin price variance 40 000 F
Sales margin volume variance 26 000 U
Direct material variances
Direct material price variance 12 500 U
Direct material quantity (usage) variance 3 000 U
Labour variances
Direct labour rate variance 30 000 U
Direct labour efficiency variance 20 000 F
Overhead variances
Variable overhead expenditure variance 15 000 U
Variable overhead efficiency variance 4 000 F
Fixed overhead expenditure variance 10 000 F
Actual net profit R47 500

12.5 Advanced variances


In the previous sections, we discussed the general kinds of variances experienced in a manufacturing
environment. We will now consider some special types of variances that may also be found. These
are direct material mix variance, direct material yield variance, direct labour mix variance and sales
mix variance.

12.5.1 Direct material mix variance


The material mix variance occurs when a number of different materials are mixed together in
proportion to produce a final product. Concrete, for example, is produced from a mixture of gravel,
sand and cement, and a cake is made from a mixture of flour, eggs, fruit, milk and sugar in a certain
proportion.
If the quantities used in production differ from the standard quantities there will be a usage
variance due to a change in the proportions of materials. The usage variance of raw materials is
called the material mix variance.
If, for instance, concrete mix requires 8 kg of gravel, 2 kg of cement and 3 kg of sand, but the
actual mix contains 6 kg of gravel, 3 kg of cement and 4 kg of sand, there will be a mix variance due
to the different composition of materials actually used – even though the total weight of 13 kg of
concrete is still the same. In the case of a product such as a cake, a minor variance in the mixture
could mean that the cake cannot be sold. This type of variance can therefore only occur when the
standard mixture can be altered to a certain extent.

EXAMPLE 12.5
Calculating direct material mix variance
Using the example of the concrete mix, let us say the standard mix for 13 kg of concrete is as
follows:
Material Quantity Standard price/kg Cost
Gravel (G) 8 kg R3,25 R26
Cement (C) 2 kg R10,00 R20
Sand (S) 3 kg R2,00 R6
13 kg R52
The actual mix is as follows:
Material Quantity Standard price/kg Cost
Gravel (G) 6 kg R3,25 R19,50
Cement (C) 3 kg R10,00 R30,00
Sand (S) 4 kg R2,00 R 8,00
13 kg R57,50
The direct material mix variance formula = (SQ – AQ) × SP
G 3,25 × (8 kg – 6 kg) = R6,50 F
C 10,00 × (2 kg – 3 kg) = R10,00 U
S 2,00 × (3 kg – 4 kg) = R2,00 U
R5,50 U
Although less gravel has been used in the actual mix, slightly more cement and sand have been
used, resulting in an unfavourable variance of R5,50 per 13 kg of concrete produced.

12.5.2 Direct material yield variance


The standard yield is the output expected from the standard input of raw materials. From time to time,
however, the actual yield may differ from the standard, giving rise to a yield variance.
The formula is:
(Standard yield of actual input – actual yield of actual input) × standard material cost
In all processes, there is an expected normal loss involved. As discussed in Chapter 10,
companies provide for a normal loss as a percentage of units that reach an inspection stage.
Assume that we are producing a product called MNT, which requires the following material mix:
N 30 kg R3
T 20 kg R6
The standard normal expected loss is 10 per cent. See Example 12.6.

EXAMPLE 12.6
Calculating direct material yield variance
In May 2012, the following mix was produced:
Standard mix (product MNT)
Raw material Quantity Cost/kg Total cost
N 30 kg R3 R90
T 20 kg R6 R120
50 kg R210
Standard loss 5 kg
45 kg
Actual mix (product MNT)
Raw material Quantity Cost/kg Total cost
N 28 kg R3 R84
T 22 kg R3 R66
50 kg R150
Actual loss 2 kg
48 kg R150
= SC × (SY – AY)
= 4,67 × (45 – 48)
= R14,01 F
Note: SC is R210/45 kg = R4,67

12.5.3 Direct labour mix variance


A variance will arise if the grades of labour used in production are different from those budgeted for.
This is illustrated in Example 12.7.

EXAMPLE 12.7
Direct labour mix variance
The standard labour force for the manufacture of product C is:
Ten grade-6 employees paid at R48 per hour for
R19 200
40 hours
Five grade-5 employees paid at R32 per hour
for 40 hours R6 400
Two grade-4 employees paid at R24 per hour
R1 920
for 40 hours
Total standard labour cost R27 500
The actual labour force employed during the week is as follows:
Eight grade-6 employees paid at R48 per hour
R15 360
for 40 hours
Six grade-5 employees paid at R32 per hour for
R7 680
40 hours
Three grade-2 employees paid at R24 per hour
R2 880
for 40 hours
Total actual labour cost R25 920
There is no labour rate variance, as the actual labour force has all been paid at the standard rate
per hour.
Direct labour mix variance = standard cost of standard mix – standard cost of actual mix
Standard labour force Actual labour force Variance
Grade-6 labour R19 200 R15 360 R3 840 F
Grade-5 labour R6 400 R 7 680 R1 280 U
Grade-4 labour R1 920 R2 880 R960 U
R27 500 R25 920 R1 600 F
The overall variance is favourable, as fewer of the more expensive grade-6 workers have been
employed and have been replaced by more of the less expensive grade-5 and grade-4 workers.

12.5.4 Sales mix variances


Usually companies sell more than one product, and each of these products will have their own profit
margins or profits, depending on the method used. It would be more meaningful to use contribution
rather than profit because an increase or decrease in sales volume affects profit due to contribution
per unit sold and not profit per unit sold.
In Example 12.8, three products are produced by the same company, Axe, Bee and Cee.

EXAMPLE 12.8
Sales mix variances
Below are the standard and actual figures for the period.

Standard cost card


Unit contribution
Product Budgeted sales units Total contribution
margin
Axe 12 000 (50%) R10 R120 000
Bee 8 000 (33,33%) R6 R48 000
Cee 4 000 (16,67%) R3 R12 000
24 000 R180 000

Actual figures
Unit contribution
Product Sales units Total contribution
margin
Axe 10 000 R10 R100 000
Bee 8 000 R6 R48 000
Cee 8 000 R3 R24 000
26 000 R172 000
The sales margin mix variance = (actual sales quantity – actual sales quantity in budgeted
proportion) × standard margin
Actual sales
Difference in Unit Standard
Actual sales quantity in
Product standard contribution margin mix
quantity budgeted
margin margin variance
proportion
Axe 10 000 (38%) 13 000 −3 000 10 R30 000 U
Bee 8 000 (31%) 8 667 −667 6 4 002 U
Cee 8 000 (31%) 4 333 +3 667 3 11 001 F
26 000 26 000 23 001 U
This variance indicates that a larger proportion of product Axe and Bee was sold, and less of
product Cee. As Cee has a lower standard margin, the sales margin mix variance was
unfavourable.
The sales quantity variance = (actual sales quantity in budgeted proportion – budgeted sales
quantity) × standard margin
Actual sales
Difference in Unit
Actual sales quantity in Sales quantity
Product standard contribution
quantity budgeted variance
margin margin
proportion
Axe 13 000 12 000 +1 000 10 R10 000 F
Bee 8 667 8 000 + 667 6 4 002 F
Cee 4 333 4 000 +333 3 999 F
26 000 24 000 15 001 F
The sales quantities of all three products were higher than expected, which gave rise to a
favourable sales quantity variance.
It can therefore be seen that the sales quantity variance is R15 001 (favourable) while the
sales mix variance was R23 001 (unfavourable).

As can be seen from the above examples, analysis of the variances in a standard costing system is
important to reveal why budgeted profit is different from actual profit. This information can be very
important to highlight areas that require attention.

CLOSING PRACTICAL EXAMPLE


In preparation for the annual general meeting of Promix Ltd, the CEO had asked the senior cost
accountant why the company’s profit figures were 10 per cent lower than the budgeted figures.
The senior cost accountant calculated the variances from the standard costing system used by
the company. He was then able to give the following analysis to the CEO in preparation for the
AGM:
Budgeted profit 120 000
Labour cost variance 20 000 U
Material quantity variance 10 000 F
Fixed cost variance 2 000 U
Actual profit 108 000
The senior cost accountant explained that the unfavourable labour cost variance was due to an
unexpectedly high wage increase negotiated with the union for the year. The favourable material
quantity variance was due to an improvement in waste product management, which allowed certain
products to be reused. Finally, the fixed cost variance was due to a larger than expected increase
in the cost of the cleaning services used in the factory.
This information was used by the CEO to explain the variance to the company’s shareholders at
the AGM.

12.6 Summary
In this chapter, we considered a standard costing system, which is a system whereby standard costs
are determined based on expected usage and expected costs. These budgeted amounts can then be
compared with the actual results.
Actual figures and predetermined standards are usually available or can be derived from financial
reports drawn up annually. A standard analysis can be readily carried out whenever necessary and
positive or negative variances can be calculated. These variances can assist the management of an
entity to identify problem areas that require attention.
Standard costing should be seen as an ongoing rather than a one-off process. If standard costs
become outdated over time, the variances will become increasingly irrelevant as costs increase.

PRACTICAL ACTIVITY
Gillian and Stephanie both have a flair for the production of modern jewellery. After testing the
market, they decided to form a small private company and pool their resources. In addition to their
existing ranges, they decided to introduce a line of ethnic jewellery, which has become very
popular.
After a year’s operation, their product ranges have become so popular that they decided to
employ a third staff member, Zandile, to be responsible for the ethnic range and to assist with
general office administration.
At the end of the last financial year, they were disappointed to find that their operating profit
had dropped by approximately 20 per cent since the previous year. They realised that should this
trend continue, they would be forced to downsize and maybe close down altogether.
The owners of the company hear that you have recently completed your national diploma in cost
and management accounting, and approach you to help them turn the business around. You notice
that the company uses no specific costing methods to control the purchase and use of material, nor
is there any system to deal with overheads. In some cases, you notice that emergency inventory has
been purchased from a selection of suppliers at a higher price per unit than that of their usual
suppliers. You also notice that selling prices are regularly adjusted to suit the demand at the time.
In some cases, overheads have not been included in the cost per unit.
You conclude that there are virtually no control systems present, and while you realise that the
entire inventory system needs overhauling, you decide to implement a standard costing system –
mainly to control raw-material purchases and use, and overheads.
Explain how you would implement this system showing the owners the use of variances to
calculate the differences between budgeted and actual information.

Short questions
1. Mantis Manufacturing Company used 35 000 actual labour hours during the period and paid a
rate of R13,00 per hour for this work. The labour efficiency variance was R15 600
favourable. Calculate the standard hours that should have been worked for the same period.
2. Twofold Production Company uses a standard costing system. The following information is
provided:
Budgeted sales 15 000 units
Budgeted selling price per unit R30
Standard variable cost R11,20 per unit
Actual sales amount to 13 500 units and the actual selling price per unit is R24. Calculate the
following variances:
• Sales margin price variance
• Sales margin volume variance
3. Aegis Chemical Plant provides the following standards per finished product for direct
materials and direct manufacturing labour:
Direct materials: 20 kg at R6 per kg R120
Direct manufacturing labour: 0,5 hours at R40 per hour R20
The company budgeted for 20 000 units to be completed.
The actual results were:
Material used: 196 146 kg
Direct labour: 9 800 hours at a total cost of R205 800
19 620 units were produced during the month
During the month, materials purchased amounted to 200 000 kg at a total cost of R620 000.
Assume that there was no opening or closing inventory.
• Calculate the price and efficiency variances for direct materials and direct labour for the
month of February.
• Comment on the price and efficiency variances for February.
4. The standard cost card for product Rubika in terms of sales figures is:
Standard sales volume 10 000 units
Standard price per unit R10
The actual cost card shows the following:
Actual sales volume 9 500 units
Actual price per unit R9,50
Prepare the following variances:
• Sales margin price variance
• Sales margin volume variance
(Source: Adapted from CIMA Pilot Paper, May 2004)
5. Maxwell Manufacturing has operated a standard costing system successfully for a number of
years.
The standard cost card for one of its products, C2X, is as follows:
Expected production for the period is 25 000 units.
Each unit is expected to take 30 minutes to manufacture.
Labour is paid at R15 per hour.
Variable overheads are paid at R10 per hour.
Actual data for the month of March 2012:
24 000 units were manufactured in March using 12 500 labour hours at a cost of R200 000.
Variable overheads cost R120 000.
Calculate the following variances:
• Labour rate variance
• Labour efficiency variance
• Variable overhead expenditure variance
• Variable overhead efficiency variance

Long questions
1. Ribok Shoe Manufacturing Company operates a standard costing system. The following
information is provided for the month of April 2012:
Standard card:
Selling price per unit R280
Direct material A12 kg at R12,50 per kg
Direct Material B 6kg at R6,50 per kg
Direct labour 9 hours at R8,50 per hour
Budgeted production and sales amount to 32 000 units
Fixed overheads amount to R1 200 000 for the year 2012 and are incurred evenly throughout
the year.
Actual data:
Production and sales 30 000 units
Selling price per unit R270
Direct material A 300 000 kg at R12 per kg
Direct material B 200 000 kg at R7 per kg
Direct labour 260 000 hours at R9 per hour
Actual fixed overheads for the month of April R80 000
a. Calculate and explain the following variances for April 2012:
• Material price variance
• Material quantity variance
• Labour rate variance
• Labour efficiency variance
• Fixed overhead expenditure variance
• Sales margin price variance
• Sales margin volume variance
b. Prepare a statement reconciling the difference between the budgeted profit and the actual
profit for April 2012
(Source: Adapted from MUT)

2. Focus (Pty) Ltd operates a standard costing system. Standard cost card information relating to
product Zap is provided below:
Unit cost
Direct material 6kg @ R4 per kg R24
Direct labour 1 hour @ R7 per hour R7
Variable overhead – 1 hour @ R3 per hour R3
Total variable production cost per unit R34
Budgeted fixed costs per month came to R100 000, while budgeted production was 20 000 units
per month.
Actual production and costs for the period were as follows:
Direct material purchased and used 113 500 kg R442 650
Direct labour 17 800 hours R129 940
Variable production overheads R58 800
Fixed production overheads R104 000
Actual production of Zap for the month was 18 000 units.
Calculate the following variances:
• Material price variance
• Material quantity variance
• Labour rate variance
• Labour efficiency variance
• Variable overhead expenditure variance
• Variable overhead efficiency variance
• Fixed overhead expenditure variance
(Source: Adapted from MUT)

3. P Wood (Pty) Ltd uses a standard costing system in the manufacture of bar stools.
Data for the standard cost per unit is as follows:
Direct material 30 metres of wood at R50 per metre
Direct labour 4 hours at R30 per hour
Variable overheads 4 hours at R0,50 per labour hour
Fixed overheads 4 hours at R15 per labour hour
Units budgeted for the period 480 bar stools
Actual results for the period:
Direct material 15 500 metres of wood at a total cost of R2 340 500
Direct labour 1 950 hours at a total cost of R60 450
Variable overheads R897
Fixed overheads R31 000
500 bar stools were manufactured
Calculate and explain the following variances:
• Material price variance
• Material quantity variance
• Labour rate variance
• Labour efficiency variance
• Variable overhead rate variance
• Variable overhead efficiency variance
• Fixed overhead expenditure variance

4. Alpha Manufacturing Company operates a standard costing system. The following


information relates to the three-month period ending 28 February 2011.
Standard cost card for the period (as the operation is mostly labour-intensive, overheads are
absorbed using direct labour hours):
Direct material per unit 20 kg at R0,50 per kg
Direct labour per unit 16 hours at R2,80 per hour
Variable manufacturing overheads 16 hours at R0,30 per hour
Fixed manufacturing overheads R80 000 per quarter
Production 10 000 units
The actual operating results for the period are as follows:
Direct material purchased and issued to
178 000 kg at R0,54 per kg
production
Total direct labour cost for the period 67 000 hours at R2,88 per hour
Variable overhead cost R18 090
Fixed overheads R7 400
Completed units 8 000
Calculate and explain the following variances:
• Material price variance
• Material quantity variance
• Labour rate variance
• Labour efficiency variance
• Variable overhead expenditure variance
• Variable overhead efficiency variance
• Fixed overhead expenditure variance

5. Remak Manufacturing Company manufactures chemicals for use in the petroleum industry.
The following standard card is provided for May 2012:
X: 12 litres at R5,20 per litre
Direct materials
Y: 13 litres at R4,20 per litre
Direct labour 4 hours at R60 per hour
Variable overheads 4 hours at R20 per hour
Overheads are allocated according to direct labour hours.

Budgeted information for May 2012


Budgeted sales 500 litres at R700 per litre
X: 6000 litres at R5,20 per litre
Direct materials
Y: 6500 litres at R4,20 per litre
Labour 2000 hours at R60 per hour
Variable overheads 2000 hours at R20 per hour
R1440 000 per annum spread evenly throughout
Fixed overheads
the year
Actual data for May 2012
Actual sales 400 litres at R725 per litre
X: 5000 litres at a total cost of R25 000
Materials
Y: 5000 litres at R20 000
Labour 1800 hours at a total cost of R117 000
Variable overheads Total cost of R41 400
Fixed overheads for May 2012 R80 000
Calculate all the necessary variances for materials, labour, variable overheads and fixed
overheads.
Prepare a reconciliation statement showing the difference between budgeted and actual
profit.
(Source: Adapted from MUT)

6. TC Floor Tiles (Pty) Ltd manufactures floor tiles in a material mix as follows:
Material Quantity (kg) Price (R) Amount (R)
A 40 1,50 60
B 30 1,20 36
C 10 1,40 14
D 20 0,50 10
Total 120
Each mix will produce 800 floor tiles. During January, 60 mixes were processed and the actual
output was 46 400 tiles from an input of:
Material Quantity (kg) Price (R) Amount (R)
A 2 200 1,60 3 520
B 2 000 1,10 2 200
C 500 1,50 750
D 1 400 0,50 700
Total 7 170
a. Calculate for each material:
• price variance
• mix variance
b. Calculate the yield variance.
(Source: Adapted from CIMA Pilot Paper (May 2004))

7. Maxwell Manufacturing has operated a standard costing system successfully for a number of
years. The standard card for one of its products C2X is as follows:
Expected production for the period is 25 000 units.
Each unit is expected to take 30 minutes to manufacture.
Labour is paid at R15 per hour.
Variable overheads are paid at R10 per hour.
Actual data for the month of March 2012
24 000 units were manufactured in March using 12 500 labour hours at cost of R200 000.
Variable overheads cost R120 000.
Calculate the following variances:
a. Labour rate variance
b. Labour efficiency variance
c. Variable overhead expenditure variance
d. Variable overhead efficiency variance

8. Tempo manufactures a product Serafin using a mix of two raw materials Alpha and Beta.
The standard mix for Serafin is as follows:
Raw material Units Price
Alpha 60 R3,25/unit
Beta 40 R6,50/unit
The standard loss per mix is 30 per cent. During March 60 mixes were processed. The actual
mix data is as follows:
Raw material Units Price Amount (R)
Alpha 3 200 R3,25/unit 10 400
Beta 2 600 R6,40/unit 16 640
Actual loss (1 740) 27 040
4 060
Calculate:
a. Direct material mix variance
b. Direct material yield variance

9. Zastron Chemicals manufactures and sells three different types of industrial cleaning liquid.
The following standard information and actual information for the month of February 2013 is
provided:
Budgeted sales
Standard selling Standard variable Actual sales
Type standard
price (per litre) cost (per litre) (litres)
quantities (litres)
A 60 000 (30%) R200 R80 R72 000
B 100 000 (50%) R160 R50 R84 000
C 40 000 (20%) R140 R44 R36 000
Calculate:
a. Sales margin mix variance
b. Sales margin quantity variance

10. Ronaldo Confectionary bakes and sells three different types of chocolate cake, namely white
chocolate cake, brown chocolate cake and dark chocolate cake. The following information is
provided for the month of January 2013:
Budgeted sales
Standard selling Standard variable Actual sales
Type standard
price (units) cost (per unit) (units)
quantities
W 30 000 (50%) R45 R15 40 000
B R40 R15 25 000
D R35 R15 15 000
Calculate:
a. Sales margin mix variance
b. Sales margin quantity variance
13 Activity-based costing
Peter Kamala

LEARNING OUTCOMES
After studying this chapter, you should be able to:
• define an activity-based costing (ABC) system;
• discuss the advantages and disadvantages of an ABC system;
• understand the steps followed in implementing an ABC system;
• compare an ABC system with a traditional volume-based system; and
• discuss when an ABC system would be appropriate for an entity.

PRACTICAL EXAMPLE
J&B Bank has increasingly replaced its human tellers with automated teller machines (ATMs). The
effect of this trend has been a reduction in labour costs. However, the machine overheads, such as
depreciation and maintenance, have increased dramatically. The bank has found that maintaining
the ATMs costs a lot of money and it needs to consider how to allocate the cost of the ATMs
accurately to each customer who uses the ATM in order to know how much each customer’s ATM
transactions cost the bank.
The bank has been considering how to allocate these costs and has come to the conclusion that
it would be inaccurate to simply divide the ATM costs equally among all ATM customers, as some
customers use the ATMs more frequently, and for more complex transactions than others.

13.1 Introduction
Traditional volume-based costing systems allocate overheads to cost objects, such as products or
services, on a single volume-related basis such as direct labour hours or machine hours. The main
assumption with volume-based costing is that overheads are related to the number of labour or
machine hours. In the modern environment, however, where overheads (indirect costs) are no longer
necessarily related to the number of hours, using a volume-related basis to allocate overheads may
distort the product costs, as it seldom meets the cause-and-effect criteria that may be required for
accurate cost allocation. As a result, the traditional volume-based costing systems tend to over-cost
high-volume products and under-cost low-volume products.
Because of increased mechanisation, among other things, a larger portion of organisations’ total
costs is now made up of indirect costs. This, together with the increase in the product range produced
or sold by a typical enterprise, which may vary significantly in the use of resources, has necessitated
the development of a more sophisticated overhead allocation system – one that recognises that
overheads are no longer always related to the volumes produced. It is in response to this need that
activity-based costing (ABC) has emerged as an alternative approach for allocating overheads more
accurately according to the activities consumed by each product or service rendered.
ABC is a sophisticated, more detailed way of allocating overheads to cost objectives such as
products, customers and services. ABC does not affect the direct costs, as they are traceable to the
cost objectives. However, it allocates the overheads to the cost objectives based on the activities that
drive (i.e. cause) these costs. ABC assumes that specific activities cause costs, and that outputs are
achieved through these activities. Costs are therefore allocated on the basis of the activities required
to produce the outputs.
The factors that cause overhead costs to be incurred are called cost drivers. Examples of cost
drivers are the number of material movements, number of inspections, number of machine setups,
number of orders received and number of batches processed (or manufactured). Different costs that
have the same cost driver are typically grouped (pooled) together in what is referred to as a cost
pool. The costs in each cost pool are then assigned to a cost objective according to how much it
makes use of the relevant cost driver.
ABC is equally applicable to manufacturing and service businesses. ABC has also been widely
applied in state entities, and non-governmental organisations, depending on the level of sophistication
that an organisation requires and can afford. Typically, ABC is particularly suitable in environments
where competition is intense, where pricing is based on product or service cost and where a large
portion of total cost is made up of indirect costs (due to mechanisation or simply the nature of the
industry). In the banking sector, for example, human tellers have been increasingly replaced by
ATMs, as illustrated in the opening example case study.

13.2 Definitions relating to ABC


In this section, we consider some of the key ideas and definitions that are used when discussing an
ABC system.

Activity-based costing is an ‘approach to the costing and monitoring of activities which involves
tracing resource consumption and costing final outputs. Resources are assigned to activities, and
activities to cost objects based on consumption estimates. The latter utilise cost drivers to attach
activity costs to outputs.’ (Chartered Institute of Management Accountants, CIMA Official
Terminology, Oxford: CIMA Publishing, 2005: 1)

From this CIMA definition, it is clear that in an ABC system there are two key concepts – activities
and cost objectives (or objects).

An activity refers to a task, action or unit of work carried out in an organisation. Examples are
maintenance, materials handling, setting up machines, ordering, verifying of orders, machining, and
so on.

The second concept, a cost objective, refers to a product, customer, service, department, customer
group, branch, etc., for which costs are accumulated or measured. Simply put, a cost object is
anything that management needs to determine a separate cost for.
13.3 Advantages and disadvantages of ABC
The advantages and disadvantages of the ABC system need to be carefully considered before
deciding whether it should be introduced or not.

13.3.1 Advantages of ABC


• The operations of modern enterprises have become more complex, with wider product ranges,
shorter product life cycles and more complex processes. By using multiple cost drivers, ABC
takes into account this complexity and leads to a better understanding of overheads.
• Given that ABC requires companies to identify the cost drivers, it affords an organisation the
opportunity to control the incidence of the cost drivers, hence the overhead costs become more
controllable.
• ABC helps identify and eliminate non-value-adding activities, or activities that do not contribute
to the final value of the product or process. Examples of non-value-adding activities include
needless inspections and duplicate processes. In so doing, ABC facilitates process improvement,
as it makes visible wasteful and non-value-added activities. This aspect improves profitability
and is also compatible with other innovative and continuous improvement programmes, such as
value chain analysis.
• By allocating costs accurately, ABC reduces cross-subsidisation, which makes visible the less
profitable products that should be eliminated from the product line, thereby increasing the
profitability of an organisation without increasing prices. Furthermore, ABC facilitates the
allocation of resources to the more profitable items, or items that use fewer resources. In the
service sector, ABC can be used for customer profitability analysis by revealing the per person
contribution to the profits.

13.3.2 Disadvantages of ABC


• It is a daunting task to implement an ABC system, and it requires substantial resources, and once
implemented, an ABC system is costly to maintain. Data concerning numerous activity measures
must be collected, checked and entered into the system.
• Given that ABC breaks down the overheads into various cost pools, it tends to generate time-
consuming and unnecessary details that not only obscure the bigger picture, but may also incur
costs that outweigh the benefits.
• ABC is not relevant to all organisations. Some organisations introduce ABC erroneously because
it is perceived as fashionable, and not because it provides meaningful additional information. In
many cases, the cost of implementing ABC far outweigh the benefits. The key determining factors
on whether it should be implemented are how many product lines there are, the nature of the
industry, what the costing information is used for, etc. For example, if insignificant inventory
levels are held and cost-plus pricing is not used, ABC may be unnecessary. More importantly, if
only one product or service is produced, ABC is completely irrelevant, as allocation of overheads
is not required.
• ABC still requires some measure of arbitrary allocation of costs, as it uses one cost driver per
cost pool, but one cost driver may not be adequate to explain the cost behaviour in a cost pool.
• ABC makes waste visible, and, therefore, some executives and managers may oppose it given that
it exposes their inefficiency.

From the above, it is clear that implementing ABC should be on the basis of cost–benefit analysis:
depending on the nature of the organisation, ABC may be crucial in some cases, but a waste of
resources in others. Again, depending on the cost–benefit analysis, organisations can choose how
homogeneous or diverse the costs in a cost pool should be.

13.4 Implementing an ABC system


There are four main steps involved when an organisation implements an ABC system. These are
discussed in the following sections.

13.4.1 Step 1: Identifying the organisation’s main activities and


grouping them into cost pools
An organisation must identify all its major activities. This may require interviewing workers,
supervisors and managers in the main departments, and determining the activities that they perform.
This often results in a very long list of small tasks, which are then grouped into meaningful activities
according to similarity. For example, all tasks involving receiving, moving and storage of materials
could be grouped together as materials handling. Their costs would therefore form a single cost pool.
Cooper & Kaplan (1999) developed a useful hierarchy for guiding organisations on how to group
activities into four levels, namely unit, batch, product and facility. The following section explains
this hierarchy.

13.4.1.1 Unit-level activities


The unit-level activities are performed each time a unit of a product is produced or a service is
rendered. The number of times unit-level activities (such as drilling holes or inspecting parts) are
performed varies according to the number of units produced, and are referred to as volume-related
activities. Accordingly, the costs related to unit-level activities will vary with the volume of products
produced or services rendered. Examples of such costs include all variable manufacturing overheads.

13.4.1.2 Batch-level activities


Batch-level activities are performed each time a batch of goods is produced. The number of times
batch-level activities (such as setting up a machine) are performed varies according to the number of
batches made and not according to the number of individual units produced. The costs of batch
activities can be assigned to individual batches but they are fixed regardless of the number of units in
the batch. Examples of batch-level activities include processing 20 applications for loans by a bank
at once or, in the case of a manufacturing enterprise, a set-up of machines, processing purchasing
orders and inspections. The related costs would include machine set-up costs, costs of processing a
purchasing order and costs of inspection of each batch, which are incurred in proportion to the
number of batches handled.
13.4.1.3 Product-sustaining activities
Product-sustaining activities are performed to support product or service lines. The related costs are
incurred regardless of the number of units or batches handled. Therefore, the costs related to these
activities are incurred to maintain individual products and product ranges, and include costs such as
design and marketing costs, which are not proportional to the number of units produced or batches
handled.

13.4.1.4 Facility-sustaining activities


Facility-sustaining activities are performed to maintain the organisation in general. In the short-term,
such activities are performed regardless of the number of units or batches produced or handled, and
regardless of the number of different products produced or services rendered. Accordingly, it is often
difficult to identify a cost driver that represents the cause-and-effect relationship between the activity
and the costs incurred. For this reason, facility-sustaining costs are usually not allocated to products
or services, but instead are treated as period costs.

13.4.2 Step 2: Identifying cost drivers for each activity


A cost driver is identified for each cost activity. It is vital that the cost driver represents a cause-and-
effect relationship between the activities in the cost pool and the cost incurred. In other words, for a
cost driver to be suitable, a direct relationship must exist between the cost driver and the activity
performed. Table 13.1 provides examples of activities, cost drivers and activity levels that are
relevant in a bank.

Table 13.1 Activities, cost drivers and activity levels in a bank


Activities Possible cost drivers Activity level
Accepting cash Number of transactions Unit level
Processing of cash deposit by
Number of transactions Unit level
bank
Supervisor’s review of clerks’
Number of reviews Batch level
deposits
Review and transfer of funds Number of accounts Batch level
Maintenance charges for bank
Number of accounts Product-sustaining
accounts
Reconciling bank accounts Number of accounts Product-sustaining
Number of advertisements on
Marketing the bank’s services Facility-sustaining
national television

Similarly, Table 13.2 provides examples of activities, cost drivers and activity levels applicable to a
manufacturing enterprise.

Table 13.2 Activities, cost drivers and activity levels in a manufacturing environment
Activities Cost drivers Activity level

Production by machines Machine hours Unit level


Production by workers Direct labour hours Unit level
Ordering Number of orders Batch level
Setting up Number of set-ups Batch level
Quality control Number of inspections Batch level
Maintenance Maintenance hours Product-sustaining
Accounting services Head count Facility-sustaining

The cost drivers can be further categorised as follows:


• Transaction driver. This is the most common type of cost driver and is based on the assumption
that overhead costs are driven by the number of times an activity is performed. For example, if the
ATM maintenance costs were found to increase with the number of withdrawals, then the number
of withdrawals would be a transaction driver.
• Duration driver. This is used where the overhead costs vary with the length of time it takes to
perform an activity. For example, if the ATM maintenance costs were found to increase with the
duration of a transaction, then the cost of maintenance could be allocated to customers on the basis
of machine hours, as the longer the customer transacts on the ATMs, the higher the maintenance
costs.
• Intensity driver. This type of cost driver recognises that certain types of products or services may
use resources more intensively than others. For instance, if a client uses an ATM machine to print
bank statements, this service should cost more than checking the account balance on the ATM
monitor.

We will now illustrate the first two steps in implementing an ABC system by means of an example.

EXAMPLE 13.1: ESTABLISHING ACTIVITY LEVELS


Sarova Ltd makes three types of sports shoes – Drexlers (D), Clydes (C) and Pippens (P). A
traditional product-costing system is used at present. The company is considering implementing
an ABC system. Details of the three products for a typical period are given in the table below.
Total production overheads are R1 309 000 and further analysis shows that the total production
overheads can be apportioned as follows depending on the activities that cause them:
Costs and related activities %
Costs relating to set-ups 35
Costs relating to running production machines 20
Costs relating to inspection 30
Costs relating to materials handling 15
100
Required
Classify each of the activities according to the cost hierarchy explained on pages 257–258, and
suggest a possible cost driver for each.

Solution
On the basis of the information provided, the following would be a reasonable classification:
Activities Cost drivers Activity level
Setting-up Number of set-ups Batch level
Running production machines Machine hours Unit level
Inspecting Number of inspections Batch level
Number of movements of
Materials handling Batch level
materials

13.4.3 Step 3: Calculating activity rates


The activity rate is calculated by dividing the total cost per activity cost pool by the organisation’s
capacity for that cost driver. For instance, the activity rate could be R50 per order, where the number
of orders (200) is the cost driver, and the ordering cost is R10 000.

13.4.4 Step 4: Allocating costs of each cost pool to cost objects


The amount of overheads allocated to the cost object depends on the number of the cost drivers that a
cost object consumes. The overheads are allocated by multiplying the activity rate by the number of
the cost drivers that a cost object consumes.

EXAMPLE 13.2: ACTIVITY RATES AND ALLOCATING COSTS


Continuing from Example 13.1, the following additional information is provided for Sarova Ltd.
Details of the three products for a typical period are as follows:
Machine hours Labour hours per Material cost per
Products Production units
per unit unit unit (R)
D 3 3 40 1 500
C 3 2 24 2 500
P 2 6 50 14 000

Direct labour costs R12 per hour and production overheads are absorbed on a machine-hour
basis. The overhead absorption rate is based on machine hours.
The following total activity volumes are associated with each product line for the period as a
whole:
Number of movements Number of materials
Product Number of set-ups
of materials inspections
D 150 24 300
C 230 42 360
P 960 174 1 340
1 340 240 2 000

Required
1. Calculate the activity rates for each activity.
2. Allocate the costs of each cost pool to cost objects.

Solution
1. Calculating activity rates for each activity Overheads
Total number of Cost per driver
Overheads Total cost (R)
drivers (R)
Costs relating to set-ups 458 150 1 340 341,90
Costs relating to machinery 261 800 40 000 6,55
Costs relating to inspection 392 700 2 000 196,55
Costs relating to materials handling 196 350 240 818,13
The activity rate is calculated by dividing the total cost per activity cost pool by the
organisation’s capacity (total number of cost drivers) for that cost driver. This comes to R341,90
per set-up, R6,55 per machine hour, R196,55 per inspection and R818,13 per material movement.

2. Allocating costs of each cost pool to cost objects


Overheads D C P Total
Costs relating to
51 285,44 78 637,68 328 226,88 458 150
set-ups (R)
Costs relating to
29 452,50 49 087,50 183 260 261 800
machinery (R)
Costs relating to
58 905 70 686 263 109 392 700
inspection (R)
Costs relating to
materials handling 19 635 34 361,25 142 353,75 196 350
(R)
Total overheads
159 277,94 232 772,43 916 949,63 1 309 000
(R)
Units produced 1 500 2 500 14 000
Overhead cost
106,19 93,11 65,50
per unit (R)
Each activity rate is multiplied by the number of cost drivers consumed by each product. For
instance, for Drexlers, the set-up cost is calculated by multiplying the activity rate of R341,90 by
150, which is the number of set-ups required by the product. This gives 51 285,44. For each
product, the cost of all activities are totalled, and then divided by the number of units produced to
allocate the overheads per unit of each product.

13.5 Comparing ABC and traditional volume-based costing systems


The principal difference between ABC and volume-based costing is the way in which overheads are
allocated to products. While traditional volume-based costing systems most commonly use two
allocation bases, namely direct labour hours and/or machine hours to allocate overheads to cost
objects, ABC uses many cost drivers, such as the number of orders, number of inspections, number of
set-ups, and so on, as bases for allocation. Therefore, the allocation of overheads is more closely
linked to the causes of the costs.
ABC allocates costs that vary with production volume by using volume-related drivers, such as
direct labour hours and direct machine hours. These are usually short-term variable overheads.
Overheads that do not vary with the volume of output but rather with some other activity are allocated
to products using transaction-based cost drivers, such as the number of set-ups and number of orders.
Given that traditional volume-based costing systems assume that all overheads are volume-related,
they arbitrarily allocate non-volume-related costs, which leads to less accurate product costs than
with the ABC method.
In the following examples, the traditional system is compared with the ABC method.

EXAMPLE 13.3
Comenc Ltd manufactures two products – X and Y. The company previously operated a
traditional volume-based costing system and allocated overheads to products using a single basis,
namely machine hours. The firm has recently introduced an ABC system. The additional
information is provided below:
Product X (R) Product Y (R) Total (R)
Direct materials per unit 300 300
Direct labour cost per unit 200 200
Overheads:
Machining costs 440 000
Purchasing costs 360 000
Set-up costs 300 000
Total overheads 1 100 000
Units manufactured 1 000 10 000 11 000
Machine hours1 4 000 40 000 44 000
Purchase orders 160 320 480
Set-ups 80 120 200
Required
1. Calculate the cost per unit of each product using the traditional volume-based costing system.
2. Calculate the cost per unit of each product using ABC principles.
3. Compare and comment on the unit costs determined by the two systems.

Solutions
1. Traditional volume-based costing system
Product X (R) Product Y (R)
Direct materials per unit 300 300
Direct labour cost per unit 200 200
Overheads (R1 100 000 ÷ 44
100 100
000) × 4
Unit production cost (volume-
600 600
based costing)
2. ABC system
Product X (R) Product Y (R)
Direct materials per unit 300 300
Direct labour cost per unit 200 200
Machining costs2 40 40
(R440 000 ÷ 44 000) × 4
Purchasing costs
(R360 000 ÷ 480) = R750 per 120 24
purchase order3
Set-up costs
(R300 000 ÷ 200) = R1 500 per 120 18
set-up4
Unit production cost (ABC) 780 582

Workings
1. Product X: 4 000 machine hours ÷ 1000 units = 4 machine hours per unit.
Product Y: 40 000 machine hours ÷ 10 000 units = 4 machine hours per unit.
2. The cost driver for machining costs is machine hours.
3. Product X: R750 × 160 ÷ 1 000 = 120; Product Y: R750 × 320 ÷ 10 000 = 24.
4. Product X: R1 500 × 80 ÷ 1 000 = 120; Product Y: R1 500 × 120 ÷ 10 000 = 18.

3. Comparison and comment on unit cost determined by using the two systems
Product X Product Y
Production 1 000 10 000
Unit production cost (volume-
600 600
based costing) (R)
780 582
Unit production cost (ABC) (R)
The traditional volume-based unit cost of product X is the same as that of product Y despite the
fact that the volumes of product Y are 10 times greater than those of product X. This is in
accordance with the notion that traditional volume-based costing tends to over-cost high-volume
products and under-cost low-volume products. ABC corrects this distortion in costs by showing
that the unit cost of product X (a relatively low-volume product) is higher than the unit cost of
product Y (a relatively high-volume product).

13.6 When is it appropriate to introduce ABC?


ABC should be introduced only if the additional information it provides will lead to an increase in
the organisation’s overall profitability. This is most likely to occur when the results of an ABC
analysis vary significantly from those of a traditional volume-based costing analysis, in a manner that
improves the cost allocation. Such an improvement could lead to better management decisions,
which, in turn, may improve profitability – for example, by lowering costs or facilitating better
pricing decisions.
Factors that would cause variation between the results of the two costing systems include the
following:
• Production overheads are high in relation to direct costs, such as direct labour costs. This could
be due to mechanisation of production where machines replace direct labour hours.
• Organisational activities are not related to production volume, such as routine maintenance of
ATMs, and depreciation calculated on a straight-line basis.
• Overhead resource input varies significantly across the product range – in other words, where the
requirements for products or services vary significantly.

Other indicators that would favour implementation of an ABC system are as follows:
• When line managers doubt the integrity of product costs.
• If the marketing department rejects reported product costs as a basis for pricing products.
• Divisional profit margins are difficult to explain.
• Certain products have suspicious profit margins.
• Unit cost of low-volume products appears to be lower than that of high-volume products.

13.7 ABC in service and retail organisations


Although ABC was first used in manufacturing, it is equally suitable in service organisations, such as
banks and post offices, where it can be used to determine the service costs associated with specific
customers or customer groups, or in comparing the profitability of different customers – also known
as customer profitability analysis. In this case, the customer is the cost object, but the service
organisations can also use the service itself (or any other relevant element) as the cost object.
ABC can also be used in the retail industry where it can be applied to determine the service costs
associated with specific departments or product lines. In this case, the individual retail departments
would be the cost objects.
CLOSING PRACTICAL EXAMPLE
In the opening example, it was explained that J&B Bank has increasingly replaced its human tellers
with ATMs. The effect of this trend has been a reduction in labour costs. However, the machine
overheads, such as depreciation and maintenance, have increased dramatically. The bank has found
that maintaining the ATMs costs a lot of money and, accordingly, it needs to consider how to
allocate the cost of the ATMs accurately to each customer who uses the ATM in order to know
how much each customer’s ATM transactions cost the bank.
After considering the matter, the bank’s financial managers decided that instead of allocating
the costs evenly to all customers, a more accurate, sophisticated way of allocating ATM costs to
the customers would be according to the number of activities or transactions that each customer
performs with the ATM, as well as the complexity of each transaction. Using this method, the cost
per customer will correspond with the number and complexity of transactions conducted by the
customer using the ATM. This is an example of an ABC system.

13.8 Summary
This chapter discussed activity-based costing, a sophisticated way of allocating overheads (indirect
costs) to cost objects, in cases where a more accurate allocation of overheads is required. ABC is
particularly important in environments where competition is intense, where pricing is based on
product cost, where a large portion of total cost is made up of indirect costs, and where an enterprise
has a wide product range consisting of products that vary significantly in their degree of resource
consumption.
Unlike traditional volume-based costing systems, which allocate overheads to cost objects on a
single volume-related basis, such as direct labour hours or machine hours, which can result in an
inaccurate allocation, ABC uses multiple cost drivers to more accurately allocate costs of activities
to cost objects according to their consumption of those activities. This requires four steps, namely
identification of activities and grouping them into cost pools; identification of cost drivers;
calculation of activity rates; and allocation of activity costs to cost objects.
Although ABC provides a more sophisticated method for allocating overheads, the overheads
should be allocated only to the extent that they are relevant to the decision that management is facing.
Just like with traditional volume-based costing systems, the ABC system has its own limitations. It is
important to note that, regardless of the method of allocation used, the total amount of overheads
incurred by the organisation does not change. Different amounts of overheads may be allocated to cost
objects depending on the basis used, but the sum of the overheads remains unaffected by a change in
allocation.
However, by using an ABC system, an enterprise may save its overhead costs if it enables
management to make better decisions through improved understanding of how costs behave. In
addition, where selling prices are determined by applying a markup to cost, more realistic selling
prices based on more accurate product costs may lead to improved profitability for the organisation.

Questions
1. Using the information given in Example 13.2:
a. Calculate the overhead cost per unit for each product using traditional volume-based
costing systems, on the basis of machine hours.
b. Compare the overhead cost per unit of each product found in Question 1 a., using the
traditional volume-based costing system, with the cost per unit for each product using
ABC principles found in Example 13.2.
c. Compare the total cost per unit using the traditional volume-based costing system with the
total cost per unit using ABC.

2. Dhansay Ltd manufactures medical equipment. Its two products, A and B, are made
specifically for use in doctors’ rooms. The company currently uses traditional volume-based
costing. The chief financial officer has recently returned from a conference where he learnt
that ABC is a more accurate system of allocating overheads where a company manufactures
different products using machinery. He is considering implementing this system in his
company. He provides you, as the cost accountant, with the following information:
Product A (R) Product B (R) Total (R)
Direct material costs 240 000 1 440 000 1 680 000
Direct labour costs 200 000 1 800 000 2 000 000
Manufacturing
overheads:
Machine set-up 400 000
Design costs 1 200 000
Machining costs 2 200 000
Total product cost 7 480 000
Other data:
Units manufactured 4 000 36 000 40 000
Machine set-ups 40 60 100
Design hours 8 000 32 000 40 000
Machine hours 20 000 360 000 380 000
a. Calculate the cost per unit of each product using the traditional volume-based costing
system, using machine hours to allocate manufacturing overheads.
b. Calculate the cost per unit of each product using activity-based costing principles.

3. Black Ltd. manufactures two products (X and Y). The overhead costs have been divided into
four cost pools that use the following activity drivers:
Number of Number of Machine Packing
Product
set-ups orders hours orders
X 40 70 2 000 150
Y 10 140 3 000 250
Cost per pool R30 000 R16 800 R240 000 R80 000
a. Compute the allocation rates for each of the activity drivers listed.
b. Allocate the overhead costs to Products X and Y using activity-based costing.
c. Compute the overhead rate using machine hours under the traditional-based costing
system.
d. Allocate the overhead costs to Products X and Y using the traditional-based costing
system overhead rate calculated in part (c).

4. XYZ Ltd manufactures plastic components for the car industry. The following budgeted
information is available for three of the company’s key plastic components:
A B C
(R) per unit (R) per unit (R) per unit
Selling price 400 366 350
Direct material 100 80 70
Direct labour 60 70 60
Units produced and
20 000 30 000 36 000
sold
The total number of activities for each of the three products for the period is as follows:
Number of purchase requisitions
Number of purchase
2 400 3 600 4 000
requisitions
Number of set-ups 480 520 600
Overhead costs have been analysed as follows:
Receiving/inspecting quality assurance R2 800 000
Production scheduling/machine set-up R2 400 000
Using ABC, calculate the budgeted profit per unit for each of the three products.

5. Exotica Ltd manufactures two products, X and Y. At present, the company uses a traditional
absorption costing system based on production volumes to allocate overheads to products.
The budgeted costing information for 2013 is as follows:
Budgeted information Cost (R) Cost driver
Machine set-ups 110 000 Number of set-ups
Material procurement 128 000 Number of orders
Quality control 105 000 Number of inspections
Material insertion 1 600 Number of parts
Product X Product Y Total
Number of set-ups 1 000 100 1 100
Number of orders 12000 800 12 800
Number of inspections 18 000 3 000 21 000
Number of parts 256 64 320
Production volumes 10 000 30 000 40 000
a. Calculate the overhead cost per unit of each product using the absorption costing system.
b. Calculate the overhead cost per unit of each product using the ABC system.
c. Comment on the results of a. and b. above.
14 Direct and absorption costing systems
Jayan CV Mathew and Johan Hefer

LEARNING OUTCOMES
After studying this chapter, you should be able to:
• differentiate between direct costing and absorption costing;
• calculate unit cost using direct costing and absorption costing;
• list the benefits and shortcomings of direct costing and absorption costing;
• draft a statement of financial performance using direct costing and absorption costing; and
• reconcile the profits realised by the two methods.

PRACTICAL EXAMPLE
Long-John Ltd manufactures a range of shoes. The company is currently preparing both its
financial- and management-accounting information for the previous financial year. A friend of
John, the managing director, has suggested that he uses direct costing for the purposes of
management accounting, as it is a better cost-accounting tool, while his accountant has informed
him that he has to use absorption costing for the purposes of his annual financial statements. John is
not sure what the difference is and has discussed the matter with his cost accountant. The cost
accountant has said he will consider the matter and give him his recommendation.

14.1 Introduction
There are two main ways to value inventory in a costing system. The first is direct costing (also
sometimes referred to as variable or marginal costing); the other is absorption costing. The two
methods differ in the manner in which fixed costs are treated when determining the value of inventory.
When using direct costing, all variable manufacturing costs are included in the costs when
determining the value of inventory. Fixed manufacturing costs, however, are not included in the costs
used to determine the cost of inventory but treated as period costs. (Refer to Chapter 2, where these
and other cost concepts are discussed.) It should be remembered that non-manufacturing variable
overheads are not included in the value of inventory and are treated as period costs (period costs are
those costs that are directly allocated to expenditure).
Wi th absorption costing, on the other hand, all variable and fixed manufacturing costs are
included in the costs used to determine the value of inventory. Therefore, fixed costs are not treated
as period costs. Absorption costing is required by the financial-accounting and taxation rules when
organisations prepare their annual financial statements. These rules do not apply to cost accounting,
however, and managers need to consider the kind of information they need for decision-making
purposes. Direct costing often gives management more valuable financial information.
The difference in the way costs are allocated using direct and absorption costing is shown in
Figure 14.1.
Figure 14.1 Cost allocation using direct and absorption costing

We will now use a simple example to illustrate the two methods.

EXAMPLE 14.1
Long-John Ltd manufactures a range of shoes. The following information pertains to the 2014
financial year:
Variable manufacturing costs R350 000
Fixed manufacturing costs R250 000
Non-manufacturing costs R180 000
Number of units produced during the period 100 000
Number of units at the beginning of the period 0
Number of units on hand at the end of the period 1 500
Determine the value of closing inventory using the direct and absorption costing methods.

Suggested solution
Using absorption costing, the total costs used to determine the value of inventory for the period
would be R600 000 (R350 000 + R250 000). Therefore a unit would be valued at R6 (R600 000
÷ 100 000). The value of closing inventory is therefore R9 000 (1 500 × 6). The non-
manufacturing costs would be treated as a period cost.
If using direct costing, the total costs to determine the value of inventory for the period would
be R350 000, therefore a unit would be valued at R3,50 (R350 000 ÷ 100 000). The value of
closing inventory would be R5 250 and fixed manufacturing costs and non-manufacturing costs
would be treated as period costs.

It should be emphasised that businesses that use direct costing also use absorption costing. Direct
costing cannot be used for the business’s financial-accounting or tax purposes and therefore the
financial accounts would be prepared using the absorption-costing system. We will now discuss the
benefits and shortcomings of both direct and absorption costing in more detail.
14.2 Benefits and shortcomings of direct costing
As explained, direct costing does not include fixed overheads in the determination of the unit cost of a
product or service. Therefore, with direct costing, inventory value does not include the fixed
overhead component, which can be a significant part of the total cost.
One of the main benefits of using direct costing is that it is more relevant for decision making
because fixed costs are very difficult to change with the volume of production in the short term and
therefore direct costing can give a better sense of the impact of differing levels of production (and
sales) on the financial performance of the entity.
In the same way, direct costing also gives management more pertinent information about cost
control because fixed costs are often not controllable to a large extent. If direct costs are monitored or
variable costs increase, any inefficiencies can be identified and remedied earlier. If absorption
costing is used, however, these costs may be obscured.
A further benefit of direct costing is that it is relatively easy to calculate and it could be argued
that it is illogical to carry forward a portion of the current year’s fixed manufacturing overhead costs.
Direct costing is also used by management accountants when performing cost–volume–profit analysis
(See Chapter 19), which is a powerful tool for determining profitability levels and breakeven points
for different volumes.
One of the shortcomings of direct costing is that it excludes fixed cost from inventory valuation.
This leads to a different valuation of inventories of finished goods and work in process from that
produced by the absorption-costing method used for financial-accounting and taxation purposes. It is
therefore important for management to reconcile and understand the different net profits produced by
the two systems.
Before we move on, we need to have a clear understanding of the important concept of
contribution margin, which is applicable to direct costing only. Contribution margin plays a major
role in the study of direct costing. We have already seen that direct costing does not include fixed
overheads in the calculation of production cost. It however includes variable non-manufacturing
costs. In order to determine the profit made from selling the product the distinction between variable
costs and fixed costs is maintained. The difference between sales and variable costs is known as
contribution margin. The following equation can be used to calculate it:

Contribution margin = sales − variable costs.

The contribution margin therefore shows how much sales contribute towards paying off fixed costs
and making the net profit. Therefore the above equation can be rewritten as follows:

Contribution margin = net profit + fixed costs.

14.3 Benefits and shortcomings of absorption costing


Absorption costing is a system that takes into consideration all costs incurred by manufacturing
providing a product or a service, irrespective of the behaviour of the cost (for more on cost
behaviour, refer to Chapter 2). Absorption costing can therefore be said to give a better picture of a
product’s cost, as it recognises that fixed manufacturing costs were also incurred in its production.
On the other hand, it could be said that the inclusion of fixed costs in the value of inventory hides
the fact that these costs were incurred during the period. This is especially true if the volume of
production varied between two periods but the fixed costs (as would be expected) remained constant.
This could lead to very different unit costs between the two periods. It could then be argued that if the
fixed costs were treated as period costs (as in direct costing), it would give a more appropriate profit
figure.
In entities where production is constant but sales are irregular, fixed costs may be included as an
element in the unit cost of inventory. Due to the fact that it is included in the cost of inventory, the
fixed cost included in the inventory is treated as an expense only when the inventory is sold. In this
type of scenario, absorption costing may give a more accurate indication of profitability.
The major disadvantage of absorption costing for the purposes of managerial decision making is
that absorption costing includes fixed costs as part of the unit cost. Fixed costs are considered
irrelevant for decision making, as their incurrence does not depend on a financial decision. As an
example, if a decision needs to be made regarding whether to accept a special order or not, the fixed
rent of the factory will not change and therefore it is not a relevant cost for the purpose of deciding
whether to accept the special order or not (for further information on these types of decisions, see
Chapter 21).
Absorption costing can also artificially boost the profits in a period, as not all the fixed costs
incurred during the period are deducted if the firm has not sold all of its products. Therefore, the
profit-and-loss statement does not reflect all the expenses incurred for the period. This can give a
misleading picture to management when analysing the profitability. This disadvantage is especially
true where production and sales are not constant, as stated before.
One of the major considerations when using absorption costing is that the validity of product costs
when one uses absorption costing depends on the correct apportionment of overhead costs. In
practice, many overhead costs are apportioned using methods that make the product cost unreliable
and unrealistic. This could lead to misleading financial information. (Refer to the discussions on this
topic in Chapter 6 regarding overheads.)
We will now consider how to compile a statement of financial performance using direct and
absorption costing.

14.4 Drafting the statement of financial performance using direct


and absorption costing
Although services are produced and consumed simultaneously, in the case of products there is often a
time difference between manufacturing and selling. As a result, some of the units may not be sold in
the same period that they are manufactured. This causes a difference in the net profit calculated using
the direct and absorption costing methods, as fixed costs are treated as period costs in direct costing,
whereas they are included in the cost of inventory in absorption costing. If all the units manufactured
were sold in the same period, there would be no difference in the net profit calculated using each
method.

Figure 14.2 Statement of financial performance – direct and absorption costing


Using direct costing, the statement of financial performance (also referred to as the income
statement) clearly differentiates between variable and fixed costs. The fixed costs are treated as
period costs and are fully written off in the same period. We now look at how the statements of
financial performance are drawn up using the two methods, and how they differ. Figure 14.2 indicates
how the fixed costs are treated and how the information presented in the two methods is different.
Let us now draw up the statement of financial performance by means of an example.

EXAMPLE 14.2
Company A produces 3 000 units of Product Z. The following costs are incurred:
Direct material R2
Direct labour R4
Variable manufacturing overheads R2
Variable selling costs R1
Fixed manufacturing overheads R12 000
Fixed selling and administration cost R6 000
In addition to the information given, assume that the selling price of the product is R18 and all
units manufactured are sold.

Required
1. Calculate the unit cost using:
a. Direct costing
b. Absorption costing
2. Prepare the statement of financial performance using:
a. Direct costing
b. Absorption costing

Solution
1. a. Unit cost (direct costing)
Direct material R2
Direct labour R4
Variable manufacturing overheads R2
Unit product cost R8
b. Unit cost (absorption costing) Direct material
Direct material R2
Direct labour R4
Variable manufacturing overheads R2
Total variable production cost R8
Fixed manufacturing overheads
R4
(R12 000 ÷ 3 000 units)
Unit product cost R12
2. a. Statement of financial performance using direct costing
Sales (3 000 units × R18) 54 000
Less variable costs (27 000)
Opening inventory 0
Add variable manufacturing cost (3 000 units × R8) 24 000
Goods available for sale 24 000
Less closing inventory 0
Variable manufacturing cost of sales 24 000
Variable selling cost 3 000
Contribution 27 000
Less fixed costs (18 000)
Manufacturing overheads 12 000
Selling and administration cost 6 000
Net profit 9 000
b. Statement of financial performance using absorption costing
Sales (3 000 units × R18) 54 000
Less cost of sales 36 000
Opening inventory 0
Add manufacturing cost (3
36 000
000 units × R12)
Less closing inventory 0
Gross profit 18 000
Less non-manufacturing costs 9 000
Variable selling cost 3 000
Fixed selling and
administration costs 6 000
Net profit 9 000

We can see from these statements that when all units produced are sold, the net profit, as
determined by both methods, is the same.

We will use the following information in addition to the information given in Example 14.2 and
redo the statement of financial performance. The difference is that the units produced are not sold
in the same period. See Example 14.3.

EXAMPLE 14.3
Units manufactured 3 000
Closing inventory 200
Selling price R18
Statement of financial performance using direct costing
R R
Sales (2 800 units × R18) 50 400
Less variable costs (25 200)
Opening inventory 0
Add variable manufacturing cost (3 000 units × R8) 24 000
Goods available for sale 24 000
Less closing inventory (200 units × R8) (1 600)
Variable manufacturing cost of sales 22 400
Variable selling cost 2 800
Contribution 25 200
Less fixed costs (18 000)
Manufacturing overheads 12 000
Selling and administration costs 6 000
Net profit 7 200
Statement of financial performance using direct costing
Sales (3 000 units × R18) 50 400
Less cost of sales 33 600
Opening inventory 0
Production (3 000 units × R12) 36 000
Goods available for sale 36 000
Closing inventory (200 units × R12) 2 400
Gross profit 16 800
Less non-manufacturing costs 8 800
Variable selling cost 2 800
Fixed selling and administration cost 6 000
Net profit 8 000
From these statements, we can establish that the difference between the two profits is R800 (R8
000 − R7 200). This difference in the net profit produced by the two methods can be identified as
the difference in the value of closing inventory.
Closing inventory as per direct costing 1 600
Closing inventory as per absorption costing 2 400
Difference 800
This difference can be attributed to the fixed manufacturing overheads included in the value of
closing inventory using absorption costing. The fixed manufacturing overheads of R800 for the
current period is deferred in inventory until the next period, when the products will be sold. The
cost of the closing inventory using absorption costing can be further analysed to establish the
fixed cost included in it.
Variable cost of closing inventory (200 units ×
1 600
R8)
Fixed manufacturing overhead cost (200 units ×
800
R4)
Total closing inventory value 2 400
This means that with the absorption costing system, only R11 200 (2 800 units × R4) of the R12
000 total fixed manufacturing overheads will be incurred in the current period. This is because
only R11 200 is included in the cost of sales when using absorption costing. The remaining R800
(200 units × R4) will be incurred in the next period, when the inventory is sold.

Let us now look at a situation where the company has an opening inventory of 100 units and the same
production cost.

EXAMPLE 14.4
Statement of financial performance using direct costing
Sales (2 900 units × R18) 52 200
Less variable costs (26 000)
Opening inventory (100 units ×
800
R8)
Add variable manufacturing
24 000
cost (3 000 units × R8)
Goods available for sale 24 800

Less closing inventory (200


(1 600)
units × R8)
Variable manufacturing cost of
23 200
sales
Variable selling cost 2 800
Contribution 26 200
Less fixed costs (18 000)
Manufacturing overheads 12 000
Selling and administration
6 000
costs
Net profit 8 200
Statement of financial performance using absorption costing
Sales (2 900 units × R18) 52 200
Less cost of sales (34 800)
Opening inventory (100 units
1 200
× R12)
Production (3 000 units ×
36 000
R12)
Goods available for sale 37 200
Closing inventory (200 units
2 400
× R12)
Gross profit 17 400
Less non-manufacturing costs (8 800)
Variable selling cost 2 800
Selling and administration
6 000
costs
Net profit 8 600
Analysing the net profit of the two methods establishes that the difference between the profits is
R400:
Net profit with direct costing R8 200
Net profit with absorption costing R8 600
Difference R400
This difference in the net profit of the two methods can be identified by the difference in the
value of inventory.
Direct costing Absorption costing Difference
Difference in closing
inventory R1 600 R2 400 R800
Less: Difference in
R800 R1 200 (R400)
opening inventory
Net difference R400
This difference can be attributed to the fixed manufacturing overheads included in the valuation of
opening inventory and closing inventory. The fixed manufacturing overhead cost of R400 is
deferred in inventory until the next period, when it will be sold.
Change in inventory:
Closing inventory 200
Less: Opening inventory 100
100
Fixed cost included in inventory (100 units ×
R400
R4)
This means that with absorption costing, only R11 600 (2 900 units × R4) of the R12 000 total
fixed manufacturing overheads will be incurred in the current period. This is because only R11
600 is included in the cost of sales when using absorption costing. The remaining R400 (100 units
× R4) will be incurred only in the next period, when the inventory is sold.
The total fixed manufacturing overheads, i.e. R12 000, are treated as a period cost and fully
recovered in the current year with direct costing.

14.5 Reconciling profits determined by direct and absorption


costing
It is evident from the above examples that the profits as established by the two methods can vary
significantly owing to the way in which fixed manufacturing overheads are treated when not all units
produced are sold in the same period. As we have seen, the direct costing approach writes off the
total fixed manufacturing overheads as a period cost in the period in which it is incurred, whereas
with absorption costing, fixed manufacturing overheads are incurred only when the product is sold. In
Example 14.3, we have seen that because of this difference in the method in which the fixed
production overheads are treated, the profits calculated using the two approaches are different when
there is inventory left at the end of the period.
Reconciliation is a statement that analyses the difference between two known figures. When the
total number of units produced is not sold, the profits calculated using direct costing and the profits
calculated using absorption costing do not tally. So there is a need to reconcile the two different
figures. We have already seen from the above examples and explanations that the reason for the
difference in the profit is the fixed overheads included in the valuation of opening and closing
inventory.
Let us first look at how we can reconcile the profits as determined in Example 14.3:
Net profit using direct costing R7 200
Add: Fixed overhead included in valuation of closing stock as per absorption
R800
costing (200 units × R4)
Net profit as per absorption costing R8 000
We will now look at the reconciliation of the profits as determined in Example 14.4:
Net profit as per direct costing R8 200
Add: Difference in inventory × fixed overhead rate (100 units × R4) R400
Net profit as per absorption costing R8 600

CLOSING PRACTICAL EXAMPLE


Long-John Ltd produces a range of shoes; the company is currently preparing both its financial-
accounting and management-accounting information for the previous financial year. The cost
accountant advises the managing director that the financial-accounting rules only allow the use of
absorption costing where fixed overheads are included in costs when calculating the unit costs of
inventory. He also confirms that direct costing may be a better source of financial information for
management-accounting purposes, as this method treats fixed costs as a period cost.

14.6 Summary
Direct costing and absorption costing are two methods that are used to calculate the production cost
and to determine the profit. Each method has its advantages and disadvantages but direct costing is
perceived to give better management information while absorption costing is required for financial-
accounting and taxation purposes.
The two methods differ in the way fixed overheads are treated. Direct costing only uses variable
costs in order to determine the product cost and writes off the total fixed overheads as a period cost.
In the case of absorption costing it includes both variable costs and fixed costs in order to determine
the product cost. The fixed overhead is incurred only when the units are sold. Therefore, a portion of
the fixed overhead will be tied up in the inventory if not all units produced are sold.

Short questions
1. Define the following:
• Direct costing
• Absorption costing
• Contribution margin
2. Are the following statements true or false?
• The difference between sales and cost of sales in the case of direct costing is known as
contribution margin.
• Direct costing is not used in managerial decision making.
• Absorption costing writes off the total fixed overhead as a period cost in the year in which
it is incurred.
• Absorption costing uses both variable costs and fixed overheads to determine the
production cost.
• Contribution margin is the difference between sales and variable costs.

Long questions
1. The following information was extracted from the books of SM Manufacturers:
Units produced 2 400
Units budgeted 2 400
Cost information
Direct materials R12 per unit
Direct labour R15 per unit
Total variable manufacturing overheads R12 000
Total fixed manufacturing overheads R16 800 (budgeted and actual)
Variable selling and administration costs R3 per unit
Fixed selling and administration costs R9 600
Selling price per unit R60
a. Calculate the following:
i. Fixed manufacturing overhead allocation rate
ii. Direct cost per unit
iii. Absorption cost per unit
b. Prepare the statement of financial performance using:
i. The direct-costing method
ii. The absorption-costing method

2. Use the information provided in Question 1. Assume that all the units produced were sold
except for 100 units.
a. Prepare the statement of financial performance using:
i. The direct-costing method
ii. The absorption-costing method
b. Account for the difference in the profit.

3. BSP Company produced 37 500 units in the first year of its operations. The selling price of
their product is R55 per unit. BSP uses production units as the basis for absorbing the fixed
manufacturing overheads. The following information is taken from the company records:
R
Sales 1 402 500
Total fixed manufacturing overheads 300 000
Total variable manufacturing cost 375 000
Total variable marketing and administration
225 000
costs
Total fixed marketing and administration costs 123 000
a. Calculate the net profit using:
i. Direct-costing method
ii. Absorption-costing method
b. Provide an explanation for the difference in net profits between direct costing and
absorption costing.

4. HH Ltd is a manufacturing business that produces one product using two different raw
materials. The following statement of financial performance is taken from the books of HH
Ltd for the year ended 28 February 2013.
HH Ltd statement of financial performance for the year ended 28 February 2013
R R
Sales (80 000 units × R24) 1 920 000
Less: Variable costs 1 440 000
Direct material (80 000 units × R5) 400 000
Direct labour (80 000 units × R7) 560 000
Manufacturing overheads (80 000 units × R3) 240 000
Selling expenses (80 000 units × R2,70) 216 000
Administrative expenses 80 000 units × R0,30) 24 000
Contribution margin 480 000
Less: Fixed costs 120 000
Selling expenses 24 000
Administration expenses 96 000
Net profit 360 000
Rewrite the statement of financial performance using absorption costing.

5. Mosutho Manufacturing Company has a normal production capacity of 22 500 units per
annum. The company provides the following cost information about its new product line:
Direct material per unit R2
Direct labour per unit R3
Variable production overheads per unit R1
Fixed production overheads for the year R45 000
Fixed marketing and administrative costs R27 000
Variable marketing and administrative cost per
R3
unit of sales
Selling price per unit R15
During the year the company’s production was at normal capacity but it sold only 20 250
units.
a. Calculate the following:
i. The value of closing inventory using direct costing.
ii. The value of closing inventory using absorption costing.
iii. The total variable costs charged to expenses for the year if the company is using
direct costing.
iv. The total fixed costs charged against the current year’s operations if the company is
using absorption costing.
v. The net profit using direct costing.
vi. The net profit using absorption costing.
g. Reconcile the difference in the profits between the two methods.

6. Comrado Manufacturers Ltd provides the following estimates for standard costs and
expenses at two different levels of production:
21 000 units 24 000 units
Manufacturing overheads R123 150 R131 100
Direct material costs are R2.30 per unit and direct labour costs are R8,40 per unit.
The company estimates the following additional expenses:
Variable marketing and administrative expenses
R3,90
per unit sold
Fixed marketing and administrative expenses R100 500
The company absorbs manufacturing overheads on the basis of a standard production
capacity of 22 500 units. Actual production for the year was 21 750 units and sales were 20
700 units.
a. Draw up the statement of financial performance using:
i. Absorption costing
ii. Direct costing
b. Account for the difference in the profits between the two methods.
PART 3
Budgets
CHAPTER 15
Operational budgets

CHAPTER 16
Cash budgets

CHAPTER 17
Flexible and fixed budgets

CHAPTER 18
Capital budgeting
15 Operational budgets
Minnette Vermaak and Johan Hefer

LEARNING OUTCOMES
After studying this chapter, you should be able to:
• define the term ‘budget’;
• explain the budgeting process;
• explain the benefits and shortcomings of budgeting; and
• prepare the following operational budgets:
– Revenue budget
– Production budget
– Direct-materials budget
– Direct-labour budget
– Manufacturing-overheads budget
– Cost-of-sales budget
– Operating-expenditure budget
– Pro forma statement of comprehensive income

PRACTICAL EXAMPLE
MV Furniture is a company that manufactures tables and chairs. The CEO wants to introduce an
operational budget, which has not been done before in this company. Senior management have set
their objectives for the following year but now need to put these into quantitative terms to ensure
that they can monitor the performance of the various managers in the organisation. The CEO,
however, is unsure how to proceed with the operational budget.

15.1 Introduction
All organisations have goals that they wish to achieve. Although many of these goals may not be
financial, especially in the case of not-for-profit entities, most of them will have financial
implications. The budgeting process sets the short-term financial targets and plans to achieve the
entity’s objectives. During the budgeting process, targets must be set for the next financial period,
keeping in mind the organisation’s objectives.

A budget is the quantitative expression of a proposed financial plan of action by management for a
specific period and an aid to coordinate what needs to be done to implement that plan.

The overall budget of the organisation is called the master budget. One of the components of this
budget is the operational budget, which is an important budget that is normally prepared before any
other budgets can be compiled, because it is the foundation on which all other budgets are built.

T h e operational budget is the financial plan for a period and is made up of various
subcomponents, including the sales budget, production budget, direct-materials budget, direct-
labour budget and manufacturing-overhead budget.

Based on the operational budget, the cash budget will be drawn up to plan for the expected cash
flows for the next year (this will be dealt with in Chapter 16). The final type of budget is a capital
budget, which is a plan for the long-term cash flows of a project (see Chapter 18).
Budgets can be drawn up at any time during the year for any short-term planning horizon. Budgets
may be drawn up for a week, a month, a quarter or the full year. If an annual budget for the following
year is prepared, the organisation will normally start the budgeting process in the last quarter of the
year. It is important that enough time is given to draw up the required budgets and have them
approved before they need to be implemented. The budgeting process will differ from entity to entity
and no set rule need be followed.
However, budgets should be set according to the long-term objectives of the organisation. Budgets
will be used as guidelines in the accounting period to ensure that the organisation achieves its long-
term goals, as set out in its strategic plan. Budgets help enterprises to focus not only on their short-
term goals, but also on their long-term objectives.
The basic information needed to prepare the budgets will serve as a benchmark against which
actual results can be compared. This will allow the monitoring of expenditure and may highlight
shortfalls in revenue or overruns in expenditure during the year so that corrective action may be taken.
Budgets can also help ensure that there is the necessary coordination between the different
departments in the enterprise. Departments need to work together to achieve the objectives of the
enterprise, and the managers in the organisations should have a clear understanding of the part they
need to play and how they will contribute towards these objectives.
It is normally best practice for management at all levels of the organisation to be involved in the
budgeting process. Managers tend to be more committed to goals if they are part of the process that
sets the goals. This also guards against unreasonable or unattainable targets being set that may act as a
demoralising factor for managers.
When preparing a budget, one should bear in mind that only a well-thought-out process, one that
takes into consideration all opinions and ensures the accuracy and reasonableness of the budget, will
result in the full benefits of the process being felt by the organisation.

15.2 The budgeting process


The budgeting process starts with the organisation’s strategic planning process. Through strategic
planning, the objectives of the organisation are set, potential strategies to reach the objectives are
identified and alternative strategic options considered. Once management have agreed on these
strategies, a course of action for the organisation can be decided upon.
After the strategic planning process has been completed, the budgeting process will translate these
objectives into quantitative terms. We will now discuss the steps in the planning and budgeting
process. We will use as an example a supermarket chain. Let us assume that the organisation has set a
goal to increase its regional market share from 10 to 15 per cent.
• Step 1. Identify the problem and uncertainties. The goal is to achieve an increase of five
percentage points in regional market share. The problem is how to achieve that objective. Some of
the uncertainties that the company will need to consider are – can it open new stores in the region?
Can the growth be achieved with its current products or might the company need to diversify?
Does it have the necessary operational capacity to increase its market penetration? How strong is
the competition?
• Step 2. Obtain information. The managers of the company must now gather the necessary
information to assess the uncertainties identified in step 1. They may find that there is an
opportunity to open a new store in a neighbouring town. They may also identify a new product line
that should boost revenue. They may find that their current capacity will not allow the revenue
growth and therefore they will need to increase their labour force and operating capacity.
• Step 3. Make predictions about the future. Now that they have the required information, they need
to predict what is likely to occur in the next financial year (which is also their planning horizon).
They may predict that opening a new store will cost R1 million rands in capital expenditure but
should increase their market share by five percentage points. The new product line should also
increase their market share by five percentage points but the increase in market share will mean a
10 per cent increase in their operating costs.
• Step 4. Make decisions by choosing from alternative options. Management may decide that
opening a new store, although requiring a large capital investment, would be more effective in
reaching the goal than introducing the new product line into its current stores.
• Step 5. Implement the decision, evaluate performance and learn. Once the company has made its
decision, it can now implement it by finding premises for the new store, laying it out, stocking it
and opening it to the public. The performance of the store should be measured on an ongoing basis
to assess whether the goals are being met or not. For example, if we assume that due to its distance
from where the company’s senior management are currently based, the store did not do as well as
expected because of poor-quality local management, this should be taken into account when future
decisions like this one need to be made.

Within this broader budgeting process, budget policy and guidelines should be followed to ensure that
all parties are aware of their responsibilities. Furthermore, the budgetary goals that are set during the
process should be negotiated with the managers who will need to implement the budget. Normally, a
budget should only be implemented once the managers who will be responsible for the
implementation have agreed to the budget.
In the budgeting process, a budgetary cycle can be identified that will be followed during the
planning period:
• Step 1. In consultation with managers, management accountants plan the performance of the entity
based on the goals of the organisation and past performance. This performance is then broken
down by subunits (such as cost centres) and discussed at various levels so that management at all
levels obtain a common understanding of what is expected.
• Step 2. Based on the forecasted performance, senior management develop a specific set of
financial or non-financial performance measures for each subunit, against which actual results will
be compared.
• Step 3. Management accountants record and measure performance during the period, and assist
management to investigate variations from the budget. Corrective actions will then be taken where
possible.
• Step 4. Based on the actual performance for the year, and taking into account changes in market
forces and economic conditions, and other factors, managers learn from analysing the variables
that have affected the budget and are able to feed the information they have obtained into the
budgeting cycle for the next planning period.

15.3 Benefits and shortcomings of budgets


There are many benefits to be derived from conducting an appropriate budgetary process in an
organisation. However, one should be remember that these benefits might not ensue if the process is
not appropriately managed and designed.
The first benefit of a budget is that it serves as a tool to help the organisation achieve its
objectives. Financial objectives are normally long term in nature and therefore need to be broken
down into short-term goals. This is where the budgetary process can assist, as shorter-term goals are
easier to accomplish.
Another benefit of a properly implemented budget is that it ensures that the enterprise has standard
information available regarding its products and services. This is especially important when
determining prices or when having to make financial decisions.
Budgets are designed to plan for operational costs, so they help to manage operational costs by
allowing these costs to be monitored on an ongoing basis. This way, remedial action can be taken if
costs are too high.
The final benefit of a budget is that, if used appropriately, it will contribute to effective
management by allowing one to measure performance against set objectives. This should be done
with care because if not, it may demoralise staff and lead to unhappiness.
Budgets also have shortcomings. One should bear in mind that a budget is only a plan, and not all
plans can be fully met. Changes in the economic environment, and internal or external conditions may
lead to the budgeted goals not being achieved.
It should be emphasised that if the managers involved in the budgeting process are not committed
to the success of the process, the budget may not be accurate. It is therefore important that the people
managing the process ensure that all views are expressed during the budget process and that all
managers participate fully in the process.
The organisation should also weigh up the cost of the budgeting process against the benefits that
may be derived from it. Preparing a detailed budget in a large company can consume a lot of time and
other resources, and this use of resources should be analysed against the proposed benefits. It may be
more appropriate to reduce the complexity of the budget so that more focus can be placed on
operational issues.

15.4 Master budgets


The document at the centre of the budgeting process is known as the master budget.

The master budget expresses management’s operating and financial plans for a specific period.

The master budget needs to take into account both operating and financing decisions. The operating
decisions concern how best to use the organisation’s limited resources. The financing decisions
consider the aspects of how best to fund the acquisition of the required resources.
The master budget comprises several components, but the precise elements it incorporates will
depend on the organisation’s specific requirements. One of the most important parts of the master
budget is the pro forma financial statements, which show forecasts of comprehensive income,
statements of financial position and statements of cash flows. The pro forma financial statements are
prepared based on the expectations for the next planning period. If the expectations prove true, the
historical financial statements should be similar.
The master budget can be broken down into two main components – the operational budget and the
financial budget:
• The operational budget contains all the operational expectations for the period. These include
revenue, production, cost of goods sold and operating-expenses budgets. These budgets will give
rise to a pro forma statement of comprehensive income.
• The financial budget contains the capital-expenditures budget, a cash budget based on the
operational and capital-expenditures budget, the budgeted statement of financial position and
statement of cash flows.
As mentioned, the exact components of the master budget will depend on the type of entity. A service
company does not usually have a production budget, and certain organisations may split their
operating-expenses budget into subcategories like a marketing budget. The important thing is that the
master budget should provide management with the particular information they require to plan
appropriately and measure the performance of the entity during the year.
Figure 15.1 is a graphic representation of a master budget.

Figure 15.1 Master budget


Example 15.1 will be used to discuss the different types of operational budgets.

EXAMPLE 15.1
MV Furniture manufactures and sells tables and chairs. The following information was compiled
for the purposes of preparing the company’s master budget for the year ending 31 December
2014.
Chairs Tables
Sales volume (u) 8 500 1 600
Selling price per unit R400 R600
Expected closing inventory 1 870 90
Expected opening inventory 170 @ R296 85 @ R471
Material requirements:
Wood R7,20 per sheet 10 sheets 8 sheets
Steel R16 per length 5 lengths 9 lengths
Labour requirements: 10 hours @ R12/hour 15 hours @ R16/hour
Budgeted fixed manufacturing
R125 000 R75 000
overheads
Budgeted variable
manufacturing overheads per R5 R4
labour hour
Wood Steel
Expected opening inventory 8 500 sheets @ R6 8 000 lengths @ R12
Expected closing inventory 10 200 sheets 1 700 lengths

The following sections discuss how to prepare the various parts of the master budget.

15.4.1 Revenue budget


An organisation normally starts compiling the budget by preparing the revenue budget because
determining the budgeted sales is required to calculate the production budget.

T he revenue budget shows the quantities of each product (expected sales volume) that the
company plans to sell and the intended selling price.

As mentioned, the forecasted sales volumes provide the basic data for creating budgets for production
costs (direct material, direct labour and manufacturing overheads), and for selling, distribution and
administration expenses. Factors that may influence sales forecasts include:
• general economic conditions;
• industry sales and profits;
• general inventory conditions;
• competitive conditions;
• internal information (i.e. history of sales and profits); and
• seasonal trends.

When preparing the revenue budget, it should be remembered that sales values may fluctuate during
the year. Furthermore, a company that manufactures or sells multiple products would need to consider
the sales volumes and expected sales values during the year for each of the products. Very large
retailers or similar companies may budget on the basis of a value rather than per product, as it may be
impossible to forecast the amount of sales of each item that they sell.
When preparing the revenue budget, the following important relationships between the variables
should be observed:
• Sales volume (U) × selling price per unit (SP/u) = total sales value (TSV)
• TSV ÷ U = SP/u
• TSV ÷ SP/u = U
EXAMPLE 15.2
Based on the information given in Example 15.1, the following revenue budget can be prepared:

Revenue budget for MV Furniture for year ended 31 December 2014


Selling price Total sales
Product Units (u)
per unit value
Chairs 8 500 × R400 = R3 400 000
Tables 1 600 × R600 = R 960 000
R4 360 000

15.4.2 Production budget


The production budget follows the revenue budget, as production volumes often depend on expected
sales. It should be remembered, however, that this may not always be the case, especially if just-in-
time or similar inventory systems are used. An entity may also produce a larger amount of items than
required by forecasted sales if it needs to increase inventory levels for a future event or a special
order. As an example, if an entity produces football jerseys and there is a national soccer competition
scheduled early in the next period, it may produce a larger number of jerseys than it expects to sell
after the end of the current planning period.

The production budget normally only expresses the volume/quantities for each product and does
not normally show any values. The costs will be calculated in the individual direct-material,
direct-labour and manufacturing-overheads budgets. The production budget is often the
responsibility of the production manager.

The objective of this budget is to ensure that production is sufficient to meet sales demand and that
economic inventory levels are maintained for completed units. The inventory levels may be linked to
the minimum inventory levels, as determined by the company’s inventory policies (see Chapter 4 for
a full discussion on minimum inventory levels).
One of the factors that should be considered when preparing the production budget is to ensure that
the entity has the necessary capacity to produce the required quantities. Production should also be
planned to ensure that the minimal resources are used to manufacture the required units. Depending on
the inventory policies, inventory should also normally be kept to a minimum to ensure that inventory-
carrying costs are minimised, but should not be so low that the entity risks running out of inventory
and production is interrupted.

EXAMPLE 15.3
Based on the information provided in Examples 15.1 and 15.2, the following production budget
can now be compiled for MV Furniture:

Production budget for year ended 31 December 2014


Chairs Tables
Sales volume
(From the sales budget)
Units to be sold 8 500 1 600
Plus: Planned closing inventory 1 870 90
Total units required for sales
10 370 1 690
and inventory
Less: Planned opening inventory (170) (85)
Units to be produced 10 200 1 605

Once the production budget has been drawn up, the direct-materials, direct-labour and manufacturing-
overheads budgets can be compiled. The information from the production budget forms the basis for
these budgets.

15.4.3 Direct-materials budget


T he direct-materials budget calculates the cost of production, as planned in the production
budget. The direct-materials budget is normally compiled according to the type of direct materials
used, as opposed to products.

Firstly, the expected volume of material will need to be determined to enable the entity to produce the
planned quantities of finished goods (referred to as the ‘usage’). These quantities are multiplied by
the expected cost per unit to determine the total direct-material costs. It is important to note that the
same price may not apply to a specific material throughout the planning period, as prices may vary.
The compilers of the budget should consider any expected changes in the material unit costs through
the year.
The usage part of the budget ensures that the entity has sufficient material to meet its production
needs. In the case of the items in Example 15.1, the budget helps the furniture manufacturer make sure
it has enough wood and steel available to manufacture the required number of tables and chairs, as set
out in the production budget.
The cost part of the budget helps management control costs during the planning period. It is also
necessary to take into account the planned raw-material inventory levels.
The usage part of the budget is normally the responsibility of the production manager and the cost
part is the responsibility of the purchasing manager – but such responsibilities depend on the nature of
the entity.

EXAMPLE 15.4
Using the information in Examples 15.1–15.3, the direct-materials budget can be compiled for
MV Furniture.

Direct-materials budget for year ended 31 December 2014

Therefore, the total purchase value for all materials is R1 785 408. These values will be fed into
the cash budget (see Chapter 16) to plan the expected cash flows based on the payment terms of
the company’s creditors.

15.4.4 Direct-labour budget


T he direct-labour budget determines the expected direct-labour needs per product that the
company intends to produce. The production manager is normally responsible for preparing this
budget.

When preparing the direct-labour budget, one should bear in mind that different grades of labour may
be required. The budgeting process should be tailored to ensure that the budget reflects as accurately
as possible the nature of the production. It should also be remembered that labour is not always a
variable cost. If an entity’s labour costs are fixed, these costs would be determined in an appropriate
fashion and added to the cost-of-sales budget.

EXAMPLE 15.5
Based on the information provided in Examples 15.1 and 15.3, the following direct-labour cost
budget can be compiled for MV Furniture:

Direct-labour cost budget for year ending 31 December 2014


Chairs Tables
Units to be produced
Units to be produced (as per the production budget)
10 200 1 605
Labour hours per unit × 10 × 15
Total budgeted labour hours needed 102 000 24 075
Labour cost per hour × R12 × R16
Total budgeted labour cost R1 244 000 R385 200
Therefore, the total budgeted labour cost is R1 629 20; total budgeted labour hours = 126 075
hours.

15.4.5 Manufacturing-overheads budget


The final part of the budget that will form part of the cost-of-sales budget is the manufacturing-
overheads budget. The expected manufacturing overheads will be determined for the year and then
allocated to production based on an allocation method. The different allocation methods are
discussed in Chapter 6. The method of allocation (often called a recovery rate) will be based on the
expected production.
It should be remembered that the nature of the overheads may also influence how this budget is
prepared. If the overheads are fixed in nature, they should simply be added to the cost of sales, as
they are not determined by production. Variable overheads, however, should be allocated in an
appropriate fashion. It should also be remembered that the manufacturing-overheads budget may be
prepared per department, in which case it is important that overheads are allocated to the various
departments in an appropriate fashion, as described in Chapter 6.

EXAMPLE 15.6
Based on the information given in Examples 15.1 and 15.5, the manufacturing-overheads budget
of MV Furniture can be prepared as follows:

Manufacturing-overheads budget for year ended 31 December 2014


Chairs Tables
Budgeted labour hours
Variable manufacturing overheads (as per the labour cost budget)
102 000 hours 24 075 hours
Overhead recovery rate per labour hour R5/labour hour R4/labour hour
Total variable manufacturing overheads R510 000 R96 300
Budgeted fixed manufacturing overheads R125 000 R75 000
Total manufacturing overheads R635 000 R171 300
Therefore, the total cost of manufacturing overheads for the year is expected to be R806 300.

15.4.6 Cost-of-sales budget


The cost-of-sales budget is a summary of the direct-materials cost budget, the direct-labour cost
budget and the manufacturing-overheads budget.

This budget also takes into consideration the value of opening and closing finished-goods inventory. It
may be necessary to calculate the value of opening and closing inventory based on the information in
the different parts of the budget. Raw-materials opening and closing inventory is not included, as the
raw-materials inventory has already been included when the direct-material costs were calculated in
the direct-materials budget.
We will now continue with the MV Furniture example to calculate the value of opening and
closing finished-goods inventory.
EXAMPLE 15.7
Based on the information given in Examples 15.1–15.6, we will now calculate the values of
opening and closing finished-goods inventory.
First, we need to calculate the unit cost of closing finished-goods inventory and we will
assume the first-in-first-out principle for simplicity’s sake.
Chairs Tables
(10 × R7,20) + (5
Direct materials R152,00 (8 × R7,20) + (9 × R16) 201,60
× R16)
Direct labour 10 × R12 R120,00 15 × R16 R240,00
Manufacturing (R125 000 ÷ 10 (R75 000 ÷ 1 605) + (15 ×
R62,25 R106,73
overheads 200) + (10 × R5) R4)
Total unit cost R344,25 R548,33
As we now have the opening unit costs of finished goods, which were given, and the closing unit
costs of finished goods, which we calculated, we can now calculate the value of opening and
closing inventories:
Opening inventory Closing inventory
Finished goods
Chairs 170 × R296 R50 320 1 870 × R344,25 R643 748
Tables 85 × R471 R40 035 90 × R548,33 R49 350
Total R90 355 R693 098

EXAMPLE 15.8
Based on the values of the opening and closing inventory, calculated in Example 15.7, we can
now compile the cost-of-sales budget using the information from Examples 15.1–15.7.

Cost-of-sales budget for year ended 31 December 2014


Refer to examples Total
Opening finished-goods
Example 15.7 R90 355
inventory
Direct materials used Example 15.4 R1 785 408
Direct labour Example 15.5 R1 629 200
Manufacturing
Example 15.6 R806 300
overheads
Cost of goods
R4 220 908
manufactured
Cost of goods available
R4 311 263
for sale
Closing finished-goods
(R693 098)
inventory
Cost of sales R3 618 165

15.4.7 Operating-expenditure budget


The next part of the master budget is the operating-expenditure budget.

The operating-expenditure budget is used to estimate the entity’s operating expenses.

This budget can be prepared in various ways, either as one budget or separate budgets for
administrative, marketing, product design and other costs.

EXAMPLE 15.9
MV Furniture’s management have determined that their expected operating expenses for the year
ended 31 December 2014 are as follows:
• Variable marketing and selling costs are expected to be 2 per cent of revenue, and fixed
marketing and selling costs are R120 000.
• Administrative costs are expected to increase by 7,5 per cent from the R150 000 of the current
year, and are all fixed costs.
• Fixed distribution costs are expected to be R32 000 for the coming year with variable
distribution costs being R2,50 per unit sold.

Based on the above information, we can now prepare the operating-expenditure budget for MV
Furniture.

Operating-expenditure budget for year ended 31 December 2014


Calculation of Variable costs
Fixed costs (R) Total costs (R)
variable cost (R)
Marketing and
R4 360 000 × 2% 87 200 120 000 207 200
selling costs
Administrative
161 250 161 250
costs
Distribution costs 10 100 × R2,50 25 250 32 000 57 250
Total 112 450 313 250 425 700

15.4.8 Pro forma statement of comprehensive income


The pro forma statement of comprehensive income, also known as the income statement, is an
important part of the master budget, as it gives an indication of the profitability that the entity can
expect for the planning period.

The pro forma statement of comprehensive income includes information from the revenue, cost of
sales and operational-expenditure budgets.

EXAMPLE 15.10
MV Furniture can now prepare a pro forma statement of comprehensive income based on the
information given in Examples 15.2, 15.8 and 15.9.

Pro forma statement of comprehensive income for year ending 31 December 2014
Revenue Example 15.2 R4 360 000
Cost of sales Example 15.8 R3 618 165
Gross profit R741 835
Operating costs
Marketing and
Example 15.9 R207 200
selling costs
Administrative
Example 15.9 R161 250
costs
Distribution costs Example 15.9 R57 250 (R425 700)
Operating income R316 135

15.4.9 Other components of the master budget


The other components of the master budget that have not yet been discussed in this chapter are the
cash budget, capital-expenditure budget, pro forma statement of financial position and pro forma
statement of cash flows.
The capital-expenditure budget will be based on the capital budgeting process, which is covered
i n Chapter 18. The capital expenditure will be included in the cash budget, which is discussed in
Chapter 16. The pro forma statement of financial position and statement of cash flows can then be
compiled based on the cash budget and the pro forma statement of comprehensive income, which are
discussed in Chapter 16.
This completes the compilation of the master budget. It should be remembered that entities should
determine the structure and content of their master budgets on the basis of their specific needs. The
steps given in this chapter simply provide guidelines and an indication of best practice.
CLOSING PRACTICAL EXAMPLE
The CEO of MV Furniture is reviewing the actual performance of the entity compared to the
expected performance. The accountant has compiled the following statement of comprehensive
income based on the results for the year:

Statement of comprehensive income for year ending 31 December 2014


Budgeted (R) Actual (R)
Revenue 4 360 000 5 250 360
Cost of sales 3 618 165 4 567 420
Gross profit 741 834 682 940
Operating costs
Marketing and
207 200 186 000
selling costs
Administrative
161 250 182 000
costs
Distribution costs 57 250 (425 700) 75 350 (443 350)
Operating income 316 135 239 590
Reflecting on the results, the CEO is encouraged by the higher-than-expected revenue due to a
large order from a retail chain. The gross margin on this order was, however, much lower than
normal and therefore the operating income was lower than expected. The company must ensure that
it prices special orders appropriately to ensure that the extra revenue also results in a larger
operating income.

15.5 Summary
In this chapter, we explained the important part that a well-structured and properly thought-out
operational budget can play in assisting an entity to achieve its goals. Operational budgets allow
management to plan quantitatively to help them achieve their goals. The operational budget also gives
managers a tool to monitor the performance of the organisation.
The budgeting process needs to be taken seriously by all involved, otherwise the information it
contains may be incorrect. Furthermore, the budgeting process should be viewed as an exercise
whereby lessons can be learnt to improve the process in future periods.
This chapter also discussed the various components of the master budget, namely:
• Revenue budget
• Production budget
• Direct-materials budget
• Direct-labour budget
• Manufacturing-overheads budget
• Cost-of-sales budget
• Operating-expenditure budget
• Pro forma statement of comprehensive income
PRACTICAL ACTIVITY
You have just been appointed as the accountant for a large manufacturer of electronic equipment.
One of your tasks is to prepare the master budget for the following financial year. Write a memo to
the production manager requesting the information relevant to enable you to compile the production
budget.

Short questions
1. Briefly discuss the parties involved in the budgeting process.
2. Briefly discuss how the strategy of an organisation and its budgeting process should interact.
3. Briefly define the following types of budgets:
• Master budget
• Cost-of-sales budget
• Operating-expenditure budget

Long questions
1. Mr Office Cleaners CC produces two cleaning products in two departments and sells the
products to industrial cleaning businesses. The budget estimates for 2014 are as follows:
Finished products
Selling price per Opening
Product Sales value Closing inventory
unit inventory
Furniture cleaner R3 400 000 R400 170 units 1 870 units
Carpet cleaner R896 000 R560 85 units 100 units
Standard direct material
Direct material per product Opening Closing
Material Cost per litre inventory inventory
Furniture Carpet
(litres) (litres)
Miracle R7,20 10 l 8l 8 500 10 200
Foamy R16,00 5l 9l 10 000 9 700
Direct labour
Direct labour hours per product Cost per direct labour
Department
Window Carpet hour
A 10 hours R12
B 15 hours R12
The manufacturing overheads have been estimated as R5 368 593. The manufacturing
overheads are allocated using a predetermined rate based on the direct labour hours.
Prepare the following budgets for 2014:
a. Sales budget
b. Production budget
c. Direct-material usage and purchase budgets
d. Direct-labour budgets
Determine the manufacturing-overheads allocation rate.

2. Smart Energy is a company that manufactures electrical components for televisions. The
following information is available for 2014:
Sales forecasts (units)
July August September October November
Mother boards 1 500 2 000 2 500 2 000 2 000
Power
3 000 4 000 5 000 4 000 5 000
suppliers
The estimated unit selling prices of the components are R60 for mother boards and R200 for
power suppliers. However, Smart Energy will increase the selling prices of both products on 1
August 2014 by 10 per cent. One completed component consists of:
Material A Material B Direct labour
Mother boards 2 kg 6 metres 4 hours
Power suppliers 4 kg 12 metres 8 hours
Material and labour costs information for 2014:
Material A R2 per unit
Material B R4 per unit
Direct labour R10 per hour
Manufacturing overheads information for 2014:
Variable overheads R2 per labour hour
Fixed overheads R600 000 per annum
Manufacturing overheads are consumed evenly throughout the year.
Inventory levels (units)
Material A Material B Mother boards Power suppliers
1 July 4 000 5 000 1 000 3 000
110% of opening 80% of opening
Month end See below See below
inventory inventory
The company will introduce a new inventory policy for finished goods as from 1 July.
Inventory levels of finished goods, at the end of each month, must equal 40 per cent of the
sales estimates of the next month.
Prepare the following budgets:
• The sales budget for July and August.
• The production budget for July and August.
• The direct-materials budget for July.
• The direct-labour budget for July.
• The manufacturing-overheads budget for July.

3. Fun Toy Company manufactures eight-wheeled toy buses. The company hopes to generate R3
220 000 from the sales for July 2014. It will sell the buses for R70 each. Closing inventory
of finished buses is expected to consist of 6 300 buses, 5 000 plastic moulded bodies, 55 000
wheels and 25 000 axles at the end of June. The company’s policy is to have a closing
inventory each month equivalent to 15 per cent of budgeted sales volume of that month. The
assembly department is planning its July production. Assembly is the last process in the
production of the buses and it requires 0,2 hours per bus at a cost of R30 per labour hour.
The department uses the following material per unit of production:
Plastic moulded body 1 R20,50
Wheels 8 R6
Axles 4 R12
The manufacturing overheads are applied at R10 per labour hour.
Prepare the following budgets for July 2014:
a. Sales budget.
b. Production budget.
c. Direct-material usage and purchase budgets.
d. Direct-labour budgets.
e. Manufacturing-overheads budget.

4. You have been given the following information for Construct Pty Ltd for August 2014:
Product Quantity (units) Price/unit (R)
A 1 000 100
Sales
B 2 000 120
C 1 500 140
Material needs Product Material X Material Y Material Z
A 4 2 0
Quantity needed
B 3 3 2
for 1 product
C 2 1 1
Cost per kg R4 R6 R9
Raw-material Opening 26 000 20 000 12 000
inventory Closing 31200 24 000 14 400
Product A Product B Product C
Direct-labour
Hours needed 2 1 3
requirements
Rate/hour 5 10 6
Product A Product B Product C
Finished-product Closing 1 100 1 650 550
inventory Opening 1 000 1 500 500
Manufacturing-overhead costs are recovered based on 50 per cent of direct-labour cost.

Prepare the following budgets for Construct Pty Ltd for August 2014:
a. Sales budget.
b. Production budget.
c. Direct-material usage and purchase budgets.
d. Direct-labour budgets for Product A only.
e. Manufacturing-overheads budget.
16 Cash budgets
Jean Struwig and Johan Hefer

LEARNING OUTCOMES
After studying this chapter, you should be able to:
• define a cash budget and understand its importance;
• prepare a cash budget;
• discuss the purpose of a cash budget; and
• understand the impact of budgeted cash surpluses and shortages on business operations.

PRACTICAL EXAMPLE
James had to see his bank manager to ask for an increase in his overdraft, as he has once again
exceeded his overdraft limit in his business account. His company is doing well but he is
struggling to manage the cash flows in the business. He is not sure why his profitable business is
not generating enough cash.
The bank manager has suggested that James use a cash budget to plan his cash flows
appropriately.

16.1 Introduction
A cash budget is a very important planning tool in a business. It combines the information in the
various budgets that form part of the master budget (refer to Chapter 15) to prepare a budget that will
reflect the expected cash flows in the business. Once the cash budget has been prepared, the business
will be able to see its net budgeted cash position and evaluate if it will have enough cash resources
or whether it may have to seek further financing.
The budgeted closing balance indicates if the organisation will have excess cash to invest or if
there will be a shortage of cash at the end of the period. This will give the organisation the
opportunity to plan how it can invest excess funds or how to address a shortage of funds.
It is essential for an organisation to do cash planning, otherwise it may not have enough cash at the
end of the month to pay employees’ salaries or its suppliers for purchases and other services
received. If a business runs out of funds, it can force it to stop operating. This is because key
suppliers may refuse to supply goods or services on credit if no payments are made, or employees
may stop working until they have been paid.

A cash budget can be defined as a schedule of planned and expected receipts and payments for a
budgeted period. It will be summarised and combined with any cash available at the beginning of
the period to show the expected cash balance at the end of the period.
16.2 Basics of drawing up a cash budget
Cash budgets can be prepared on a weekly, monthly or quarterly basis, depending on the requirements
of the organisation. If an organisation has a lot of weekly receipts and payments, then a weekly budget
would be more effective than a monthly budget.
A cash budget makes use of the cash-flow information from all the operational budgets in the
business, such as the sales, material, labour and other budgets (see Chapter 15), as well as the capital
budget, as discussed in Chapter 18.

Figure 16.1 Information used to prepare cash budget

It is important to remember that only cash-flow items are included in the cash budget. In other words,
if there is not a receipt or a payment transaction, then that item will not be shown in the cash budget.
Non-cash-flow items, like depreciation, provisions and bad debts, are not included in a cash budget
because these are accounting entries and not payments or receipts.
It is also important to remember that all cash flows in a business must be shown in the cash
budget, and not only those that affect the income statement. Examples of such cash flows are new
machinery that has been paid for, capital-budgeting cash-flow items (as discussed in Chapter 18) and
repayment or receipt of loans.
There are two basic questions that have to be answered when preparing a cash budget. Firstly, you
need to determine whether a cash flow will occur or not and, secondly, when the cash flow will
occur.
A cash budget normally consists of three sections – receipts, payments and summary:
• The receipts section deals with the cash amounts that we estimate will be received during the
budget period. It includes budgeted values for cash sales, collections from debtors for credit
sales, interest received, sales of non-current assets and any other planned receipts. For all these
receipts, the amount planned to be received must be used, and not the amount that is accrued for
financial accounting purposes.
• The payments section deals with the cash amounts that are estimated to be paid during the period.
It includes all planned payments for the period, such as cash material purchases, payment to
creditors for credit purchases, labour costs, salaries, overhead payments, purchase of new
machinery and equipment, interest paid, taxes paid, repayment of loans and any other planned
payments. For all these payments, the amount planned to be paid must be used and not the amount
that is payable.
• The summary section is where the planned closing cash balance is calculated and the cash
shortage or surplus will be identified. The cash balance available from the previous period must
be combined with the receipts and payments of the current period to calculate the planned closing
balance at the end of the period. The closing balance is calculated as follows:
Total receipts XXX
Less total payments XXX
= Net cash flow for period XXX
Plus opening balance XXX
= Closing balance XXXX
The closing balance will indicate if the organisation is likely to have excess cash to invest or a
shortage of cash at the end of the period. This will give management the opportunity to plan how it
can invest excess funds or how to address a shortage of funds.

16.3 Purpose of a cash budget


A cash budget is used to make sure that enough cash, but not excessive amounts of cash, is available
at all times in an organisation to meet the requirements that are outlined in the various other budgets it
has prepared.
Because cash budgeting is based on planned and expected receipts and payments, and not actual or
guaranteed receipts and payments, it is important to make provision for more than the minimum cash
required in the business. The planned and expected receipts and payments will normally be drawn up
from the information provided in other budgets and from the entity’s experience of receiving money
from its customers and paying its suppliers.
It is important to prepare realistic cash budgets, to give the organisation the opportunity to plan
how to invest a possible surplus of funds or how to address a possible shortage of funds. A shortage
may be addressed by borrowing money, arranging for a bank overdraft or reducing payments to
creditors. A surplus of funds may be addressed by investing the money in a short-term investment
where interest can be earned on idle funds or repayment of loans. Excess cash can also be used to pay
dividends, buy back shares or invest in new ventures.

EXAMPLE 16.1
The accounting clerk at Mandy’s Clothing has prepared the following information for the current
financial year:

Actual sales:
April R40 000
May R36 000
June R80 000
Budgeted sales:
July R80 000
August R85 000
September R85 800
Sales are normally received in the following manner:
• All of the sales are credit sales.
• The sales are collected during the month after the sale.
Budgeted purchases:
July R12 000
August R13 000
September R11 800
October R11 500
Purchases of inventory made in the month must be paid for in the month of the purchase. Monthly
salaries and wages are R15 000 per month. Monthly rent expenses are R4 000. Other sundry
monthly payments are R2 500.
Mandy’s Clothing had a positive balance of R2 500 in the bank at the end of June.

Required
Prepare a monthly cash budget for Mandy’s Clothing for July, August and September.

Solution
Cash budget for Mandy’s Clothing:
July (R) August (R) September (R)
Receipts
Credit sales 80 000 85 000 85 000
80 000 85 000 85 000
Payments
Cash purchases 12 000 13 000 11 800
Salaries and wages 15 000 15 000 15 000
Rent paid 4 000 4 000 4 000
Sundry payments 2 500 2 500 2 500
33 500 34 500 33 300
Summary
Receipts 80 000 85 000 85 000
Payments (33 500) (34 500) (33 300)
Net cash flow 46 500 50 500 60 700
Opening balance 2 500 47 500 98 000
Closing balance 47 500 98 000 158 700
As can be seen in the example, Mandy’s Clothing expects to have a large cash balance by the end
of September and the company may wish to invest this or pay out a dividend to shareholders.

Let us look at another example of a cash budget.

EXAMPLE 16.2
Zama’s accountant has prepared the following information for the current financial year, which
has been extracted from the components of the master budget:

Actual sales:
January R280 000
February R275 400
March R296 000
Budgeted sales as per the sales budget:
April R260 000
May R310 000
June R240 800
July R260 000
Based on past experience, sales are normally received in the following manner:
• 31 per cent of the sales are cash sales.
• The remaining sales are collected in equal amounts during the following three months.
• All cash sales receive a 5 per cent discount.

Purchases of inventory are made in the month of sale and are equal to 54 per cent of sales. All
purchases are on credit. Sixty-five per cent of the purchases must be paid 30 days after the
purchase and the balance must be paid 60 days after the purchase.
According to the labour budget, salaries and wages for March are R60 000 per month. In May
the employees will receive their annual salary and wage increase of 6 per cent.
According to the overheads budget, selling expenses are 1 per cent of total sales and are paid
one month after they are incurred. Monthly rent expenses are R15 000. The monthly
administrative and other expenses of R2 850 include depreciation of R650.
Zama applied for a one-year loan of R55 000. This will be received in May. Interest at 12 per
cent per year will be paid at the end of the one-year period.
Zama had a positive balance of R25 664 in the bank at the end of March.

Required
Prepare a monthly cash budget for Zama for April, May and June.

Solution
Cash budget for Zama
Calculations April (R) May (R) June (R)
Receipts
Cash sales C1 76 570 91 295 70 916
Credit sales C2 64 400 63 342 68 080
C3 63 342 68 080 59 800
C4 68 080 59 800 71 300
Loan 55 000
272 392 337 517 270 096
Payments
Salaries and
C5 60 000 63 600 63 600
wages
Selling expenses C6 2 960 2 600 3 100
Rent 15 000 15 000 15 000
Administrative and
C7 2 200 2 200 2 200
other expenses
Purchases (after
C8 103 896 91 260 108 810
30 days)
(after 60 days) C8 52 051 55 944 49 140
236 107 230 604 241 850
Summary
Receipts 272 392 337 517 270 096
Payments 236 107 230 604 241 850
Net cash flow 36 285 106 913 28 246
Opening balance 25 664 61 949 168 862
Closing balance 61 949 168 862 197 108
Calculations:
C1 260 000 × 31% × 95% = 76 570
C2 280 000 × 69% ÷ 3 = 64 400
C3 275 400 (Feb) × 69% ÷ 3 = 63 342
C4 296 000 (March) × 69% ÷ 3 = 68 080
C5 May R60 000 × 1,06 = 63 000
C6 April March sales of R296 000 × 1% = 2 960
C7 2 850 – 650 (not cash flow) = 2 200
Purchases total in April 54% × R260 000 = R140 400
C8 Pay 65% after 40 days in May = R 91 260
Pay 35% after 60 days in June = R 49 140
Purchases total in March 54% × R296 000 = R159 840
C8 Pay 65% after 30 days in April = R103 896
Pay 35% × R159 840 after 60 days in May = R55 944

CLOSING PRACTICAL EXAMPLE


James’s business is doing well, but he is struggling to manage the cash flows in the company. He
was not sure why his profitable business was not generating enough cash. His bank manager
suggested that he compile a cash budget.
Once James had completed the cash budget for his business, he realised that the large
repayments on a bank loan were negatively affecting his cash flows. He was able to renegotiate the
repayment terms with the bank, which now allows him to manage his cash better.

16.4 Summary
This chapter explained why it is important for companies to create a cash budget, discussed the
purpose of cash budgets and explain the main aspects to remember when preparing a cash budget. It
also explained the importance of evaluating and managing the budgeted cash surpluses and shortages
in a business. The purpose of a cash budget is to make sure that enough cash, but not excessive
amounts of cash, is available at all times in a business to meet the requirements that are outlined in the
various other budgets prepared by the business.
When preparing a cash budget, the cash-flow information is used from all the business’s
operational budgets, as discussed in Chapter 15 – for example, the sales, material, labour and other
budgets, as well as the capital budget, as discussed in Chapter 18.

PRACTICAL ACTIVITY
You have recently been appointed as the management accountant of a large manufacturing firm. All
the components of the master budget have been compiled except for the cash budget. Write a
memorandum to the credit manager (who is responsible for collecting credit sales from customers)
asking for any relevant information that will assist you in compiling the receipts section of the cash
budget.

Short questions
1. What type of payments are shown in a cash budget?
2. Is it better to prepare cash budgets on an annual or a monthly basis? Motivate your answer. 3.
Why is there a difference between the sales figure in the income statement and the receipts
from sales in the cash budget of most businesses?
4. Which other budgets influence the cash budget?
5. How will one month’s cash budget influence the next month’s cash budget?

Long questions
1. The following budgeted data for Pens & Paper, a stationery shop that sells a standard
stationery pack, is available:
August September October November
Sales units 1 000 1 400 1 550 990
Sales price R50 R55 R60 R60
General costs R14 000 R15 400 R16 300 R12 000
• The budgeted cost of the standard stationery pack will be 40 per cent of the sales price. It
will be purchased in the month of sale.
• Creditors must be paid 30 days after purchases.
• 70 per cent of sales are expected to be cash sales and the balance of the sales is expected
to be collected after 30 days.
• Included in the general costs is R500 monthly depreciation. All general costs that must be
paid will be paid the month the expenses are incurred.
• Sales commissions consist of 10 per cent of sales and are paid one month after the sale is
made.
• Pens & Paper plans to sell off its old furniture in August for R2 000. The company expects
to receive the cash in September. It plans to replace it with new furniture at a cost of R10
500. It has credit arrangements with the supplier to pay it in November.
• Pens & Paper has R500 in the bank at the end of August.

Prepare a cash budget for September, October and November. Round off to two decimals.
2. Collins CC has prepared the following information for the current financial year:
Actual sales:
March R450 000
April R399 400
May R254 000
Budgeted sales:
June R430 000
July R390 400
August R350 800
In the past, sales were collected as follows:
• 30 per cent of the sales are cash sales. These customers will receive a 5 per cent discount.
• 40 per cent of the debtors pay 30 days after their purchase.
• 50 per cent of the debtors pay 60 days after the transaction.
• The balance of the sales is considered as bad debts.
Purchases of inventory are equal to 55 per cent of sales. All purchases are made one month
before the planned sale. All purchases are on credit. Forty per cent of the purchases must be
paid 30 days after the purchase, and the balance must be paid 60 days after the purchase.
Salaries and wages for June are R45 000. In July the employees will receive their annual
increase of 5 per cent. Selling expenses are 5 per cent of total sales and are paid one month
after they are incurred.
Budgeted and actual total overheads:
May June July August
R21 000 R22 000 R21 500 R22 500
Sixty per cent of the overheads refer to variable expenses that must be paid in the month in
which the expenses are incurred. The fixed portion of the overheads includes monthly rent of
R4 600 and monthly depreciation of R650. According to the rental contract, the rent must be
paid in advance every six months. The previous rent payment was made in February. The
balance of the fixed overheads must be paid one month after the cost was incurred.
Collins plans to buy new computers to the value of R11 000 in August. The company will
pay cash for these computers and will write off depreciation at 25 per cent per year on the
reducing balance method.
Provisional tax of R12 000 must be paid in August. Collins has an unfavourable bank
balance of R50 390 at the end of May. Collins expects to receive a five-year loan of R120
000 in July. Interest on this loan will be paid at the end of each year at a rate of 9 per cent.
Prepare a monthly cash budget for Collins for June, July and August. Round off to two
decimals.
3. Based on actual results and budgets, Bulela’s accountant has prepared the following
information for the current financial year:
• Sales. Thirty per cent of all the sales are for cash. Sixty per cent of the credit sales are
collected in the month of sale. The cash sales will be given a 5 per cent discount, and
credit sales that are received in the month of sale will receive a discount of 2 per cent.
Thirty-five per cent of the credit sales are received two months after the sale. The balance
of the credit sales will be considered as bad debts. The company sells all the products it
produces each month.
Actual and budgeted sales
January R430 000
February R350 000
March R420 000
April R470 000
May R450 000
June R467 000
July R480 000
• Material. The cost of raw materials equals 40 per cent of each sale. The company requires
a monthly ending inventory equal to 50 per cent of the next month’s production
requirements. Of its raw material purchases it pays 50 per cent in the month of purchase
and the balance one month later. The value of the starting inventory for materials in
January is R20 000.
• Labour. Direct workers have agreed to have their wages paid into their bank accounts on
the second working day of each month in respect of the previous month’s earnings. The
direct workers earn R24 per hour and at the beginning of April they received a 12 per cent
increase.
Expected working hours
January 3 600
February 3 700
March 3 750
April 3 650
May 3 666
June 3 400
• Production overheads. Production overheads will be 20 per cent of the labour cost per
month and must be paid in the month the cost was incurred.
• Equipment. Bulela plans to sell old equipment with a book value of R21 600 for cash in
April for R22 800. It will purchase replacement equipment at a cost of R85 000 in March.
Payment for this will be made during the two months immediately after the purchase in two
equal payments. The depreciation on the new equipment will be calculated at 20 per cent
per year.
• Dividends. Bulela declared dividends of R150 400 on 28 February, which will be paid to
shareholders on 8 April.
• Overheads. Selling overheads are all fixed and will be R378 000 for the year from
January. These overheads include depreciation on a delivery van at 30 per cent per year
on a reducing balance method. The cost price of the van on 1 January is R18 000 and the
book value R15 000. All selling overheads are incurred monthly, except rent of R48 500,
which is payable annually in one instalment in March.
• Interest. Interest paid on an investment of R34 500 is R2 460 per year. The company
receives the interest every two months, and the payments started in January.
• Opening balance. The opening balance on 1 March is R1 500.

Prepare a monthly cash budget for March, April, May and June, showing all your calculations.
17 Flexible and fixed budgets
Melissa McGill

LEARNING OUTCOMES
After studying this chapter, you should be able to:
• explain the importance of flexible budgets in terms of budgetary control;
• explain the disadvantages of fixed budgets in terms of budgetary control;
• prepare a flexible budget;
• explain the terms ‘responsibility accounting’ and ‘performance report’;
• identify situations where fixed or flexible budgetary control would be appropriate;
• flex a budget to a given level of volume;
• calculate simple variances between a flexed and fixed budget; and
• evaluate performance using fixed and flexible budget reports.

PRACTICAL EXAMPLE
Multinational cosmetics company Dermaface Ltd produces skincare products, which it supplies to
beauty parlours and spas. The company’s line managers have recently received their annual
performance reports in order to justify all variances noted in their expenditures. These
performance reports compare their actual results with the original budgeted results, as reported by
the managers at the beginning of the year.
After investigating the performance report, the product manager in charge of the face-cream
range confronted the finance team, as the variances reported on the manufacturing overheads and
other expenses were extremely high.
The product manager said to the finance division that he did not understand how performance
could be measured on the basis of the original budgeted figures. The reason his department’s
manufacturing overhead expenses were much higher was that the production output levels were
much higher than had been anticipated. Production increased as a consequence of the increase in
demand for the product – the market grew as a result of people becoming more aware of healthy
skincare. Was it not obvious that the production manager’s expenses would be greater than
budgeted for if the production volume turned out to be more than budgeted for? Should the finance
division not take into account the level of activity that increased significantly?
The finance team said that to address this discrepancy, they would definitely look into
preparing flexible budgets in future and inform the product manager of their decision.

17.1 Introduction
The budgeting process is a plan that a company puts in place for a forthcoming period. The figures are
therefore an estimate of what the company thinks might happen in the future. The master budget, as
explained in Chapter 15, is a fixed budget prepared on the basis of the company’s planned level of
activity. When budgetary control is exercised and performance reports are prepared, it is not always
feasible to compare this fixed budget with the actual figures because the actual level of activity might
turn out to be very different from the figures that the master budget was based on. In order to
overcome this problem, companies ‘flex’ the budget, giving management the opportunity to control as
well as plan.

17.2 Differences between fixed and flexible budgets


In Chapter 15, we discussed and explained how to prepare operational budgets, which are classified
as fixed budgets. A fixed budget is prepared for a fixed level of planned activity. The master budget
is prepared at the beginning of a financial period. Refer to the fixed budget of Dermaface for the year
ended 30 June 2012 (Example 17.1).

EXAMPLE 17.1: DERMAFACE LTD FIXED BUDGET FOR YEAR


ENDED 30 JUNE 2012
Number of units produced and sold 150 000
Sales (150 000 @ R25) R3 750 000
Direct materials (150 000 @ R2) R300 000
Direct labour (150 000 @ R12) R1 800 000
Variable overheads:
Electricity (150 000 @ R4) R600 000
Other overheads (150 000 @ R6) R900 000
Contribution R150 000
Fixed overheads
Face-cream factory rental R12 000
Depreciation on machinery R6 000
Face-cream line casual cleaner R36 000
Net profit before tax R96 000

The level of activity for Dermaface for the year ended 30 June 2012 has been budgeted for at 150
000 units. All variable overhead costs will therefore be based on the same budgeted number of
units produced, as the production equals sales.
Management’s concern is that when they compare actual figures with the fixed budget, the
variances that arise cannot be accurately explained, as the level of activity differs. Is the variance
a result of the activity level only? Can the variance perhaps be partly attributed to poor cost
management?
Consequently, the fixed budget is sufficient for planning purposes, but does not evaluate how
well costs are controlled. We determine the amount of variance attributable to cost management
by flexing the budget.
17.3 Preparing a flexible budget
A flexible budget takes any movement in the level of activity into account. It is prepared for a
range of levels of activities. Think of a flexible budget as a series of fixed budgets for different
levels of activities.

Variable overhead costs per unit, however, remain the same as originally budgeted for. Fixed costs
remain the same irrespective of whether the level of activity changes within activity levels. Once the
upper end of an activity level is reached, further fixed costs would have to be incurred as activity
increases. This principle was explained in Chapter 5 and is important to remember when preparing
flexible budgets.
Flexible budgets can be prepared at the beginning of a financial period. Refer to the flexible
budget prepared for Dermaface (Example 17.2). The company is of the opinion that sales might be as
low as 140 000 or as high as 170 000 units for the period. The company has therefore prepared a
contingency flexible budget and will then assess the outcomes at the end of the financial period.

EXAMPLE 17.2: DERMAFACE LTD FLEXIBLE BUDGET FOR YEAR


ENDED 30 JUNE 2012
Number of
units produced Cost formula 140 000 150 000 160 000 170 000
and sold
Sales 25 3 500 000 3 750 000 4 000 000 4 250 000
Direct material 2 280 000 300 000 320 000 340 000
Direct labour 12 1 680 000 1 800 000 1 920 000 2 040 000
Variable
overheads
Electricity R4 560 000 600 000 640 000 680 000
Other
R6 840 000 900 000 960 000 1 020 000
overheads
Contribution 140 000 150 000 160 000 170 000
Fixed
overheads:
Face-cream
12 000 12 000 12 000 12 000
factory rental
Depreciation on
6 000 6 000 6 000 6 000
machinery
Face-cream line
36 000 36 000 36 000 36 000
casual cleaner
Net profit 86 000 96 000 106 000 116 000
before tax
The direct costs and sales are calculated by multiplying the expected units by the cost formula.
These costs will remain the same whether they are in the fixed budget or the flexible budget. For
example, the direct-material unit cost in the fixed budget is R2 per unit. As can be seen in the
flexible budget, the direct material cost remains at R2 per unit. The fixed costs remain the same at
all levels of activity, as no upper end of activity has been reached that would cause the fixed
costs to increase.

The variable costs increase with the level of activity, as can be seen in Example 17.2. For example,
direct-material costs are R280 000 (R2 × 140 000) at 140 000 units. If the company sells 160 000
units, the budgeted material cost then increases to R320 000 (R2 × 160 000).
The important thing to remember when preparing a flexible budget is that if a figure, either a cost
or revenue, was variable when the original budget was prepared, then that amount remains variable
and will need to be recalculated when preparing a flexible budget. Conversely, if the cost was
identified as a fixed cost, no changes are made in the budgeted amount when the flexible budget is
prepared. Differences may occur in fixed expenses, but they are not related to changes in activity
within the relevant range.
Alternatively, a company can prepare a flexible budget at the end of the reporting period, when
actual figures are available. The flexible budget can then be compared against the actual results. The
budget will then only have one level of activity, which will reflect the actual number of units
produced.

17.4 Disadvantages of fixed budgets


A fixed budget does not adjust its figures during the year, regardless of the company’s performance.
For example, if the budget for sales commission in a small business is R20 000, this budget will
remain the same even if the sales team break sales records. The cap on the company’s budget helps
the business control its costs and provides a level of certainty for the finances. This allows the
business to maximise savings and ensure it spends every rand as wisely as possible.
However, the lack of mobility in the budget means there isn’t the flexibility to allocate resources
to support underperforming areas of the business or provide additional capital in the event of
equipment failure or to seize a new market opportunity, for example. This may lead to a decrease in
the revenue stream that causes the company to operate at a loss for the period. A company that is
losing money can cause a knock-on effect whereby investors choose other business opportunities.
In summary, the fixed budget has a role to play in the management planning function. It does not
enable management, however, to control activities and costs within the company.

17.5 Budgetary control: Responsibility accounting and


performance reports
The concept of responsibility accounting is a performance-measurement system. Companies may
decentralise, or separate their companies into manageable divisions.
Responsibility accounting allows responsibility to be assigned to managers that have the most
influence over key elements within their divisions.

Managers are given decision-making authority and responsibility for each activity occurring within a
specific area of the company. For example, Dermaface has various divisions for its product lines,
such as face cream, cleansing products, toners and sun screen. Each division has a manager who is
responsible for the control of costs within that division. The managers of each division will therefore
need to explain any variances that occur in their performance reports.
Each division might be further subdivided so that the manager responsible for sales, for example,
will report on sales only and the manager responsible for production will report on production-
related performance concerns.
The underlying concept of responsibility accounting is referred to as controllability. The thinking
behind controllability is that a manager should only be held responsible for those elements that he or
she can control.

A performance report is one in which the budgeted figures are compared with the actual figures.
These reports are used in the control process in order to measure and evaluate the performance of
both the manager as well as the operations for which the manager is responsible.

Performance reports contain variances that will need to be investigated and explained by the manager
responsible for that unit or division. The reports are also used to identify areas that need
improvement.
In Example 17.3, we will prepare a performance report using the fixed budget for Dermaface, as
given in Example 17.1
The budgeted number of units remains 150 000. The actual number of units for Dermaface,
however, was reported at 165 000. All costs in the actual figures are therefore based on the actual
number of units produced.
In the Dermaface example, the variances do not indicate to the manager whether the cost control
was effective or not. All costs are higher than in the original budget, which is to be expected, as the
actual output was 15 000 units more than the budgeted output.
A manager therefore cannot be held responsible for unfavourable variances in a fixed-budget
performance report, as such a budget is created purely for planning purposes. A fixed budget is
therefore more reliable when determining the performance of fixed costs, and not variable costs, as
fixed costs remain constant irrespective of a change in activity.

EXAMPLE 17.3: DERMAFACE LTD FIXED-BUDGET PERFORMANCE


REPORT FOR YEAR ENDED 30 JUNE 2012
Fixed budget Actual Variance
Number of units
150 000 165 000
produced
Sales and sold 3 750 000 4 200 000 450 000 F
Direct materials 300 000 335 000 35 000 U
Direct labour 1 800 000 1 850 000 50 000 U
Variable overheads:
Electricity 600 000 620 000 20 000 U
Other overheads 900 000 990 000 90 000 U
Contribution 150 000 405 000
Fixed overheads:
Face-cream factory
12 000 12 500 500 U
rental
Depreciation on
6 000 6 300 300 U
machinery
Face-cream line casual
36 000 34 500 1500 F
cleaner
Net profit before tax 96 000 351 700 255 700F
Note: ‘F’ denotes a favourable variance and ‘U’ denotes an unfavourable variance.

To determine whether the actual cost of producing 165 000 units was higher than it should have been,
we now need to flex the budget. When we refer to flexing the budget, this means that the budgeted
figures are adjusted to reflect what the cost would have been at the actual number of units sold and
produced. By doing this, management can then determine whether they have spent too much on the
production. Refer to Example 17.4 for the flexible-budget performance report prepared for
Dermaface.
The variance shown after flexing the budget will exclude the variance that relates to the change in
the level of activity, as the budgeted figures are now based on the actual units sold or produced.

EXAMPLE 17.4: DERMAFACE LTD PERFORMANCE REPORT FOR


YEAR ENDED 30 JUNE 2012
Fixed budget Flexible budget Actual Variance
Number of units
150 000 165 000 165 000
produced and sold
Sales 3 750 000 4 125 000 4 200 000 75 000 F
Direct materials 300 00 330 000 335 000 5 000 U
Direct labour 1 800 000 1 980 000 1 850 000 13 000 F
Variable
overheads:
Electricity 600 000 660 000 620 000 40 000 F
Other overheads 900 000 990 000 990 000 0
Contribution 150 000 165 000 405 000
Fixed overheads:
Face-cream
12 000 12 000 12 500 500 U
factory rental
Depreciation on
6 000 6 000 6 300 300 U
machinery
Face-cream line
36 000 36 000 34 500 1500 F
casual cleaner
Net profit before
96 000 111 000 351 700 123 700F
tax
Note: ‘F’ denotes a favourable variance and ‘U’ denotes an unfavourable variance.

The budgeted selling price per unit remains the same as in the fixed budget (3 750 000 ÷ 150 000
= R25). The actual activity level was then used to calculate the flexible-budget sales figure at
(R25 × 165 000 = R4 125 000).
Fixed overheads remain the same as per the fixed budget. Profit for the actual output should
have been R111 000. Profit, however, was reported at R351 700, much higher than expected.
This is due to better cost management and a higher sales price. As mentioned earlier, the variance
relating to the change in activity is excluded in a flexible budget.
Let us now look at the total cost variances:
Variance Fixed budget variance Flexed budget variance
Direct materials 35 000 U 5 000 U
Direct labour 50 000 U 13 000 F
Electricity 20 000 U 40 000 F
Other variable overheads 90 000 U 0
Face-cream factory rental 500 U 500 U
Machinery depreciation 300 U 300 U
Casual cleaner’s salary 1500 F 1500 F
Total cost variances 194 100 U 48 700F
As can be seen with the flexible-budget favourable variance, Dermaface has good cost control.
The total variance is 194 100 unfavourable in the fixed budget, meaning that the variance relating
to the change in activity is 242 800 unfavourable (194 100 + 48 700) when comparing the flexed
and the fixed budgets.
The same comparison be done with revenue. The flexible-budget variance was calculated at
75 000 favourable. This means that the sales price was higher than originally budgeted for. We
know this, as flexing the budget has eliminated the changes in the activity level. This variance is
also known as the sales price variance.
The difference between the original fixed-budget profit and the actual budget can be explained
as follows:
• The change in the activity level – a favourable movement of 15 000 units, known as the sales-
quantity variance.
• Savings in the cost and increase in the sales price.

17.6 Flexible budgets and overheads


Overhead costs, along with salaries and wages, form the major portion of a company’s expenses.
From the glue that is used for the assembly of tables to the disposable coffee cups in a customer
waiting area, overhead costs are normally made up of a number of small costs. These costs can be
variable, fixed or mixed. This is why overhead costs are more difficult to control than the direct
materials and direct labour used in production. Flexible budgeting can assist in overcoming the
difficulty in controlling overhead costs.
Now that you are familiar with performance reports and how to determine the variances, we will
discuss variances in overheads in more detail. These variances were briefly discussed in Chapter 12.
When preparing flexible budgets, there is the option to use either the actual labour or machine
hours needed to produce the product, or the standard hours that should be used.
In Chapter 12, we determined that the cost control overhead variance can be due to:
• spending too much on resources; and/or
• using the resources inefficiently.

This leads to two variances that need to be calculated, namely the spending variance and the effi-
ciency variance.
When using the actual hours used in producing the product in a flexible budget, one can only
calculate the spending variance. If we use the standard hours that should be used in producing the
product, the spending as well as the efficiency variance can be calculated.
Let us return to the Dermaface example. Refer to the flexible budget for the overheads (Example
17.2). Assume that the number of machine hours required to produce a product was identified as the
cost driver to be used in allocating overheads in Dermaface.
Recall that overhead costs are assigned to products and services using a predetermined overhead
rate (POHR). The POHR is calculated as follows:

The predetermined overhead rate is used to apply manufacturing overheads to a product.


Applied overhead = POHR × standard activity

In this example, we will assume that the actual hours worked for the period are 152 000 hours.
Standard hours that should have been worked are 150 000 hours.
Total variable Variable overhead Total fixed Fixed overhead
Machine hours
overheads rate overheads rate
150 000 R1 500 000 R10 54 000 0,36
165 000 R1 650 000 R10 54 000 0,33
Note that the variable overhead rate remains constant at all levels of activity, and the fixed overhead
rate decreases as the activity increases. The total predetermined overhead rate is the sum of the fixed
(R3,60) and variable (R10) rates for the given activity level.
17.6.1 Variable overhead variances
The variable overhead spending variance is useful if the level of activity is the actual hours worked.
The flexible budget for variable overheads will then be an indication of what the entity should have
spent on variable overheads for the level of activity. You can now calculate the variance by
comparing the actual variable overheads spent with what should have been spent.
Refer back to Example 17.4 where the total actual variable overhead cost was recorded as:
Electricity R620 000
Other variable overheads R990 000
Total R1 610 000

In total there is an unfavourable variance of R110 000, as the actual expenditure is more than what the
company should have spent.
Assuming that 152 000 actual machine hours were used in the production process, the flexible
budget for the variable overhead rate will remain R10, as recorded in the flexible budget. Remember
that the variable cost per unit remains constant even though the level of activity changes.
From the calculation above, it is clear that the company has spent R90 000 more on variable
overheads than it should have spent. It was also determined that the company has used more hours
than it should have in producing the period’s output, given the R20 000 unfavourable effi-ciency
variance reported. Take note that the efficiency variance is not an indication of the use of the
overheads but rather of the use of the base itself, namely the machine hours.

17.6.2 Fixed overhead variances


Two variances can be calculated for fixed overheads, namely the budget variance and the volume
variance.

T he budget variance is calculated by comparing the actual fixed overheads spent with the
budgeted fixed overheads, as contained in the flexible budget. The budget variance indicates to
management where they have over- or underspent on overheads.

The volume variance, however, is not an indication of over- or underspending of overheads. We have
explained how fixed overhead costs remain the same regardless of whether the level of activity
changes. This variance relates to the activity only, as the monetary value does not change. In other
words, if the denominator activity that is used in the predetermined overhead rate and the standard
hours allowed for the number of units produced are the same, there will be no volume variance. An
unfavourable volume variance would indicate that the plant facilities are not used at the optimum
level.
With reference to the example under variable overhead variances, we assume that the standard
hours that should have been worked were 150 000. Budgeted hours worked remain the same at 152
000. Total actual fixed costs are as follows:
Rental R12 500
Depreciation R 6 300
Salary R34 500
Total R53 300
The predetermined overhead rate should therefore be calculated as R54 000 ÷ 152 000 = R0,35

From the calculation, it is clear that the company has spent R700 less on fixed overheads than it had
expected. However, it operated at an activity level lower than the denominator activity, hence the R1
500 unfavourable volume variance.

CLOSING PRACTICAL EXAMPLE


After his recent win at the Olympic Games in London, Haram Ulain has returned to his home
country and opened a running-shoe company called Pike.
After discussions with his accountant, he was not happy with the first year’s financial results,
as the variances in his performance reports have shown that the company has over-spent on all
expenses.
His accountant informed him that the budget prepared by him was purely for planning purposes.
The sales of Pike’s limited-range running shoes that were available for one month only led to
actual sales being much higher than what had been planned for at the beginning of the year.
The budget for the period ended 31 December 2012 is as follows:
Number of units sold and produced 400
Sales 856 000
Direct material 140 000
Direct labour 120 000
Variable overhead expenses 30 000
Contribution 566 000
Fixed cost 230 000
Net profit 336 000
Actual sales came to 740 pairs of running shoes. These shoes were sold for R2 850 a pair.
Actual expenses were as follows:
Direct materials 240 000

Direct labour 190 000


Variable overhead expenses 90 000
Fixed costs 260 000
The flexible budget is as follows:
Budget Cost formula Flexed budget Actual Variance
Number of units 400 740 740
Sales 856 000 2 140 1 583 600 2 109 000 525 400 F
Direct
140 000 350 259 000 240 000 19 000 F
materials
Direct labour 120 000 300 222 000 190 000 32 000 F
Variable
overhead 30 000 75 55 500 90 000 34 500 U
expenses
Contribution 566 000 1 047 100 1 589 000
Fixed cost 230 000 230 000 260 000 30 000 U
Net profit 336 000 817 100 1 329 000 511 900 F

Workings:
Sales
Cost formula: 856 000 ÷ 400 = 2 140
Flexed budget: 2 140 × 740 = 1 583 600
Actual: 740 × R2 850

Direct material
Cost formula: 140 000 ÷ 400 = 350
Flexed budget: 350 × 740 = 259 000

Direct labour
Cost formula: 120 000 ÷ 400 = 300
Flexed budget: 300 × 740 = 222 222

Variable overhead costs


Cost formula: 30 000 ÷ 400 = 75
Flexed budget: 75 × 740 = 55 500

17.7 Summary
A flexible budget can be used to control manufacturing costs. The budget is set up using the actual
activity level in order to determine what should have been spent to produce the actual output.
If a cost is variable, the flexible budget amount will be the actual level of activity multiplied by
the budgeted cost per unit. The fixed-cost elements in a flexible budget will, however, remain
constant within certain activity limits. Using flexible budgets, performance reports can be prepared
from which variances similar to those discussed in Chapter 12 can be calculated. This information
can then be used to manage performance in the organisation and identify inefficiencies and waste.

PRACTICAL ACTIVITY
After the product manager’s recent discussion with the finance division, the finance team have
investigated the possibility of implementing a flexible-budgeting process to evaluate performance.
They met with the production manager to discuss how a flexible budget should be set up to take
into account what the budgeted figure would be for the actual level of activity.
The face-cream division of Dermaface currently has a specific recipe that it uses to make face
cream. This recipe has a standard cost card, which includes the quantity that is needed per unit as
well as the budgeted cost for each item listed on the cost card.
The division will therefore draw up a budget for the actual number of face-cream tubes
produced at the budgeted price per unit (the price listed on the cost card). The result of the flexible
budget will then be compared to the actual results and any variances will be investigated.
This new flexible budget implementation will make it easier for the production manager to plan
and control the production of the face cream, as the variances will be based on the actual number
of units produced and not the budgeted number, which might be higher or lower.
It will now be possible to determine whether variances arise from the change in activity or due
to poor cost management.

Short questions
1. A fixed budget …
a. should not be prepared in a company.
b. shows planned results at the actual activity level.
c. is useful in evaluating performance by comparing actual and planned variable costs. d.
shows planned results at the original budgeted activity level.
2. What is the primary difference between a fixed budget and a flexible budget?
a. The fixed budget contains only fixed costs, while the flexible budget contains only
variable costs.
b. The fixed budget is adjusted for different activity levels, while a flexible budget is
prepared for a single level of activity.
c. The fixed budget is prepared for a single level of activity, while a flexible budget is
adjusted for different levels of activities.
d. The fixed budget is prepared at the end of a period, while a flexible budget is prepared at
the beginning of a period.
3. Which of the following statements is false?
a. A flexible budget is used for control purposes and a fixed budget is used for planning
purposes.
b. A flexible budget is prepared at the end of the period and a fixed budget is prepared at
the beginning of the period.
c. A flexible budget is not useful for controlling variable costs.
d. A fixed budget provides budgeted estimates for one level of activity.
4. Pratly (Pty) Ltd had the following information available for the expected cost and selling
price based on 5 000 units:
Variable manufacturing cost per unit R32
Fixed manufacturing cost per unit R20
Selling price per unit R70
Expected production level 5 000 units
In the flexible budget at 10 000 units, what is the total manufacturing cost?
a. R250 000
b. R700 000
c. R520 000
d. R420 000
5. Peter Computers (Pty) Ltd’s variable selling and administrative expenses are R48 000 at a
production level of 6 000 units. What are the variable selling and administrative expenses? a.
R56 000
b. R80 000
c. R64 000
d. R48 000

Long questions
1. When choosing an activity base on which to construct a flexible budget, discuss three criteria
that should be considered.
2. When comparing budgeted data with actual data in a performance report for variable
overheads, which variances will be produced if the budgeted data is based on actual hours
worked?
3. A partially completed flexible overhead budget for Alpha (Pty) Ltd is shown below:
Activity level in units
Cost formula 8 000 12 000 16 000
Variable
overheads
Supplies ? ? R108 000 ?
Utilities ? ? R60 000 ?
Repairs ? ? R24 000 ?
Total variable
? ? R192 000
overheads
Fixed overheads:
Depreciation ? R15 000 ?
Salaries ? R96 000 ?
Rent ? R44 000 ?
Total fixed
? R155 000 ?
overheads
Total
manufacturing ? R347 000 ?
overheads
Fill in the missing data.

4. Bedford (Pty) Ltd has provided the following information regarding its June results:
Revenue and cost formula Actual results
Revenue R13/unit R28 000
Conversion cost R3,25/unit R7 000
Salaries R8 000 R7 600
Utilities R600 +R0,50/unit R1 550
Rent R5 000 R5 000
Miscellaneous R800 + R0,80/unit R2 500
a. Prepare the company’s planning budget assuming that 2 000 units were manufactured.
b. Assuming that 2 100 units were actually manufactured, prepare the flexible budget for this
level of activity.
c. Prepare a flexible-budget performance report.

5. Bedridden (Pty) Ltd has a relevant range of 15 000 to 25 000 labour hours per year. The cost
formulas for Bedridden’s manufacturing overheads are listed below:
Utilities R0,20 per labour hour
Indirect labour R34 000 + R0,90 per labour hour
Supplies R0,15 per labour hour
Maintenance R12 000 + R0,25 per labour hour
Rent R75 000
Prepare a flexible budget in increments of 6 000 labour hours.

6. Bettacouch (Pty) Ltd manufactures couches for a niche market. The company makes use of a
standard-costing system. The standard cost card for a two-seater couch is as follows:
Standard cost card – Type 1 Leather P
Direct material (3 metres at R6 per metre) R18
Direct labour (4 hours at R7,75 per hour) R31
Variable manufacturing overheads (4 hours at
R6
R1,50 per hour)
Fixed manufacturing overheads (4 hours at R5
R20
per hour)
Total standard cost per unit R75
During the year, Bettacouch manufactured 9 500 Type 1 Leather P couches and worked a total
of 37 000 hours. Manufacturing overhead costs are applied to production on the basis of
direct labour hours.
The following information relating to fixed manufacturing overhead costs is available:
Actual fixed overhead
R198 700
cost
R? Budget variance
Flexible budget fixed
?
overhead cost
R10 000U Volume variance
Fixed overhead cost
applied to work in ? hours × R?
process
a. What were the standard hours allowed for the year’s production?
b. What were the fixed overhead costs contained in the flexible budget for the year?
c. What was the fixed overhead budget variance for the year?
d. What denominated activity level did the company use in setting the predetermined
overhead rate for the year?

7. Trappers (Pty) Ltd manufactures hiking boots. The following information is available:
Actual Fixed budget
Units sold 110 units 100 units
Sales R3 200 R3 000
Variable expenses R1 920 R1 800
Contribution margin R1 280 R1 200
Fixed expenses R680 R700
Operating income R600 R500
a. Determine what the operating income should have been for the actual units sold.
b. Reconcile the difference between the fixed budget and actual operating income.

8. Ethiopia (Pty) Ltd has made the following information available with regard to its monthly
revenues and costs:
Variable cost formula Fixed cost formula
Revenue R15/unit
Cost of material R7,25
Wages and salaries R20 000
Utilities R0,45/unit R1 200
Rent R10 000
Miscellaneous R0,90 R2 000
a. Prepare the company’s planning budget, assuming that 10 000 units were manufactured.
b. Assume that 9 900 units were actually manufactured. Prepare the flexible budget for this
level of activity.
c. Prepare a flexible-budget performance report for the company using the actual income
statement information shown below:
Revenue R149 200
Cost of materials R73 200
Salaries R19 500
Utilities R5 800
Rent R10 000
Miscellaneous R12 000
18 Capital budgeting
Peter Taylor

LEARNING OUTCOMES
After studying this chapter, you should be able to:
• discuss the various types of investment projects; and
• evaluate capital budgets using the following techniques:
– Cash flows
– Accounting rate of return
– Payback period
– Discounted payback period
– Net present value
– Internal rate of return.

PRACTICAL EXAMPLE
A Life General Hospital in the Greater Durban area is considering purchasing new state-ofthe-art
MRI and CTC scanning equipment. Although the existing equipment can operate for a further three
years, it is expected that the new, more efficient equipment will reduce the number of scans
required by patients. As a further benefit, labour costs may also be reduced. The existing scanner
will have a zero terminal value in three years, but it is expected that if it were sold immediately it
would result in a net cash inflow of R85 000.
The new equipment will require an initial investment of R2,4 million plus extra investment in
working capital and spare parts for the new equipment, amounting to R80 000. This sum could be
set off against the expected cash inflow from the sale of the existing scanner.
The new scanner is expected to have a useful life of five years and it will have no terminal
value at the end of its working life. The hospital manager expects annual cash flows of
approximately R800 000. These cash flows will occur throughout the year.
The hospital manager is now faced with an important capital-budgeting decision – whether to
continue operating the existing scanning equipment or to purchase the new scanning equipment and
replace the existing equipment. (Adapted from Horngren, Datar & Foster, 2011)

18.1 Introduction
According to Drury (2012), decisions concerning capital investment are normally the most important
ones that an organisation makes, since large sums of money are invested that cannot be reimbursed.
The capital budget forms part of the overall master budget that is used by entities when planning their
long-term strategies.
A major question regarding capital investment is how much money the organisation should invest
in the initial stages of the investment. The investment also impacts on the business profile of the
organisation. If an organisation has surplus cash to invest, it must consider the opportunity cost
forgone of an alternative investment for the cash.
An organisation places itself at financial risk if a large capital investment relative to the size of the
organisation fails. The risk will be smaller if the organisation makes smaller investments in several
projects.
Two other risks associated with capital investments that need to be considered are:
• Technology risk. There is always a danger that technology may become out of date.
• Cash-flow risk. There is a risk that the cash flows of a project may be incorrectly estimated owing
to differences in inflation rates, exchange rates or other variables used to project the cash flows.

When evaluating capital projects and the long-term allocation of funds, management should not only
select the most profitable project, but should also look at maximising long-term returns and, therefore,
long-term success. This means that, although a financial analysis may result in a preferred project
ranking, a strategic analysis by top management is essential to prioritise projects in terms of the
organisation’s long-term strategy.

18.2 Types of investment projects


When entities are faced with investment decisions, there are a number of different types of investment
projects that they can consider. Understanding the classifications of projects helps establish the level
of risk involved.

18.2.1 Replacement projects


Plant and machinery reach the end of their lifespan through wear and tear and the process of
obsolescence.

Obsolescence occurs when new items enter the market, which produce better products more
efficiently and at a lower cost per unit.

The original plant will need to be replaced under such circumstances, but before the purchase is made
management must be sure that the cash flows generated from it will justify the purchase of the new
plant. Replacement projects tend to be lower-risk projects, as there is an established market for the
product or service.

18.2.2 Expansion projects


New products and opportunities need to be considered if a business is to grow.

Expansion projects entail introducing a new product or service, or introducing an existing product
or service into a new area.
The aim of expansion projects is different from that of replacement projects. By means of an
expansion project, a business aims to increase its share of the market, whereas with a replacement
project the company looks to maintain its market share. The expected cash flows of expansion
projects are more difficult to forecast, as the success of the product or service is not known.
Expansion projects are therefore generally seen as higher risk than replacement projects.

18.2.3 Mutually exclusive projects


With mutually exclusive projects, the acceptance of one project precludes the acceptance of
another.

To illustrate with an example, consider that a company is planning to invest in a new plant. Accepting
one plant will mean that the company cannot purchase another similar plant at this stage. In this case,
the financial feasibility of both plants needs to be tested using various appraisal methods to be
discussed later. The plant that offers the best financial result will be accepted, meaning that the other
similar plant will be rejected.

18.2.4 Independent projects


In contrast to mutually exclusive projects, independent projects are those where the acceptance or
rejection of one project does not affect the acceptance or rejection of another project.

If an entity is considering two investment proposals, either proposal or both proposals could be
accepted if they are independent projects.

18.3 Determining accounting profits


A simple accounting calculation illustrates how accounting profits and the resulting cash flows are
calculated:
R
Sales turnover 450 000
Cost of sales 50 000
Gross profit 400 000
Expenses (including depreciation) 100 000
Net profit before tax 300 000
Taxation at 28% 84 000
Net profit after tax 216 000
Accounting profits can change due to depreciation calculations, taxation rates, and adjustments.
Therefore, various profit levels can be obtained, which will distort decision making when deciding
which projects to accept. Therefore, for the purpose of this chapter we will use profits before tax as
the relevant accounting profits.
As depreciation is a book entry for taxation purposes, no money is paid out and therefore
depreciation does not affect the cash outflow. In practice, depreciation may be calculated using a
number of different methods. Simply recalculate the depreciation and then add this depreciation back
to the accounting profit to calculate the cash flows. If cash flows are provided, it will be necessary to
subtract depreciation to revert to accounting profit. Some methods of capital appreciation deal with
cash flows, whereas others deal with accounting profits. (The only method that uses accounting
profits is the accounting rate of return, while all others use cash flows.)
Other factors that may affect cash flows are as follows:
• Cash flows must be relevant and include accruals, prepayments and depreciation.
• Sensitivity of cash flows needs to be considered. A conservative approach to the calculation of
cash flows is important.
• Cash flows from overseas projects may be affected by changes in exchange rates and inflation.

18.4 Methods for making capital-budgeting decisions


Managers need to make investment decisions based on the information available to them. Forecasts
are normally made of the expected cash flows or accounting profits, and various methods can then be
used to determine if the projects should be accepted or rejected.
It is useful to briefly review the concept of the time value of money before discussing the specific
methods that can be used to value capital projects.

18.4.1 Time value of money


The value of money is not constant over time. For example, if you invest R1 today, you would expect
to receive more than R1 one year from now, because as an investor, you hope to receive interest on
money at the prevailing interest rate. An investor would receive R1 today plus interest at, say, 5 per
cent per annum, which will be equal to R1,05 in one year. Working backwards, with the same interest
rate, R1 in one year’s time is equal to R0,9095 today. The value is therefore discounted back to the
present at the prevailing interest rate. This concept, namely that R1 today is not equal to R1 in the
future, is called the time value of money.
Some investment-appraisal methods make use of the time value of money, and some methods do
not. Methods that use the time value of money will always produce more realistic results than those
methods that do not. Time value of money calculations are normally done using spreadsheets,
published discount tables or financial calculators. However, for the purpose of understanding such
calculations, we will use formulas to explain the concept.
In order to discount a cash flow back to the present value (PV), the following formula is used:
PV = future cash flow × 1 ÷ (1 + i)n
where ‘i’ is the applicable discount rate and ‘n’ is the number of periods to be discounted.
Example 18.1 illustrates this formula.
EXAMPLE 18.1
If the cash flow in year 3 is R25 000 and it must be discounted to its present value using a
discount rate of 10 per cent, the formula is applied as follows:

The discount factor of 0,7513 can either be found in published present values of lump-sum rates
or it can be calculated using a financial calculator.

18.4.2 Discount rates


The discount rate used to discount cash flows back to their present values is known as the firm’s cost
of capital. This rate is the opportunity cost of an investment because if cash is invested in a capital
project, it cannot be invested elsewhere (i.e. the opportunity for an alternative investment is not
available). A company will only invest in a project if it will earn a return on the investment greater
than the opportunity cost of the investment.

18.4.3 Accounting rate of return


T he accounting rate of return (ARR) method measures the average accounting profit that an
investment will generate and expresses it as a percentage of the average annual investment in a
project.

The ARR can be defined thus:

The average investment is calculated as the investment plus the residual value of the asset divided by
two. If the investment has no residual value, the average investment will be equal to the average book
value of the investment (i.e. cost of the investment divided by two). It should be noted that the ARR
method is not used consistently in practice and various slightly different definitions may be found.
The new asset will generate profits annually over its useful life and to find the average annual
profits, we divide the sum of the annual profits by the useful life of the asset. Management may accept
a project that meets a certain ARR because of its effect on the company’s rate of investment. A
benchmark ARR will be decided on before a project is accepted or rejected.

EXAMPLE 18.2
Mattison Engineering Co (Pty) Ltd operates a machine shop in the greater Durban area. It is
considering the replacement of one of its metal-cutting machines, which is now obsolete. The
following quotation was received from Tinker Heavy Machinery (Pty) Ltd.
The new machine would require an initial investment of R500 000 and the machine will have
a useful life of five years. It will also have a salvage value of R60 000 at the end of its life. The
expected profits earned by the machine are as follows:
Year 1 R50 000
Year 2 R70 000
R120
Year 3
000
R100
Year 4
000
Year 5 R90 000
The machinery will depreciate using the straight-line method over five years. The company’s cost
of capital is 10 per cent. The company needs to calculate the ARR.

Solution

In order to decide whether 30,71 per cent return is acceptable, it must be compared to the
business requirement. If the firm has a target ARR of 30 per cent, then this investment is
borderline and may not be accepted. If, however, the target ARR is 20 per cent, the project would
be accepted.

The ARR should never be used in isolation, but in addition to other appraisal methods that use the
time value of money.
Advantages of ARR:
• It is simple to calculate.
• The necessary information can be easily acquired.

Disadvantages of ARR:
• The time value of money is ignored. Longer projects may be rejected due to the uncertainty of high
cash flows occurring towards the end of a project’s life.
• Accounting book values are used rather than cash flows.

18.4.4 Payback period


The payback period (PBP) makes use of cash flows, and not profits (as used in the ARR method). The
profits need to be converted to cash flows by adding back non-cash-flow items and considering the
capital outflows. The main example of a non-cash-flow item is depreciation, which is an adjustment
for the use of an item at every financial year end. When calculating the cash flows, we add back the
depreciation calculated for the year to the profits in order to arrive at cash flows for the year.
Similarly, the salvage, or terminal, value of a proposed capital investment should be treated as an
inflow of cash at the end of the life of the asset.

The PBP determines the length of time that is required for a stream of cash flows to recover the
initial investment.

The PBP is then determined by adding the cash flows in successive years until the total cash flows to
date equal the cost of the investment. In most cases, the sum of cash flows will exceed the cost of the
investment, meaning that the repayment term will be successive years plus a fraction of a year. This is
illustrated in Example 18.3.

EXAMPLE 18.3
Using the same information as in Example 18.1, the profits are now converted to cash flows by
adding back depreciation, which is a non-cash-flow item.

Solution
Depreciation is calculated as following formula:

Profits (R) Depreciation (R) Cash flows (R)


Year 1 50 000 88 000 138 000
Year 2 70 000 88 000 158 000
Year 3 120 000 88 000 208 000
Year 4 100 000 88 000 188 000
Year 5 90 000 88 000 178 000
+ salvage 60 000 60 000
Initial investment (500 000)
Less year 1 cash flow 138 000
(362 000)
Less year 2 cash flow 158 000
(204 000)
Less year 3 cash flow 208 000

The cash flow in year 3 will pay off more than the balance owing, amounting to R204 000.
Therefore the PBP will be:
Advantages of the PBP:
• It is quick and easy to use.
• This method is widely used by many investors.

Disadvantages of the PBP:


• It does not take into account cash flows that are earned after the payback date. It is therefore
possible that the PBP would distort the real figures owing to high cash flows in the later years not
being taken into account.
• The PBP does not take the time value of money into account. Therefore, future cash flows cannot
be accurately compared with the initial outlays, and consequently it may produce an unsatisfactory
result.

18.4.5 Discounted payback period


The discounted payback period method is an attempt to overcome the disadvantage of the PBP by
applying the time value of money to reduce cash flows to their present values. All future cash flows
will now be discounted back to their present values.

The same method used in the case of the PBP is then used to recover the initial investment cash
outlay.

EXAMPLE 18.4
Using the information in Examples 18.1 and 18.2, we now discount the cash flows calculated in
Example 18.2 to their present values using a discount rate of 10 per cent:
Discounted cash flows
Cash flows (R) Discount rate (10%)
(R)
Year 1 138 000 0.9091 125 456
Year 2 158 000 0.8264 130 571
Year 3 208 000 0.7513 156 270
Year 4 188 000 0.6830 128 404
Year 5 238 000 0.6209 147 774
Calculate the discounted payback period in the same way as for the payback period as
follows:
Initial investment (500 000)
Less year 1 cash flow 125 456
(374 544)
Less year 2 cash flow 130 571
(243 973)
Less year 3 cash flow 156 270
(87 703)
Less year 4 cash flow 128 404

Therefore, the discounted payback period is:

Note that the discounted payback period is slightly longer than the straight payback period. This is
because when the discounted payback period is used, the annual cash flows have been discounted to
their present values and are more realistic than those used for the payback period.
Advantages of discounted payback period:
• This method is quick and easy to use.
• It makes use of the time value of money and therefore the discounted cash flows will be more
realistic than in the case of the payback period.

Disadvantages of discounted payback period:


• As in the case of the payback period, this method does not take the cash flows into account after
the payback date. The result, although more accurate than the payback period because of the use of
the time value of money, could therefore still produce distorted results because cash flows in later
years are not taken into account.

18.4.6 Net present value


When using the net present value (NPV) method, the present values of all the future cash flows over
the life of the investment are compared with the investment outlay. If the sum of the discounted cash
flows is greater than the investment amount, the NPV will be positive. Conversely, if the sum of the
present values of all future cash flows is less than the initial investment, the NPV will be negative. If
the NPV is positive, the investment will be accepted, and if negative the investment will be rejected.
Any number of investments can be compared in this manner and those having positive NPVs will
be considered. Should the firm require only one investment, then the one with the highest NPV will be
accepted. In other words, investments can be ranked, which helps the decision-making process.
Using the discounted cash flows in Example 18.4 for the discounted payback period, the NPV is
calculated as follows:

EXAMPLE 18.5
Discounted cash flows (as for the discounted
Discount rate (10%)
payback period)
Years R
1 125 456
2 130 571
3 156 270
4 128 404
5 147 774
Total discounted cash flows 688 475
Less initial investment 500 000
NPV + 188 475
As the NPV is positive, the above investment is acceptable.

Advantages of NPV:
• The time value of money is taken into account when preparing the net cash flows.
• Comparing the NPVs of various projects is easy to do.

Disadvantages of NPV:
• It may be difficult to estimate future cash flows reasonably accurately. A conservative approach to
cash-flow calculation is preferred.
• The time periods used in determining the useful life of the investment may be difficult to calculate.

18.4.7 Internal rate of return


The internal rate of return (IRR) is the discount rate that, when used to discount future cash
flows, will equate the present value of cash flows to the initial investment. In other words, the IRR,
when calculated, will result in the NPV being zero.

Using interpolation, the formula for calculating the IRR is:

Where
A = low discount rate that will produce a positive NPV. As the discount rate is increased, so the NPV
will decrease as cash flows are discounted faster than for a low discount rate.
B = high discount rate that will produce a negative NPV.
C = the NPV, which must be positive when the low discount rate in A is used.
D = the NPV, which must be negative when the high discount rate in B is used.

EXAMPLE 18.6
The IRR is calculated as follows:

Note that the NPV when a discount rate of 24 per cent is used is negative (−R16 138)
18.4.7.1 Decision criteria
The IRR must at least be greater than the company’s cost of capital or the appropriate discount rate.
Therefore, if the IRR is greater than the company’s discount rate, accept the project, but if the IRR is
less than the cost of capital, reject the project.
The interpolation method gives an approximation of the IRR. However, in this example, the cost of
capital is only 10 per cent and therefore an accurate calculation is unnecessary. The project will be
accepted.

18.4.7.2 Trial-and-error method


Computers can be programmed to produce fast and accurate results for the IRR. In the absence of such
technology, manual calculations can be done, which is tedious but not impossible. Results calculated
by the trial-and-error method will be slightly more accurate than the formula method. However as
stated above, in many cases an accurate IRR is not required, in which case the formula method would
be acceptable.
In order to calculate the IRR by trial and error, find the highest possible discount rate before the
NPV of cash flows becomes negative. Remember that if the discount rate is increased, the NPV will
decrease and vice versa. Recalculate the NPV by adding 1 per cent to the rate used in the previous
calculation. The NPV will now be negative. Using interpolation, the IRR can now be calculated as
shown in Example 18.7.

EXAMPLE 18.7
Annual cash flows Discount rate (22%) Discounted cash flows
Year 1 138 000 0,8197 113 119
Year 2 158 000 0,6719 106 160
Year 3 208 000 0,5507 114 546
Year 4 188 000 0,4514 84 863
Year 5 238 000 0,3700 88 060
506 748
Less initial investment
(500 000)
Positive NPV 6 748
Annual cash flows Discount rate (23%) Discounted cash flows
Year 1 138 000 0,8130 112 194
Year 2 158 000 0,6610 104 438
Year 3 208 000 0,5374 111 779
Year 4 188 000 0,4369 82 137
Year 5 238 000 0,3552 84 538
495 086
Less initial investment
(500 000)
Negative NPV (4 914)
By interpolation, the discount rate is:

In the above examples, both IRR results are similar figures – e.g. 22,90 per cent by formula and
22,58 per cent by trial and error. When using the formula method, the larger the gap between the
higher and the lower rates, the greater the inaccuracy of the IRR.

As the trial-and-error method is a long calculation, which may need a number of test calculations
before the correct two discount rates are found, the result using the formula would usually be
sufficient.
Normally one uses a financial calculator or software like Microsoft Excel to calculate the IRR of
cash flows. Figure 18.1 shows an example of a Microsoft Excel spreadsheet to calculate the IRR and
NPV.

Figure 18.1 Calculating IRR using Excel


A B C D E
1 Year Outflow 1 2 3
2 Cash Flow –500,000 138,000 158,000 208,000
3
4 Formula
5 NPV 188,492 =NPV(0.1, C2:G2)+B2
6 IRR 22.57% =IRR(B2:G2)

CLOSING PRACTICAL EXAMPLE


You will remember from the opening case study that a Life General Hospital in the Greater Durban
area is considering the purchase of a new MRI and CTC scanner. Although the existing equipment
can operate for a further three years, it is expected that the new, more efficient equipment will
reduce the number of scans required by patients. As a further benefit, labour costs may also be
reduced. The existing machine would have a zero terminal value in three years, but it is expected
that an immediate sale will result in a net cash inflow of R85 000.
The new equipment will require an initial investment of R2,4 million plus an extra investment
in working capital and spare parts for the new equipment, amounting to R80 000. This sum could
be set off against the expected cash inflow from the sale of the existing scanning device.
The new scanner is expected to have a useful life of five years and it will have no terminal
value at the end of its life. The hospital manager expects annual after-tax cash flows to amount to
approximately R800 000. These cash flows will occur throughout the year.

The hospital manager is faced with an important capital-budgeting decision, namely:


– to continue operating the existing equipment without change; or
– to purchase the new scanning equipment and replace the existing equipment.

To make this decision, the hospital may decide to use the NPV with an appropriate discount rate of
12 per cent. The following cash flows are expected:
Cash flow NPV at 12%
Year 1 800 000 714 286
Year 2 800 000 637 755
Year 3 800 000 569 424
Year 4 800 000 508 414
Year 5 (annual cash flow plus working capital) 880 000 499 335
Total discounted cash flow 2 929 214
Cash outflow from purchase of new machine (80 000 + 2
(2 480 000)
400 000)
Cash inflow from sale of old machine 85 000
Net present value 534 214
As the new machine has a positive NPV of R534 214, the hospital should purchase the new
machine. (Adapted from Horngren, Datar and Foster, 2011)

18.5 Summary
Investment decisions, also called capital-budgeting decisions, entail a commitment of financial
resources to capital projects with the purpose of securing future cash inflows for the entity.
The combination of an increase in revenue and a decrease in cost by improving production
efficiency will increase the firm’s productivity and expand its production capacity.
The entity needs to use the appropriate capital-budgeting method to decide which projects to
accept and which to reject. This chapter discussed several of the techniques used to evaluate capital
budgets.

PRACTICAL ACTIVITY
According to the company auditors, management has recently made a number of poor decisions
with regard to capital appraisal, resulting in several investments being unprofitable.
As a specialist in this field, you are asked to explain critically the use of at least five project-
evaluation techniques. In your explanation, include a discussion of techniques using the time value
of money and those that do not use time value. Then explain to management how you would
implement this process with the aim of improving decisions regarding future investment projects.

Short questions
1. The payback period method of capital appraisal is used extensively, but it has a number of
serious deficiencies. Which of the following are deficiencies of the payback period?
a. Cash flows after recovery of the initial investment are not taken into account.
b. The method does not take the time value of money into account.
c. It is easy to calculate and cash flows are readily available.
d. As an appraisal method, it should always used in conjunction with other methods.
2. The discounted payback period is an attempt to overcome the deficiencies of the payback
period. However, which of the following deficiencies remain with this method?
a. Cash flows after recovery of the initial investment are still not taken into account.
b. Short-term profitability is considered rather than profitability over the life of the project.
c. It does not distinguish between projects of different sizes.
d. The payback standard is still determined subjectively.
3. Which of the following are advantages of the NPV method?
a. The time value of money is integral to the calculation of the NPV.
b. All cash flows throughout the life of the asset are considered.
c. Projects being considered can be easily ranked from most profitable to least profitable,
thus simplifying decision making.
d. It is difficult to calculate.
4. Which of the following are disadvantages of the IRR?
a. Where positive and negative cash flows are present, numerous IRRs can occur, resulting
in incorrect decision making.
b. The IRR is difficult to calculate without a financial calculator or the use of a computer
program.
c. The IRR is easy to understand and thus apply.
d. It takes the time value of money into account.

Long questions1
1. A project costing R46 000 yields the following cash inflows:
Year 1 R12 000
Year 2 R16 000
Year 3 R20 000
Year 4 R24 000
Year 5 R16 000
Installation costs amount to R6 000. The opportunity cost of the investment is 20 per cent.
Using the following investment-appraisal methods, determine whether this project should be
accepted or not:
a. Discounted payback period
b. Net present value
c. Internal rate of return

2. Saturn Ltd has an opportunity to invest in a project with the following estimated future cash
flows:
R
Year 1 3 000 000
Year 2 4 000 000
Year 3 5 000 000
Year 4 2 000 000
The overall cost of the project is R11 000 000 and it has no residual value. The
company’s required rate of return is 15 per cent. Ignore taxation. Evaluate the above project
using the following methods:
• Payback period
• Discounted payback period
• Net present value
• Internal rate of return, using the formula method.
Write a report to the board of directors outlining in detail whether this project should be
accepted or not.

3. Trefoil Enterprises Ltd wishes to purchase a new cutting machine for its machine shop.
Agents from two different companies, A and B, have supplied the following information. Due
to limited funds, only one project can be accepted.
A B
Initial cost R200 000 R180 000
Expected life span 5 years 4 years
Scrap value expected R10 000 R8 000

Expected profits R R
End year 1 42 000 12 000
2 32 000 22 000
3 27 000 52 000
4 22 000 57 000
5 17 000 0
The company estimates its cost of capital to be 18 per cent and it uses the straight-line
method to calculate depreciation.
Using the above information, evaluate the projects using the following appraisal methods:
• Accounting rate of return
• Payback period
• Net present value
Which project should be accepted? Give your reasons.
4. Maxwell Ltd has the opportunity to invest in a project with the following estimated future
cash flows:
Year R000
1 2 400
2 3 000
3 4 000
4 3 200
5 1 800
The project cost is R8 000 000. The company’s required rate of return (cost of capital) is 14
per cent. The project has zero residual value. Ignore taxation.
Determine the following for the project:
• Payback period
• Net present value
• Internal rate of return
Would you accept the project or not? Give your reasons.

5. Modex Ltd is considering a new investment project. The following facts relate to this
project:
Cost R4 300 000
Useful life 5 years
Residual value in five years 700 000
Expected sales per annum 4 600 000
Expected costs per annum 2 800 000
Tax rate 30%
The equipment qualifies for the section 12C depreciation allowance of 20 per cent per annum
using the straight-line method of calculating depreciation. Modex’s cost of capital is
estimated to be 20 per cent. We will assume that all cash flows, apart from the cost of
acquisition, occur at the end of each year.
Calculate the following:
• Depreciation allowance
• Operating cash flows
• NPV of the project
Would you accept or reject the project based on the NPV calculation? Explain your
reasoning.

6. Consider the following two mutually exclusive projects, A and B.


Cash flows A Cash flows B
Year 0 Investment –175 000 –20 000
Year 1 Cash flows 10 000 10 000
Year 2 25 000 5 000
Year 3 25 000 3 000
Year 4 375 000 1 000
You require a 15 per cent return on your investment. Both projects are depreciated on the
straight line over the useful life of the asset. Project A has a salvage value of R10 000;
project B has no salvage value.
Apply the following capital-budgeting techniques to evaluate both projects:
• Accounting rate of return
• Payback period
• Net present value method
Which project would you choose and why?

7. Consider the following two projects that Dee Cee Investments is considering:
Project A Project B
Cost of plant and equipment R180 000 R120 000
Useful life 3 years 3 years
Salvage value R10 000 R5 000
Expected cash flows:
Year 1 R50 000 R15 000
Year 2 R45 000 R29 500
Dee Cee Investments uses the straight-line method of calculating depreciation for all fixed
assets. The estimated cost of capital is 10 per cent.
Calculate for each project:
• Accounting rate of return
• Payback period
• Net present value
• Internal rate of return
Which investment would you choose and why?

8. Consider the following two mutually exclusive projects:


Cash flows A Cash flows B
Investment (350 000) (40 000)
Year 1 120 000 20 000
Year 2 150 000 15 000
Year 3 50 000 8 000
Year 4 60 000 2 000
You require a 15 per cent return on your investment. Both projects are depreciated on the
straight line over their useful life. Project A has a salvage value of R10 000; project B has no
salvage value.
Apply the following capital-budgeting techniques to evaluate both projects:
• Accounting rate of return
• Payback period
• Net present value
• Internal rate of return
State which project you would choose and why.

9. According to the company auditors, management has recently made a number of poor
decisions with regard to capital appraisal, resulting in a number of investments being
unprofitable. As a specialist in this field, you are asked to explain critically the use of the net
present value method and the internal rate of return method as sophisticated techniques of
capital appraisal in an attempt to improve decisions regarding future projects. Use examples
in your evaluation.

10. Madlala Engineering Co (Pty) Ltd operates a machine shop in the greater Durban area. It is
considering the replacement of one of its metal cutting machines which is now obsolete. The
following quotation was received from Linca Heavy Machinery (Pty) Ltd. The new machine
would require an initial investment of R500 000 and the machine will have a useful life of 5
years. It will also have a salvage value of R60 000 at the end of its life. The expected profits
earned by the machine are as follows:
Year 1 R50 000
Year 2 R70 000
Year 3 R120 000
Year 4 R100 000
Year 5 R90 000
The machine will depreciate using the straight-line method over 5 years.
The company’s cost of capital is 10%.
You are required to present an evaluation for the company directors as to whether this
machine should be purchased or not. In your evaluation use the following methods of capital
appraisal:
• Accounting Rate of Return
• Payback Method
• Discounted Payback Method
• Net present Value Method
• Internal Rate of Return using:
– the Formula Method
– verify your answer using the Trial and Error Method.

Note
1 Questions 1–9 are adapted from Financial Management IV B.Tech exam papers at Mangosuthu

University of Technology.
PART 4
Decision making
CHAPTER 19
Cost–volume–profit analysis

CHAPTER 20
Linear programming

CHAPTER 21
Relevant costing

CHAPTER 22
Pricing decisions
19 Cost–volume–profit analysis
Peter Kamala and Johan Hefer

LEARNING OUTCOMES
After studying this chapter, you should be able to:
• explain the importance of cost–volume–profit analysis as a management tool;
• show that a marginal (variable) costing income statement provides the necessary information for
cost–volume–profit-analysis;
• calculate the contribution (marginal income) and the contribution ratio (marginal income ratio);
• compute the breakeven point using formulas and graphs;
• calculate the margin of safety and margin of safety ratio;
• estimate expected profit using cost–volume–profit analysis in a manufacturing environment; and
• list the limitations of breakeven analysis.

PRACTICAL EXAMPLE
James Mbuweni, who runs a small manufacturing business, opens a letter from one of his major
suppliers. The letter states that, due to unforeseen increases in input costs, the supplier has to
increase the selling price of its products by 25 per cent.
This comes as a shock to James, as he knows that he may not be able to increase his own
selling prices by that margin. He also has shareholders who are very demanding and have set
targeted levels of profits that he has to meet. His nephew, who is on holiday from university, tells
him about a planning tool known as cost–volume–profit analysis and how it might assist him with
his problem.

19.1 Introduction
Hardly a day goes by without a screaming headline in the newspapers, both locally and
internationally, about companies filing for bankruptcy or retrenching workers. Indeed, the business
environment has become very competitive as many companies have experienced a reduction in their
market share, or their profits have fallen due to rising costs or declining income because of the
challenging economic climate. As a result, an increasing number of businesses are struggling to cover
their fixed and variable costs, let alone make profits. When a business is just able to cover its costs
(i.e. it neither makes a profit nor a loss), it is said to break even. The level of activity (also known as
sales volume) at which a business neither makes a profit nor a loss is known as the breakeven point.
To ensure that their businesses at least break even, many company executives use planning tools to
enable them to understand the relationship between their volume of activity, costs and profits. One
such tool that has been increasingly employed in short-term planning is cost–volume–profit (CVP)
analysis (also known as breakeven analysis), which is the study of the effects of costs (both fixed and
variable), sales price and volumes, and sales mix (the number of units of each product line that is
sold) on profitability.

Simply put, CVP analysis is based on the interrelationship between cost, volume and eventual
profit. This interrelationship implies that a change in costs or sales will influence the profits.
Hence there is a causal relationship between these variables and profitability.

This chapter highlights the importance of CVP analysis as a management tool, shows that a marginal
income statement provides the necessary information for a CVP analysis, explains how to calculate
the contribution and the contribution margin ratio, and how to compute the breakeven point using
formulas and graphs. Furthermore, we will calculate the margin of safety and margin of safety ratio,
apply a CVP analysis to determine expected profits and discuss the limitations of breakeven analysis.
It should be borne in mind that CVP analysis is done within a specific planning horizon, normally
annually. Therefore, units produced and sold, total costs and all other factors are annual figures. We
will also assume that the sales volume and production volume are equal.
In the first part of this chapter, we will assume that only one product or service is provided by the
entity to simplify the explanation of the key concepts. CVP in an entity with multiple product lines is
discussed separately in Section 19.6.3.

19.2 CVP analysis (breakeven analysis)


CVP analysis helps managers understand the underlying relationship between the following factors:
• Sales price of a product
• Sales volume (or activity level)
• Variable cost per unit
• Total fixed costs
• Sales mix
• Estimated profit

Therefore, managers must analyse each of these elements and understand their impact on the
profitability of the entity.

19.2.1 Why is CVP analysis important?


By analysing and understanding the interrelation between these factors, management is able to
provide answers to the following questions:
• How many units must be sold to break even?
• How many units must be sold to achieve a targeted profit?
• What profit will be made at a given sales volume?
• How will a change in sales volume affect profits?
• How will a change in variable costs affect profits?
• How will a change in fixed costs affect profits?
• How will a change in price affect profits?

When the cost and management accountant has answered the above questions by means of a CVP
analysis, the results can inform key management decisions, such as determining the selling price. For
example, if an entity receives notification that the major input into its product is going to increase in
cost, the entity can use the analysis to assess the impact of the increase in variable costs on
profitability. If the increase has a large impact, the entity may decide to increase the selling price to
achieve its targeted profit.

19.2.2 CVP analysis and cost behaviour


When conducting a CVP analysis, it is important to understand how costs behave as volumes change.
These concepts were more fully discussed in Chapters 2 and 6, but are repeated here briefly. Total
variable costs change in proportion to changes in volumes produced and sold, whereas the variable
cost per unit remains constant. Furthermore, total fixed costs remain constant (within a relevant range)
as the production and sales volumes change, whereas fixed costs per unit decrease as the volumes
increase (see Figure 19.1).

Figure 19.1 Cost behaviour – fixed and variable costs

19.2.3 Comparison of accounting and economist’s models of CVP


The accounting model of CVP assumes that the behaviour of cost and revenue is linear. Accordingly,
the model is not intended to provide a completely accurate picture of the behaviour of revenues and
costs for all ranges of output, but only within a relevant range. In a relevant range, the assumptions
regarding the behaviour of cost and revenue remain valid. As an example, the rent paid for a factory
will remain constant unless the premises become too small because of increased volumes, in which
case a larger factory will have to be used to provide extra capacity for the increase in volume. This
will lead to an increase in fixed costs, which would significantly impact on the CVP model.
By contrast, the economist’s model of CVP assumes that the behaviour of cost and revenue is
curvilinear, as some costs do not increase proportionately owing to, for instance, quantity discounts
on bulk purchases as well as economies of scale. Equally, an increase in sales volume might not lead
to a proportionate increase in sales revenue, as such an increase may be the result of sales discounts
to customers. In these cases, a curvilinear pattern will emerge. Nevertheless, within a relevant range,
the differences between the accounting and economist’s models will be negligible. The accounting
model of CVP deliberately simplifies the economist’s model so that we can do CVP analysis (See
Figure 19.2)

Figure 19.2 Accounting and economist’s models of CVP compared


Accounting model

Key:
BEP = Breakeven point
R = Revenue
TC = Total cost
TFC = Total fixed cost
TR = Total revenue

Economist’s model

19.2.4 Other assumptions of CVP analysis


Given the complexity of the real business environment, it is unlikely that any model can fully
represent the real world. A CVP analysis model, like any other model, simplifies the real world by
making the following assumptions:
• All costs can be accurately distinguished as fixed or variable costs.
• Fixed costs remain constant regardless of the level (volume) of activity, whereas the variable
costs vary in direct proportion to the volume. (These ensure straight cost lines in the break-even
graph, which will be discussed later.)
• There is no change in effectiveness of the production factors that affect variable costs. Similarly,
all other variables remain constant, as factors such as supplier volume discounts and economies of
scale are ignored. These ensure linearity of variable cost functions.
• A business sells a single product or maintains a constant sales mix where it sells different types of
products.
• Costs and sales revenue are only influenced by one factor, namely the volume produced or sold.
• Inventory levels remain constant.

It should be emphasised here that these assumptions are required to enable CVP to function. If these
assumptions were not be made, CVP would not be possible, as a model representing all the variables
would simply be too complex to be useful. This means that the usefulness of the CVP model is limited
to the validity of the assumptions listed above. Accordingly, the assumptions are also the limitations
of the CVP model (as will be discussed at the end of the chapter). Nevertheless, the model remains a
very useful management tool for short-term decision making and profit planning.
To illustrate the limitations of CVP, let us consider an example. Assume that a company pays R2
per kilogram for the material it uses in its manufacturing process. The supplier, however, gives a
discount for bulk orders that changes the price to R1,95 per kilogram. Due to the variable nature of
demand for the product, the company can sometimes take advantage of the discount, while at other
times it cannot. This means that the variable costs are not totally linear, but change if larger volumes
are purchased. However, for the purposes of CVP, we have to assume that the variable costs are
linear, otherwise we would have to formulate a highly complex model that takes into account this
constraint.
Therefore, we accept the constraints of the accounting CVP model, which is easy to use and
accurate enough for the purpose it is intended for.

19.3 Marginal (variable) costing and absorption-costing income


statements
As mentioned above, one of the key assumptions of the CVP analysis model is that costs can be
accurately distinguished as fixed or variable. This assumption is very important, as it forms the basis
of an income statement, employed widely by the management of organisations. This statement is
referred to as the marginal (variable) costing income statement. Its format is provided in Table 19.1,
where it can be compared with the traditional format, known as the absorption-costing income
statement. (Refer to Chapter 14 for further information on marginal and absorption costing.)

Table 19.1 Comparison of absorption-costing and marginal-costing income statements


Absorption-costing income statement
Sales XX
Less cost of sales (XX)

Gross profit XX
Less operating expenses (XX)
Marketing XX
Administration XX
Net profit XX
Marginal-costing income statement
Less total variable costs (XX)
Total manufacturing variable
XX
costs
Direct material XX
Add direct labour XX
Add variable manufacturing
XX
overheads
Add total variable operating
XX
expenses
Sales commission paid XX
Add variable marketing cost XX
Add variable administration cost XX
Contribution XX
Less total fixed costs (XX)
Net profit XX

CVP analysis relies on the marginal (variable) costing income statement. In this income statement,
only variable production costs are included in the value of inventory. This differs from the absorption
(full) costing income statement, in which both variable and fixed manufacturing costs are included in
the value of inventory. As the International Financial Reporting Standards (IFRS) require that the
inventory is valued at full cost, the absorption-costing income statement, which complies with the
IFRS requirements, is employed in external reporting.
For the purposes of internal management decision making, fixed costs are assumed to be fixed
over the planning horizon. Therefore, including fixed costs in the valuation of inventory when an
absorption-costing income statement is prepared makes this type of statement less ideal for CVP
analysis. A marginal (variable) costing income statement, which excludes the fixed costs when
valuing inventory, is therefore more suitable for informing internal management for the purposes of
short-term planning, controlling and decision-making tasks, such as price setting and manufacturing or
purchase analysis.

19.4 Contribution (marginal) income


As shown in Table 19.1, contribution, or marginal, income is the difference between sales and
variable costs. Contribution is first used to cover the fixed costs and the remainder ‘contributes’ to
the profit of the period, hence the term. If the contribution is not adequate to cover the fixed costs,
there will be a loss. Contribution can be expressed in three ways:
• In total
• Per unit
• As a ratio or percentage (known as the contribution margin ratio)
These different ways of expressing the contribution are discussed in Sections 19.4.1, 19.4.2 and
19.4.3 using the information provided in Example 19.1.

EXAMPLE 19.1
XYZ (Pty) Ltd produces and sells a single type of iron box, which it calls Dax. The selling price
of each unit of Dax is R100. For the period ended 31 December 2013, the company manufactured
and sold 35 000 units. The sales representatives are entitled to a 10 per cent commission, in
addition to their fixed salaries, which are included in the fixed administration costs. The
following additional information is also available:
Per unit Total
Direct material R17,50
Direct labour R15,00
Variable manufacturing
R7,50
overheads
Sales commission R10,00
Fixed manufacturing overheads R100 000
Fixed administration overheads R150 000
Fixed marketing costs R125 000

Required
Calculate the following:
a. The total contribution
b. The contribution per unit
c. The contribution margin ratio

19.4.1 Total contribution


Total contribution refers to the excess of sales revenue over total variable costs. The total
contribution is used to cover the fixed costs. Where the total contribution exceeds the fixed costs, the
excess contributes to the profit. The formula for total contribution is as follows:

Total contribution = sales revenue – total variable costs

Solution (a)
R R
Sales revenue (R100 × 35 000) 3 500 000
Total variable costs: (1 750 000)
Direct material (R17,5 × 35 000) 612 500
Direct labour (R15 × 35 000) 525 000
Variable manufacturing
262 500
overheads (R7,5 × 35 000)
Sales commission (R10 × 35
350 000
000)
Total contribution 1 750 000

19.4.2 Contribution per unit


Contribution per unit is the excess of the selling price of each unit over its variable costs. For every
unit sold, its contribution is used to cover the fixed costs. Once all the fixed costs have been covered,
profit increases by the contribution per unit for every additional unit sold. The formula for
contribution per unit is as follows:

Selling price per unit – variable cost per unit

Solution (b)
R R
Selling price per unit 100
Total variable costs: (50)
Direct material per unit 17,5
Direct labour per unit 15
Variable manufacturing
7,5
overheads per unit
Sales commission per unit 10
Contribution per unit 50

19.4.3 Contribution margin ratio


T he contribution margin ratio is the percentage of contribution over sales. This ratio can be
expressed as a percentage of total contribution over total sales or as a percentage of contribution per
unit over selling price per unit. It can also be interpreted as the proportion of every R1 of sales that is
available to cover fixed costs. If, for example, the contribution margin ratio is 75 per cent, this is
interpreted as for every R1 of sales, R0,75 is available to cover the fixed costs. Upon reaching the
breakeven point, R0,75 of every R1 of sales is contributed to the profit. The following formulas can
be used to calculate the contribution margin ratio:
Solution (c)
• First formula (total contribution as a percentage of total sales)

• Second formula (contribution per unit as a percentage of selling price)

As you can see, both formulas give the same answer. This should not come as a surprise, as total
sales and total variable costs vary directly with the volumes sold, but both the unit selling price and
variable cost remain constant. The interpretation of the above results is that for every unit that XYZ
(Pty) Ltd sells, 50 per cent of the selling price contributes to covering the fixed costs. Once the fixed
costs have been covered, any extra amount contributes towards the company’s profit.
Just as the contribution is expressed as a ratio, the variable costs too can be expressed as a ratio.
The percentage of variable costs over total sales is known as the variable cost ratio or the marginal
cost ratio. In the above example, in which the contribution margin ratio is 50 per cent, the variable
cost ratio will be 50 per cent (i.e. 100 per cent sales minus the contribution margin ratio of 50 per
cent). This means that for every R1 of sales, R0,50 of variable costs will be incurred. If, however,
the contribution margin ratio had been (say) 60 per cent, the variable cost ratio would have been 40
per cent.

19.5 Breakeven point


The breakeven point refers to the sales volume at which the total contribution exactly equals the
fixed costs. It is the point at which neither a profit nor a loss is made. Simply put, the breakeven
point is the level of sales, be it in quantity or value, at which the profit is zero.

If contribution exceeds the fixed costs then a profit is made, but if the fixed costs exceed the total
contribution, then there will be a loss. Therefore, the breakeven quantity or volume is the minimum
quantity of a product that must be sold in a given period – normally a year – to ensure that all fixed
costs are covered and that a business does not experience a loss.
It is important for companies to know their breakeven point, as this helps them to avoid incurring
losses when making short-term decisions – for example, determining whether to accept a special
order at a discounted selling price or not. Failure to break even puts the survival of a company at risk.
As already alluded to, the breakeven point can be expressed in two ways, namely in units (i.e.
quantity or volume) and in sales value (i.e. rands).
The breakeven point can be illustrated with a line graph (see Figure 19.3).

A breakeven graph is a visual representation of cost and revenue at different levels of activity,
which shows the profit or loss at the different levels within a range. The graph is used to estimate
the point at which sales revenue equals total cost, where neither a profit nor a loss is made.

Figure 19.3 Breakeven graph

In Figure 19.3, the breakeven point (P) is at the intersection of the total revenue (A) and the total costs
(C).
The horizontal axis represents the sales volume/output in units or value; the vertical axis represents
sales revenue and costs.
The line OA represents revenue at varying levels of production and sales volumes. OB represents
the fixed costs. An increase in output leads to an increase in variable costs incurred, meaning that the
total costs (fixed + variable) also increase. At lower levels of output, the costs are greater than the
revenue. At the point of intersection, P, costs are exactly equal to income, and hence neither a profit
nor a loss is made. This is the breakeven point.

19.5.1 Breakeven units (quantity or volume)


The breakeven units figure can be calculated using the following formula:

Returning to the information given in Example 19.1, the breakeven units can be calculated using the
above formula, as shown in Example 19.2.

EXAMPLE 19.2
Total fixed costs R
Fixed manufacturing overheads 150 000
Fixed administration overheads 100 000
Fixed marketing costs 125 000
Total fixed costs 375 000

Contribution per unit


Thus breakeven units = 375 000 ÷ 50 50
= 7 500 units
This means that XYZ Pty Ltd has to sell 7 500 units to break even. To show this, we can prepare
a marginal income statement as follows:
Sales (7 500 × 100) R750 000
Less total variable costs: (7 500 × 50) R375 000
Total contribution R375 000
Less total fixed costs R375 000
Net profit R0
For every unit that XYZ Pty Ltd sells, R50 (the contribution per unit) contributes towards
covering the fixed costs. Once the fixed costs are covered, any additional unit sold contributes
R50 to the profit. We can show this as follows. Assuming that the actual sales and production
units were 7 501 (one unit above the breakeven point), the marginal income statement would
appear as shown below:
Sales (7 501 × 100) R750 100
Less total variable costs: (7 501 × 50) R375 050
Total contribution R375 050
Less total fixed costs R375 000
Net profit R50
From the above, we can conclude that profit made equals the number of units sold in excess of the
breakeven point, multiplied by the contribution per unit. Therefore, in this example the profit for
selling 8 500 units (1 000 units more than the breakeven point) would be R50 000 (1 000 × 50).

19.5.2 Breakeven value


The breakeven value is the rand value of the units sold at the breakeven point. In other words, it is
the sales value when neither a profit nor a loss is made.

Two formulas can be used to determine the breakeven value as follows:

Or alternatively:

Breakeven point units × selling price per unit


Using the figures in Example 19.1, the breakeven value can be calculated as follows:
Total fixed costs
Contribution margin ratio R375 000
Breakeven value = R375 000 ÷ 0,5 50% or 0,5
Alternatively: = R750 000
7 500 units × R100 = R750 000

19.6 Application of CVP analysis


CVP analysis has many applications. We will now discuss some of the most widely used
applications, including calculating the margin of safety; determining sales volume to achieve a
targeted profit; considering multiple products; and giving an indication of the impact of a change in
selling price, variable costs, fixed costs or sales mix on profitability.

19.6.1 Margin of safety


Margin of safety is the difference between the actual or budgeted sales volume or value and the
breakeven sales volume or value.

The margin of safety is used to indicate the extent, in units or as a percentage, to which the actual or
budgeted sales volume or value can drop before a company incurs a loss (or, put differently, when the
sales volume or value falls below the breakeven point volume or value). The margin of safety can
thus be expressed in three different ways, namely:
• Margin of safety units
• Margin of safety value
• Margin of safety ratio

Continuing with Example 19.1, the margin of safety can be calculated as shown in Example 19.3.

EXAMPLE 19.3
Margin of safety units
Actual or budgeted sales units – breakeven sales units
35 000 – 7 500
= 27 500 units

Margin of safety value


Actual or budgeted sales value – breakeven sales value
R3 500 000 – R 750 000
= R 2 750 000
Margin of safety ratio

Knowing the margin of safety is vital to management, especially when facing financial difficulties, as
it acts as a risk measure. The lower the margin of safety, the higher the risk of incurring losses. In
Example 19.3, if sales were to decrease by more than 78,57 per cent, XYZ Pty Ltd would incur a
loss. This means that the company is financially healthy, as it has a high margin of error and a decline
in sales of less than 78,57 per cent would still leave the company profitable.

19.6.2 Expected or targeted profit


The CVP technique can be used to predetermine the sales volume that will produce a certain net
profit. In this regard, the following formula is used:

EXAMPLE 19.4
Supposing XYZ Pty Ltd has a targeted profit of 25 per cent on its initial capital investment of R2
million. Using the formula above, determine the number of units to be sold in order to achieve the
targeted profit (ignore tax).

Solution
First, we determine the targeted profit as follows:
25% × R 2 000 000 = R 500 000
Second, we apply the formula as follows:

To determine the sales value that will achieve the targeted profit, we do the following:
17 500 units × selling price per unit
17 500 × R100 = R 1 750 000
This could also be calculated using the following formula:
The above formula may be adjusted to determine the sales volume that will achieve a targeted
profit after tax as follows:

EXAMPLE 19.5
Supposing XYZ Pty Ltd has an after-tax targeted profit of 25 per cent on its initial capital
investment of R2 million. Using the formula above, determine the number of units to be sold in
order to achieve the targeted profit; assume a tax rate of 20 per cent.

Solution

This could be expressed as a sales value (in rands) by multiplying the units by the selling price
per unit as shown below:
20 000 units × 100 = R2 000 000

19.6.3 Breakeven analysis for multiple products


In cases where a company sells more than one product, breakeven analysis is more complicated
because each product has its own contribution per unit or contribution margin ratio. Therefore, a sales
mix (i.e. the ratio of volumes of different products) must first be determined to be able to calculate the
contribution per unit or contribution margin ratio on a weighted average basis.

EXAMPLE 19.6
ABC Ltd manufactures and sells three different products: X, Y and Z. Additional information
about the products is provided below:
Product X Product Y Product Z Total
R R R R
Selling price per
20 30 40 90
unit
Variable costs per
unit 14 24 36 74

Fixed costs in total 112 000


Product X Product Y Product Z
Sales mix in units 20 000 12 000 8 000

Required
Calculate the breakeven quantity and value.

Solution
The sales mix can be expressed as a simple ratio:
Product X Product Y Product Z
Sales mix ratio 5 3 2
We now need to determine the weighted average contribution per unit.
Contribution per unit
Mix Total (R)
(R)
Product X 6 5 30
Product Y 6 3 18
Product Z 4 2 8
Average per unit (R56
5,6 10 56
÷ 10)
Breakeven quantity:

Breakeven quantity for each product according to the sales mix of 5 : 3 : 2:


X: × 20 000 = 10 000 units
Y: × 20 000 = 6 000 units
Z: × 20 000 = 4 000 units

The breakeven value can be determined by multiplying the breakeven units by selling prices.
X: 10 000 units × 20 = R200 000
Y: 6 000 units × 30 = R180 000
Z: 4 000 units × 40 = R160 000
R 540 000
19.6.4 Changes in selling price, variable cost per unit and fixed costs
In the discussion up to now, we have assumed that factors such as selling prices, variable cost per
unit and fixed costs remain constant over the planning horizon. In the real world, however, these
change over time. These changes may be due to normal price increases from suppliers, larger than
expected labour cost increases or a decrease in selling price due to increased competition. Therefore,
it is important to examine the effect of changes in these factors on breakeven calculations. The
following examples illustrate how this is calculated.

EXAMPLE 19.7: CHANGE IN SELLING PRICE


Martin Payne Ltd produces and sells a single product called Rex. The financial director of the
company provides you with the following information regarding product Rex:
R
Selling price per unit 20
Variable costs per unit 12
Contribution per unit 8
Total fixed cost 80 000
The directors of the company have decided to increase the selling price by 10 per cent with the
aim of achieving a targeted profit of R40 000 (ignore tax).

Required
– Determine the number of units that must be sold to break even.
– Calculate the number of units that must be sold to achieve the targeted profit.

Solution
Current (R) After price increase (R)
Selling price per unit 20 22 (R20 + 10%)
Variable costs 12 12
Contribution per unit 8 10
1. Breakeven in units:

= 10 000 units = 8 000 units


2. Number of units that must be sold to achieve the targeted profit (R40 000):

= 15 000 units = 12 000 units


EXAMPLE 19.8: CHANGE IN SELLING PRICE
Drakes Pty Ltd bakes and sells a single type of cake. The variable cost of production is R30 per
unit and the current selling price is R50 per unit. The fixed costs are R5 200 per month, and the
annual profit for the company at the current sales volume is R72 000. The volume of sales is
constant throughout the year.
The sales director is considering whether to raise the selling price to R58, bearing in mind
that the price increase might result in some loss of sales.

Required
Determine the minimum sales volume required each month that would justify increasing the
selling price from R50 to R58.

Solution
The minimum sales volume that would justify a price increase to R58 is one that would at least
leave the total profit at the same level as before, R6 000 (R72 000 ÷ 12). To determine the sales
volume, the profit must be converted into the required production, as follows:
R
Monthly fixed costs 5 200
Minimum monthly profit required 6 000
Current monthly contribution 11 200
The minimum volume of sales required after the increase in the selling price will be an amount
that earns a contribution of R11 200 per month, which equals the current contribution. At the new
price of R58, the contribution per cake will be R28 (R58 – R30).

EXAMPLE 19.9: CHANGE IN VARIABLE AND FIXED COSTS


Laface Ltd makes a product with a variable production cost of R16 per unit and variable selling
costs of R4 per unit. Fixed costs are R80 000 per annum, the selling price per unit is R36, and the
current volume of units produced and sold is 12 000 units per annum.
The company is considering whether to hire a new machine that is expected to reduce the
variable production cost per unit to R12. This would cost the company R20 000 annually in hiring
fees.

Required
Assuming the machine is hired:
– Determine the number of units that must be produced and sold to achieve the same profit as is
currently earned.
– Calculate the annual profit if the output and sales remain at 12 000 units per annum.

Solution
The current contribution per unit R36 – (16 + 4) = R16
1.
R
Current contribution (12 000 × R16) 192 000
Less current fixed costs 80 000
Current profit 112 000
If the machine is hired, the fixed costs will increase from R20 000 to R100 000 per annum.
The variable cost per unit will fall to R(12 + 4) = R16 and the contribution per unit will be R20.
R
Required profit as currently earned 112 000
Fixed costs after hiring the new machine 100 000
Required contribution 212 000
Contribution per unit = R20
Sales required to earn R112 000 profit = 212 000 ÷ 20 = 10 600 units

2. Annual profits if sales are 12 000 units


R
Contribution (12 000 × R20) 240 000
Less fixed costs 100 000
Profit 140 000
Alternatively:
Profit at 10 600 units of sale (see 1.) 112 000
Contribution from sale of extra 1 400 units (1
28 000
400 × R20)
Profit 140 000

19.7 Other methods of conducting a breakeven analysis


Breakeven analysis can also be conducted using the algebraic method, the equation method and the
graphical method. The most widely used of these, the algebraic method, is discussed and illustrated in
the next section.

19.7.1 Algebraic method


To determine profit, we subtract the variable costs and fixed costs from the sales. In algebraic terms,
this can be expressed as:

Sales – variable costs – fixed costs = profit

With the aid of letter symbols, this elementary equation can be transformed to calculate the unknown
variables in a breakeven analysis. This is because the equation contains every component of a
condensed marginal costing income statement, as follows:
Sales S
Less variable costs V
= Contribution C
Less fixed costs F
= Profit P
By rearranging the symbols, each of the above components of an income statement can be determined
by using simple equations as follows:
Sales S = V+ F + P
Variable costs V= S – F – P
Contribution C=S–V
Fixed costs F = S – V– P
Profit P = S – V– F
By expanding the above formulas further, other unknowns in the breakeven analysis can also be
determined as follows:

EXAMPLE 19.10: BREAKEVEN ANALYSIS USING ALGEBRAIC


METHOD
The financial director of UOT Ltd is concerned about the comapny’s declining sales. He provides
you with the following information:
Sales R50 000
Variable costs R30 000
Margin of safety ratio 40%

Required
Using algebraic calculations, compute the following:
1. Contribution
2. Contribution margin ratio
3. Net profit
4. Fixed costs
5. Breakeven value
6. Margin of safety

Solutions
1. Contribution C = S – V
= R50 000 – R30 000
= R20 000
2. Contribution margin ratio (CMR)
3. Net profit P = S – V – F
= R50 000 – R30 000 – F
Given that there are two unknowns, an alternative way of determining the profit must be found.
The solution is found in the margin of safety ratio (MSR), which in this case is 40 per cent. This
means that the contribution can decrease by 40 per cent before any losses will occur. Simply put:

P = C × MSR
= R20 000 × 40%
= R8 000

F = S – V– P
4. Fixed costs = R50 000 – R30 000 – R8 000
= R12 000
5. Breakeven value

MS = S – BEV
6. Margin of
= R50 000 – R30 000
safety (MS)
= R20 000

19.8 Limitations of CVP analysis


Although CVP analysis can be a useful tool for managers given that it provides simple and quick
estimates that can be represented graphically, it does, however, have certain limitations. These are as
follows:
• It only applies to a single product or single mix of products.
• It assumes that fixed costs remain constant over the volume range.
• It assumes that production and sales volumes are the same, so it ignores inventories.
• It assumes that the selling price and variable cost per unit are constant over the entire volume
range.
• It assumes that total variable costs fluctuate in direct proportion to volume.
• It assumes that volumetric increase is the only factor affecting costs.
• It ignores the uncertainty in the estimates of fixed costs and variable cost per unit
• It assumes that the efficiency in the use of resources will remain constant over the period.

CLOSING PRACTICAL EXAMPLE


James Mbuweni, who owns a small manufacturing business, opens a letter from one of his largest
suppliers. The letter states that, due to unforeseen increases in input costs, the supplier has to
increase the selling price of its products by 25 per cent.
Thanks to the help of his nephew, James has been able to use CVP analysis to:
– determine the impact of the increase in variable costs on his company’s profitability;
– determine the selling price if he wishes to maintain the same profitability; and
– determine the volume of sales required to generate the same profitability.

19.9 Summary
This chapter has demonstrated how CVP analysis can provide managers with a useful short-term
planning tool. CVP analysis can be used to calculate target profits, and reach pricing and volume
decisions.
Central to CVP analysis is determining the breakeven point, which can be done using simple
equations, a breakeven graph and an algebraic method.
CVP analysis can also be applied to determine the breakeven points of multiple products, where
the sales mix ratio is employed to calculate the breakeven points of individual products after the total
breakeven point has been calculated using a weighted-average contribution margin.
Although a useful technique for financial planning, CVP analysis depends on a number of
underlying assumptions, which means it has certain limitations. Understanding the impact of these
assumptions and limitations is critical to performing a valid and reliable CVP analysis. It is
reassuring to note that only significant deviations from the abovementioned assumptions are likely to
impact on the validity and reliability of the analysis.

Questions
1. Rearrange functional format income statement below as a behavioural format income
statement and calculate the following:
a. Variable cost per output unit
b. Variable cost ratio and variable cost percentage
c. Contribution margin per output unit
d. Contribution margin ratio and contribution margin percentage
e. Breakeven units
f. Breakeven value in rands
g. Margin of safety
h. Margin of safety ratio
i. How many units must be sold in order to obtain an income of R240 000 before tax?
j. How many units must be sold in order to obtain an income of R240 000 after tax?
Sales (sales price = R100) 800 000
Cost of goods sold:
Direct materials 160 000
Direct manufacturing labour 240 000
Variable factory overhead 48 000
Fixed factory overhead 80 000 528 000
Gross profit 272 000
Selling gen. & adm. expenses:
Variable SG&A expense 32 000
Fixed SG&A expense 50 000 82 000
Income before tax 190 000
Income-tax expense 57 000
Net income 133 000

2. Green Point Lawn Chair Ltd produces and sells single high-priced lawn chairs. In 2013
the company produced and sold 60 000 units. The company’s income statement for the year
ended 2013 is presented below:
Green Point Lawn Chair Ltd income statement for year ended 2013 Total R
Sales 3 600 000
Variable costs 2 700 000
Contribution margin 900 000
Fixed costs 480 000
Income before tax 420 000
Tax expense 126 000
Income after taxes 294 000
Determine the following:
a. The breakeven point in units.
b. The margin of safety in units.
c. If the company’s sales in units were to increase by 30 per cent, what would be the
percentage increase in profits before taxes?
d. The sales in units to achieve R540 000 in profits before taxes.
e. Assume the company is expecting to experience a shortage of its main raw material.
This situation is expected to result in an increase in the company’s variable production
costs by R6 per unit. Given this scenario, and assuming that the company does not
believe that it can increase its selling price, determine the company’s breakeven point
and new safety margin.
f. Management has decided to raise the price of its product to R130 per unit. It will also
spend an additional R204 000 per year on advertising. Although it has never paid
commissions before, the company has decided to begin paying sales personnel R2 per
unit for every unit sold. Determine the new breakeven point. Also determine the safety
margin of the company under this plan if sales only reach 54 000 units.

3. PDD Ltd owns the MusicFinder, a sophisticated satellite radio. The company has
experienced a steady growth in sales for the past five years. However, Mr J, PDD’s CEO,
believes that to maintain the company’s present growth will require an aggressive
advertising campaign next year. To prepare for the campaign, the company’s accountant,
Mr Z, has prepared and presented to Mr J, the following data for the current year, 2013:
Variable costs:
Direct labour (per unit) R200
Direct materials (per unit) R90
Variable overheads (per unit) R40
Total variable costs (per unit) R330

Fixed costs (annual):


Manufacturing R1 600 000
Selling R1 200 000
Administrative R1 600 000
Total fixed costs (annual) R4 400 000
Selling price (per unit) R800
Expected sales revenue, Year 1 (25 000 units) R20 000 000
PDD Ltd has an income tax rate of 35 per cent. PDD has set the sales target for the
following year, 2014, at a level of R22 400 000 (or 28 000 radios).

Calculate the following:


a. The breakeven point in units for 2013.
b. Mr J believes that to attain the sales target (28 000 radios) will require additional
selling expenses of R600 000 for advertising in 2014, with all other costs remaining
constant. What will be the after-tax operating profit for 2014 if the firm spends the
additional R600 000?
c. What will be the breakeven point sales in rands for 2014 if the firm spends the
additional R600 000 on advertising?
d. If the firm spends the additional R600 000 on advertising in 2014, what is the sales
value level in rands required to equal the 2013 after-tax operating profit?
e. At a sales level of 28 000 units, what is the maximum amount the company can spend
on advertising to earn an after-tax operating profit of R1 500 000?

4. You are provided with the following information for KB Ltd for March 2014:
Sales R720 000
Fixed manufacturing costs R70 000
Fixed marketing and administrative costs R50 000
Total fixed costs R120 000
Total variable costs R240 000
Unit price R180
Unit variable manufacturing cost R110
Unit variable marketing cost R10
Calculate the following:
a. Breakeven point in units.
b. Number of units sold that would produce an operating profit of R240 000.
c. Sales value in rands required to earn an operating profit of R40 000.
d. Number of units sold that would produce an operating profit of 40 per cent of sales in
rands.

5. Digolo Limited provides you with the following information:


Selling price per unit R158.00
Variable manufacturing cost per unit:
Direct material R25.00
Direct labour R47.20
Manufacturing overheads R29.80
Total fixed manufacturing cost:
Manufacturing overheads R190 000
Non-manufacturing cost:
Fixed marketing cost R60 000
Fixed administration cost R90 000
Sales commission 10% of the selling price per unit
By treating each question independently, answer the following:
a. Prepare a complete marginal income statement.
b. Calculate the breakeven sales in units and breakeven sales in rands.
c. What is the margin of safety units if Digolo Ltd sells 13 900 units during the period?
Why is the margin of safety ratio useful to the management of a company?
d. A marketing research consultant has advised the company that a 15 per cent decrease in
selling price will give the company an advantage over the competitors, as its prices
will then be lower than those of its competitors. The 15 per cent decrease in sales price
will increase the sales volume to 14 250 units compared to the current sales volume of
11 250 units. The increase in sales volume will then result in an increase profits. Do
you agree with the consultant’s advice? Assuming that this is the only change, prepare a
marginal income statement and use calculations to support your answer.
20 Linear programming
Marina Bornman

LEARNING OUTCOMES
After studying this chapter, you should be able to:
• understand the circumstances in which linear programming is used;
• outline the main concepts and principles of linear programming;
• understand and list what is meant by limiting factors, or constraints;
• construct and interpret a linear-programming solution for a two-variable problem using a graph;
and
• discuss the limitations of linear programming.

PRACTICAL EXAMPLE
A company manufactures liquid hand soap and bars of soap. The same base raw material
(glycerine) is used in both products, but the supply of glycerine is limited. The same machine is
used to mix the ingredients, but different quantities of ingredients and different mixing times are
needed for the two products. The machine hours are limited to a certain number of hours per
month, as there is only one machine. The decisions that management need to make, given these
constraints, are how to allocate these resources and how many units of each product they should
produce to maximise profit.

20.1 Introduction
Organisations often need to make decisions regarding their product mix. For example, they need to
decide how many units of product A and how many units of product B should be produced in a certain
period. It may be that the demand for a product or service is greater than the organisation is able to
deliver. A firm may be restricted by, for example, the number of machines available or the number of
workers within a given period, or the amount of raw material at its disposal. Similarly, when
rendering a service, an organisation may need to decide on the optimum mix of services in a specific
period. An organisation may be restricted by the number of labour hours available to meet the demand
for the service.
Scarce resources, such as limited labour hours, machine hours or supply of materials, are
referred to as limiting factors or constraints. A constraint means that the number of units that can be
produced in a certain period is limited to the availability of the resources necessary for production
during the period.
In situations where an organisation is constrained by scarce resources, it becomes necessary for
management to make a decision on how to make use of these scarce resources in order to maximise
profit.
Often more than one scarce resource limits production or constrains the provision of a service.
Take the example of an entrepreneur who operates rubble-removal and tree-felling services. He only
has two workers available per day and limited hours per day for the use of the chain saw. The
business owner is therefore constrained by two scarce resources: labour hours and machine hours. He
must make a choice between how many trees to fell and how many rubble removals to do per day
(assuming there is a demand for both every day).
Linear programming is a technique used to determine the optimum number of units to produce of
each type of product, or which services to render, in a situation where the resources cannot meet the
expected demand and there is a choice to be made with regard to the combination (or mix) of
products or services. Linear programming only needs to be applied in a situation where there is more
than one scarce resource limiting production or the provision of services.
This chapter introduces the concepts and principles of linear programming so the reader can
understand the technique, and explains the steps to follow when calculating the optimum combination
of two products in order to maximise profit. The focus is on the graphical method, with a maximum of
two manufactured products.

20.2 Concepts and principles of linear programming


There are a number of important terms associated with linear programming. Those that are most
important, and which need to be understood, are defined here:
• Limiting factors (also called constraints) – any input resource that is limited in supply and puts a
constraint on the number of units that can be manufactured or the number of services that can be
provided (e.g. production capacity of machinery).
• Contribution per unit – you will recall that a product’s (or service’s) selling price less its
variable cost gives the contribution per unit. Therefore, this is equivalent to the amount remaining
when the variable cost per unit is deducted from the selling price per unit. Contribution per unit
must be determined for each product or service to be used in the linear-programming model. If the
contribution per unit for both products or services is exactly the same, it is not necessary to apply
the technique of linear programming, as the combination of units will not influence the profit
maximisation.
• Optimum product composition – the linear-programming technique will result in determining the
optimum combination (mix) and number of units to produce of each product in order to maximise
profit.

In practice, results obtained by using the linear-programming technique might not be followed by
management. Other factors may influence the decision on the number and mix of units to produce or
services to provide, such as the personal choice of management, strategic decisions about the focus of
the company, marketing strategies for a specific product and responding to competitors’ strategies.
One should remember that linear programming is merely a tool to be used in the decision-making
process and in certain circumstances, as explained.

20.3 Understanding limiting factors


Some limiting factors to be considered in the linear-programming model that are typically
experienced by manufacturing companies are as follows:
• Raw materials – certain materials may be in scarce supply because of seasonal factors, import
restrictions, suppliers’ capacity, etc.
• Machine hours – machines used in manufacturing have capacity constraints in terms of the number
of hours they can function. A machine cannot necessarily operate 24 hours a day, especially if run
by operators, and machinery also needs regular downtime for maintenance.
• Labour – the number of direct labour hours available will depend on the number of workers hired.
Labour can also be limited if workers with specific scarce skills are required.
• Space constraints – a factory may be limited in its production capacity because of space available
for its machinery and workers.
• Demand for the product – this is an important consideration and key element in the linear-
programming model. The demand for a product will set the maximum limit for the number of units
to be produced in the optimum combination. It is, however, important to note that demand is not an
input, and is therefore different from the other factors mentioned here, as it is not a resource.

Limiting factors in a non-manufacturing (service) environment are similar to those mentioned above,
with the exception that no raw materials are transformed in the case of a service. However, materials
may be used in the rendering of a service, and such materials may also be short in supply for the
reasons mentioned above.
Example 20.1 illustrates how raw material can be a limiting factor in production.

EXAMPLE 20.1
Miriam makes curtains and duvets using the same material. One duvet uses 2 m of material and
one curtain uses 4 m. Miriam buys the material on a 50 m roll and needs to plan how many duvets
and how many curtains to make from a roll.
Construct an Excel spreadsheet as follows:
A B C D
Duvet Curtain
1 No. of metres of No. of duvets No. of metres of No. of curtains
material allocated material allocated
2 50 =A2/2 =50-A2 =C2/4
3 40 =A3/2 =50-A3 =C3/4
4 30 =A4/2 =50-A4 =C4/4
5 20 =A5/2 =50-A5 =C5/4
6 10 =A6/2 =50-A6 =C6/4
7 0 =A7/2 =50-A7 =C7/4

You can see from the spreadsheet how the allocation of material dictates the number of duvets or
curtains that can be made. For example, in row 4, if 30 m of material is allocated to duvets, 15
duvets can be made, but that means only 20 m of material is available for curtains, which results
in only five curtains that can be made.
20.4 The linear programming model
The graphic method in linear programming is used where a decision must be made on the optimum
combination between a maximum of two products. The x and y axes of the graph represent the two
products (see Figure 20.1). The number of constraints, however, need not be limited. It should be
remembered that all units need to be standardised. For example, one product’s labour constraint
cannot be expressed in hours and the other in minutes.
The model used to produce the graph and solve the problem consists of the following five steps:
• Step 1. State the objective function. This is determined by asking the question, what is the goal or
objective with this production decision? Usually it is to maximise contribution (profit).
• Step 2. Write down all the limiting factors or constraints in the form of a linear equation with
product A and B as the variables. As we know that production cannot be negative, the assumption
will always be that A≥0 and B≥0.
• Step 3. The constraints are then plotted on the graph as straight lines. An area is identified that
falls within all the limits. This is known as the feasible area. The optimal solution will lie on one
of the corners of the feasible area.
• Step 4. Draw the objective function as a dotted line on the graph.
• Step 5. The optimal solution can be read from the graph by moving the dotted line in a parallel
fashion to where it touches an intersection of the constraint lines at the furthest point. This point
will represent the optimal (i.e. profit-maximising) number of units of product A (read from the x
axis) and the optimal number of units of product B (read from the y axis).

This process is shown by means of an example.

EXAMPLE 20.2
A company manufacturing two types of vehicle tyres provides the following information:
Tyre A Tyre B
Contribution per unit R150 R120
No. of labour hours per unit 2 hours 1.5 hours
Raw material used per unit 4,5 kg 3 kg
The raw material costs R45 per kg and its supply is restricted to 2 700 kg per month. The total
available manufacturing capacity for an average month is 800 hours of labour. The maximum
demand for tyre A is 380 tyres per month and for tyre B 450 tyres.

Required
Use the graphical linear programming to determine the optimum combination for production in
order to maximise profit.

Solution
• Step 1. State the objective function: Maximum profit = 150A + 120B. The objective is to
determine the number of units of tyres A and B to produce per month to result in the maximum
profit.
• Step 2. Identify all the constraints and write linear equations in terms of the limit: Material:
Material: 4,5A + 3B = 2 700
Labour hours: 2A + 1,5B = 800
Demand for tyre A: A = 380 units
Demand for tyre B: B = 450 units
• Step 3. Draw the graph. The x and y axes represent the number of units of product A and B.
Find the intersections on the x and y axes by using the linear equation for each limiting factor.
e.g. for material: 4,5A + 3B = 2 700
by making A = 0: 3B = 2 700 Therefore B = 2 700 ÷ 3 = 900
by making B = 0: 4,5A = 2 700 Therefore A = 2 700 ÷ 4,5 = 600

Do this for every equation and draw all the straight lines for each limiting factor. Remember to
name all the lines.
• Step 4. Draw the objective function.
The objective function was: 150A + 120B = profit
The profit ratio in units between product A and B is 150:120 if one unit of each is sold. Use
these values as intersections with the x and y axes. In a case where the given contributions are
too small to use directly on the graph, use multiples of the values to get a significant
intersection on the axis. For example, a multiple of 150:120 is 300:240. Draw this objective
function as a dotted line on the graph.
The area enclosed by points K, L, M and N on the graph represents the feasible area. This
means that the combination of units A and B to obtain maximum profit will lie in this area.
Any combination that lies outside this area is not possible because of the limited resources.
• Step 5. Find the optimal combination of number of units for product A and B by moving the
objective line in a parallel fashion until it touches the furthest point of the enclosed area. As
can be seen from Figure 20.1, this is at point L. At this point, you can read the number of units
of product A and B from the x and y axes.

Figure 20.1 Linear programming with a graph


Notes: Point K is the intersection of the demand for B line with the y axis; Point L is the
intersection of the labour line with the demand for B line; Point M is the intersection of the labour
line with the demand for A line; Point N is the intersection of the demand for A line with the x
axis.

In Figure 20.1, points K, L, M and N enclose an area in which all the possible combinations of
numbers of units to be produced of A and B is found. No combination of units outside this area
can be produced, as one of the limiting factors will limit such a combination. For example, the
demand for product B is limited to 450 units, therefore production of B should not exceed 450
units and should therefore not lie above the line where B = 450.
From the graph, we can see that the optimal combination is about 60 units of product A and
450 units of product B.
It may be difficult to achieve an accurate result using only the graph. Therefore, a
mathematical method can be used to reach the final solution. Point L is the point where the two
lines meet, representing the demand for product B (B = 450) and the labour constraint (2A + 1,5B
= 800). At the point where two lines meet, the values for A in both equations are equal. So are the
values for B equal to each other in the two equations. We can find the values for A and B by
solving the two equations, as follows:
Equation 1: B = 450
Equation 2: 2A + 1,5B = 800
By using the value for B (450) in equation 2, we can solve A:
2A + 1,5(450) = 800
2A = 800 – 675 = 125
Therefore A = 125 ÷ 2 = 62,5 units.
The final (accurate) solution is therefore 62,5 units of product A and 450 units of product B to
be manufactured in order to maximise profit. Half a unit cannot be produced, however, so the
number should be rounded down to 62 units of product A.
The total contribution can now be calculated by replacing the values for A and B in the
objective function:
Objective function = 150A + 120BA
= 150(62) + 120(450)
= R63 300

20.5 Limitations of linear programming


Although a useful planning tool, linear programming has the following limitations:
• Accuracy: The optimum solution predicted by the model can only be as accurate as the information
provided to set up the model. In real-world scenarios, it is not always possible to reduce
variables to a linear equation.
• Division of resources: It is not always possible to allocate resources, such as labour and machine
hours, at will between different products. For example, the constraint may state that a machine can
produce eight units of product A or eight units of product B per day, but can it handle, say, four
units of each per day?
• Extent of product dependability: The model does not take into account that demand for one product
may influence the demand for the other product.
• Only one goal can be satisfied: The objective function can only be expressed in a format to satisfy
one goal in the construction of the model (e.g. to maximise profit). As mentioned, a company may
have other goals or strategies (e.g. growth in market share, quality of service, cost minimisation),
which cannot be satisfied simultaneously using this model.

Therefore, linear programming should only be used as a technique for assisting management in their
decision-making process concerning the mix of products or services to produce.

EXAMPLE 20.3
A company manufactures liquid hand soap (P) and bars of soap (Q). The same raw material
(glycerine) is used in both products, but the supply of glycerine is limited. The same type of
machine is used to mix the ingredients, but different quantities of ingredients and different mixing
times are needed for the two products. The machine hours are limited to a certain number of hours
per month, as there is only one machine.
P and Q both require processing in the machining department. The machining department has
eight machines with a daily capacity each of 15 hours. In one hour, four units of product P or three
units of product Q can be processed.
Supply of glycerine is limited to 18 kg per day. Product P requires 400 g per unit; product Q
requires 500 g per unit.
The contribution per unit for product P is R4,00; for product Q it is R5,00.

Required
Determine the optimum combination of products P and Q to be produced daily in order to
maximise contribution.

Solution
• Step 1. Objective function is to maximise profit.
Profit = 4P + 5Q
• Step 2. Constraints.
Machining: 4P + 3Q = 120
Material: 0,4P + 0,5Q = 18
• Step 3. Compiling the graph.
Machining: if P = 0, Q = 40; If Q = 0, P = 30
Material: if P = 0, Q = 36; if Q = 0, P = 45
Demand for Q = 40
Use the points for P and Q on the x and y axes to draw the constraint lines.
• Step 4. Draw the objective function and move the line parallel up until it touches the furthest
point of the enclosed area.
4P + 5Q: use multiples, e.g. P8 + 10Q or 20P + 25Q

Although multiple objective lines are illustrated in the graph (Figure 20.2), it is only done for
explanation purposes and only one objective line would have sufficed.

Figure 20.2 Linear programming graph (products P and Q)


Move the objective function (profit line) parallel until it touches the furthest point of the area
enclosed by points A, B, C and D. That will be point B.
• Step 5. Use point B to read from the x axis the number of units of product P, and from the y
axis the number of units of product Q to make up the optimum combination to produce in order
to maximise profit. Graphically, the solution is 8 units of product P and 30 units of product Q.
Also use the mathematical method to obtain the accurate number of products and determine the
contribution that will be made daily in using this product mix:
Point B is the point where the machining constraint line and the material constraint line
intersect:
4P + 3Q = 120 (1)
0,4P + 0,5Q = 18 or 4P + 5Q = 180 (2)
To solve P and Q, deduct equation (1) from (2), then 2Q = 60 and therefore Q = 30.
Replace Q = 30 in equation (1): 4P + 3(30) = 120 then P = 7,5.
The optimum combination to maximise profit, is therefore 7,5 units of P and 30 units of Q

CLOSING PRACTICAL EXAMPLE


Returning to the example at the beginning of this chapter, it should now be clear that the company
manufacturing the liquid soap and bars of soap is faced with limiting factors – namely raw
materials and machine hours. Both products use the same raw material (glycerine), but the
glycerine is limited in supply and needs to be allocated between the two products in such a manner
that maximum profit can be made from the sale of these products. Machine hours are also limited
because there is only one machine that can be used to mix the ingredients. The products require
different quantities of the mixture and different mixing times, so the machine can only be used to
mix the ingredients of one type of product at a time.
Linear programming, a decision-making technique, can be used by management to determine the
optimum number of units of liquid soap and bars of soap to produce when faced with a situation
where the resources cannot meet the expected demand.
The starting point in using the linear-programming technique is to define the objective of
management as regards the sale of these products. If the objective of management in this case is to
maximise profit, the objective function will be expressed as a function of the contribution per unit
and the number of units to be sold. The second step is to write down the maximum availability or
capacity of the limiting factors (in this case, the raw material and the machine hours) as a function
of the requirements per product. The linear-programming model is then built step by step until the
graph is constructed and the optimum combination of soap products is determined.

20.6 Summary
This chapter has demonstrated the use of the linear-programming model and graph to determine the
number of units to produce of two different products in a situation where resources used for both
products are in scarce supply. The same model applies to making a choice between the number of
services to provide, in the case of two different services using the same scarce resources. Resources
such as labour, materials and machine hours may be in limited supply and constrain the number of
units that can be produced.
Linear programming also has its limitations and it is important to realise its use is limited to the
following situations:
• Where limiting factors are known
• Different products have different contributions per unit
• One goal can be satisfied (e.g. maximisation of profit)

PRACTICAL ACTIVITY
JP Clay recently started his own small business specialising in the production of fibreglass kitchen
sinks and bathroom wash basins. He attended a short course on entrepreneurship presented by the
local college and learnt about linear programming. The course presenter created a linear-
programming model for him but he can’t remember what it all means. He wrote you an email and
presented you with the model (see below). You are requested to reply to JP’s email and answer his
questions.

To: costaccountingstudent@univ.ac.za
Subject: Re: Linear programming model
Dear Cost Accounting Student,
Please can you explain the LP model from the email below?
• In equation 1, what is the meaning of 500 and 460?
• What do all the equations under point 2 mean?
• How do I use this model to determine the combination of sinks and basins to produce?
I would really appreciate your help.
With kind regards,
JP

To: jpclay@gomail.com
Subject: Linear Programming Model
Dear JP
As discussed during the short course, please find the LP model that you can use to determine the
number of units you can manufacture per day. A represents the kitchen sinks, and B the bathroom
basins:
• Profit = 500 A + 460 B
• Fibreglass (litres): 12 ℓ A + 9 ℓ B = 630 ℓ
Labour hours: 2H A + 1,5 H B = 96 H
A ≤ 50
B ≤ 65
You can use this information to construct the graph and then determine your optimum combination
of sinks and basins for production.
With kind regards,
Andrew

Short questions
1. Define the concept ‘limiting factor’.
2. What is meant by the term ‘optimum product composition’?
3. Explain which factors (other than limiting resources) may influence management’s decision
to produce a certain number of products.
4. Peter’s Paints employs four painters and they specialise in painting palisade fencing and tile
roofs. A standard fence (F) of 50 m requires two hours of labour per painter and a standard
roof (R) of 80 m2 requires three hours of labour per painter. On a normal day, the painters
have collectively 32 hours of labour available.
Which equation represents the limiting factor of labour:
a. 50F + 80R = 3 200
b. 2F + 3R = 32
c. 20F + 60R = 128
d. 2F + 3R = 8

Long questions
1. Portia Ndau produces tablecloths and aprons, hand-embroidered with flowers. She sells
them at a local market and has a fixed demand from a local guest house for three aprons and
one tablecloth each week.
An apron uses 1,5 m of material and a tablecloth 1,2 m. Portia limits her purchases of
material to 20 m per week. Portia and three employees work five days a week for eight hours
per day, and can either produce four aprons or two tablecloths per day. The contribution per
unit for an apron is R120 and for a tablecloth R100.
Use linear programming to construct a model for determining the optimum combination of
aprons and tablecloths in order to maximise profits. Use a graph to determine the combination
and verify it with a mathematical calculation.

2. A company manufactures bicycle chains. One type is made from stainless steel and a more
expensive type is made from an aluminium alloy. Both types are manufactured using the same
machinery and the total daily capacity of the machines is 72 hours. It takes 45 minutes to
produce one stainless-steel chain and one hour for an aluminium chain. The raw material
used in the stainless-steel chain is unlimited in supply, but the aluminium is limited to a daily
supply of 20 kg. One aluminium chain uses 400 g of aluminium. A fixed order demands that a
minimum of 10 stainless-steel chains must be produced daily.
The contribution per unit is R20 per steel chain and R30 per aluminium chain.
Construct a linear-programming model and draw the graph to determine the optimum
combination of the different chains in order to maximise profit.

3. A company manufacturing steel garden chairs and tables has two departments in the
manufacturing process – the cutting department and the welding department. Four chairs or
10 tables can be processed in the cutting department each hour. In the welding department,
eight chairs or five tables can be processed each hour. Both departments have a capacity of
eight machine hours per day.
The supply of wooden seats used in the chairs is limited to 30 seats per day. Chairs are
sold at R58.00 each and tables at R184.00 each.
The total variable cost per unit is as follows:
Chairs R22.00
Tables R52.00
Use linear programming to construct a model for determining the optimum combination of
chairs and tables in order to maximise profits. Use a graph to determine the combination and
verify it with a mathematical calculation.

4. A toy company produces remote-controlled racing cars and remote-controlled speedboats.


Management’s strategic planning has indicated that the company must aim to sell a minimum
of 100 racing cars and 60 speedboats per week. The battery-operated engines used in both
models are limited in supply, and the supplier can only deliver 250 batteries per week.
The contribution per unit for the racing cars is R40, and for the speedboats R50.
Construct a linear-programming model and graph to determine the optimum combination
of cars and boats in order to maximise profits.

5. Bond Ltd produces two types of leather cricket balls, A and B. The raw material costs R25
per kilogram and labour is remunerated at R20 per hour.
The costs per unit are as follows:
Type A Type B
Selling price R120 R85
Direct material R25 R40
Direct labour R55 R20
Only 220 kg of the raw material and 360 labour hours are available per production run.
Construct a linear-programming model and graph to determine the optimum combination
of ball A and ball B in order to maximise profits. Remember to first determine the
contribution of each product.

6. A company produces two products, K and L, each manufactured from the same raw material
and they undergo a machining process using the same type of machines. The resources are in
short supply and the company needs to determine the best combination of unit numbers in
order to maximise profit.
The raw material is limited to 5 240 kg per production period. Raw material is purchased
at R15 per kg. To produce one unit of product K, 4 kg of raw material is needed; product L
requires 2 kg. Machine hours are limited to 480 hours per production period. In one hour,
four units of product K or two units of L can be manufactured.
A company decision states that a maximum of 1 000 units of each product may be
manufactured in any production period.
Other relevant information:
Product K Product L
Selling price R240 R190
Variable cost:
Material R? R?
Direct labour R45 R40
Manufacturing overheads R25 R55
a. Determine the contribution per unit.
b. Construct a linear-programming model to take all the constraints into account.
c. Use a graph to determine the optimum combination of product K and L to be produced to
maximise contribution.
d. Test your answer by solving the equations mathematically.
21 Relevant costing
Joset Jordaan-Marais

LEARNING OUTCOMES
After studying this chapter, you should be able to:
• distinguish between financial and non-financial factors in terms of decision making;
• identify non-financial factors that should be considered when decisions are made;
• distinguish between relevant and irrelevant incomes and costs in decision making;
• prepare an analysis showing whether a special order should be accepted or rejected;
• prepare an analysis showing whether a task should be outsourced;
• prepare an analysis showing whether to keep or replace old equipment;
• determine the most profitable use of a limited resource; and
• prepare an analysis showing whether a product should be discontinued.

PRACTICAL EXAMPLE
We all need to make decisions on a daily basis. To be able to make these decisions, we need
information. Think about some of the decisions that you have made recently. What were the things
that you took into account when you decided to further your studies? Some of the factors you
considered were likely to have been financial in nature, whereas others would have been non-
financial. Take a moment to complete Table 21.1, which lists the factors that you considered
before starting your studies.

Table 21.1 Financial and non-financial factors


Financial factors Non-financial factors
How much money will I need to pay for my
Do I want to obtain a tertiary qualification?
studies and monthly expenses?
How will I pay for my studies? What are my interests?
How will I pay for my monthly expenses? Where do I see myself in five years?
Will I need to get a part-time job to have

spending money?
Should I rather work full time and study part

time?
Just as you need information to help you make key decisions, managers of organisations also need
information of a financial and non-financial nature to make informed choices for the benefit of their
organisations.

21.1 Introduction
This chapter examines how to analyse certain relevant factors that should be considered when
decisions need to be made by businesses in terms of their costs and incomes. Although emphasis is
placed on the financial factors that must be considered, it is important to remember that certain non-
financial factors may also influence a final decision. For example, when a business decides to
outsource one of its functions to an external supplier, the factors to be considered are not purely
financial. The business will also need to consider how reliable the external supplier is in terms of
delivering the goods and services in the correct quantities, at the correct time and of the correct
quality. As discussed in Chapter 1, a key function of management accounting is to provide the
financial information on which managers can base their decisions. Firstly, it is important to
understand what is meant by ‘relevant financial factors’.

21.2 Relevant financial factors


When making business decisions, only relevant costs and incomes should be considered. Irrelevant
costs and incomes should be ignored. To distinguish between relevant and irrelevant costs and
incomes, only one question needs to be answered: will the incurrence of the cost or income be
influenced by the current decision? If the answer is yes, it means that the cost or income is relevant to
the decision that needs to be made and should be considered. If the answer is no, it means that the cost
or income is irrelevant to the decision and should not be considered.
A relevant cost or income is always a future cost or income. The relevant cost or income is caused
as a result of the decision and will differ for the alternatives under consideration. A cost or income
that is not changed by a decision and which stays the same no matter which alternative is chosen or
which decision is made will be irrelevant to that decision. It will, in other words, not be considered
when the decision is made.
For example, if you have to decide between going to the movies on Friday night or staying at home
to watch television, the ticket price of the movie will be a relevant cost. This is relevant, as this cost
will only be incurred on Friday night (in the future) if you decide to go to the movies. You will incur
this cost because you have decided to go to the movies. It is caused by your decision. It will also
differ for the alternatives under consideration. The two alternatives are to either go to the movies or
to stay at home and watch television. If you decide to go to the movies, you will have to pay for the
ticket; if you decide to stay at home, you will save this cost. The rent that you pay each month, for
example, will be irrelevant to your decision. The decision as to whether to go to the movies or to stay
at home will not influence this cost. It will stay the same regardless of which alternative is chosen.
Costs that have already been incurred in the past and which stay the same no matter what decision
is made are called sunk costs. Sunk costs are always irrelevant when decisions are made. Let’s
continue with our previous example. Assume that you bought a car last year and that you will use this
car to drive to the movies on Friday night. The amount of money that you paid for the car will be
irrelevant to your current decision as to whether to go to the movies or not. The amount paid for the
car was paid in the past and it will not change no matter whether you use the car on Friday night to go
to the movies or not.
Figure 21.1 illustrates the thought process to follow when you need to distinguish between
relevant and irrelevant costs. It is important to be able to make this distinction correctly.
Once you understand the difference between relevant and irrelevant costs, you will be able to
make informed decisions as a manager and also in your personal life.
Figure 21.1 Flow chart to distinguish between relevant and irrelevant costs

21.3 Making financial and non-financial decisions


As mentioned, managers need to make a multitude of decisions to steer their organisations in the right
direction. Examples of financial decisions include whether a special order should be accepted or not;
whether a product or component should be produced internally or bought externally; and whether a
product line should be continued or discontinued. We will explore how some of these decisions
should be made in the form of examples. We will help Kai, who is the newly appointed manager of a
well-known local music group, Mzansi Groove, to make some important financial decisions in her
new business.
Kai has had to learn about the music industry, and especially how music albums are recorded,
manufactured and promoted to be able to act as manager of the group. She has learnt that music
albums are recorded in studios with the help of a sound engineer. After recording, the album is mixed
and then mastered by a master engineer. The album is then ready to be pressed. This is the
manufacturing phase, during which the album is duplicated and packaged. Artwork needs to be
completed beforehand and is also reproduced and included in the album packaging at this stage. After
the manufacturing phase, the album is ready for distribution. It will then be promoted by means of
advertising (e.g. radio, print or web campaigns) and other forms of publicity.
Kai has compiled a cost calculation for the recording, pressing and promotion of Mzansi Groove’s
new album, The sun never sets (see Table 21.2).

Table 21.2 Cost calculation for The sun never sets


Recording costs R
Studio time 80 hours at R800 per hour 64 000
Sound engineer 80 hours at R200 per hour 16 000
Mixing and mastering R1 000 per song, 12 songs 12 000
Manufacturing costs
Artwork design 3 000
Duplication R2 per copy, 1 000 copies 2 000
Packaging, including printing of
R5 per CD, 1 000 CDs 5 000
album cover and CD case
Promotion costs
Launch party 3 000
Advertising 4 000
Radio campaign 7 000
Publicity 2 000
118 000

EXAMPLE 21.1: SPECIAL ORDERS


Let’s assume that Mzansi Groove recorded their album and that the cost calculation that Kai
prepared was accurate. Rich Brandon, a well-known international businessman, approached Kai
with the following proposal via email.

From: Rich Brandon


To: Kai
Subject: Order

Afternoon, Kai,
I have recently seen Mzansi Groove perform and have become a great fan of their music. I have
decided to buy a copy of the group’s album, The sun never sets, for each of my three hundred
employees as a thank-you gift for their hard work during the year. Due to budget constraints, I
will only be able to pay R30 a copy. It would also be appreciated if you could change the cover
design to include my company logo and a personal message from myself to my employees.
Could you please assist me?
Kind regards,
Rich

Kai was very excited when she received the email, as this order would give Mzansi Groove great
international exposure. However, she did not know what factors should inform her decision to
accept the order or not. Would accepting this special order be to the advantage of the group when
financial factors are taken into account? The group normally sell their albums for R150 each at
their performances and there were only 300 of the original 1 000 copies manufactured left. Kai
did not want to sell these albums to Rich, as the group needed stock to sell to their local fans
during their upcoming national tour. This would mean that a new batch of albums would need to
be manufactured specially for this order.
To help Kai decide whether to accept the order or not, first we need to identify the relevant
costs. Look at the costs listed in Table 21.2. Which of these costs are relevant to the decision in
question? In other words, which of these costs are future costs that will change if the decision is
made to accept the special order?

Table 21.3 Identifying relevant costs


Relevant?
Recording costs
Studio time
Sound engineer No
No
Mixing and mastering No
Manufacturing costs
Original artwork design No
Duplication Yes
Packaging, including printing of album cover
Yes
and CD case
Promotion costs
Launch party No
Advertising No
Radio campaign No
Publicity No
The recording costs are irrelevant, as these costs will not be changed by accepting the order. No
additional recording costs will be incurred by accepting the special order. They are sunk costs,
as they have been incurred in the past.
The same applies for the design cost of the original artwork. This cost has been paid in the
past and is therefore also a sunk cost and irrelevant to the current decision. Kai should, however,
take into account the cost of the new artwork that will be needed for this order when she makes
her decision, as this is an additional cost that will be incurred if she decides to accept the special
order. It is a future cost, as it will only be incurred if the special order is accepted, and is thus
relevant to the decision. Kai has obtained a quotation for the new artwork and has established that
it will cost R1 800. This sum needs to be included when calculating the total relevant cost and
should therefore inform the decision-making process.
Accepting the special order would not result in any additional promotion cost. The promotion
costs included in Kai’s calculation in Table 21.2 are past costs and are therefore also classified
as sunk costs, and hence are irrelevant to the current decision as to whether to accept the order or
not. These promotion costs will not change as a result of the current decision.
As for the relevant costs, the order requires 300 copies of the album to be manufactured if it is
accepted. Therefore, 300 copies will need to be duplicated and packaged at a cost of R7 (R2 for
duplication plus R5 for packaging) per album. This is a future cost, as it will only be incurred
when the special order is accepted. This cost will result directly from the decision to accept the
special order and is therefore a relevant cost that should be considered.
Now that we have identified the relevant costs associated with accepting this order, we can
identify what the relevant income will be. Rich is prepared to pay R30 per album and wants to
order 300 copies. The relevant income is therefore R9 000. It is classified as relevant because it
will be received in the future if the special order is accepted, and it results directly from the
decision to accept the special order.
Let us help Kai to prepare an analysis to enable her to make an informed decision (see Table
21.4).

Table 21.4 Analysis of relevant income and costs


R
Relevant income
Album sales (300 copies @ R30 a copy) 9 000
Less relevant costs
Duplication (300 copies @ R2 per copy) 600
Packaging (300 copies @ R5 per copy) 1 500
New artwork design 1 800
Net advantage 5 100
Accepting the special order will generate an additional income of R5 100. Financially, it would
be to Mzansi Groove’s advantage to supply the 300 copies of their album to Rich Brandon.
Accepting the special order will also have non-financial benefits, as it will give the group
international exposure. Based on these financial and non-financial factors, it is clear that Kai
should accept the special order.

EXAMPLE 21.2: OUTSOURCING


Kai is in the process of obtaining quotations for new Mzansi Groove T-shirts to sell to fans. In the
past, the group have always made their own T-shirts. They normally buy plain black T-shirts and
print the band’s emblem on each shirt using stencils and fabric ink. This process is labour-
intensive, and they are not sure whether they will have time to make a new batch of T-shirts
before their upcoming tour, as they are currently busy with activities to promote the tour.
The group members have mentioned their concerns to Kai, who then began sourcing external
suppliers to manufacture the T-shirts. She agrees with the group that outsourcing would be a more
feasible alternative to manufacturing the T-shirts themselves, but is not sure whether this decision
would make financial sense. She is also concerned that fans will not be satisfied with the mass-
produced T-shirts, as the personal touch added to each T-shirt handmade by the group members
has always been an important selling point in the past.
Kai has obtained the following quotation from The T-Shirt Guys, an external supplier that will
be able to manufacture the T-shirts:
Quotation
The T-Shirt Guys
To: Kai
Unit price Total
Description Quantity
R R
Preparation of artwork 1 700
Plain black T-shirts 100 30 3 000
Printing on T-shirts 100 10 1 000
Delivery 300
Grand total 6 000
The group leader, Cheyne, wrote Kai a note in which he prepared a cost calculation to show Kai
the total cost of manufacturing the T-shirts themselves:

Dear Kai,
These are our costs if we manufacture the shirts ourselves:
Plain black T-shirts (100 shirts @ R20 per shirt): R2 000
Fabric ink (5 colours × 1 litre each @ R100 per litre): R500
Paintbrushes: R250
Stencils: R1 500
We still have two 1-litre containers of ink left from the last time we made a batch of T-shirts. We
can use this if we are going to make the T-shirts ourselves. We will, however, need new
paintbrushes. We had the stencils made a few years ago and do not need new ones for the new T-
shirts.
We are worried that we will not be able to manufacture the T-shirts ourselves before we
leave on our tour next month, as it takes about 30 minutes to print each T-shirt. This means that
we will have to spend a total of 50 hours on printing the T-shirts. If all five of us work on the T-
shirts, this would take up 10 hours each. We simply do not have this time, as our diary is fully
booked. We would have to give up at least one performance in order to make time for printing the
shirts. This would of course mean that we would lose the money that we could have earned
performing, which normally amounts to at least R3 000.
Could you please look into this matter urgently?
Thanks,
Cheyne

Firstly, the decision that Kai must make is whether it would be better if the group manufactures
the T-shirts themselves or if it will be better to outsource this task to an external supplier. There
are financial and non-financial factors that should be considered, and only relevant factors should
be taken into account.
First, we should compare the relevant costs of the two options:
– Option 1: The group manufacture the T-shirts themselves.
– Option 2: The task is outsourced to The T-Shirt Guys.

Remember that when you want to identify the relevant costs, you need to ask yourself: will the
cost be influenced by the current decision? Relevant costs are future costs that will differ for the
alternatives under consideration.

Table 21.5 Relevant costs for option 1


R
Plain black T-shirts (100 shirts @ R20 per
2 000
shirt)
Fabric ink (3 litres @ R100 per litre) 300
Paintbrushes 250
2 550
Opportunity cost of performance missed 3 000
Relevant cost 5 550
A hundred plain black T-shirts, three litres of ink and paintbrushes would need to be purchased if
the group manufacture the T-shirts themselves. These are future costs, as they will only be
incurred when the group manufacture the T-shirts. These costs will also result from the decision
to manufacture the T-shirts internally. Therefore, these costs are relevant and should inform the
decision to outsource or not.
The income lost from the missed performance should also be included because this loss
results from the decision to manufacture the T-shirts internally. This is an opportunity cost,
which is always a relevant cost. This type of cost represents the opportunity that is lost when one
investment opportunity is chosen, as opposed to another. By choosing to manufacture the T-shirts
themselves, the group forgo the income that they could have earned from the performance.

Table 21.6 Relevant costs for Option 2


R
Preparation of artwork 1 700
Plain black T-shirts 3 000
Printing on T-shirts 1 000
Delivery 300
Relevant cost 6 000
All of the costs included in the external supplier’s quotation are relevant, as they are all future
costs. These costs will only be incurred if the manufacture of the T-shirts is outsourced. These
costs will also result from the decision to outsource the manufacture of the T-shirts to the external
supplier.
It will cost less if the group manufacture the T-shirts. This would therefore be the
recommended option to choose when considering only financial factors. Kai should, however,
also weigh up non-financial factors when making her decision. For example, how would the
group react to the decision? It might be bad for the group’s morale if they are compelled to spend
their time printing T-shirts instead of performing. They are musicians, after all, and Kai should
discuss the matter with them before she makes her final decision.

EXAMPLE 21.3: KEEPING OR REPLACING OLD EQUIPMENT


Mzansi Groove has been using an old minibus for the last five years to travel to and from
performances. The vehicle started to have mechanical problems. Kai has to decide whether the
vehicle should be repaired and used for another five years or whether the group should purchase
a new minibus. A car dealer has offered to be the group’s sponsor and is prepared to give the
group a 50 per cent discount on one of its new minibuses in exchange for marketing opportunities
with the group.
The information shown in Table 21.7 about the vehicles was given to Kai by the group’s
accountant.
Table 21.7 Vehicle costs
Old minibus New minibus
R R
Price after 50%
Original cost 164 500 148 450
discount
Remaining book value 65 800 Expected usage 5 years
Disposal value in five
Remaining life 5 years 124 500
years
Disposal value now 55 000 Annual running cost 6 000
Disposal value in five
0
years
Annual running cost 12 000
Cost to repair 18 000
To make the decision about whether or not to replace the old vehicle, Kai needs to compare the
relevant costs and incomes for the two options:
– Option 1: Keep the old minibus for a further five years.
– Option 2: Sell the old minibus, buy the new minibus and use it for the next five years.

The relevant costs and income to consider are those future costs and incomes that differ between
the two options. Would it be to the group’s advantage or disadvantage to buy the new minibus?
What will be the financial effect if the new minibus is bought?

Table 21.8 Analysis showing the effect of buying the new vehicle (discounted price)
R
Disposal value of old minibus 55 000
Disposal value of new minibus in five years 124 500
Saving on running cost (R6 000 per year* × 5
30 000
years)
Saving of repair cost 18 000
Less
Cost of new minibus 148 450
Net advantage 79 050
*R12 000 – R6 000

Note that the original cost of the old minibus is a sunk cost, as this cost was incurred in the past.
The current decision will not change the price originally paid for the old minibus. This cost is
therefore irrelevant to the current decision. The depreciation written off each year (shown in the
book value of the old minibus) represents the cost of the minibus distributed across its lifetime
and is also irrelevant to the current decision.
Remember to ask yourself whether incurring the cost or receiving the income will be
influenced by the current decision when determining whether a cost or income is relevant to a
decision. Which costs and incomes (or savings in this example) will result from the decision to
buy the new minibus?
Buying the new minibus will result in the sale of the old minibus. The disposal value of the
old minibus is therefore relevant to the decision, as this cash inflow results directly from the
decision to buy the new minibus. The disposal value of the new minibus is also relevant, as this
cash inflow in five years’ time will also result from choosing to buy the new minibus today.
Buying the new minibus leads to certain savings. The new minibus has a lower running cost
than the old minibus. The old minibus will also not need to be repaired if Kai decides to buy the
new minibus. These savings are therefore relevant to the decision, as they result from choosing to
buy the new minibus.
From this analysis, it is clear that buying the new minibus makes financial sense, as it will
result in a net advantage of R79 050. Kai should, once again, however, consider the nonfinancial
factors involved in this decision. To be able to buy the new minibus at the discounted price, the
group will need to be involved in the car dealer’s marketing campaign. Is this an organisation
with whom the group want to be associated? Will they have time to fulfil the marketing
commitments?
How would not taking the discount from the car dealer influence the decision? If the discount
were not available, it would be better financially to keep the old minibus (see Table 21.9).

Table 21.9 Analysis showing the effect of buying the new minibus at full price
R
Disposal value of old minibus 55 000
Disposal value of new minibus in five years 124 500
Saving in running cost (R6 000 per year × 5
30 000
years)
Saving of repair cost 18 000
Less
Cost of new minibus (full price) 296 900
Net disadvantage (69 900)

EXAMPLE 21.4: LIMITED RESOURCES


Kai is busy preparing the group’s work programme for the coming year. Since the group’s
popularity has increased over the last couple of months, they are in great demand. Kai has
received various requests for appearances and performances by the group. She cannot accept all
of these requests, as the group only has a limited amount of time available. Kai is not sure how to
decide which requests to accept and which to reject.
For Youth Day, 16 June, alone, she has received three requests. After taking travel and
preparation time into consideration, it is estimated that the group will only have six productive
hours available on this day. The following requests were received:
– Request 1. The Gauteng Children’s Hospital has asked the group to visit on the day to help
raise funds for the renovations needed by the hospital. The group will be required to visit for
at least one hour. Even though they will not be paid for the appearance, Kai estimates that they
will get free publicity worth R1 850 from it. The group will need to provide their own
transport to and from the hospital at a cost of R300. They will also donate R1 000 towards the
cause.
– Request 2. A local radio station is hosting a full-day concert on the day and has asked the
group to perform for two hours. The fee will be R5 000. The group will need to hire special
equipment for the performance at R1 800 and will need to buy special outfits for the occasion
costing R1 500. Transport costs to and from this event will amount to R90.
– Request 3. The group has been offered the opportunity to appear on a popular television game
show as contestants. The recording of the show will be done on the day and will take four
hours. The group will be paid R3 000 for the appearance. A make-up artist will be hired at a
rate of R450 to prepare the group for the appearance. The transport cost to and from the studio
will be R110.

When a resource is limited (known as a constraint), there is less of the resource available than
what is needed. When dealing with a limited resource, the best financial option is to maximise
profit from the resources available. The group’s time, in this case, is the limited resource, as they
do not have enough time available for all the appearances and performances that have been
requested. This time constraint forces them to accept some offers and reject others.
It should be the group’s goal to earn as much money as possible in the limited time they have
available. The question is, how much money can they earn per hour for each of the three
opportunities above? The earnings per hour should be calculated, as the number of hours
available is the constraint.
– Step 1: Calculate the contribution (income less all variable expenses) that can be earned for
each request.

Table 21.10 Contribution per request


Request 1 Request 2 Request 3
(R) (R) (R)
Income 1 850 5 000 3 000
Less
Transport 300 90 110
Donation 1 000
Equipment 1 800
Outfits 1 500
Make-up artist 450
Contribution 550 1 610 2 440
Looking at these calculations, it would first appear that request 3 is the best option, as it gives the
highest total contribution. But remember, these requests do not take up the same amount of time.
Request 1 requires one hour of the group’s limited resource (time), request 2 requires two hours
and request 3 requires four hours. We need to take this into consideration when deciding how to
use the limited amount of hours available.
– Step 2: Calculate the contribution per unit of the limiting factor – in other words, the
contribution that can be earned in an hour by each of the requests.
– Step 3: Rank the requests according to the contribution per hour, which has been calculated in
the previous step. The request that generates the highest contribution per hour will be ranked
in the number 1 position, as it is the most favourable option.

Table 21.11 Contribution per hour and ranking of requests


Request 1 Request 2 Request 3
(R) (R) (R)
Contribution 550 1 610 2 430
Hours needed 1 2 4
550 ÷ 1 1 610 ÷ 2 2 440 ÷ 4
Contribution per hour
550 805 610
Ranking 3 1 2
It is now evident that request 2 is the better use of resources, as it gives the highest contribution
per hour. Since the number of hours available is the limitation, it makes sense to try to earn as
much as possible in the six hours available.
Kai needs first to accept request 2, as it is ranked number 1. Afterwards she can accept the
other requests, depending on the number of hours still available. After accepting request 2, there
are still four of the six hours left. Request 3 (ranked in the number 2 position) can therefore also
be accepted. After accepting request 3, there will be no time left. Request 1 will therefore have to
be rejected.
By accepting requests 2 and 3, and rejecting request 1, Kai will maximise the income that the
group can earn in the limited hours they have available.

EXAMPLE 21.5: CONTINUING OR DISCONTINUING PRODUCTS


Kai would like advice regarding discontinuing one of Mzansi Groove’s products, which is still in
production. The group recorded a DVD a few years ago and have been selling copies through
retail outlets ever since. It has recently come to Kai’s attention that the sales of the DVD have
dropped significantly.
The accountant has prepared an income statement for the most recent financial year for all of
Mzansi Groove’s products (see Table 21.12).

Table 21.12 Income statement for Mzansi Groove’s products


All products DVD
(R) (R)
Sales 130 000 1 800
Less cost of sales
Direct material 6 000 120
Direct labour (variable) 30 000 600
Variable manufacturing
2 800 56
overhead
Building rent 5 000 110
Depreciation: Factory
1 400 28
equipment
Gross profit 84 800 886
Delivery expenses 560 80
Depreciation: Office equipment 330 140
Salaries: General manager 60 000 1 200
Marketing 10 000 1 000
Net profit/(loss) 13 910 (1 534)
All of the products are manufactured in the same building and use the same equipment. Building
rent, depreciation and the general manager’s salary are allocated to products using various bases.
Delivery and marketing expenses can be traced directly to specific products. Discontinuing the
DVD will have no effect on the sales of the other products.
Kai needs to decide whether it is worth continuing to manufacture and sell the DVD, as it is
currently making a loss. Again, only the relevant income and costs should be considered when
making the decision. The relevant income and costs will be those that are influenced by the
decision. Kai needs to determine which incomes and costs will change due to the decision. Which
of the costs are avoidable? In other words, which of the costs can be saved if the decision is
made to discontinue the DVD?

Table 21.13 Avoidable costs


Avoidable Not avoidable
(R) (R)
Direct material 120
Direct labour (variable) 600
Variable manufacturing
56
overheads
Building rent 110
Depreciation: Factory
28
equipment
Delivery expenses 80
Depreciation: Office equipment 140
Salaries: General manager 1 200
Marketing 1 000
Total 1 856 1 478
The avoidable costs will no longer be incurred if the decision is made to discontinue the
production of the DVD. The income from the DVD sales will also be lost if this decision is made.
The possible savings should now be compared with the forgone income before the decision can
be made.

Table 21.14 Analysis showing whether the DVD should be discontinued


R
Avoidable costs that can be saved 1 856
Income that will be lost (1 800)
Net advantage 56
The cost savings from discontinuing the DVD are greater than the income that will be lost. It is
therefore financially sound to end the production and sales of the DVD.

CLOSING PRACTICAL EXAMPLE


At the beginning of this chapter, we discussed the importance of decision making in both your
private and professional lives. In Table 21.1, you made a list of all of the things you took into
account when you decided to further your studies.
In this chapter, we spoke about the factors that should be considered when making decisions,
including financial and non-financial factors. Look back at your list. Did you consider both of these
before making your decision? We also discussed the difference between relevant and irrelevant
costs and incomes. Your list should only contain factors that were relevant to your decision – in
other words, factors that you needed to consider when making your decision. Would you classify
all of the factors on your list as relevant to your decision? This example has showed how the
consideration of relevant financial and non-financial factors can help you make better-informed
decisions. These principles can be applied to any decision that you may face.

21.4 Summary
This chapter analysed how financial decisions should be made by helping, Kai, the manager of a
well-known local music group, Mzansi Groove, to make some important business decisions.
When making such decisions, it is important to distinguish between relevant and irrelevant
financial factors, and to consider only those that are relevant to the decision.
To determine whether an income or cost is relevant to a decision, one should keep in mind that a
relevant cost or income:
• will be influenced by the current decision;
• is a future cost or income; and
• will differ for the alternatives under consideration.

Opportunity costs are always relevant costs. This type of cost represents the opportunity that is lost
when one investment opportunity is chosen, as opposed to another. Sunk costs are always irrelevant
costs, as they represent past costs that will not be influenced by the current decision.
Relevant non-financial factors should also be considered when such financial decisions are made.
PRACTICAL ACTIVITY
You have just been appointed as the management accountant for a small manufacturing firm. The
firm needs to determine whether or not to accept a special order. You need to put together all the
relevant data to enable management to make a decision. The production manager that needs to
supply some of the information has asked you to help him understand the information you require.
Write a memorandum to him explaining how to decide whether a cost is relevant.

Short questions
1. Frozen Treats (Pty) Ltd has received a special order to manufacture 10 litres of homemade
ice cream. The following resources are required to make 1 litre of ice cream:
Milk 2 litres at R10 per litre
Sugar 2 kg at R5 per kg
Direct labour 30 minutes at R30 per hour
Fixed manufacturing overheads R20 per direct labour hour
Given this information, what is the relevant cost for the special order?
a. R55
b. R550
c. R450
d. R80

2. Which qualitative (non-financial) factor should be considered when when you decide to
make a product yourself or outsource it:
a. The quality of the external supplier’s production.
b. Whether the external supplier can supply the goods when needed.
c. Whether the external supplier can supply the goods in the quantities needed.
d. All of the above.

3. Sunk costs are ___


a. historical costs.
b. always relevant when decisions are made.
c. contribution lost due to choosing one alternative instead of another.
d. future costs.

Long questions
1. ZZZ Ltd will lose revenue from their regular customers if they accept a special order while
operating at capacity.
Identify the type of cost of which this is an example, and discuss whether this cost will be
relevant to the decision on whether to accept the special order.
2. Three Cheeses Pizza bought their current pizza oven two years ago for R10 500. It has one
operating year remaining and a book value of R3 500. The owner could purchase a new oven
for R2 200, but it would only last one year. The new oven will result in a saving of R3 000
annually compared to operating with the old one.
Prepare an analysis to show whether the new pizza oven should be purchased or not.

3. Rolypoly (Pty) Ltd manufactures 20 000 rolls of paper each period. The paper is used as an
input for producing several other products that the company manufactures. The full
manufacturing costs for a batch of 100 rolls of paper are as follows:
Direct materials R540
Direct labour R400
Variable manufacturing overheads R400
Fixed manufacturing overheads R700
The fixed manufacturing overheads include depreciation expenses related to previous
investments in facilities and equipment that are used in the manufacturing of the paper. These
assets have no other use than for manufacturing the paper.
An outside supplier has offered to sell the company the 20 000 rolls of paper necessary to
meet production needs this period for a lump sum of R150 000.
Advise management, based on financial factors, on whether the paper should be bought
from the outside supplier or manufactured internally.

4. Yum Yum, a popular restaurant chain, needs to determine whether it would be cheaper to
bake its own bread rolls or to purchase bread rolls from a nearby bakery. Yum Yum requires
2 000 bread rolls per month. The bakery will charge R0,50 per bread roll.
Cost information on internal production of 2 000 bread rolls is as follows:
Direct materials R0,20 per bread roll
Direct labour R0,10 per bread roll
Manufacturing overheads:
Variable R0,05 per bread roll
Fixed R3 000
Twenty per cent of the fixed overheads can be avoided if the bread rolls are purchased
externally.
Discuss which alternative would be more cost-effective and by how much.

5. Doodlebop (Pty) Ltd manufactures four products using the same machinery. The following
details relate to the products:
Product A Product B Product C Product D
(R per unit) (R per unit) (R per unit) (R per unit)
Selling price 28 30 45 42
Direct material 5 6 8 6
Direct labour 4 4 8 8
Variable 3 3 6 6
overheads
Fixed overheads 8 8 16 16
Profit 8 9 7 6
Labour hours (per
1 1 2 2
unit)
Machine hours
4 3 4 5
(per unit)
Maximum demand
200 180 250 100
per week (units)
There is a maximum of 2 000 machine hours available per week.
Determine the product mix that will maximise the weekly profit of the company and do a
calculation showing the total contribution your mix will yield.

6. Sparkling Jewellers is considering a special order for 20 handcrafted gold bracelets to be


given as gifts to members of a wedding party. The normal selling price of a gold bracelet is
R190 and its unit cost is made up of:
R
Direct material 85
Direct labour 45
20
Manufacturing overheads
(R4 is variable)
The customer who is interested in the special order would like a special stone to be added to
each bracelet. The stone will increase the cost of each bracelet by R2 and would require the
acquisition of a special tool costing R250 that would have no other use once the special
order is completed.
The customer is prepared to pay a maximum of R170 per bracelet.
Due to limited capacity, the company would not be able to produce five normal bracelets
if this special order is accepted. The customer has, however, indicated that he would be
interested in placing similar orders in future, at a higher price, if the company is willing to
supply this initial order.
Advise management on whether the order should be accepted or not.
22 Pricing decisions
Christo Hurter and Johan Hefer

LEARNING OUTCOMES
After studying this chapter, you should be able to:
• describe the impact of customers, competitors and costs on pricing decisions;
• determine the price of a product or service from a market-based approach, calculating:
– the target price; and
– the target cost, and consider ways to reduce costs to achieve the target cost; and
• calculate the cost price of a product or service based on the cost method, including:
– determining the appropriate cost factor to use; and
– determining the appropriate markup to achieve the required return on investment.

PRACTICAL EXAMPLE
Burger City is a fast-food outlet. The following is the expected unit cost based on normal capacity
for a hamburger:
R
Material 10
Direct labour 5
Variable overheads 3
Fixed overheads 2
Variable selling cost 2
Fixed administrative cost 1
Total 23
Burger City needs to determine the most appropriate selling price of its hamburgers. It is in a very
competitive industry with multiple competitors operating close to it. The manager is unsure how to
determine the selling price.

22.1 Introduction
One of the most important, but difficult, issues facing a business is to decide how much to charge for
its products or services. There is no single accurate way to determine a pricing strategy, but there are
some guidelines that may help in the decision making. The objective of this chapter is to consider
methods that can be used to help determine the selling price of an entity’s products or services.
As explained in the discussion of cost–volume–profit analysis in Chapter 19, pricing decisions
have a direct impact on profitability. If prices are too high, sales will be lost; too low, and sufficient
earnings on investment won’t be achieved. Various methods and strategies may be used to calculate
selling prices.
There are three major influences on pricing decisions: customers, competitors and costs. The
entity should determine what selling price its customers would be willing to pay for its products or
services, and how sensitive these customers may be to changes in the selling price. If a company sells
a luxury product like a tablet computer, which does not have competition, the main consideration in
setting the selling price would be to determine what price the prospective customers would be
willing to pay. The company should also consider not only the demand for its products or services,
but also the demand for its products or services at different price points.
The second major influence on pricing decisions is competitors. If a business is in a very
competitive market, it is important to assess its competitors and their pricing. In the market for certain
products, such as certain minerals (e.g. coal), the supplier has almost no control over the selling price
of its products, as prices are set by the open market. These types of entities are known as price takers.
Another important factor in today’s global economy is exchange rates. If a South African company
competes in European markets where the currency is the euro, a weakening rand would increase its
selling price in rand terms, while its costs will remain constant, therefore allowing for the company
to increase its profitability. The converse is also true. If the euro weakens against the rand, the selling
price in euros would decrease in rand terms and the profitability of the company may fall.
When analysing competitors, the positioning of the entity is also important. If price is a key part of
a company’s market position – in the case of a discount store, for example – then it is crucial to keep
prices as low as possible (lower than the competitors), while still making the required profit.
If the business sells exclusive luxury products, too low a price may damage the company’s image
and lead to a drop in sales. Certain customers really do believe the idea that you get what you pay for.
They believe that if you pay more, the quality is better. For example, when a prospective customer
weighs up two cameras, one of which is more expensive than the other, he or she may decide that the
more expensive camera must be better and may purchase the more expensive product.
It is also important to consider where a product is in its life cycle. If the product is brand new, a
higher price can often be asked, but prices are likely to decrease over time. This is especially true in
the case of high-technology products.
The final important factor in a pricing decision is costs. No matter which pricing method is used,
managers need to understand cost behaviour and cost drivers (see Chapter 2), as the entity needs to
ensure that its selling price is in excess of its cost price.

22.2 Short-term and long-term pricing decisions


One needs to differentiate between short- and long-term pricing decisions. Short-term pricing
decisions typically relate to special orders, customised products or special offers over a short
period. These decisions should be taken by following the approaches described in Chapter 21, and
will not be discussed in detail in this chapter. Bear in mind, however, that the pricing principles
described in this chapter may also be appropriate in the context of a short-term pricing decision.
When an entity determines the selling price of a product or a service for the purposes of normal
trading, it makes a long-term pricing decision. When this long-term decision needs to be made, the
entity would normally use one of the two following pricing strategies:
• Market-based pricing. A market-based strategy determines selling price by asking the question,
what price will our customers be prepared to pay? Once this price is set, costs are analysed and
reduced to the point where the targeted return on investment can be achieved.
• Cost-based pricing. A cost-based strategy determines the cost price of a product and then
calculates the selling price based on a predetermined markup percentage.

Note that many entities do not have the option of applying a cost-based approach. In a very
competitive market (especially for price takers, as discussed in the introduction), a cost-based
approach is impossible, as the most significant factor in determining the price of a product is the
current market price. Due to increasing competition in most markets in recent years, market-based
strategies have become more relevant and practical.
In the following sections, market-based and cost-based pricing methods are discussed.

22.3 Market-based pricing


With market-based pricing, first a target price is determined. The target price is set by establishing
the price that prospective customers would be willing to pay for the product or service. This will
depend on the customer’s perceived value of the product or service. With this pricing method, the
first two of the main pricing influences, namely customers and competitors, play a very important part
in determining the target price, for the following reasons:
• Competition from low-cost producers (especially in China, India and other developing countries)
is continually putting pressure on other producers to reduce prices.
• In recent years, products tend to be on the market for less time due to the increase in competition.
This gives an entity less opportunity to recover from pricing mistakes (which leads to loss of
market share and therefore less profitability).
• Due to increased choice and information available to them, customers have become more
knowledgeable and demanding when they make purchases. Therefore, substandard products, or
ones that are not competitively priced, will be rejected, which has a negative impact on
profitability, especially in competitive markets.

These factors should be analysed by understanding customers’ perceived value, conducting


competitor analysis and generally making sure that management understand the market that the
business operates in.
Once the targeted selling price has been established and based on the targeted operating income
(in other words, the target operating profit) a target cost price can be set. If the actual cost price is
less than the targeted cost price, the entity will be able to make a larger than expected profit or reduce
its selling price to be more competitive. Conversely, if the actual cost price is more than the targeted
cost price, the costs should be analysed to identify ways to bring them down to the targeted costs.
Using target pricing is a complex problem, as illustrated in Example 22.1.

EXAMPLE 22.1
The following is the expected unit cost based on normal capacity for a Burger City hamburger:
R
Material 10
Direct labour 5
Variable overheads 3

Fixed overheads 2
Variable selling cost 2
Fixed administrative cost 1
Total 23
After conducting market research, Burger City found that the target market price for its burgers is
R20. Assuming that a profit of 10 per cent on the selling price is required, the target cost will be
as follows:
Selling price R20
Required profit (10% on selling price) 2
Target cost R18
The projected total cost, however, is R23. It is therefore necessary to look more closely at each
cost element to see if it is possible to reduce the actual cost in order to achieve the target cost of
R18.
The company should now perform a cost analysis, which analyses the specific costs of the
product or service to identify ways in which they can be reduced.

It is appropriate to point out at this stage that costs should not be reduced on aspects that are critical
for customers. If, for example, the burgers are promoted as 100 per cent beef, then reducing the cost
by introducing soya into the patties would reduce customers’ perceived value of the product.
Therefore, the company should look at reducing only non-critical costs.
Ways of reducing the costs of these items might include changing suppliers to reduce the material
costs (without lowering quality); reducing the cost of direct labour by reducing the number of staff;
reducing variable overhead costs by acquiring more efficient manufacturing equipment; or reducing
the selling costs by reducing the quality of packaging material. These are examples of cost-cutting
actions that may allow their company to achieve its target cost. All these potential savings should take
into account their impact on the quality of the product, as reduction in demand will negate the cost
reduction.
One of the ways in which large corporations achieve these savings is through value engineering.
Value engineering is a systematic evaluation of all aspects of the value chain, with the objective of
reducing costs and achieving a quality level that satisfies customers.
There are many risks inherent in implementing cost-saving strategies, including dissatisfied
employees, reduced quality, which leads to dissatisfied customers and loss of market share caused by
reduced marketing budgets. Therefore, the process needs to be managed in an appropriate fashion by
identifying these risks and guarding against them.

EXAMPLE 22.2
Burger City’s target cost for its burgers is R18 but the projected cost is R23. The following
suggestions have been made:
– Reducing the quality of the ingredients used in the burger would achieve a saving of R3 per
burger. Alternatively, a cost saving of R2 per burger can be achieved by means of improved
supply-chain management without affecting the quality.
– Changing the preparation process can reduce the direct-labour costs by R1 per burger and the
variable overhead costs by R0,50.
– Other costs reductions of R1,50 per burger have been identified in the other cost components.

Based on the above cost-reduction approaches, without reducing the product quality, projected
costs can be reduced to R18. This achieves the target cost and means that the target price can be
used to sell the burgers.

As should be evident from the example of using the market basis to determine the selling price, this
method will often focus more on cost reduction than any other consideration because the selling price
will often be less than what would have been considered in a cost-based system. It should also be
apparent that even if market values are not used to determine selling prices, costs should be
continually analysed to ensure optimum performance.
In the next section, we will discuss how to use the cost-based method to determine selling prices.

22.4 Cost-based pricing


Unlike market-based pricing, where the price that a product can be sold for to the customer is first
determined, with cost-based pricing the selling price of a product or service is established on the
basis of the costs of that product or service. An important factor to take into account when using this
method of pricing is which costs to include as components. The organisation needs to decide from a
strategic point of view which costs are relevant when formulating its pricing decisions.
When using the cost-based pricing method, a markup (profit percentage) is added to the cost to
give the selling price. This method is useful when customised products are manufactured. A good
example is a toiletries manufacturer that tailor-makes products specially for hotel chains.
Some categories of costs that may be used when making pricing decisions are:
• total cost;
• production cost (which includes fixed overheads); and
• variable cost.

Example 22.3 illustrates the cost-based method of pricing.

EXAMPLE 22.3
Returning to the example of Burger City, the following figures are the expected unit cost based on
normal capacity for producing a burger:
R
Material 10
Direct labour 5
Variable overheads 3
Fixed overheads 2
Variable selling cost 2
Fixed administrative cost 1
Total 23

Required
Calculate the selling price using the cost-based method for the following variables:
1. Markup of 20 per cent on production cost.
2. Markup of 25 per cent on variable production cost.
3. Markup of 15 per cent on total cost.
4. Markup of 25 per cent on variable cost.

Solutions
1. Production cost plus 20% = selling price
R20 + (20% × R20) = R24
2. Variable production cost plus 25% = selling price
R18 + (25% × R18) = R22,50
3. Total cost plus 15% = selling price
R23 + (15% × R23) = R26,45
4. Variable cost plus 25% = selling price
R20 + (25% × R20) = R25

It is self-evident that if different cost categories are used as a basis to formulate the selling price,
it will result in different selling prices. The company needs to decide which of these cost bases
are appropriate to use. Production costs are typically used as the basis in an environment where
the largest cost component is the production cost and other non-manufacturing overheads are not
as significant. By contrast, total costs are often used if non-manufacturing overheads form a large
portion of the costs.
In Example 22.3, we can see that using variable production overheads as the basis results in a
selling price that is less than the actual costs (R23). Clearly, this would be financially
problematic.
A company should therefore establish which category of costs best illustrates the costs of
running the business, and then determine an appropriate markup based on that category to achieve
its targeted return on investment.
It is also important to remember that many of the fixed costs are determined based on a set
level of production or sales. The expected volume might therefore be an important aspect to
consider. (You may wish to refer to Chapter 19 on cost–volume–profit analysis to refresh your
memory.)
From Example 22.3, it becomes clear that it is important to determine the markup percentage
in a way that allows good pricing decisions to be made. Although there are other ways to
calculate the markup percentage, it is often calculated using the expected return on investment,
also referred to as return on capital.
The suppliers of capital to a business (the equity and debt holders) require a return for the
capital they have invested. The management of a business can calculate the figure that gives the
minimum return an entity needs to be able to give the suppliers of capital the return they expect.
This approach is discussed in Example 22.4.

EXAMPLE 22.4
Using the information given in Example 22.3, let us assume that R345 000 was invested into this
business and the expected rate of return is 20 per cent per annum. The company has forecast that
30 000 hamburgers will be produced and sold during the year.

Solution
Expected return on investment
R345 000 × 20% = R69 000

Expected total cost

R23 × 30 000 units = R690 000

Markup percentage = expected return ÷ total cost


R69 000 ÷ R690 000 = 10%

The selling price per unit can now be calculated:


Selling price per unit = total cost plus 10%
= R23 + 10% × R23
= R25,30

Compiling the income statement will show that a selling price of R25,30 per unit, and with 30
000 units sold, will provide a 20% return on R345 000 invested:
Sales = 30
R759 000
000 × R25,30
Cost = 30 000
690 000
× R23,00
Profit R69 000 (20% × R345 000)

CLOSING PRACTICAL EXAMPLE


Burger City needs to determine the most appropriate selling price for its burgers. It is in a very
competitive industry with multiple competitors in the area. Due to the competitive nature of the
industry, a market-based pricing strategy would be most appropriate. After conducting market
research, the company found that the target price should be R20 and that the target cost is R18.
Burger city will need to reduce its costs by R5 per unit to enable it to be competitive and
achieve its financial goals. These cost reductions can be achieved by various means, including
reducing the cost of materials, improving the productivity of labour and reducing overhead
expenditure.

22.5 Summary
In this final chapter of the book, we looked at methods for pricing products or services. Pricing
products and services is a key decision that managers need to make. Especially in competitive
markets, when formulating prices care should be taken to achieve the goals of the entity.
Prices can be determined either by the cost incurred in producing the product or providing the
service, or the price that customers are willing to pay for it. Therefore, an analysis of customers,
competitors and costs is very important.
Formulating a price, whether the method applied is based on the market or costs, is largely
determined by the projected costs. If the selling price is based on the market, these costs will need to
be reduced to achieve the target costs. On the other hand, if the selling price is based on costs, the
costing information is critical in determining the selling price.

Short questions
1. Briefly differentiate between market based and cost based pricing strategies.
2. Briefly define value engineering.
3. Assume that a product’s total production cost is R100 and the company uses a cost based
pricing strategy with a 30% markup on total production costs. Calculate the selling price per
unit for the product.
4. A company uses a market-based approach for calculating the selling prices of its products.
They have determined that an appropriate selling price for a product is R25. Based on their
required return the target cost for the product is R18, however the expected cost per unit is
R20. Briefly describe three ways in which the expected cost can be reduced to the target
cost.
5. Briefly describe the impact of customers on pricing strategy.

Long questions
1. JXT Chemicals produces shampoo. The expected unit costs based on normal capacity per
bottle is as follows:
R
Material 15
Direct labour 3
Variable overheads 7
Fixed overheads 4
Variable selling costs 4
Fixed administrative costs 1
Total 34
Calculate the selling price using the cost-based method for the following variables:
a. Markup of 30 per cent on production cost.
b. Markup of 35 per cent on variable production cost.
c. Markup of 20 per cent on total cost.
d. Markup of 30 per cent on variable cost.

2. Furniture Galore is a retailer selling furniture. One of their most successful ranges is their
‘Golden’ range of lounge suites. A lounge suite costs R3 500 per suite when purchased from
their supplier. They are currently reviewing their selling prices for the range.
The lounge suites currently sell for R4 500 per suite but their market research has
indicated that the quantity of sales will not be affected up to a price of R4 800 per suite. They
currently sell 40 lounge suites per annum. To achieve their required rate of return the gross
profit earned from the range needs to be R49 000 per annum.
a. Determine the selling price of the Golden range of lounge suites to achieve the required
gross profit based on the cost based pricing strategy.
b. Compare the value calculated in a) above to the market based price of R4 800 per lounge
suite and advise the company on the most appropriate price for the lounge suites.

3. KP Consulting services are trying to determine an appropriate hourly rate for their consulting
services. They have calculated that the consultant’s hourly cost to company is R150 per hour
and their variable costs per hour is R20. Their fixed costs are R120 000 per annum and they
believe that they will bill 8 000 hours per annum. The owners have invested R1 million into
the business and require a return of 30% per annum.
Determine the appropriate hourly rate of KP Consulting to enable the owners to achieve
their required return.

4. Wood Manufacturers Limited makes wooden dog houses. They use a market based pricing
policy and have set a target cost per unit based thereon. As the target cost is R20 lower than
the current cost per unit the following proposals have been made to achieve the target cost:
a. They could change their supplier of wood which will reduce their unit costs by R15 per
unit although the new supplier will require them to place larger orders.
b. They could use a poorer quality wood that will affect the longevity of the dog houses.
This will reduce the unit costs by R10 per unit.
c. They could change the manufacturing process of the dog houses without significantly
affecting the quality of the product which will reduce the unit costs by R5 per unit. This
will however require a once off capital cost of R15 000.
d. They could reduce their overhead costs by R8 per unit by reducing the number of staff
they employ.
Assessing each proposal individually discuss whether the proposal should or should not be
implemented. Clearly identify the benefits and drawbacks of every proposal.

5. King Dry Cleaners is the sole dry cleaning business in their area while Queen Dry Cleaners
operate in an area where there are a large number of competitors. The average price per dry
cleaning is currently R20.
Discuss the impact of the difference in their respective competitive environment would
have on the pricing strategies that they would follow when determining their average price
per dry cleaning.
References
CIMA (Chartered Institute of Management Accountants), 2005. CIMA Official Terminology . Oxford:
CIMA Publishing.
Cooper, R & Kaplan, R S, 1999. Design of Cost Management Systems, 2nd ed. Upper Saddle River:
Prentice Hall.
Drury, C, 2012. Management and Cost Accounting, 8th ed. Melbourne: Cengage Learning.
Els, G (ed.), 2010. Corporate Finance: A South African Perspective. Cape Town: Oxford University
Press.
Horngren, C T, Datar, S M & Foster, G. 2011. Cost Accounting: A Managerial Emphasis, 14th ed.
Prentice Hall.
Impala Platinum Holdings, 2012. Impala Platinum Holdings Limited Annual Report 2012. Illovo:
Impala Platinum Holdings.
International Accounting Standards Board, 1993. IAS 2. London: International Accounting Standards
Board.
Lovemore, F C H & Brummer L M, 2003. The ABC of Financial Management. Pretoria: Van Schaik.
Roos, S-A, 2008. Principles of Management Accounting. Cape Town: Oxford University Press.
Glossary
Abnormal gain: Losses during the manufacturing process that are lower than expected.
Abnormal loss: Losses during the manufacturing process that are higher than expected.
Absorption costing: Process whereby all variable and fixed manufacturing costs are included in the
costs used to determine the value of inventory.
Accounting rate of return (ARR): Average accounting profit that an investment will generate,
expressed as a percentage of the average annual investment in a project.
Activity: Task, action or unit of work carried out in an organisation.
Activity-based costing: An ‘approach to the costing and monitoring of activities which involves
tracing resource consumption and costing final outputs. Resources are assigned to activities, and
activities to cost objects based on consumption estimates. The latter utilise cost drivers to attach
activity costs to outputs’ (CIMA, 2005: 1).
Applied overheads: Overheads capitalised to a job using a predetermined method.
Average inventory: Normal amount of inventory on hand.
Batch-level activities: Activities performed each time a batch of goods is produced (e.g. machine
set-up).
Breakeven graph: Visual representation of cost and revenue at different levels of activity that shows
the profit or loss at the different levels within a range.
Breakeven point: Level of activity, expressed in value or quantity, at which a business neither makes
a profit nor a loss.
Breakeven units (quantity or volume): Lowest quantity of a product that must be sold in a particular
period to cover the total costs.
Breakeven value: Value of the units sold at breakeven point.
Budget: Quantitative expression of a proposed financial plan of action by management for a specific
period and an aid to coordinate what needs to be done to implement that plan.
Budget variance: Difference between actual overheads and budgeted overheads. Gives an indication
of the monetary value that is over- or underspent.
By-products: Products that are incidental to the production of products in a manufacturing process
and contribute a relatively small amount to the market value of the process outputs.
Capital budgeting: Analysis of investment opportunities in order to secure the most lucrative
alternative available to the entity. This helps the company to allocate its scarce resources.
Cash budget: Schedule of planned and expected receipts and payments for a budgeted period. It is
summarised and combined with any cash available at the beginning of the period to show the
planned cash balance available at the end of the period.
Certified work: Statements confirming the level of completion of a project issued by a qualified
professional.
Clock card: Document used to record time worked.
Completed contract method: Employed when profit is only recognised when the project is
completed, irrespective of the length of time taken to work on the project.
Consignment inventory: Inventory that is owned by one party but is in the possession of another
party.
Constraints: See ‘limiting factors’.
Contract costing: Type of job costing used where the job is on a large scale and for a long duration.
Contract costs: Those costs that are incurred in the contracts of the entity.
Contract price: Amount receivable by the contractor according to the agreement for the completion
of the contract.
Contribution margin: Difference between sales and variable costs.
Contribution margin ratio: Percentage of contribution over sales. This ratio can be expressed as a
percentage of total contribution over total sales or as a percentage of contribution per unit over
selling price per unit.
Contribution per unit: Amount remaining when the variable cost per unit is deducted from the selling
price per unit.
Cost accounting: System that measures, analyses and provides financial and non-financial
information regarding the costs incurred by the organisation.
Cost-based pricing strategy: One that determines the cost price of a product and then calculates the
selling price based on a predetermined markup percentage.
Cost centre: Production or service unit (department) whose costs can be allocated to that
department.
Cost driver: Factor that causes overhead costs to be incurred.
Cost objective: Product, customer, service, department, customer group, branch, etc., for which
costs are accumulated or measured.
Cost of production: Sum of all the direct and indirect costs incurred in the manufacturing of a
product or the rendering of a service.
Cost of production statement: Shows the calculation of the cost of production.
Cost of sales budget: summary of the direct-materials cost budget, the direct-labour cost budget and
the manufacturing-overheads budget.
Cost of sales statement: Shows the calculation of cost of sales.
Cost per unit: Total cost incurred to produce one product unit.
Cost pool: Grouping of different costs that have the same cost driver.
Cost to employer: Total of the gross remuneration of the employee plus all other costs relating to
labour that the employer incurs.
Cost–volume–profit (CVP) analysis: Study of the effects of costs (both fixed and variable), sales
price and volumes, and sales mix (number of units of each product line that is sold) on
profitability. CVP analysis is used to examine the relationship between sales volume, total costs,
total revenues and profits in a particular period.
Direct allocation method: Method whereby inter-service department allocations are ignored and all
service department costs are directly allocated to production departments.
Direct contract costs: Costs that can be directly traced to a particular contract.
Direct costing: Method where only variable manufacturing costs are included in the costs when
determining the value of inventory.
Direct costs: Total of direct material, direct labour and other direct expenses.
Direct-labour budget: Determines the expected direct-labour needs per product that the company
intends to produce.
Direct labour efficiency variance: Difference between the standard hours required for the actual
production and the hours taken at the standard labour rate per hour. It shows with the efficiency of
the labour force in the production process.
Direct labour mix variance: If the grades of labour used in production are different from those
budgeted for, there will be a labour mix variance.
Direct material mix variance: Difference between the total quantity of material in standard
proportion and the actual quantity of material, both measured at the standard price.
Direct material quantity variance: Difference between the standard quantity specified for actual
production and the actual quantity used at the standard purchase price.
Direct material yield variance: Difference between the standard yield of the actual material input
and the actual yield of the actual material input, both valued at the standard material cost.
Direct materials: Key components that are converted into the finished product (also known as
primary materials).
Direct-materials budget: Calculates the cost of the materials required for production, as planned in
the production budget.
Discounted payback period (DPBP): Number of years it takes to recover the initial investment using
discounted cash flows.
Duration driver: Cost driver used where the overhead costs vary with the length of time it takes to
perform an activity.
Economic inventory: Value or quantity of inventory that represents the amount of inventory that an
entity owns.
Economic order quantity (EOQ): Optimal inventory holding and reorder point.
Engineering studies: Analysis of each production operation to identify material specifications,
labour hours and variable overheads in order to set up the standard for a specific output.
Equivalent units: Partially completed units expressed as the equivalent of fully completed units.
Expansion project: One that entails introducing a new product or service, or introducing an existing
product or service into a new area.
Expected outputs: Number of units that should be produced based on the number of units that were
put into the process.
Extra work: Additional work requested by the client for a contractor to perform that is not included
in the contract price.
Facility-sustaining activities: Activities performed to maintain the organisation in general.
Financial accounting: System that focuses on the preparation of annual financial statements that fairly
represent the financial position and performance of the entity to external institutions, such as tax
authorities, banks, suppliers, shareholders and investors.
Financial factors: Quantitative factors that can be expressed in monetary terms.
Finished goods: Goods that are completed in the production process and ready for resale.
First in, first out (FIFO) method: Inventory is assumed to be issued in the order in which it is
received.
Fixed budget: Budget prepared at the beginning of a period for the planned level of activity.
Fixed overhead expenditure variance: Difference between the actual fixed overheads and the
budgeted fixed overheads. In the short run, these overheads are constant or independent of
production activity.
Fixed overheads: Overheads fixed within a specific range for a specific period and are not linked to
the number of units manufactured during that period.
Flexible budget: One that takes into account the change in the level of activity. It can be prepared for
a range of levels of activities at the beginning of the financial period or prepared at the end of the
financial period for the actual level of activity.
General head office costs: Costs incurred at the head office that are necessary for the company to
function, but are not project costs.
Goods-received note: Document used to acknowledge and record receipt of inventory.
Gross remuneration: Total of the amounts received as normal wages or salaries, overtime wages or
salaries and any other allowances received by the employee.
Idle time: Worker’s time that is not spent on the production process.
Independent projects: An investment made in one project does not prevent the business from
investing in another project. A business may therefore invest in a number of independent projects
simultaneously.
Indirect contract costs: Costs that cannot be attributed to a specific project.
Indirect costs: Expenses that are part of the manufacturing overheads.
Indirect labour: Cost of employees indirectly involved with production.
Indirect materials: Components used in the conversion of the primary material(s) into a finished
product (also known as secondary materials).
Infrastructure overheads: Depreciation of machinery, rental of machinery or factory premises,
repairs and maintenance of equipment, electricity and water costs etc.
Integrated accounting system: One in which cost and financial accounts are kept in the same set of
accounting records.
Intensity driver: Cost driver that recognises that certain types of products or services may use
resources more intensively than others.
Interim profit: Expected profit earned on an incomplete contract at the end of each accounting period
to avoid excess fluctuation in reported profits.
Interlocking accounting system: One in which two separate sets of accounting records, for cost and
for financial accounting, are kept.
Internal rate of return (IRR): The discount rate that equates the future value of cash flows over the
lifetime of the project to the initial investment. At this discount rate, the NPV (see below) of the
investment would be zero.
Inventory: The trading assets of an entity that are available for sale to customers, or are in the
process of manufacture, or the components used to manufacture a product.
Inventory holding costs: Total cost of holding inventory, including storage, labour and insurance
costs, and inventory losses.
Inventory in transit: Inventory that has been purchased but has not yet been received.
Inventory ordering costs: The costs incurred by ordering inventory, such as the labour cost of the
personnel responsible for placing orders, receiving inventory and paying for orders received and
administrative costs, like telephone bills.
Irrelevant income and cost: Income or cost that is not changed by financial decision.
Job card: Control document generated for each job.
Job costing: Calculating the cost of a specific job.
Job description: Specifications regarding a job that needs to be done.
Joint costs: Those that accumulate before the split-off point; made up of the total manufacturing costs.
Joint products: Items that are produced in large quantities and make a material contribution to the
market value of all outputs of the manufacturing process.
Just-in-time inventory system: One that keeps inventory holding costs to a minimum by organising
the delivery of inventory, and beginning the manufacture of a product at the precise point when it
is required.
Labour rate variance: Difference between the standard and actual direct labour rate per hour for the
total hours worked.
Lead time: Time it takes from when inventory is ordered to when it is delivered.
Leasing fees: Money paid to the owner of an asset (such as equipment) to acquire the right to use the
asset.
Limiting factors: Any input resource that is limited in supply and puts a constraint on the number of
units that can be manufactured or the number of services that can be provided (e.g. production
capacity of machinery) – also known as constraints and scarce resources.
Linear-programming graph: Graph constructed from the linear-programming model (see below);
can only be used for a maximum of two different products.
Linear programming model/technique: Short-term decision-making technique used to determine the
optimum combination of units to produce in a situation where scarce resources are consumed by
products.
Management accounting: System that measures, analyses and provides financial and non-financial
information to management.
Manufacturing overheads: Those that are incurred directly in the manufacturing process.
Margin of safety: Difference between the actual or budgeted sales volume or value and the
breakeven sales volume or value.
Market-based pricing strategy: One that determines selling price by estimating the price customers
are prepared to pay.
Markup: Percentage of profit that is to be added to cost to determine the selling price.
Master budget: An expression of management’s operating and financial plans for a specific period.
Material price variance: Difference between the actual and standard price per unit of materials for
the actual quantity of materials purchased.
Material quantity (or usage) variance: Difference between the actual quantity of materials used in
the production process and the standard quantity of materials that should have been used for the
specific output multiplied by the standard price of material.
Maximum inventory: Greatest possible quantity of a particular item that can be held by an entity at a
point in time.
Medical-aid-fund contribution: Contribution made by the employee to a medical-aid fund; the
amount is based on the number of dependants of the employee.
Mutually exclusive projects: Cash outlay in respect of one project results in the exclusion of a
second project. All similar mutually exclusive projects need to be evaluated to test their
profitability to the business.
Net present value (NPV): Difference between the present values of all future cash flows and the
initial investment. A positive NPV would indicate the profitability of the project.
Net remuneration: Amount received by an employee after certain deductions have been made from
gross remuneration.
Non-financial factors: Qualitative factors that do not have a monetary value.
Non-manufacturing overheads: Selling costs, distribution costs and administrative costs.
Normal costing: System in which total costs are calculated as actual direct material plus direct
labour plus applied overheads.
Normal hours: Minimum working hours that an employee needs to work in a specified period, as
determined by the contract of employment for the contracted wage or salary.
Normal inventory: Level of inventory that needs to be on hand to ensure that normal production can
continue.
Normal loss: A loss that is expected to be sustained during the normal manufacturing process.
Operating-expenditure budget: One that is used to estimate the operating expenses of the entity.
Operational budget: Financial plan for a period made up of various subcomponents.
Opportunity cost: Relevant cost that represents the opportunity that is lost when one financial option
is chosen instead of another.
Overhead costs: All costs incurred in the production process that cannot be allocated economically
directly to specific products, projects or services.
Overstocking: Situation that occurs when the amount of inventory held is above the quantity justified
by the volume of production.
Overtime hours: Hours worked during a specific period in excess of normal hours.
Overtime premium: Amount in excess of the normal rate per hour paid for overtime.
Pay as you earn (PAYE): Employee’s tax that the employer withholds from the employee’s
remuneration and pays to SARS.
Payback period (PBP): Number of years it takes to recover an investment using future cash flows.
Pension-fund contribution: Payment made employees from their salaries to a pension fund.
Percentage of completion method: System whereby profit is recognised in relation to the work
completed in each period.
Performance report: One in which the budgeted figures are compared with the actual figures.
Period costs: Those costs that are directly allocated to expenditure.
Periodic system: One in which acquisitions and reductions in inventory are not recorded in the
inventory account but are accounted for on a periodic basis.
Perpetual system: Type of inventory account that contains a current record of all inventory
transactions.
Process costing: System for calculating the average cost per unit in mass production.
Production budget: One that expresses the volume/quantities for each product expected to be
produced during the period.
Product-sustaining activities: Those activities that are performed to support product or service
lines.
Profit maximisation: Process of determining the optimum combination of units to produce in order to
make the maximum profit.
Raw materials: Component parts that are converted into the finished product in the production
process.
Relevant cost: A type of cost that is specifically linked to a financial or business decision under
consideration; excludes costs that are irrelevant to that decision and which could cloud the
decision-making process.
Remuneration: Compensation paid by employers to their employees in exchange for the services
they provide to the employer.
Repeated distribution method: One whereby service department costs are allocated repeatedly in
specified percentages until the remaining amount is too small to be material.
Replacement projects: Process in which an entity eliminates a project at the end of its useful life
(obsolescence) and substitutes it for another.
Requirement planning system: One that manages the ordering process to ensure inventory is
received on time.
Retail inventory: Finished products obtained from factories or wholesalers for resale purposes.
Retention money: A sum withheld in accordance with the contract until work has been completed in
a satisfactory manner or until a specified period has elapsed after the completion of the contract.
Allows the client to protect itself against defective work by contractors.
Revenue budget: A budget that sets out expected sales volumes of each product the company plans
to sell and the intended selling price.
Safety inventory: Amount of inventory that needs to be held to ensure that production can continue in
the event of an unforeseen situation that might inhibit the ability to procure raw materials in the
normal time frame.
Safety margin ratio: Expresses the margin of safety as a percentage of sales.
Salary: Remuneration paid to employees earning a fixed amount per month, as determined by their
contract of employment.
Sales margin price variance: Difference between the actual selling price and the standard selling
price for the actual volume of sales.
Sales margin volume variance: Difference between the actual sales volume and budgeted volume at
the standard contribution margin. The sales effort is taken into account by valuing it at the
contribution margin.
Sales mix variance: Difference between the actual sales quantity and the actual sales quantity in
budgeted proportion, valued at the standard margin.
Scarce resources: See ‘limiting factors’.
Semi-variable overheads: Overhead costs that have a variable and a fixed component.
Simultaneous equation method: One whereby an algebraic equation is formulated to solve the
amount of overheads that should be allocated to each production cost centre from service cost
centres.
Skills Development Levy (SDL): Compulsory levy scheme for the purpose of funding education and
training.
Specified order of closing method: One whereby management determines in which order service
department costs are closed off to the production cost centres.
Speculative inventory: Inventory that is held for economic reasons.
Split-off point: Point in a manufacturing process where one can identify different products.
Standard cost method: Inventory valuation by means of a predetermined standard unit price.
Standard costs: Targeted costs to be incurred under efficient operating conditions.
Standard hours: Number of hours that should be used in the manufacturing process under efficient
operating conditions.
Step-fixed overheads: Overheads that remains constant within a specific range of production but
increase to a new constant value if the number of units produced increases to a specific range.
Stock ledger card: Document to control raw-material inventory and completed goods inventory.
Stockout: Situation that occurs when inventory is not available for production purposes or when
there is demand for a product that is not available.
Stores issue voucher: Source document prepared by the inventory manager that is used to record
issues from materials store.
Strategic inventory: Inventory that is held for strategic reasons – for example, when supply may
become interrupted.
Sunk cost: Irrelevant cost incurred in the past and which will not influence a financial/business
decision.
Supplier invoice: Document prepared from the invoice received from the supplier.
Technical inventory: Value or quantity of inventory that is physically in the company’s possession.
Total contribution: Excess of sales revenue over total variable costs.
Total estimated cost: The most recent estimation of the total cost to be incurred to complete the
entire contract.
Total estimated profit: The contract price less the total estimated costs.
Total labour variance: Difference between the standard labour cost for production and the actual
labour cost.
Total material variance: Difference between the standard material cost for production and the actual
cost of material.
Total productive hours: Total number of hours available for work less annual leave, sick leave,
public holidays and idle time.
Total variable overhead variance: Difference between the actual variable overhead incurred and
the standard variable overheads charged to production.
Transaction driver: Cost driver that is based on the assumption that overhead costs are driven by the
number of times an activity is performed.
Uncertified work: Work completed at the end of a contractor’s financial accounting period of work
that has progressed since issuing the last certificate.
Understocking: Situation that occurs when the level of inventory falls below the quantity required by
the volume of production.
Unemployment Insurance Fund (UIF) contribution: payment made paid by the employer to the
government UIF fund; used to insure employees’ income.
Unit-level activities: Activities performed each time a unit of a product is produced or a service is
rendered.
Value engineering: Systematic evaluation of all aspects of the value chain, with the objective of
reducing costs and achieving a quality level that satisfies customers.
Variable cost ratio or marginal cost ratio: Percentage of variable costs over total sales.
Variable overhead efficiency variance: Depends on the overhead allocation method used. If direct
labour hours are used, this variance will show the difference between the standard hours required
for the output and the actual hours required at the standard overhead rate.
Variable overhead expenditure variance: Difference between the actual variable overhead costs
and the budgeted overhead costs for production.
Variable overheads: Overheads that vary directly in relation to the number of units produced.
Volume-based system: One that allocates overheads to cost objects using a single volume-related
basis, such as direct labour hours or machine hours. Such a system assumes that all overheads are
incurred in proportion to production volumes.
Volume variance: Difference between the overheads applied to inventory and the budgeted
overheads. Indicates that the planned activity and the denominator are not at the same level.
Wages: Remuneration paid to employees determined by the hours they worked during a certain
period (e.g. in a week) or as a predetermined amount for completing a certain task.
Weighted average method (WAM): Inventory valuation that uses the actual average purchase price
(weighted by the applicable quantities) to calculate the issue price.
Work in process (WIP): Inventory that comprises products that are in the process of being
manufactured.
Index
Page numbers in italics point to illustrations.

A
abnormal gains 188–189
accounting entries for recording wages and salaries 73–76
accounting for work in process losses in a WIP system 200–205
accounting profits, determining 335
accounting rate of return (ARR) 337–338
activity-based costing (ABC)
activity rates 260
advantages and disadvantages 255–256
appropriate time to introduce 263–264
definitions 254–255
implementation 256–261
in service and retail organisations 264
vs traditional volume-based costing systems 261–263
advanced inventory management systems 57–59

B
behavioural and technical considerations 8
breakeven point 361–364
breakeven value 363–364
budgetary control, flexible and fixed budgets 320–323
budget, definition 285
budgeting process 286–287
budget variance, defined 325
by-products 213, 220–222

C
capital budgeting
accounting profits, determining 335
capital-budgeting decisions, methods for making 336–344
investment projects, types of 333–335
cash budgets
basics of drawing up 306–307
defined 306
purpose 307–311
certified work 155
CIMA definition of management accounting 5–6
classification of costs related to products 17
completed contracts 160–161
comprehensive income, pro forma statement 298–299
constant growth profit percentage method 219–220
constraints see limiting factors
continuing or discontinuing products 404–406
contract costing
certified work 155
challenges 154–155
completed contracts 160–161
contract account 159
contract costs, recording 158–165
contract debtor account 160
direct contract costs 157
elements of contract costs 156
extra work 156
general head office costs 158
incomplete contracts 161–165
indirect contract costs 157–158
interim profit on incomplete contracts, recognising 165–172
material on site 156
retention money 156
uncertified work 155
see also job costing; process costing
contribution (marginal) income 358–361
contribution per unit 379–380
conversion cost of inventory 28
cost accounting
defined 5
guidelines 8
cost-accounting procure in manufacturing firms 119–121
cost allocation, in direct and absorption costing systems 269
cost and management accounting ethical issues 9
vs financial accounting 5
cost-based pricing 414–417
cost behaviour and CVP analysis 354–355
cost-benefit approach 8
cost centres
allocation of overheads to 94–96
described 92
see also cost objectives
cost classifications in manufacturing firms 116–117
cost, defined 12
cost drivers
in ABC systems 258–259
defined 92
cost elements see inventory, accounting for; inventory management and control; labour; overheads
cost flow and source documents for job costing 138
costing systems
job costing 136–148
timelines 91–92
cost management and accounting, roles and objectives 6
cost objectives
assigning costs to 14–17
categorising costs 13–14
see also cost centres
cost objects, in ABC systems 260–261
cost of production and cost of sales statement 122–123
cost of production, defined 114
cost pools, in ABC systems 256–258, 260–261
costs related to products, classification 17
cost statements
cost-accounting procedure in manufacturing firms 119–121
cost classifications in manufacturing firms 116–117
cost of production and cost of sales statement 122–123
flow of costs in manufacturing firms 117–118
integrated accounting systems 125–126
interlocking accounting systems 126–128
manufacturing process 115–116
product cost flow 114
reconciliation of cost-accounting and financial accounting profits 128–129
statement of financial performance/income statement 124–125
cost to employer 67–68
cost-volume-profit analysis (CVP)
accounting vs economist’s models 355–356
algebraic method of conducting breakeven analyses 371–373
analysis 353–357
application 364–370
breakeven point 361–364
breakeven value 363–364
changes in selling price 368–369
changes in variable and fixed costs 370
contribution (marginal) income 358–361
described 352–353
expected profit 365–366
limitations of 373
marginal (variable) costing and absorption-costing income statements 357–358
margin of safety 364–365
for multiple products 367–368
targeted profit 365–366

D
direct and absorption costing systems benefits and shortcomings 270–272
cost allocation 269
reconciling profits 277–278
statements of financial performance, drafting 272–277
direct costs
contracts 157
defined 14, 117
vs indirect, fixed and variable costs 16 direct labour activities 65
direct labour budgets 294–295
direct labour cost, recovery 71–72
direct labour mix variance 242
direct labour variances 235–236
direct material costs 25
direct material mix variance 240–241
direct materials budgets 293–294
direct materials in inventory 26
direct material variances 233–235
direct material yield variance 241
discounted payback period 340–341
discount rates 337

E
economic order quantity 53–57
equivalent units 194–196
ethical issues in cost and management accounting 9
expansion projects 334
expected profit 365–366
extra work 156

F
financial accounting
vs cost and management accounting 5
defined 4
financial and non-financial decisions in relevant costing 394–406
finished goods, inventory 18
first in first out method 31–32, 35–36
fixed costs
changes, and CVP analysis 370
as cost category 14
vs variable, direct and indirect costs 16
fixed manufacturing overheads 88
fixed overhead variances 325–326
flexible and fixed budgets
budgetary control 320–323
compared 317–318
fixed budgets, disadvantages 320
flexible budgets and overheads 323–326
flexible budgets, preparing 318–319
responsibility accounting and performance reports 320–323
variable overhead variances 324–325
flow of costs in manufacturing firms 117–118

G
general head office costs 158
gross remuneration 67

I
idle time 67
income, defined 12
income statement/statement of financial performance 124–125
incomplete contracts 161–165
interim profit, recognising 165–172
independent projects 334–335
indirect costs
contracts 157–158
defined 15, 117
vs direct, fixed and variable costs 16
indirect labour activities 66, 87
indirect materials 26, 87
infrastructure overheads 86
integrated accounting systems 125–126
interim profit on incomplete contracts, recognising 165–172
interlocking accounting systems 126–128
internal and external distribution and consumption of inventory 52–53
internal rate of return (IRR) 342–344
inventory, accounting for
accounting treatment 27–30
components in a manufacturing entity 25–26
conversion cost 28
cost of 27–30
defined 25
direct material costs 25
finished goods 18
ledger cards 33–38
materials 18
primary materials 26
purchase cost 27–28
retail inventory 18, 26
secondary materials 26
types of 18
valuation of 30–33
work in progress (WIP) 18, 26
inventory management and control
advanced systems 57–59
cost of holding inventory 53
cost of ordering inventory 54–55
economic order quantity 53–57
inventory-management cycle 45–53
just-in-time systems 58–59
need to order, identifying 49
norms for inventory levels, setting 46–49
ordering 49–50
reorder point, calculating 56–57
requirement planning systems 57–58
storage 50–52
investment projects, types 333–335

J
job costing
actual overheads 146–147
allocation of overheads 143–144
application 138
cost flow and source documents 138
job cards 137
job description and source documents used 139–147
labour, recording 142–143
material issued 140–142
normal costing 137
process costing 19, 137
purchase and receipt of materials 139–140
sale of finished goods 146
transfer to finished goods 144–145
see also contract costing; process costing
job description and source documents used 139–147
joint costs, methods of allocating 214–220
joint products 213, 214
joint products and by-products, process costing see process costing, joint products and by-products
just-in-time systems 58–59

L
labour
accounting entries for recording wages and salaries 73–76
as a cost element 64–68
direct labour cost, recovery of 71–72
recording, in job costing 142–143
remuneration payable, calculating 68–70
and total cost of production or service 72–73
labour costs as cost category 14
labour recovery rate 71–72
labour standards 230
limiting factors 379, 380–381
linear programming
concepts and principles 379–380
limitations as planning tool 384–387
limiting factors to consider 379, 380–381
model 381–384
long- and short-term pricing decisions 412

M
management accounting
defined 4, 5–6
opportunity cost 20
relevant cost 20
sunk cost 20
managing of organisations see organisational management
manufacturing overheads 85
manufacturing-overheads budgets 295–296
manufacturing process 115–116
marginal (variable) costing and absorption-costing income statements 357–358
margin of safety 364–365
market-based pricing 412–414
master budgets
cost-of-sales budgets 296–297
defined 288
direct labour budgets 294–295
direct materials budgets 293–294
manufacturing-overheads budgets 295–296
operating-expenditures budgets 297–298
other components 299
production budgets 292–293
pro forma statement of comprehensive income 298–299
revenue budgets 290–291
material costs, direct and indirect 25
material on site 156
materials
as cost category 13–14
inventory 18
material issued, in job costing 140–142
purchase and receipt of, in job costing 139–140
material standards 229–230
mutually exclusive projects 334

N
need to order, identifying 49
net present value 341–342
net realisable value method 218–219
net remuneration 67
non-manufacturing overheads 87–88, 90
normal hours 66
normal losses, process account 183–186
norms for inventory levels, setting 46–49

O
old equipment, keeping or replacing 400–402
operating-expenditures budgets 297–298
operational budgets
benefits and shortcomings 287–288
budgeting process 286–287
defined 285
master budgets 288–300
optimum product composition 380
ordering of inventory 49–50
organisational hierarchy of manufacturing cost variances 231
organisational management 3–4
overhead rates
for production cost centres, computing 100–102
products and services, allocating overhead costs to 102–103
overheads
actual, in job costing 146–147
allocating costs 91–108, 143–144
cost behaviour 88–89, 90
as cost category 14
defined 84
over- or under-absorption of, calculating 103–104
types of 85–88
overtime hours 66
overtime premium 66

P
payback period (PBP) 338–340
performance reports and responsibility accounting 320–323
periodic system for valuation of inventory 30–31
perpetual system for valuation of inventory 30–31
physical measure method 215–216
pricing decisions
cost-based pricing 414–417
market-based pricing 412–414
short- and long-term pricing decisions 412
primary materials in inventory 26
process costing
see also contract costing; job costing
process costing for joint products and by-products by-products 213, 220–222
constant growth profit percentage method 219–220
joint costs, methods of allocating 214–220
joint products 213, 214
net realisable value method 218–219
physical measure method 215–216
sales decisions 222–224
sales value at split-off point method 217–218
process costing for single products 177–206
abnormal gains 188–189
abnormal losses 185–187
accounting for incomplete production (work in process) 192–205
accounting for work in process losses in a WIP system 200–205
equivalent units 194–196
expected production: normal and abnormal losses, and abnormal gains 180–182
normal losses 183–186
process account 182–193
scrap value 183–189
work in process, opening 196–200
process costing vs job costing 19
process of allocating overhead costs 92–108
product cost flow 114
production budgets 292–293
production cost centres, computing overhead rates for 100–102
production cost variances
direct labour variances 235–236
direct material variances 233–235
fixed overhead expenditure variance 238
sales margin price variance 238
sales margin volume variance 238–239
standard cost cards and production budgets 231–233
total variable overhead variance 237
variable overhead efficiency variance 237
variable overhead expenditure (rate) variance 236–237
production departments, allocation of service department costs to 96–100
products, classification of costs related to 17
pro forma statement of comprehensive income 298–299
purchase cost of inventory 27–28
purpose of information 8

R
raw materials see materials
reconciliation of cost-accounting and financial accounting profits 128–129
reconciling profits in direct and absorption costing systems 277–278
relevant costing
continuing or discontinuing products 404–406
financial and non-financial decisions 394–406
limited resources 402–404
old equipment, keeping or replacing 400–402
outsourcing 398–400
relevant financial factors 393–394
special orders 395–398
remuneration payable, calculating 68–70
reorder point, calculating 56–57
replacement projects 333–334
requirement planning systems 57–58
responsibility accounting and performance reports 320–323
retail inventory 18, 26
retention money 156
revenue budgets 290–291

S
salaries 65
sale of finished goods, in job costing 146
sales decisions for joint or by-products 222–224
sales mix variances 242–244
sales value at split-off point method 217–218
scrap value 183–189
secondary materials 26
selling price change, and CVP analysis 368–369
semi-variable overheads 89
service department costs, allocation to production departments 96–100
short- and long-term pricing decisions 412
single products, product costing see process costing for single products
special orders and relevant costing 395–398
standard cost cards and production budgets 231–233
standard costing
advanced variances 239–244
method of valuing inventory 33
production cost variances 230–239
reconciliation statements 239
standard costs
defined 228
labour standards 230
material standards 229–230
standard overhead rate 230
standard hours 230
standard overhead rate 230
statement of financial performance, drafting 272–277
statement of financial performance/income statement 124–125
step-fixed overheads 89
storage of inventory 50–52
sunk costs 20, 394

T
timelines in a costing system 91–92
time value of money 336
total cost of production or service, labour cost element 72–73
transfer to finished goods 144–145
trial-and-error method 343–344

U
uncertified work 155
under- or over-absorption of overheads, calculating 103–104
unit costs 17–18

V
valuation of inventory 30–33
value-added tax (VAT) 20–21
variable costs
changes, and CVP analysis 370
as cost category 14
and fixed, direct and indirect costs 16
variable manufacturing overheads 88–89
variable overhead efficiency variance 237
variable overhead expenditure (rate) variance 236–237
variable overhead variances 324–325
volume-based costing systems, traditional vs activity-based costing 261–263
volume variance 325–326

W
wages 65
wages as a cost element, recording 75
weighted average method (WAM) 32–33, 36–37
work in process (WIP)
inventory 18, 26
opening 196–200

You might also like